You are on page 1of 369

Mathematical Olympiad in China (2011–2014) Downloaded from www.worldscientific.

com
by 91.124.253.25 on 05/31/18. For personal use only.

10113_9789813143746_tp.indd 1
25/1/18 5:05 PM
Mathematical Olympiad Series
ISSN: 1793-8570

Series Editors: Lee Peng Yee (Nanyang Technological University, Singapore)


Xiong Bin (East China Normal University, China)

Published

Vol. 15 Mathematical Olympiad in China (2011–2014):


Problems and Solutions
Mathematical Olympiad in China (2011–2014) Downloaded from www.worldscientific.com

edited by Bin Xiong (East China Normal University, China) &


Peng Yee Lee (Nanyang Technological University, Singapore)

Vol. 14 Probability and Expectation


by Zun Shan (Nanjing Normal University, China)
by 91.124.253.25 on 05/31/18. For personal use only.

translated by: Shanping Wang (East China Normal University, China)

Vol. 13 Combinatorial Extremization


by Yuefeng Feng (Shenzhen Senior High School, China)

Vol. 12 Geometric Inequalities


by Gangsong Leng (Shanghai University, China)
translated by: Yongming Liu (East China Normal University, China)

Vol. 11 Methods and Techniques for Proving Inequalities


by Yong Su (Stanford University, USA) &
Bin Xiong (East China Normal University, China)

Vol. 10 Solving Problems in Geometry: Insights and Strategies for


Mathematical Olympiad and Competitions
by Kim Hoo Hang (Nanyang Technological University, Singapore) &
Haibin Wang (NUS High School of Mathematics and Science, Singapore)

Vol. 9 Mathematical Olympiad in China (2009–2010)


edited by Bin Xiong (East China Normal University, China) &
Peng Yee Lee (Nanyang Technological University, Singapore)

Vol. 8 Lecture Notes on Mathematical Olympiad Courses:


For Senior Section (In 2 Volumes)
by Xu Jiagu (Fudan University, China)

The complete list of the published volumes in the series can be found at
http://www.worldscientific.com/series/mos

Suqi - 10113 - Mathematical Olympiad in China.indd 1 19-01-18 10:39:14 AM


Mathematical Olympiad in China (2011–2014) Downloaded from www.worldscientific.com
by 91.124.253.25 on 05/31/18. For personal use only.

10113_9789813143746_tp.indd 2
25/1/18 5:05 PM
Published by
East China Normal University Press
3663 North Zhongshan Road
Shanghai 200062
China

and

World Scientific Publishing Co. Pte. Ltd.


5 Toh Tuck Link, Singapore 596224
USA office: 27 Warren Street, Suite 401-402, Hackensack, NJ 07601
UK office: 57 Shelton Street, Covent Garden, London WC2H 9HE
Mathematical Olympiad in China (2011–2014) Downloaded from www.worldscientific.com

Library of Congress Cataloging-in-Publication Data


Names: Xiong, Bin. | Lee, P. Y. (Peng Yee), 1938–
Title: Mathematical Olympiad in China (2011–2014) : problems and solutions / editors, Bin Xiong,
East China Normal University, China, Peng Yee Lee, Nanyang Technological University, Singapore.
by 91.124.253.25 on 05/31/18. For personal use only.

Description: [Shanghai, China] : East China Normal University Press ; [Hackensack, New Jersey] :
World Scientific, 2017. | Series: Mathematical olympiad series ; vol. 15
Identifiers: LCCN 2017002034| ISBN 9789813143746 (hardcover : alk. paper) |
ISBN 9813143746 (hardcover : alk. paper) | ISBN 9789813142930 (pbk. : alk. paper) |
ISBN 9813142936 (pbk. : alk. paper)
Subjects: LCSH: International Mathematical Olympiad. | Mathematics--Problems, exercises, etc. |
Mathematics--Competitions--China.
Classification: LCC QA43 .M31453 2017 | DDC 510.76--dc23
LC record available at https://lccn.loc.gov/2017002034

British Library Cataloguing-in-Publication Data


A catalogue record for this book is available from the British Library.

Copyright © 2018 by East China Normal University Press and


World Scientific Publishing Co. Pte. Ltd.

All rights reserved. This book, or parts thereof, may not be reproduced in any form or by any means,
electronic or mechanical, including photocopying, recording or any information storage and retrieval
system now known or to be invented, without written permission from the Publisher.

For photocopying of material in this volume, please pay a copying fee through the Copyright Clearance
Center, Inc., 222 Rosewood Drive, Danvers, MA 01923, USA. In this case permission to photocopy
is not required from the publisher.

For any available supplementary material, please visit


http://www.worldscientific.com/worldscibooks/10.1142/10113#t=suppl

Typeset by Stallion Press


Email: enquiries@stallionpress.com

Printed in Singapore

Suqi - 10113 - Mathematical Olympiad in China.indd 2 19-01-18 10:39:14 AM


Pr
efac
e
Mathematical Olympiad in China (2011–2014) Downloaded from www.worldscientific.com

Thef
irs
ttime Ch
ina pa
rti
cipa
tedi
nIMO wa
sin1985,when
twos
tuden
tswe
res
entt
othe26
thIMO.S
ince1986,Ch
inaha
s
by 91.124.253.25 on 05/31/18. For personal use only.

at
eamo
fsi
xst
uden
tsa
teve
ryIMOexcep
tin1998wheni
twa
s
he
ldi
n Ta
iwan.Sof
ar,upt
o2014,Ch
inaha
sach
ievedt
he
numbe
rone r
ank
ing for 20 t
ime
sint
eam e
ffor
t. A g
rea
t
ma
jor
ityo
fst
uden
tsr
ece
ivedgo
ld meda
ls.Thef
actt
hatCh
ina
ob
tai
neds
uchencour
agi
ngr
esu
lti o,ont
sduet heonehand,
Ch
ine
ses
tuden
ts􀆳ha
rdworkandpe
rseve
rance,andont
heo
the
r
hand,t
hee
ffor
toft
eache
rsi
nschoo
lsandt
het
rai
ningo
ffe
red
by nat
ionalcoache
s. We be
lieve i
ti sals
o ar es
ultoft he
educa
tion s
yst
em in Chna,i
i n par
ticu
lar,the empha
sis on
t
rai
ningo
fba
sics
kil
lsi
nsc
ienceeduca
tion.
Thema
ter
ial
soft
hisbookcomef
romas
eri
eso
ffou
rbooks
(
inCh
ine
se)onFo
rwa oIMO :ACo
rdt lle
cti
ono
fMa
thema
tica
l
Ol
ymp
iadPr
obl 2011 2014).I
ems ( tisaco
llec
tionofpr
obl
ems
ands
olu
tion
soft
hema
jorma
thema
tica
lcompe
tit
ionsi
nCh
ina.
I
tprov
ide
sag
limps
eofhowt
heCh
inana
tiona
lteami
sse
lec
ted
and formed. Fi
rst, t
her
e i
s t
he Ch
ina Ma
thema
tica
l
Compe
tit
ion,ana
tiona
leven
t.I
tishe
ldont
hes
econdSunday
of Oc
tobe
reve
ry yea
r.Throught
hecompe
tit
ion,abou
t300
s
tuden
tsa
res
ele
ctedt
ojo
int
heCh
ina Ma
thema
tica
lOl
ymp
iad
(
common
lyknowna
sthewi
ntrcamp),ori
e tCMO,i
nshor n


Ma
thema
tic
alOl
ymp
iadi
nCh
ina

t
hef
oll
owi
ngJ
anua
ry.CMOl
ast
sforf
iveday
s.Bo
tht
het
ype
and the d
iff
icul
tyofthe pr
oblems match t
hos
e of IMO.
Simi
lary,s
l tudent
sar
egiventhr
ee prob
lemst osol
vein4.5
hour om CMO,abou
seach day.Fr t 50 t
o 60 s
tuden
tsa
re
s
ele
ctedt
oform ana
tiona
ltr
ain
ingt
eam.Thet
rai
ningt
ake
s
pl
acefortwo week
si nthe monthof Ma
rch.Af t
erfourt
osix
Mathematical Olympiad in China (2011–2014) Downloaded from www.worldscientific.com

t
ests,p
lustwoquali
fyingexami
natons,s
i ixs
tudentsa
refi
nal
ly
s
ele
ctedt
oformt
hena
tiona
lteam,t
aki
ngpa
rti
nIMOi
nJu
ly
oft
hatyea
r.
In v
iew oft
he d
iff
erence
sin educa
tion, cu
ltu
re and
by 91.124.253.25 on 05/31/18. For personal use only.

economy of we
ste
rn pa
rto
f Ch
inai
n compa
ris
on wi
tht
he
coa
sta
lpa
rti
nEa
stCh
ina,ma
thema
tica
lcompe
tit
ionsi
n We
st
Ch
inad
idno
tdeve
lopa
sfa
sta
sint
her
estoft
hecoun
try.In
orde
rtopr
omo
tet
heac
tiv
ityofma
thema
tica
lcompe
tit
ionand
t
oenhanc
ethel
eve
lofma
thema
tica
lcompe
tit
ion,s
tar
tingf
rom
2001,Ch
ina Ma
thema
tica
lOl
ymp
iad Commi
tteeor
gan
ize
sthe
Ch
ina We
ste
rn Ma
thema
tica
lOl
ymp
iad.Thet
optwo wi
nne
rs
wi
llbeadmi
ttedt
othe na
tiona
ltr
ain
ingt
eam.Throught
he
CWMO,t
her
e have beentwos
tuden
tsen
ter
ing t
he na
tiona
l
t
eamandr
ece
ivi
ngGo
ld Meda
lsfort
hei
rpe
rformancea
tIMO.
S
ince1995,f
ora qu
itel
ongpe
riodt
her
e wa
snof
ema
le
s
tuden
tint
he Ch
ine
se na
tiona
lteam.In orde
rtoencour
age
more femal
e student
s par
tic
ipa
ting i
n the ma
thema
tica
l
compe
tit
ion, s
tarting f
rom 2002, Ch i
na Mathema
tica
l
Ol
ymp
iadCommit
teeconduct
edtheChina Gi
rls
􀆳Mathemat
ical
Ol
ymp
iad.Aga
in,thetoptwowi
nnerswil
lbeadmit
teddir
ectl
y
i
ntot
hena
tiona
ltr
ain
ingt
eam.
Theau
thor
soft
hisbooka
recoache
soft
heCh
inana
tiona
l
t
eam.Theya
reXi n,LiShenghong,LengGang
ongBi song,Wu
J
ianp
ing, Chen Yonggao, Li We
igu, Yu Hongb
ing, Zhu


Pr
efa
ce

i,Feng Zh
Huawe igang, Li
u Sh
ixong, Qu Zhenhua, Wang
i
Weiye, and Zhang Si
hui. Thos
e who took partint he
t
ransl
ati
on work a r
e Xi ong Bin, Wang Shanp
ing, Liu
Yongmi
ng. Wea
reg
rat
efu
lt u Zonghu, Wang J
o Qi ie, Wu
J
ianp
ing,andPanChengb
iaofort
hei
rgu
idanceanda
ssi
stanc
e
t
oau
thor
s.Wea
reg
rat
efu
lto NiMi
ngofEa
stCh
ina Norma
l
Mathematical Olympiad in China (2011–2014) Downloaded from www.worldscientific.com

Un
ive
rsi
tyPr
ess.The
ire
ffor
tha
she
lped makeourj
obea
sie
r.
Wea
rea
lsog
rat
efu
ltoZhangJ
iof Wor
ldSc
ien
tif
icPub
lish
ing
f
orhe
rha
rdworkl
ead
ingt
othef
ina
lpub
lica
tionoft
hebook.
by 91.124.253.25 on 05/31/18. For personal use only.

XiongBin
Oc
tober2017


b2530   International Strategic Relations and China’s National Security: World at the Crossroads

This page intentionally left blank


Mathematical Olympiad in China (2011–2014) Downloaded from www.worldscientific.com
by 91.124.253.25 on 05/31/18. For personal use only.

b2530_FM.indd 6 01-Sep-16 11:03:06 AM


Int
roduc
tion
Ea
rlyday
s
Mathematical Olympiad in China (2011–2014) Downloaded from www.worldscientific.com

The In
terna
tiona
l Ma
thema
tica
l Ol
ymp
iad (IMO ),
by 91.124.253.25 on 05/31/18. For personal use only.

f n 1959,i
oundedi sone oft
he mo
stcompe
tit
iveand h
igh
ly
i
nte
llec
tua
lac
tiv
iti
esi
nthewor
ldforh
ighs
choo
lst
uden
ts.
Even be
f e IMO,t
or her
e we
rea
lready many coun
tri
es
wh
ich had ma
thema
tic
scompe
tit
ion.They we
re ma
inl
ythe
coun
tri
esi
n Ea
ste
rn Eu
ropeandi
n As
ia.In add
iti
ont
othe
popul
ariza
tion of mathemati
cs and the conver
gence in
educa
tionalss
ytemsamong d
iffer
entcoun
tri
es,thesucc
essof
ma
thema
tica
lcompe
tit
ion
satt
he na
tiona
lleve
l prov
ided a
founda
tion f
or t
he s
ett
ing
-up o
f IMO. The coun
tri
est
hat
a
sse
rtedg
rea
tinf
luenc
ear
e Hunga
ry,t
hef
orme
rSov
ietUn
ion
andt
heUn
itedS
tat
es.He
rei
sabr
iefr
evi
ew o
ftheIMO and
ma
thema
tica
lcompe
tit
ioni
nCh
ina.
In1894,t
heDepa
rtmen
tofEduca
tioni
nHunga
rypa
sseda
mo
tionandde
cidedt
oconduc
ta ma
thema
tica
lcompe
tit
ionfor
t
he s
econda
rys
choo
ls. The we
ll-known s
cien
tit, J. von
s
Et s,wa
övö stheMi
nis
tero
fEduca
tiona
ttha
ttime.Hi
ssuppor
t
i
nthe even
t had made i
tas
ucce
ss and t
husi
t wa
s we
ll
pub
lic
ized.Inadd
iton,t
i hesucce
ssofh
isson,R .vonEtövös,
whowa sal
soaphysi
cit,
s inprov
ingt
hepri
ncil
peofequi
valence
o
fthe gene
ralt
heor
yofr
ela
tiv
ity by A .Ei
nst
eint
hrough


Ma
thema
tic
alOl
ymp
iadi
nCh
ina

expe
r t,hadbrough
imen tHunga
ryt
othewor
lds
tagei
nsc
ience.
The
rea
ftr,t
e hepr
izef
orma
thema
tic
scompe
tit
ioni
n Hunga
ry
s named “Et
wa övö
s pr
ize”. Th
is wa
sthe f
irs
t forma
lly
or
gan
ized ma
thema
tica
lcompe
tit
ion i
nthe wor
ld.In wha
t
fo
llows,Hunga
ry hadi
ndeed pr
oduced al
otof we
ll-known
s
cien
tis
tsi
ncl
udng L .Fe
i jér,G .S
zegö,T.Radó,A .Haa
r
Mathematical Olympiad in China (2011–2014) Downloaded from www.worldscientific.com

and M .Ri
esz(
inr
ealana
lys
is),D .Kön
ig(
incomb
ina
tor
ics),
T.von Kármán (
inae
rodynami
cs),andJ.C .Ha
rs i(
any i
n
gamet y),whohada
heor lsowont
heNobe
lPr
izeforEconomi
cs
n1994.Theya
i llwe
ret
he wi
nne
rsof Hunga
ry ma
thema
tica
l
by 91.124.253.25 on 05/31/18. For personal use only.

compe
tit
ion.Thet
op s
cien
tif
ic gen
iuso
f Hunga
ry,J.von
Neumann,wa
soneo
fthel
ead
ing ma
thema
tic
ian
si he20
nt th
c
ent
ury.Neumann wa
sove
rsea
s wh
ilet
he compe
tit
ion t
ook
l
pace.La
terhed
idi
thims
elfandi
ttookh
im ha
lfanhourt
o
comp
let
e. Ano
the
r ma
thema
tic
ian wor
th men
tion
ing i
sthe
h
igh
lyproduc
tivenumbe
rtheor
istP.Erdö
s.Hewa
sapup
ilof
Fe
jér and a
lso a wi
nne
roft
he Wo
lf Pr
ize.Erdö
s wa
sve
ry
pa
ssi
ona
te abou
t ma
thema
tica
l compe
tit
ion and s
ett
ing
compe
tit
ionque
sti
ons.Hi
scon
tri
but
iont
odi
scr
etema
thema
tic
s
wa
s un
ique and g
rea
tlys
ign
ifi
can
t.The r
api
d pr
ogr
ess and
deve
lopmen
tofd
isc
ret
e ma
thema
tic
s ove
rthe s
ubs
equen
t
decade
shadi
ndi
rec
tlyi
nfl
uencedt
het
ype
sofque
sti
onss
eti
n
IMO.Ani
nte
rna
tiona
llyr
ecogn
ized pr
ize nameda
fte
r Erdö
s
wa
stohonourt
hos
e whohadcon
tri
but
edt
otheeduca
tionof
ma
thema
tica
lcompe
tit
ion.Pr
ofe
ssor Qi
u Zonghu f
rom Ch
ina
hadwont
hepr
i n1993.
zei
In1934,af
amou
s ma
thema
tic
ian B.De
lone conduc
teda
ma
thema
tica
lcompe
tit
ionf
orh
ighs
choo
lst
uden
tsi
nLen
ing
rad
(
nowS
t.Pe
ter
sbug).In1935,Mo
r scowa
lsos
tar
tedor
gan
izi
ng
s
ucheven
t.Ot
hert
hanbe
ingi
nte
rrup
teddur
ingt
he Wor
ld Wa
r


In
troduc
tion

I,t
I hes
eeven
tshad beenca
rri
edonun
tilt
oday.Asfort
he
Ru
ssi
an Ma
thema
tica
lCompe
tit
ion (
lat
err
enameda
stheSov
iet
Ma
thema
tica
lCompe
tit
ion),i
t wa
s no
tst
art
ed un
til1961.
s,t
Thu hef
orme
rSov
ietUn
ionand Ru
ssi
abecamet
hel
ead
ing
powe
rso
f Ma
thema
tica
lOl
ymp
iad.A l
otofg
randma
ste
rsi
n
ma
thema
tic
sinc
lud
ingt
he g
r t A .N .Ko
ea lmogo
rov we
rea
ll
Mathematical Olympiad in China (2011–2014) Downloaded from www.worldscientific.com

ve
ry en
thu
sia
sti
c abou
tthe ma
thema
tica
lcompe
tit
ion.They
wou
ld pe
rsona
llyi
nvo
lve i
nse
tti he que
ng t sti
ons for t
he
compe
tit
ion. The forme
r Sov
iet Un
ion even ca
lled i
tthe
Ma
thema
tica
l Ol
ymp
iad, be
liev
ing t
hat ma
thema
tic
sist
he
by 91.124.253.25 on 05/31/18. For personal use only.


gymna
sti
cso
fth
ink
ing”.The
se po
int
sofv
iew gavea g
rea
t
impac
tont
heeduca
tiona
lcommun
ity.Thewi
nne
roft
heFi
eld
s
Meda
lin1998,M .Kon
tse
vich,wa
sonc
ethef
irs
trunne
r-upof
t
he Ru
ssi
an Ma
thema
tica
l Compe
tit
ion. G . Ka
spa
rov,t
he
i
nte
rna
tiona
lche
ssg
randma
str,wa
e soncet
hes
econdr
unne
r-
up.Gr
igo
riPerelman,t
hewi nne
roftheFiel
ds Medalin2006
(bu
thedecl
ined),whosol
vedthePo
inca
ré􀆳
sConjecture,wasa
go
ldmeda
lis
to n1982.
fIMOi
In t
he Un
ited S
tat
esof Ame
rica, due t
othe ac
tive
promoti
onbyt her
enowned ma
themat
ican G .
i D.
Bir
kho f and
f
hi
ss on,toge
ther wi
th G .Pólya,the Pu
tnam mathemati
cs
compe
tit
ion wa
sor
gan
iz n1938f
edi orj
uni
orunde
rgr
adua
tes.
Manyoft
he que
sti
ons we
re wi
thi
nthes
cope o
fhi
ghs
choo
l
s
tuden
ts.Thet
opf
ivecon
tes
tan
tsoft
hePu
tnam ma
thema
tica
l
compe
tit
ion wou
ldbeen
tit
ledt
othe membe
rsh
ip ofPu
tnam.
Many o
fthe
se we
re even
tua
lly ou
tst
and
ing ma
thema
tic
ians.
The
rewe
ret
hef
amou ynman (
sR .Fe winne
roft
heNobe
lPr
ize
forPhy
sis,1965),K .Wi
c lson (
winne
roft
heNobe
lPr
izefor
Phy
sis,1982),J.Mi
c l r(
no wi
nne
roft
heFi
eld l,1962),
sMeda
D .Mumf
ord (wi
nne
roft
he Fi
eld l,1974),and D .
s Meda


Ma
thema
tic
alOl
ymp
iadi
nCh
ina

Qu
ilen (
l wi
nneroft
heFie
ldsMedal,1978).
Snce1972,i
i norde
rt o pr
epa
refort heIMO,t
he Un
ited
S
tat
es of Ame
rica Ma
thema
tica
l Ol
ymp
iad (USAMO ) wa
s
or
gan
ized.Thes
tanda
rd of que
sti
ons po
sed wa
s ve
ryh
igh,
pa
ral
lelt
otha
toft
he Wi
nte
rCampi
nCh
ina.Pr
iort
oth
is,t
he
Un
ited S
tat
es had or
gan
ized Ame
rican Hi
gh Schoo
l
Mathematical Olympiad in China (2011–2014) Downloaded from www.worldscientific.com

Ma
thema
tic
s Exami
nat
ion (AHSME ) for t
he h
igh s
choo
l
s
tuden
tss
i e1950.Th
nc iswa
satt
hej
uni
orl
eve
lye
tthe mo
st
popu
larma
thema
tic
scompe
tit
ioni
nAme
rica.Or
igi
nal
ly,
itwa
s
pl
anned tose
lec t 100 con
t abou t
est
ant
sf rom AHSME t o
by 91.124.253.25 on 05/31/18. For personal use only.

par
tic
ipa
teinUSAMO.Howeve r,duetothedi
scr
epancyi
nthe
l
eve
lofd
iff
icu
lty be
tween t
he two compe
tit
ion
s and o
the
r
r
est
ric
tions,f
rom 1983 onwa
rds, an i
nte
rmed
iat
eleve
l of
compe
tit
ion, name
ly, Ame
rican Inv
ita
tiona
l Ma
thema
tic
s
Exami
nat
ion (AIME ), wa
sin
troduced. Hence
for
th bo
th
AHSMEandAIMEbe
camei
nte
rna
tiona
lly we
ll-known.Af
ew
c
iti
esi
n Ch
ina had pa
rti
cipa
tedi
nthe compe
tit
ion and t
he
r
esu
ltswe
reencour
agi
ng.
S
imi
lar
lya
sint
hef
orme
rSov
ietUn
ion,t
he Ma
thema
tica
l
Olympiadeduca
tion was wi
delyrecogni
zedin Ame
rica.The
book “How toSolveit”writen by Geo
t rePo
g l
ya al
ong wi
th
many o
the
rti
tle
s had been t
rans
lat
ed i
nto many d
iff
eren
t
l s.Ge
anguage ore Po
g lya prov
ided a who
les
eri
es of gene
ral
heur
ist
icsfors
olv
ingpr
obl
emsofa
llk
ind
s.Hi
sinf
luencei
nthe
educa
tiona
lcommun
ityi
nCh
inashou
ldno
tbeunde
res
tima
ted.

I
nte
rna
tiona
lMa
thema
tic
alOl
ymp
iad
In1956,t
heEa
stEu
ropeancoun
tri
esandt
heSov
ietUn
ion
t
ookt
hei
nit
iat
ivet
o or
gan
izet
heIMO forma
lly.Thef
irs
t
In
terna
tiona
l Ma
thema
tica
l Ol
ymp
iad (
IMO ) wa
s he
ldi
n


In
troduc
tion

Br
asov,Roman
ia,i
n1959.Att ime,t
het her
ewe
reon
lys
even
pa
rti
cipat
ingcountri
es,namely,Romana,Bu
i l
gar
ia,Poland,
Hungary, Czecho
slovak
ia, Eas
t Germany and the Sovi
et
Un
ion.Sub
sequen
tly,t
he Un
ited S
tat
esof Ame
rica, Un
ited
ngdom,Fr
Ki ance,Ge
rmanyanda
lsoo
the
rcoun
tri
esi
ncl
udi
ng
t
hos
efr
om As
iaj
oined.Today,t
heIMOhadmanagedt
oreach
Mathematical Olympiad in China (2011–2014) Downloaded from www.worldscientific.com

a
lmo
sta
llt
hedeve
lopedanddeve
lop
ingcoun
tri
es.Exc
epti
n
t r1980 duet
heyea ofi
nanc
iald
iff
icu
lte
isf
aced by t
he ho
st
coun
try,Mongo
lia,t
her
ewe
rea
lready49 Ol
ymp
iad
she
ldand
97coun
tri
espa
rti
cipa
ting.
by 91.124.253.25 on 05/31/18. For personal use only.

The ma
themat
ica
lt op
icsinthe IMO i
ncl
ude Algebra,
Combi
nat
ori
cs,Geomety, Numbe
r rt heor
y.Theseareas had
prov
idedguidanc
ef orsett
ingquest
ionsfort
hecompetit
ions.
Otherthant
hef i
rstfew Olymp
iad ,
s eachIMOi snorma
llyheld
i
nmi
d-J
ulyeve
ryyea
randt
het
estpape
rcons
ist
sof6que
sti
ons
i
nal
l.Theac
tua
lcompe
tit
ionl
ast
sfor2day
sforat
ota
lof9
hour
swhe
re pa
rti
cipan
tsa
rer
equ
iredt
ocomp
let
e3 que
sti
ons
eachday.Eachque
sti
oni
s7po
int
swi
tht
ota o42po
lupt int
s.
Thef
ulls
cor
eforat s252 ma
eam i rks.Abou
tha
lf oft
he
par
tici
pan
ts wi
ll be awa l, whe
rded a meda re 1/12 wi
ll be
awardedagoldmedal.Thenumbersofgo
ld,si
lverandbronze
meda
lsawa
rdeda
rei
nther
ati
oof1:
2:3appr
oxima
tel
y.Int
he
ca
se when a pa
rti
cipan
t prov
ide
sa be
tte
rso
lut
ion t
han t
he
of
fic
ialan r,as
swe pec
ialawa
rdi
sgi
ven.
Eachpa
rti
cipa
tingcoun
try wi
llt
aket
urnt
oho
stt
heIMO.
Theco
sti
sbornebyt
heho
stcoun
try.Ch
inahads
ucc
ess
ful
ly
ho
st he31
edt stIMOi
n Be
iji
ng.Theeven
thad madea g
rea
t
impac
tont
hema
thema
tica
lcommun
ityi
nCh
ina.Ac
cord
ingt
o
t
her
ule
sandr
egu
lat
ion
so heIMO,a
ft llpa
rti
cipa
tingcoun
tri
es
a
rer
equ
ired t
osend a de
lega
tion cons
ist
ing o
f al r,a
eade

􀃸
Ma
thema
tic
alOl
ymp
iadi
nCh
ina

depu
tyl
eade
rand6con
tes
tan
ts.Thepr
obl
emsa
recon
tri
but
ed
byt
hepa
rti
cipa
tingcoun
tri
esanda
rel
ate
rse
lec
tedca
ref
ull
yby
t
heho
stcoun
tryf
ors
ubmi
ssi
ont
othei
nte
rna
tiona
ljur
yse
tup
by t
he ho
st coun
try.Even
tua
lly,on
ly 6 pr
obl
ems wi
ll be
ac
cep
tedf
oru
sei
nthecompe
tit
ion.Theho
stcoun
trydoe
sno
t
provide any quest
ion. The s hort
-li
sted probl
ems are
Mathematical Olympiad in China (2011–2014) Downloaded from www.worldscientific.com

subs
equent
lytrans
laed,i
t f nece
ssary,i n Eng
lih, Fr
s ench,
rman,Ru
Ge ssanando
i the
rwork
ingl
anguage
s.Af
tert
hat,t
he
t
eam l
eade
rs wi
llt
ran
sla
tet
he prob
lems i
ntot
hei
r own
l
anguage
s.
by 91.124.253.25 on 05/31/18. For personal use only.

The answe
rsc
rit
pso
f each pa
rti
cipa
ting t
eam wi
ll be
ma
rkedbyt
het
eam l
eade
randt
hedepu
tyl
eade
r.Thet
eam
l
eade
rwi
lll
ate
rpr
esen
tthes
cri
ptso
fthe
ircon
tes
tan
tst
othe
coord
ina
tor
sfora
sse
ssmen
t.I
fthe
rei
sanyd
ispu
te,t
hema
tte
r
wi
llbes
ett
ledbyt
hej
ury.Thej
uryi
sformedbyt
heva
riou
s
t
eaml
eade
rsandanappo
int
edcha
irmanbyt
heho
stcoun
try.
Thej
uryi
sre
spon
sib
lef
orde
cid
ingt
hef
i l6prob
na lemsfort
he
compe
tit
ion.The
ir du
tie
sal
soi
ncl
udef
ina
liz
ing t
he g
rad
ing
s
t rd,en
anda sur
ing t
he ac
cur
acy o
fthe t
ran
sla
tion of t
he
probems,s
l t
andard
izing r
epl
iest o wri
tten queri
esr a
ised by
part
icipan
tsdur
ingt hecompet
iton,s
i ynchroniz
ing d
ifferenc
es
i
ngr
adi
ngbe
tweent
het
eaml
eade
rsandt
hecoord
ina
tor
sand
a
lsodec
idi
ngont
hecu
t-o
ffpo
int
sfort
hemeda
lsdepend
ingon
t
hecon
tes
tan
ts􀆳r
esu
ltsa
sthed
iff
icu
lti
eso
fprob
lemseachyea
r
a
red
iff
eren
t.
Ch
inahadpa
rti
cipa
tedi
nforma
llyi
nthe26
t n1985.
hIMOi
On
lytwos
tuden
ts we
res
ent.S
tar
ti rom 1986,excep
ng f tin
1998whent
heIMO wa
she
ldi iwan,Ch
nTa inahada
lway
ssen
t
6 of
fic
ial con
tes
tan
tst
othe IMO. Today, t
he Ch
ine
se
con
tes
tan
tsno
ton
lype
rformedou
tst
and
ing
lyi
ntheIMO,bu
t

􀃹
In
troduc
tion

a
lsoi
ntheIn
terna
tiona
lPhy
sis,Chemi
c sty,Inf
r orma
tis,and
c
Bi
ology Ol
ymp
iad
s.Th
iscanber
ega
rdeda
sani
ndi
cat
iont
hat
Ch
ina pay
sgr
eata
tten
tiont
othet
rai
ning of ba
sicsk
ill
sin
ma
thema
tic
sands
cienc
eeduca
tion.

Wi
nne
rso
ftheIMO
Mathematical Olympiad in China (2011–2014) Downloaded from www.worldscientific.com

Amonga
llt
heIMO meda
lis
ts,t
her
e we
re many oft
hem
whoeven
tua
lly becameg
rea
t ma
thema
tic
ian
s.They we
rea
lso
awa
rdedt
heFi
eld l,Wo
sMeda lfPr
izeandNevan
linnaPr
ize (
a
promi
nen
tma thema
tic
spri
zeforcomput
ingandinforma
tis).
c
by 91.124.253.25 on 05/31/18. For personal use only.

Inwhatfol
lows,wenamesomeofthewinne
rs.
G . Ma
rgu
lis,a s
ilve
r meda
lis
to n 1959, wa
f IMO i s
awa
rdedt
heFi
eld
s Meda
lin1978.L .Lova
sz,who wont
he
Wo
lfPr
i n1999,wa
zei sawa
rdedt
heSpec
ialAwa
rdi
nIMO
cons
ecu
tive
l n1965and1966.V .Dr
yi ine
fld,ago
ldmeda
lis
tof
n1969,wa
IMOi sawa
rdedt
heF
iel
ds Meda
lin1990.J.-C .
Yoc
coz and T.Gowe
rs,who we
re bo
th awa
rdedt
he Fi
eld
s
Meda
lin1998,we
rego
ld meda
lis
tsi n1974and1981
nIMOi
r
espec
tive
ly.As
ilve
rmeda
lis
to n1985,L .La
fIMOi ffo
rgue,
wont
heFi
eld
sMeda
lin2002.Ago
ldmeda
lis n1982,
tofIMOi
Gr
igo
riPe
relmanf
rom Ru
ssa,wa
i sawa
rdedt
heF
iel
dsMeda
lin
2006f
ors
olv
ingt
hef
ina
lst
ep oft
hePo
inca
récon
jec
tur
e.In
1986,1987,and1988,Te
r eTao wonabronze,s
enc il r,and
ve
go
ld meda
lres
pec
tivel
y.He wa sthe younge
st par
tic
ipantto
da
teintheIMO,fi
rstcompe
tingattheageoft en.Hewa sal
so
awa
rdedt
heF
iel
dsMeda
lin2006.Go
ld meda
litofIMO1988
s
and1989, Ngo Bau Chao, won t
he Fi
eld
s Meda
lin 2010,
t
oge
the
r wi
th t
he bronze meda
lit of IMO 1988, E.
s
Li
nden
str
aus
s.Go
ld meda
lis
tofIMO 1994and1995,Ma
ryam
Mi
rzakhan
iwont
heF
iel
ds Meda
lin2014.A go
ld meda
lis
tof

􀃺
Ma
thema
tic
alOl
ymp
iadi
nCh
ina

n1995,Ar
IMOi turAv
ilawont
heFi
eld
sMeda
lin2014.
As
ilve
rmeda
lis
to n1977,P.Sho
fIMOi r,wa
sawa
rded
t
he Nevan
linna Pr
ize.A go
ld meda
lis
to n1979,A .
fIMO i
Razbo
rov,wa
sawa
rdedt
he Nevan
linna Pr
ize.Ano
the
r go
ld
meda
lis
tofIMOi rnov,wa
n1986,S.Smi sawa
rdedt
heCl
ay
Re
sea
r rd.V .La
ch Awa ffo
rgue,a go
ld meda
lis
tofIMO i
n
Mathematical Olympiad in China (2011–2014) Downloaded from www.worldscientific.com

1990,wa
sawa
rdedt
heEur
opean Ma
thema
tica
lSoc
iet
ypr
ize.
sL .La
Hei ffo
rgue􀆳
syounge
rbr
othe
r.
Al
so,a f
amou
s ma
thema
tic
ian i
n numbe
rt y, N .
heor
El
kis,whoi
e sal
so a pro
fes
sora
t Ha
rva
rd Un
ive
rsi
ty, wa
s
by 91.124.253.25 on 05/31/18. For personal use only.

awa
rdedago
ld meda n1982.Ot
lofIMOi herwi
nne
rsi
ncl
ude
P.Kr
onhe
ime
rawa
rdedas
ilve
rmeda
lin1981andR .
Tay
lora
con
tes
tan
to n1980.
fIMOi

Ma
thema
tic
alc
ompe
tit
ioni
nCh
ina
Duet
ova
riou
srea
sons,ma
thema
tica
lcompe
tit
ioni
nCh
ina
s
tar
tedr
ela
tive
lyl
atebu
tisprog
res
singv
igorou
sly.
“Wea
rego
ingt
ohaveourown ma
thema
tica
lcompe
tit
ion
oo!”s
t aid HuaLuog
eng.Huai
sahou
se-ho
ldnamei
nCh
ina.
Thef
irs
t ma
thema
tica
lcompe
tit
ion wa
she
ldconcur
ren
tlyi
n
Be
ijng,Ti
i ani
jn,Shangha n 1956.Duet
iand Wuhani othe
pol
iti
cals
ituat
iona
tt ime,t
het h
iseventwasinte
rrup
tedaf ew
t
imes.Un ti ,
l1962 whenthe pol
iti
calenv
ironmentsta
rtedto
improve, Be
iji
ng and o
the
rci
tie
sst
art
ed or
gan
izi
ng t
he
compe
tit
ion t
hough no
tregu
lar
ly.In t
he e
ra of Cu
ltur
al
Revo
lut
ion,t
he who
le educa
tiona
lss
ytem i
n Ch
ina wa
sin
chao
s.Thema
thema
tica
lcompe
tit
ioncamet
oacomp
let
eha
lt.
Incon
tra
st,t
hema
thema
tica
lcompe
tit
ioni
nthef
orme
rSov
iet
Un
ionwa
sst
il
lon
-go
ingdur
ingt
hewa
randa
tat
imeunde
rthe
d
iff
icu
ltpo
lit
ica
lsi
tua
tion.Thecompe
tit
ionsi
n Mo
scow we
re

􀃻
In
troduc
tion

i
nte
rrup
tedon
ly3t
ime tween1942and1944.I
sbe twa
sindeed
commendab
le.
In 1978,i
t wa
sthe s
pri
ng o
fsc
ience. Hua Luog
eng
conduc
tedt
he Mi
ddl
eSchoo
l Ma
thema
tica
lCompe
tit
ionfor8
prov
inc
esi
n Ch
ina.The ma
thema
tica
lcompe
tit
ioni
n Ch
ina
wa
sthenmak
ingaf
reshs
tar
tandemba
rkedonaroado
frap
id
Mathematical Olympiad in China (2011–2014) Downloaded from www.worldscientific.com

deve
l t.Hua pa
opmen ss n 1985.In commemor
ed awayi a
ting
him,acompet
itonnamedHuaLuog
i eng Go
ldCupwasse
tupi n
1986fors
tuden
tsi
nGr
ade6and7andi
tha
sag
rea
timpac
t.
The ma
thema
tica
lcompe
tit
ion
sin Ch
inabe e1980can
for
by 91.124.253.25 on 05/31/18. For personal use only.

becons
ide
red a
sthei
nit
ial pe
riod.The pr
obl
ems we
res
et
wi
thi
nthes
copeo
f mi
ddl
eschoo
ltex
tbook
s.Af
ter1980,t
he
compe
tit
ionswe
reg
radua
lly mov
ingt
owa
rdst
hes
eni
or mi
ddl
e
s
choo
ll l.In 1981, t
eve he Ch
ine
se Ma
thema
tica
l Soc
iet
y
de
cidedt
o conduc
tthe Ch
ina Ma
thema
tica
l Compe
tit
ion,a
na
tiona
leven
tforh
ighs
choo
ls.
In1981,t
heUn
itedS
tat
esofAme
rica,t
heho
stcoun
tryof
IMO,i
ssuedani
nvi
tat
iont
oCh
inat
opa
rti
cipa
tei
ntheeven
t.
On
l n 1985, Ch
yi ina s
ent two con
tes
tan
tst
o pa
rti
cipa
te
i
nforma
llyi
ntheIMO.Ther
esu
ltswe
reno
tencour
agi
ng.In
v
iew oft
his,ano
the
r ac
tiv
ity ca
lled t
he Wi
nte
r Camp wa
s
conduc
ted a
fte
rthe Ch
ina Ma
thema
tica
l Compe
tit
ion. The
Wi
nte
rCamp wa
sla
terr
enamed a
sthe Ch
ina Ma
thema
tica
l
Ol
ymp
iadorCMO.The wi
nni
ngt
eam wou
ldbeawa
rdedt
he
Che
rnSh
iing
-Shen Cup.Ba
sedont
heou
tcomea
tthe Wi
nte
r
Camp,a s
ele
cti
on wou
ld be made t he 6
o form t -membe
r
na
tiona
lteam f om 1986 onwa
orIMO.Fr rds,o
the
rthant
he
year when IMO was or
gani
zedi iwan, Ch
n Ta i
na had been
send
inga6 -membe
rteamt oIMO.Upto2011,Chinahadbeen
awa
rdedt
heove
ral
lteamchamp
i or17t
onf ime
s.


ⅴⅱ
Ma
thema
tic
alOl
ymp
iadi
nCh
ina

In1990,Ch
inahads
ucc
ess
ful
ly ho
st he31s
edt tIMO.I
t
s
howedt
hatt
hes
tanda
rdo
fma
thema
tica
lcompe
tit
ioni
nCh
ina
ha
sleve
ledt
hatofo
the
rlead
ingcoun
tri
es.F
irt,t
s hef
actt
hat
Ch
inaach
ieve
stheh
ighe
stma
rksa
tthe31s
tIMOfort
het
eam
i
sanev
idenceoft
hee
ffe
cti
vene
sso
fthepy
rami
dappr
oachi
n
s
ele
cti
ng t
he con
tes
tan
tsi
n Ch
ina.Second
ly,t
he Ch
ine
se
Mathematical Olympiad in China (2011–2014) Downloaded from www.worldscientific.com

ma
thema
tic
ian
shads
imp
lif
iedand mod
ifi r100pr
edove obl
ems
ands
ubmi
tt hemt
edt othet
eaml
eade
rso
fthe35coun
tri
esfor
t
hei
rpe
rus
al.Even
tua
lly,28pr
obl
emswe
rer
ecommended.At
heend,5pr
t obl
emswe
recho
sen (
IMOr
equ
ire
s6 pr
obems).
l
by 91.124.253.25 on 05/31/18. For personal use only.

Th
isi
sano
the
rev
idencet
oshowt
hatCh
inaha
sach
ievedt
he
h
ighe
stqua
lit
yins
ett
ingpr
obl
ems.Th
ird
ly,t
heanswe
rsc
rit
ps
oft
hepa
rti
cipan
tswe
rema
rkedbyt
heva
riou
steaml
eade
rsand
a
sse
ssedbyt
hecoord
ina
tor
swho we
renomi
nat
edbyt
heho
st
coun
tri
es.Ch
ina had formed a g
roup 50 ma
thema
tic
ianst
o
s
er scoord
vea ina
tor
swho wou
lden
sur
etheh
ighac
cur
acyand
f
airne
ssi
nma
rki
ng.Thema
rki
ngproc
esswa
scomp
let
edha
lfa
dayea
rli
ert
hani
twa
sschedu
led.Four
thy,t
l hatwa
sthef
irs
t
eve
rIMO or
gan
izedi
n As
ia.Theou
tst
and
ingpe
rformanc
eby
Ch
inahadencour
agedt
heo
the
rdeve
lop
ingcoun
tri
es,e
spec
ial
ly
t
hos
ein As
ia.The or
gan
izi
ng andcoord
ina
ting work oft
he
IMObyt
heho
stcoun
try wa
sal
sor
eas
onab
lygood.
In Ch
ina,t
he ou
tst
and
ing pe
rformancei
n ma
thema
tica
l
compe
tit
ioni
sar
esu
ltof many con
tri
but
ion
sfr
om t
he a
ll
quar
tersof ma
themat
ical communi
ty. Ther
ea rethe ol
der
gener
ati
onofmathema
tic
ians,midd
le-agedmathemat
ici
ansand
a
lsot
he mi
ddl
eande
lemen
tar
yschoo
lteache
rs.The
rei
sone
pe
rsonwhode
ser
vesas
pec
ialmen
ti sHuaLuog
onandhei eng.
Hei
nit
iat
edandpr
omo
tedt
hema
thema
tica
lcompe
tit
ion.Hei
s
a
lsot
he au
thoro
fthef
oll
owi s:Beyond Yang hu
ng book i􀆳s


ⅴⅲ
In
troduc
tion

Tr
iang
le,Beyondt i of Zu Chong
he p zhi,Beyondt
he Mag
ic
Compu
tat
ion of Sun
-zi, Ma
thema
tica
l Induc
tion, and
Ma
thema
tica
lProb
lems of Bee Hi
ve.The
se we
reh
is books
de
rivedf
rom ma
thema
tic
scompe
tit
ion
s.When Ch
inar
esumed
ma
thema
tica
lcompe
tit
i n 1978,he pa
oni rti
cipa
tedi
nse
tti
ng
prob
lemsandg
ivi
ngc
rit
iquet
oso
lut
ion
soft
heprob
lems.Ot
her
Mathematical Olympiad in China (2011–2014) Downloaded from www.worldscientific.com

ou
tst
and
ing books de
rived f
rom t
he Ch
ine
se ma
thema
tic
s
compe
tit
ionsa
re:Symme
tr fu,La
yby DuanXue tti
ceandAr
ea
by Mi
nShe,OneS
i troke Dr
awi
ng and Po
stman Prob
lem by
J
iangBo
ju.
by 91.124.253.25 on 05/31/18. For personal use only.

Af
ter1980,t
he younge
r ma
thema
tic
iansi
n Ch
ina had
t
aken ove
rfrom t
he o
lde
r gene
rat
ion o
f ma
thema
tic
iansi
n
r
unn
ing t
he ma
thema
tica
l compe
tit
ion. They worked and
s
tri
vedha
rdt
obr
ingt
hel
eve
lof ma
thema
tica
lcompe
tit
ioni
n
Ch
inat
oa new he
i t.Qi
gh u Zonghu i
sones
uch ou
tst
and
ing
r
epr
esen
tat
ive.Fromt
het
rai
ningofcon
tes
tan
tsandl
ead
ingt
he
eam 3t
t ime
stoIMO t
othe or
gan
izi
ng o
fthe31
thIMO i
n
Ch
ina,hehadcon
tri
but
edpr
omi
nen
tlyandwa
sawa heP.
rdedt
Erdö
spr
ize.

Pr
epa
rat
ionf
orIMO
Cur
ren
tly,t
hes
ele
cti
onproc
essofpa
rti
cipan
tsf
orIMOi
n
Ch
inai
sasfo
llows.
Fi
rst,t
he Ch
ina Ma
thema
tica
l Compe
tit
ion,a na
tiona
l
compe
tit
ionforh
ighSchoo
ls,
isor
gan
izedont
hes
econdSunday
i
n Oc
tobe
r eve
ry yea
r.The ob
jec
tive
sae:t
r oinc
rea
set
he
i
nte
res
tofs
tuden
tsi
nlea
rni
ng ma
thema
tis,t
c o pr
omo
tet
he
deve
lopmen to
fco -
curri
cula
racti
vit
iesin ma
thema
tics,tohe
lp
improve t
he teaching of ma
thematic
si nh i
gh schoo
ls,to
d
iscove
randcu
lti
vat
ethet
alen
tsanda
lsot
o pr
epa
ref
ort
he


ⅰⅹ
Ma
thema
tic
alOl
ymp
iadi
nCh
ina

IMO.Th
is happens s
ince 1981. Cur
ren
tlyt
her
ear
e abou
t
200,
000pa
rti
cipan
tst
aki
ngpa
rt.
Throught
heCh
ina Ma
thema
tica
lCompe
tit
ion,a
round350
o
fst
uden
tsa
res
ele
ctedt
otakepa
rti
ntheCh
ina Ma
thema
tica
l
Ol
ymp
iadorCMO,t
ha s,t
ti he Wi
nte
rCamp.TheCMOl
ast
s
for5 day
sandi
she
ldi
nJanua
ry eve
ryyea
r.Thet
ype
sand
Mathematical Olympiad in China (2011–2014) Downloaded from www.worldscientific.com

d
iff
icu
lti
eso
fthe prob
lemsi
n CMO a
res
imi
lart
otheIMO.
The
rea
rea
lso3prob
lemst
obecomp
let
edwi
thi 5hour
n4. seach
r,t
day.Howeve hescor
eforeach prob
lemis21 ma rkswh
ich
addupto126 ma
rksintot
al.Sta
rtingfrom 1990,t
he Winte
r
by 91.124.253.25 on 05/31/18. For personal use only.

Camp inst
itu
t he Che
ed t rn Sh
iing-
Shen Cup for team
champ
ionsh
ip.In 1991, t
he Winte
r Camp was off
ici
all
y
r
enameda
sthe Ch
ina Ma
thema
tica
lOl
ymp
iad (CMO).I
tis
s
imi
lart
otheh
ighe
stna
tiona
lma
thema
tica
lcompe
tit
ioni
nthe
forme
rSov
ietUn
ionandt
heUn
itedS
tat
es.
The CMO awa rdst
he f
irt,s
s econd and th
ird pri
zes.
Amongthepa
rti
cipant
sofCMO,about60studen
tsaresel
ected
t
opa
rti
cipa
tei
nthet
rai
ningf
orIMO.Thet
rai
ningt
ake
spl
ace
i
n Ma
rcheve
ryyea
r.Af
ter6t
o8t
est
sandano
the
r2round
sof
qua
lif
yingexami
nat
i s,on
on ly6con
tes
tan
tsa
reshor
t-l
ist
edt
o
f
ormt
heCh
inaIMO na
tiona
lteamt
otakepa
rti
ntheIMOi
n
J
uly.
Be
side
sthe Ch
ina Ma
thema
tica
l Compe
tit
ion (
for h
igh
s
choo
ls),t
heJ
uni
or Mi
ddl
eSchoo
l Ma
thema
tica
lCompe
tit
ion
i
sal
sodeve
lop
ing we
ll.S
tar
tingf
rom1984,t
hecompe
tit
ioni
s
or
gan
izedi
nApr
ileve
ryyea
rbyt
hePopu
lar
iza
tionCommi
ttee
oft
he Ch
ine
se Ma
thema
tica
lSoc
iet
y.Theva
riou
sprov
i s,
nce
c
iti
esandau
tonomou
sreg
ion
swou
ldro
tat
etoho
stt
heeven
t.
Ano
the
rma
thema
tica
lcompe
tit
ionf
ort
hej
uni
ormi
ddl
eschoo
ls
i
sal
so conduc
tedi
n Apr
ileve
ry yea
r by t
he Mi
ddl
e Schoo
l



In
troduc
tion

Ma
thema
tic
s Educa
tion Soc
iet
yoft
he Ch
ine
se Educa
tiona
l
Soc
iet
ysnce1998t
i il
lnow.
The HuaLuog
eng Go
ldCup,acompe
tit
ionbyi
nvi
tat
ion,
had a
lso been s
ucc
ess
ful
ly conduc
ted s
i e 1986. The
nc
pa
rti
cipa
tings
tuden
tscompr
isee
lemen
tar
ysi
xandj
uni
ormi
ddl
e
ones
tuden
ts.The f
orma
t oft
he compe
tit
ion cons
ist
s of a
Mathematical Olympiad in China (2011–2014) Downloaded from www.worldscientific.com

pr
elimi
nar
yround,semi-
fina
lsinvar
iou
sprov
i s,c
nce iti
esand
au
tonomousr
egi s,t
on henthefi
nal
s.
Ma
thema
tica
lcompe
tit
ioni
nCh
inaprov
ide
sap
lat
formf
or
s
tuden
tst
oshowca
set
hei
rta
len
tsi
nma
thema
tic
s.I
tencour
age
s
by 91.124.253.25 on 05/31/18. For personal use only.

l
earn
ing o
f ma
thema
tic
s among s
tuden
ts.I
t he
lps i
den
tif
y
t
alen
ted s
tuden
ts and t
o prov
ide t
hem wi
th d
iff
eren
tia
ted
l
earn
ing oppor
tun
ity.I
t deve
lops co
-cur
ricu
lar ac
tiv
iti
esi
n
ma
thema
tic
s.F
ina
lly,i
tbr
ing
sabou
tchange
sint
het
each
ingof
ma
thema
tic
s.


ⅹⅰ
b2530   International Strategic Relations and China’s National Security: World at the Crossroads

This page intentionally left blank


Mathematical Olympiad in China (2011–2014) Downloaded from www.worldscientific.com
by 91.124.253.25 on 05/31/18. For personal use only.

b2530_FM.indd 6 01-Sep-16 11:03:06 AM


Con
ten
ts
Mathematical Olympiad in China (2011–2014) Downloaded from www.worldscientific.com

Pre
face ⅲ
I
ntr
oduct
ion
by 91.124.253.25 on 05/31/18. For personal use only.

Ch
inaMa
thema
tic
alCompe
tit
ion 1

2010 (
Fui
jan) 1
2011 ( i)
Hube 13
2012 ( i)
Shaanx 23
2013 (
Ji
lin) 35

Ch
inaMat
hema
tic
alCompe
tit
ion (
Comp
lemen
tar
y
Tet)
s 44

2010 (
Fui
jan) 44
2011 ( i)
Hube 51
2012 ( i)
Shaanx 56
2013 (
Ji
lin) 62

Ch
inaMa
thema
tic
alOl
ymp
iad 68

2011 (
Changchun,J
iln)
i 68
2012 (
Xian,Shaanx
i) 77
2013 (
Shenyang,Li
aon
ing) 86
2013 (
Nan
jing,J
iang
su) 99


ⅹⅲ
Ma
thema
tic
alOl
ymp
iadi
nCh
ina

Ch
inaNa
tiona
lTe
amS
ele
cti
onTe
st 109
2011 (
Fuzhou,Fu
jian) 109
2012 (
Nanchang,J
i i)
anx 119
2013 (
Jiangy
in,J
iang
su) 131
2014 (
Nan
jing,J
iang
su) 145
Mathematical Olympiad in China (2011–2014) Downloaded from www.worldscientific.com

Ch
inaGi
rls
􀆳Ma
thema
tic
alOl
ymp
iad 160
2010 (
Shi
jiazhuang,Hebe
i) 160
2011 (
Shenzhen,Guangdong) 174
2012 (
Guangzhou,Guangdong) 190
by 91.124.253.25 on 05/31/18. For personal use only.

2013 ( i,Zhe
Zhenha jiang) 199

Ch
ina We
ste
rn Ma
thema
tic
alOl
ymp
iad 211
2010 (
Tayuan,Shanx
i i) 211
2011 (
Yushan,J
i i)
angx 221
2012 ( t,Inne
Hohho rMongo
lia) 230
2013 (
Lanzhou,Gan
su) 240

Ch
inaSou
the
ast
ern Ma
thema
tic
alOl
ymp
iad 252
2010 (
Lukang,Changhua,Ta
iwan) 252
2011 (
Ningbo,Zhe
jiang) 265
2012 (
Put
ian,Fu
jian) 278
2013 (
Ying
tan,J
i i)
angx 286

I
nte
rna
tiona
lMa
thema
tic
alOl
ymp
iad 299
2011 (
Ams
terdam,Ho
lland) 299
2012 (Ma
rDe
lPl
ata,Ar
gen
tina) 311
2013 (
San
ta Ma
rta,Co
lomb
ia) 321
2014 (CapeTown,TheRepub
lico
f
Sout
hAfrca)
i 336


ⅹⅰⅴ
Ch
inaMa thematica
l
Competi
tion
Mathematical Olympiad in China (2011–2014) Downloaded from www.worldscientific.com
by 91.124.253.25 on 05/31/18. For personal use only.

2010 (
Fui
jan)

Commi
ssi
oned by Ch
inese Ma
thema
tica
l Soc
iey, Fu
t jian
Mat
hematica
l Soci
etyorgan
ized t
he 2010 Ch
ina Ma
thema
tica
l
Compe
tit
ionheldonOct
ober17,2010.
Compa
redt
othecompe
tit
ionsi
nthep
rev
iousyea
rs,wh
ilet
he
t
estt
imeandp
rob
lemt
ypesr
ema
inunchangedi
nth
iscompe
tit
ion,
t
hea
lloca
tiono
fma
rkst
oeachp
rob
lemi
sad
jus
teds
ligh
tlyt
omake
i
tmo
rer
easonab
le.
Thet
imef
ort
hef
ir
str
oundt
esti
s80mi
nues,andt
t hatf
ort
he
supp
lemen
tar
ytes
tis150mi
nut
es.
2 Ma
thema
tic
alOl
ymp
iadi
nCh
ina

Pa
rtI Sho
rt-An
swe
r Qu
est
i s(
on Qu
est
i s1 8,e
on igh
tma
rks
e
ach)
1 Ther ff(
angeo x)= x -5 - 24 -3x i
s .
So
lut
ion.I
tisea
syt
oseet
hatf(
x)i
sinc
rea
singoni
tsdoma
in
[
5,8].The
ref
ore,i
tsr s [-3, 3].
angei
Mathematical Olympiad in China (2011–2014) Downloaded from www.worldscientific.com

2 Themi
n fy = (
imumo acos2x -3)
sinxi
s -3.Thent
he
r
angeo
frea rai
lnumbe s .
So
lut
ion.Le
tsi
nx = t.Theexpr
ess
ioni
sthenchangedt
o
g(
t)= (-a
t +a -3)
t,or
by 91.124.253.25 on 05/31/18. For personal use only.

g(
t)= -a
t3 + (
a -3)
t.

From -a a -3)
t3 + ( t ≥ -3,wege
t

t(
-a t -1)-3(
2
t -1)≥ 0,
t -1)(-a
( t(t +1)-3)≥ 0.

S
incet -1 ≤ 0,wehave -a
t(t +1)-3 ≤ 0,or

a(
t2 +t)≥ -3. ①

Whent =0,-1,
expr
ess
ion ① a
lway
sho
lds;when0 <t ≤
1
1,wehave0 <t2 +t ≤ 2;andwhen -1 <t < 0,- 2
≤t +
4
3
t < 0.The
ref
ore,- ≤a ≤ 12.
2

3 Thenumbe
rofi
nteg
ralpo
ins(
t e.,t
i. hepo
int
swho
sex
-
-coord
andy ina
tesa
rebo
thi
ntege
rs)wi
thi
nthea
rea (
not
i
ncl
udi
ngt
hebounda
ry)enc
los
edbyt
her
ightbr
anchof
hype
rbo
lax2 -y2 = 1andl
inex = 100i
s .
So
lut
ion.Bysymme
try,weon
lyneedt
ocons
ide
rthepa
rtof
t
hea
r hex
eaabovet -ax
is.Suppo
sel
iney = k i
nte
rcep
tst
he
Ch
inaMa
thema
tic
alCompe
tit
ion 3

r
ightbr
anchoft
hehype rbo
laandlinex = 100a
tpointsAk and
Bk (
k = 1,2, ...,99),r e
spec
tive
ly.Thent he numbe
ro f
i
nteg
ralpo
int
swi
thi
nthes
egmen s99 -k.The
tAkBk i re e,
for
t
henumbe
rofi
nteg
ralpo
int
swi
thi
nthea
r hex
eaabovet -ax
isi
s
99

∑ (99 -k)= 99 ×49 = 4851.


k=1
Mathematical Olympiad in China (2011–2014) Downloaded from www.worldscientific.com

Fi
nal
ly,we ob
tai
nthe t
ota
l numbe
rofi
nteg
ral po
int
s
wi
thi
nthewho
lea
r s2 ×4851 +98 = 9800.
eaa

4 I
tisknownt
hat{
an }i
sana
rit
hme
tics
equencewi
thnon
-
by 91.124.253.25 on 05/31/18. For personal use only.

ze
rocommond
iff
erenceand {
bn }ageome
tri
csequence,
s
ati
sfi
ynga1 =3,
b1 =1,a2 =b2 ,3a5 =b3 ;
fur
the e,
rmor
t
her
ear
econs
tan
tsα andβ s
ucht
hatforeve
ry po
sit
ive
i
nt rn,we havean = l
ege ogαbn + β.Thenα + β =
.
So
lut
ion.Le
tthecommon d
iff
erenceof {
an }bed andt
he
commonr
ati
oof{
bn }beq.Then

3 +d =q, ①

3(
3 +4d)=q2 . ②

Sub
sti
tut
ing ① i
nto ② ,wehave9 +12d = d2 +6d +9.
td = 6andq = 9.
Thenwege
The
re e,3 +6(
for n -1)= l n -3 = (
ogα9n-1 +βor6 n-
1)
logα9 +βho
ldsf
oreve
rypo
sit
ivei
nt rn.Le
ege ttngn =1and
i

rn,wef
3
n = 2i
ntu indt
hatα = 3andβ = 3.

y,
3
Con
sequen
tl α +β = 3 +3.

5 Func
tionf(
x)=a2x +3ax -2 (
a >0,a ≠1)r
eache
sthe
max
imumva
lue8oni
nte
r l[-1,1].Theni
va tsmi
nimum
4 Ma
thema
tic
alOl
ymp
iadi
nCh
ina

va
lueont
hisi
nte
rva
lis .
So
lut
ion.Le
ta x
=y.Theor
igi
nalf
unc
tioni
sthenchangedt
o
æ 3
g(
y)= y + 3y - 2,wh r ç - ,+ ∞ ÷ .
2 ö
ichi
sinc
rea
singove
è 2 ø
a,a-1]and
When0 < a < 1,wehavey ∈ [

1
g(
y)max = a +3a -2 = 8⇒a = 2⇒a = .
-2 -1 -1
Mathematical Olympiad in China (2011–2014) Downloaded from www.worldscientific.com

Then

æ 1 ö2 1 1
g(
y)min = ç ÷ +3 × -2 = - .
è2ø 2 4
by 91.124.253.25 on 05/31/18. For personal use only.

Whena > 1,wehavey ∈ [


a-1 ,a]and

g(
y)max = a +3a -2 = 8⇒a = 2.
2

1
g(
y)min = 2 +3 ×2 -2 = - .
-2 -1
Then
4

1
I
nsumma
ry,t
hemi
nimumv
alu
eoff(
x)onx ∈ [-1,1]
is- .
4

6 Twope
rson
sro
lltwod
icei
ntu
rn.Whoeve
rge
tst
hes
um
numbe
rgr
eat
e han 6 f
rt irs
t wi
ll wi
nthe game.The
probab
ili
tyf
ort
hepe
rsonr
oll
ingf
irs
ttowi
nis .
So
lut
ion.Theprobab
ili
tyf
orr
oll
ingtwod
icet
oge
tthes
um
21 7
numbe
rgr
eat
e han 6i
rt s = .The
ref
ore,t
her
equ
ired
36 12
probab
ili
tyi
s

7 æ 5 ÷ö2 7 æ 5 ÷ö4 7 … 7 1 12
+ç × +ç × + = × = .
12 è12ø 12 è12ø 12 12 25 17
1-
144
Ch
inaMa
thema
tic
alCompe
tit
ion 5

7 Thel
eng
thso
fthen
ineedge
sofr
egu
lart
riangu
larpr
ism
ABC A1B1C1 a
reequ
al,Pi
sthemi
dpo
in fCC1,andt
to he
d
ihed
ralang
leB A1P B1 =α.Thens
inα = .
So
lut
ion 1. Le
t t
he l
ine
t
hroughs tAB bex
egmen -ax
is
wi
tht
he or
ign O be
i ing t
he
Mathematical Olympiad in China (2011–2014) Downloaded from www.worldscientific.com

mi
dpo
intofAB andl
ett
hel
ine
t
hroughs tOC bey
egmen -ax
ist
o
e
stab
lis
h as
pacer
ect
angu
lar
coord
ina
te s
yst
em shown i
n
by 91.124.253.25 on 05/31/18. For personal use only.

Fi
g.7.1.Assuming t
hel
eng
th
,
s 2 we have
of an edge i
1,0,0), B1(
B( 1, 0, 2), F
ig.7
.1

A1(
-1,0,2)
,P(
0,3,1)
.Th
en

→ = (-2,0,2),B→
BA P = (-1, 3,1),
1

→ = (-2,0,0),B →
B1A1 1P = (-1, 3,-1).

x1 ,y1 ,z1)andn = (
sm = ( x2 ,y2 ,z2)be
→ →
Le
tve
ctor

pe
rpend
icu
la oBA1P andB1A1P ,r
rt espec
tive
ly.Wehave

→ = -2x +2
m ·BA

z1 = 0,
1 1

{
m ·B→

P = -x1 + 3y1 +z1 = 0,
→ = -2x = 0,
n·B A

1 1 2

{
n·B1→

P = -x2 + 3y2 -z2 = 0.

tm = (
1,0,1),n = (
0,1, 3).
→ →
Wecant
hena
ssumet
ha
om|m ·n|=|m |·|n||c
osα|,wehave
→ → → →
Fr

6
3 = 2·2|c
osα|⇒|cosα|= .
4
6 Ma
thema
tic
alOl
ymp
iadi
nCh
ina

10
The
ref e,s
or inα = .
4
So
lut
ion2.Ass
eeni
nFi
g .2,wehave
.7
PC =PC1 ,PA1 =PB .Suppo
seA1Band
AB1i
nte
rse
c tO .Then wege
ta tOA1 =
OB ,OA = OB1 ,A1B ⊥ AB1 .
Mathematical Olympiad in China (2011–2014) Downloaded from www.worldscientific.com

S
incePA = PB1 ,t
henPO ⊥ AB1 .
The
ref eAB1 ⊥ p
or lanePA1B .
Onp
lanePA1B t
hroughO dr
awl
ine
OE ⊥ A1P wi
thf
ootpo
intE . F
ig.7
.2
by 91.124.253.25 on 05/31/18. For personal use only.

Conne
ctngB1E , ∠B1EO i
i sthent
he p
laneang
leoft
he
d
ihedr
alang
leB A1P B1 .As
s ngAA1 =2,
umi iti
sea
syt
ofi
nd

t
hatPB = PA1 = 5,A1O = B1O = 2,PO = 3.
Inr
ightt
riang
le△PA1O ,wehaveA1O·PO = A1P·OE ,

6
or 2· 3 = 5·OE .SoOE = .
5

AsB1O = 2,wehave

6 45
B1E = B1O2 +OE2 = 2+ = .
5 5

Fi
nal
ly,

B1O 2 10
s
inα = s
in∠B1EO = = = .
B1E 45 4
5

8 Thenumbe
rofpo
sit
ivei
ntege
rso
lut
ionsofequa
tionx +
y +z =2010wi
thx ≤ y ≤zi
s .
So
lut
ion.I
tisea
syt
ofi
ndt
hatt
henumbe
rofpo
sit
ivei
ntege
r
s
olu
tion fx +y +z = 2010i
so 2009 = 2
sC2 009 ×1004.
Wenowc
las
sif
ythe
ses
olu
tionsi
ntot
hreeca
tegor
ies:
Ch
inaMa
thema
tic
alCompe
tit
ion 7

(
1)x =y =z,
thenumbe
rint
hisca
tegor
yisobv
iou
sly1;
(
2)t
her
ear
eexac
tlytwot
hata
r lamongx,y,z —
eequa
t
henumbe
rint
hisca
tegor
yis1003;
(
3)x,y,z a
red
iff
eren
tfr
om eacho
t r—s
he uppo
set
he
numbe
rint
hisca
tegor
yisk.
Fr
om
Mathematical Olympiad in China (2011–2014) Downloaded from www.worldscientific.com

k = 2009 ×1004,
1 +3 ×1003 +6

wehave

6
k = 2009 ×1004 -3 ×1003 -1
by 91.124.253.25 on 05/31/18. For personal use only.

=2006 ×1005 -2009 +3 ×2 -1


=2006 ×1005 -2004.

Wege
t k = 1003 ×335 -334 = 335671.

The
ref
ore, t
he numbe
r of po
sit
ive i
ntege
r s
olu
tions
s
ati
sfi
yngx ≤ y ≤zi
s

1 +1003 +335671 = 336675.

Pa
rtI
I Wo
rdPr
obems (
l 16 ma
rksf
or Qu
est
ion9,20 ma
rks
e
achf
or Qu
est
i s10and11,andt
on hen56 ma
rksi
n
t
otl)
a
9 I
tisknownt
hatf( a ≠0),and
x)=ax3 +bx2 +cx +d (
'(
|f x)|≤ 1for0 ≤ x ≤ 1.P
lea
sef
indt
he max
imum
va
lueofa.
So
lut
ion1.f'(
x)= 3ax2 +2
bx +c.Wehave

'(
ìïf 0)=c,
ï æ1ö 3
'ç ÷ = a +b +c,
íf
ï è2ø 4
ï ()
' 1 = 3a +2
îf b +c.
8 Ma
thema
tic
alOl
ymp
iadi
nCh
ina

Then

æ1ö
3 '(
a = 2f 0)+2f
'(1)-4f
'ç ÷ .
è2ø

Wege
t

æ1ö
'(
3|a| = 2f 0)+2f
'(1)-4f
'ç ÷
è2ø
Mathematical Olympiad in China (2011–2014) Downloaded from www.worldscientific.com

æ 1 ö÷
'(
≤ 2|f 0)|+2|f
'(1)|+4 f
'ç ≤ 8.
è2ø

8
The
ref
ore,a ≤ .Fur
the e,i
rmor tisea
syt
ofi
ndt
hat
3
by 91.124.253.25 on 05/31/18. For personal use only.

8
f(
x)= x3 -4x2 +x +m (
whe
remi
sanycon
st t)s
an ati
sfi
es
3
8
t
heg
ivencond
iti
on.The
re e,t
for hemax
imumva
lueofai
s .
3
So
lut
i tg(
on2.Le x)=f
'(x)+1.Then0 ≤g(
x)≤2for0 ≤
z +1
x ≤ 1.Le
tz = 2x -1.Thenx = and -1 ≤z ≤ 1.Le
t
2

æz +1ö÷ 3a 2 3a +2
b 3
a
h(
z)= g ç = z + z+ +b +c +1.
è 2 ø 4 2 4

I
tisea
syt
ocheckt
hat0 ≤h(
z)≤2and0 ≤h(-z)≤2for
-1 ≤z ≤ 1.

h(
z)+h(-z)
The
re e,0 ≤
for ≤ 2for -1 ≤ z ≤ 1.And
2
t
hati
s

3
a 2 3a
0≤ z + +b +c +1 ≤ 2.
4 4

3a 3
a
Thenwehave +b +c +1 ≥ 0and z2 ≤ 2.Fr
om0 ≤
4 4
8
z2 ≤ 1wege
ta ≤ .
3
8
Asf(
x)= x3 -4x2 +x + m (
whe
rem i
sanycons
t t)
an
3
Ch
inaMa
thema
tic
alCompe
tit
ion 9

s
ati
sfi
est
heg
ivencond
iti
on.Weob
tai
ntha
tthemax
imumva
lue
8
ofai
s .
3

10 Gi
ventwo mov
ingpo
insA (
t x1 ,y1)andB (
x2 ,y2)on
pa
rabo
lacu
rvey2 =6x wi
thx1 +x2 =4andx1 ≠x2 ,and
tABi sx
-ax
Mathematical Olympiad in China (2011–2014) Downloaded from www.worldscientific.com

t
hepe
rpend
icu
larb
isec
toro
fsegmen nte
rsec
t is
a
tpo
intC .F
indt
hemax
imuma
reaof△ABC .
So
lut
ion1.Le
tthemi
dpo
intofAB beM (
x0 ,y0).Thenx0 =

x1 +x2 y1 +y2
= 2andy0 = .Wehave
by 91.124.253.25 on 05/31/18. For personal use only.

2 2

y2 -y1 y2 -y1 6 3
kAB = = 2 = = .
x2 -x1 y2 2 y1 y2 +y1 y0
-
6 6

Theequa
tiono
fthepe
rpend
icu
larb
ise
ctorofABi
s

y0
y -y0 = - ( x -2). ①
3

I
tisea
syt
ofi
ndt
hatones
olu
tiono
fi sx = 5,y = 0.
ti
The
ref e,t
or hei
nte
rsec
tionC i
saf
ixed po
intwi
thcoord
ina
te
(
5,0).
3
From ① ,weknowt
heequa
tionofl
ineABi
sy -y0 =
y0
(
x -2),or

y0 (
x = y -y0)+2. ②
3

Sub
sti
tut
ing ②i ty2 = 2y0(
ny2 = 6x,wege y -y0)+12,
or

y -2y0y +2y0 -12 = 0. ③


2 2

Asy1 andy2 a
retwor
ealroo
t f③ andy1 ≠y2 ,wehave
so
10 Ma
thema
tic
alOl
ymp
iadi
nCh
ina

0 -4(
Δ = 4y2 2y2
0 -12)= -4y2
0 +48 > 0.

The
re e,-2 3 < y0 < 2 3.Thenwehave
for

|AB | = (
x1 -x2)
2
+(
y1 -y2)
2

æy0 ö2 ö
1+ ç ÷ ÷ ( y1 -y2)
æç 2
=
è è3ø ø
Mathematical Olympiad in China (2011–2014) Downloaded from www.worldscientific.com

2
y0 ö
1 + ÷ [(y1 +y2) -4
y1y2]
æç 2
=
è 9ø
2
y0 ö
1+ ÷ ( y0 -4(y0 -12))
æç
= 4 2
2 2

è 9ø
by 91.124.253.25 on 05/31/18. For personal use only.

2 (
= 0)
9 +y2 ( 0)
12 -y2 .
3 F
ig.10
.1

Thed
ist
anc
efr
om po
intC(
5,0)t
os tABi
egmen s

h =|CM |= (
5 -2)
2
+(
0 -y0)
2
= 9 +y2
0.

The
ref
ore,

1 1 (
S△ABC = |AB |·h = 0)
9 +y2 ( 0)
12 -y2 · 9 +y2
2 3
0

1 1(
= 0)
9 +y2 ( 0)
24 -2y2 ( 0)
9 +y2
3 2

1 æç9 +y0 +24 -2y0 +9 +y0 ö÷


2 2 2 3
1

3 2è 3 ø
14
= 7.
3

Theequa
lit
yho
ldsi
fandon
lyi 0 =2
f9 +y2 0,
4 - 2y2 i.
e.

y0 = ± 5.Thenwege
t

A ç6 + 35, 5 + 7 ÷ ,B ç6 - 35, 5 - 7 ÷
æ ö æ ö
è 3 ø è 3 ø

A ç6 + 35,- (5 + 7)÷ ,B ç6 - 35,- 5 + 7÷ .


æ ö æ ö
and
è 3 ø è 3 ø
Ch
inaMa
thema
tic
alCompe
tit
ion 11

14
Con
sequen
tly,t
hemax
imuma
reaof△ABCi
s 7.
3
So
lut
ion 2. S
imi
lart
o So
lut
ion 1, we ge
tt t C ,t
ha he
i
nte
rse
cti
onoft
hepe
rpend
icu
larb
ise
ctorofAB andt
hex
-ax
is,
i
saf
ixedpo
intwi
thcoord
ina
te(
5,0).
Le 1,
tx1 =t2 2,
x2 =t2 t1 >t2 ,
t21 + 2 =4.
t2 ThenS△ABCi
sthe
ab
sol
uteva
lueof
Mathematical Olympiad in China (2011–2014) Downloaded from www.worldscientific.com

5 0 1
1 t2 6t1 1 ,
1
2
t2 6t2 1
by 91.124.253.25 on 05/31/18. For personal use only.

s
o

æç 1 ( ö2
S2△ABC = 5 6t1 + 6t2
t2 - 6t1t2
2 -5 6t2)÷
è2
1
ø
3(
= t1 -t2)
2
(
t1t2 +5)
2

2
3(
= t1t2)(
4 -2 t1t2 +5)(
t1t2 +5)
2
3 çæ14÷ö
3
≤ .
2è3ø

14
The
ref e,S△ABC ≤
or 7andt
heequa
lit
yho
ldsi
fandon
ly
3
7+ 5
f(
i t1 -t2)
2
=t1t2 +5andt2
1 + 2 =4.Wet
t2 tt1 =
henge
6
7 - 5,
andt2 = - wh
ichimp
lie
sei
the
r
6

A ç6 + 35, 5 + 7 ÷ ,B ç6 - 35, 5 - 7 ÷
æ ö æ ö
è 3 ø è 3 ø

or

A ç6 + 35,- (5 + 7)÷ ,B ç6 - 35,- 5 + 7 ÷ .


æ ö æ ö
è 3 ø è 3 ø
12 Ma
thema
tic
alOl
ymp
iadi
nCh
ina

14
Fi
nal
ly,t
hemax
imuma
reaof△ABCi
s 7.
3

11 Pr
ovet
hatequa
tion2x3 +5x -2 = 0ha
sexac
tlyoner
eal
r t (deno
oo t s r),and t
ed a her
eis a un
ique s
tri
ctl
y
2
i
ncr
eas
ings
equ
ene{
c an }
sucht
hat =ra1 +ra2 +ra3 + … .
5
Mathematical Olympiad in China (2011–2014) Downloaded from www.worldscientific.com

So
lut
i tf(
on.Le x)= 2x3 +5 '(
x -2.Then wehavef x)=
6x2 +5 > 0, wh
i s f(
ch mean x) i
sst
ric
tly i
ncr
eas
ing.
æ1ö 3
Fu
rthe e,f(
rmor 0) = - 2 < 0,f ç ÷ = > 0.Ther
e e,
for
è2ø 4
by 91.124.253.25 on 05/31/18. For personal use only.


f(
x)ha tr ∈ ç0, ÷ .From2 r -2 =0,
r3 +5
æ
saun
iquer
ealr
oo
è 2ø
wehave

2 r
=r +r +r +r + … .
4 7 10
=
5 1 -r3

The
re e,s
for n -2 (
equencean = 3 n = 1,2,...)s
ati
sfi
est
he
r
equ
iredcond
iti
on.
As
sumet
her
ear
etwod
iff
eren
tpo
sit
ivei
ntege
rsequence
s

a1 < a2 < … < an < … andb1 <b2 < … <bn < …

s
ati
sfi
yng

2
ra1 +ra2 +ra3 + … =rb1 +rb2 +rb3 + … = .
5

De
let
ingt
het
ermst
hatappea
ratt
he bo
ths
ide
soft
he
expr
ess
ion,wehave

rs1 +rs2 +rs3 + … =rt1 +rt2 +rt3 + …,

whe
res1 <s2 <s3 < …,
t1 <t2 <t3 < … wi
tha
l hesiandtj
lt
d
iff
eren
tfr
omeacho
the
r.
Wemaya
swe
lla
ssumet
hats1 <t1 .Then
Ch
inaMa
thema
tic
alCompe
tit
ion 13

rs1 <rs1 +rs2 + … =rt1 +rt2 + …,


1 <rt1-s1 +rt2-s1 + … ≤r +r2 + …
1 1
= -1 < -1 = 1.
1 -r 1
1-
2

I
tisacon
trad
ict
ion.Th
isprove
st t{
ha an }
isun
ique.
Mathematical Olympiad in China (2011–2014) Downloaded from www.worldscientific.com

2011 (
Hubei)
by 91.124.253.25 on 05/31/18. For personal use only.

Comm
iss
ion
edb
y Ch
ine
se Ma
thema
tic
alSo
cie
ty,Hube
iMa
thema
tica
l
Soc
iet
yor
gan
izedt
he2011Ch
inaMa
thema
tica
lCompe
tit
ionhe
ldon
Oc
t r16,2011.
obe

Pa
rtI Sho
rt-An
swe
r Qu
est
i s(
on Qu
est
i s1 8,e
on igh
tma
rks
e
ach)
1 tA = {
Le a1 ,a2 ,a3 ,a4}.Suppo
set
hes
etofs
umsofa
ll
t
hee
lemen
tsi
neve
ryt
erna
rys
ubs
e sB = {-1,3,
tofAi
5,8}.ThenA = .
So
lut
ion.Obv
iou
sly,eve
rye
l tofA appea
emen rst
hreet
ime
s
i
nal
lthet
erna
rys
ubs
ets.Thenwehave

a1 +a2 +a3 +a4)= (-1)+3 +5 +8 = 15,


3(

ora1 +a2 +a3 +a4 = 5.The


ref
ore,t
hefoure
lemen
tsofA a
re
5 - (-1)= 6,5 -3 = 2,5 -5 = 0,5 -8 = -3,r
espec
tive
ly.
Theanswe
risA = {-3,0,2,6}.

x2 +1
2 Theva
luef
iel
doff(
x)= is .
x -1
14 Ma
thema
tic
alOl
ymp
iadi
nCh
ina

π π π
So
lut
ion.Le
tx anθ,-
=t <θ < andθ ≠ .Wehave
2 2 4

1
cosθ 1 1
f(
x)= = = .
t
anθ -1 snθ -c
i osθ πö
2s
inθ - ÷
æç
è 4ø

πö
tu = 2s
inçθ - ÷ .Then - 2 ≤ u < 1andu ≠ 0.
æ
Mathematical Olympiad in China (2011–2014) Downloaded from www.worldscientific.com

Le
è 4ø
1 2 ùú ∪ (
e,f(
x)= 1,+ ∞ ).
æ
The
refor ∈ ç - ∞ ,-
u 2û
ú
è
by 91.124.253.25 on 05/31/18. For personal use only.

1 1
3 Suppo
seaandba
repo
sit
iver
ealnumbe
rss
ati
sfi
yng +
a b
≤ 2 2and (
a -b) = 4(
ab).Thenl
2
ogab =
3
.

So
lut
ion.From 1 + 1 ≤22,wehavea +b ≤22ab.Ont
he
a b
o
t rhand,
he

(
a +b)
2
ab + (
=4 a -b)
2
ab +4(
=4 ab)
3

≥ 4·2 a ab) = 8(
b·( ab),
3 2

andt
hatmeans

a +b ≥ 2 2ab. ①

The
ref e,
or

a +b = 2 2ab. ②

Theequa
lit
yin ① ho
ldson
ly whenab = 1.As
soc
iat
ingi
t
wi
th ② ,wef
ind

a = 2 -1, a = 2 +1,
{ b = 2 +1,
and
{
b = 2 -1.

Sot
hean
swe
risl
ogab = -1.
Ch
inaMa
thema
tic
alCompe
tit
ion 15

4 Suppo
s o
ecs5θ - s
in5θ < 7(
si os3θ),θ ∈ [
n3θ - c 0,2π).
Thent
her
angeofθi
s .
So
lut
ion.Fromtheinequa
lit
y

c
os5θ -s
in5θ < 7(
si s3θ),
n3θ -co

wehave
Mathematical Olympiad in China (2011–2014) Downloaded from www.worldscientific.com

1 5 1 5
s
in3θ + si
nθ > c
os3θ + co
sθ.
7 7

1 5
S
incef(
x)= x3 + x isi
ncr
eas
i r (- ∞ ,+ ∞ ),
ngove
7
by 91.124.253.25 on 05/31/18. For personal use only.

hens
t inθ > c
osθ,andt
hatmeans

π 5π
kπ +
2 kπ + (
<θ < 2 k ∈ Z).
4 4

æ π 5πö
tθ ∈ [
Bu 0,2π),
sot
her sç , ÷ .
angeofθi
è4 4 ø

5 Sevens
tuden
tsa
rea
rrangedt
oat
tendf
ives
por
tingeven
ts.
I
tisr
equ
iredt
hats
tuden
tsA andBcanno
tat
tendt
hes
ame
t,eve
even ryeven
tisa
ttendedbya
tlea
stones
t t,and
uden
eachs
tuden
t mu
sta
ttendoneandon
ly oneeven
t.Then
t
henumbe
roft
hea
rrangemen
tpl
ansmee
tingt
her
equ
ired
cond
iti
oni
s .(
theanswe
rshou
ld be g
iveni
n
nume
rica
lva
lue)
So
lut
ion.Therearetwopos
sib
leca
sest
hats
ati
sfyt
her
equ
ired
cond
iti
ons:
(
1)t
her
eisaneven
tat
tendedbyt
hrees
tuden
ts— t
hisca
se
7 ·5! - C
sC3 5 ·5! = 3600p s;
1
ha lan
(
2)t
her
ear
etwoeven
tseacha
ttendedbytwos
tuden
ts—
1 2 · 2)· !
t
hisca
s s (
eha 5 ·5! = 1
C7 C5 5 - C2 1400p
lans.
2
16 Ma
thema
tic
alOl
ymp
iadi
nCh
ina

The
ref e,t
or her
eae3600+11400 =15000p
r lanst
hatmee
t
t
her
equ
iredcond
iti
on.

6 Gi
venat
etr onABCD ,i
ahedr tisknownt
hat ∠ADB =
∠BDC = ∠CDA =6°,AD =BD =3andCD =2.Then
0
t
he r
adi
us of t
he s
phe
re c
ircums
cri
bing ABCD i
s
Mathematical Olympiad in China (2011–2014) Downloaded from www.worldscientific.com

.
So
lut
ion.Le
tthe c
ent
ero
fthe
s
phe
rec
ircums
cri
bingABCD beO .
Then O i
s on t
he ve
rti
call
ine of
by 91.124.253.25 on 05/31/18. For personal use only.

l
pane ABD t
hrough po
intN t
he
c
ircumc
ent
erof△ABD .
Iti
sknown
t
hat△ABD i
sregu
lar,s
oN i
sthe
cen
tero
fi tP and M bet
t.Le he
mi
dpo
int
s of AB and CD , F
ig.6
.1

r
espec
tive
ly.ThenN i
sonDP wi
th
ON ⊥ DP andOM ⊥ CD .
tθdeno
Le tet
heang
l tweenCD andp
ebe laneABD .From

∠CDA = ∠CDB = ∠ADB = 6°,


0

1, 2
wef
indc
osθ = s
inθ = .
3 3
1 2 2 3
S
inceDM = CD =1,DN = ·DP = · ·3 = 3,
2 3 3 2
byco
sinet em wehave,i
heor n△DMN ,

MN2 = DM 2 + DN2 -2·DM ·DN ·cosθ


1
= 1 + (3) -2·1· 3· = 2,
2 2

t
ha sMN = 2.Ther
ti adi
uso
fthes
phe
rec
irc
ums
cri
bingABCD
i
sthen
Ch
inaMa
thema
tic
alCompe
tit
ion 17

MN 2
OD = = = 3.
s
inθ 2
3

Thean
swe
risR = 3.

7 nex - 2y - 1 = 0and pa
Li rabo
lay2 = 4x i
nte
rsec
tat
Mathematical Olympiad in China (2011–2014) Downloaded from www.worldscientific.com

po
insA ,B ,po
t intC i
sont heparabo
la,and ∠ACB =
90
°.Thent
hecoord
ina
teofCi
s .
So
lut
i tA (
on.Le x1 ,y1),B (
x2 ,y2),C(
t ,2
t).From
2

x -2y -1 = 0,
by 91.124.253.25 on 05/31/18. For personal use only.

{y = 4x,
2

weg
ety2 -8y -4 = 0,whi
chme ansy1 +y2 = 8,y1·y2 =-4.
Si
ncex1 = 2y1 +1,x2 = 2y2 +1,wehave

x1 +x2 = 2(
y1 +y2)+2 = 18,
x1 ·x2 = 4y1 ·y2 +2(
y1 +y2)+1 = 1.

Fur
the
rmor °,wehaveC→
e,by ∠ACB = 90 A ·C→
B = 0,
wh
ich means

t2 -x1)(
( t2 -x2)+ (
2t -y1)(
2t -y2)= 0,

t
hati
s

t4 - (
x1 +x2) t2 -2(
t2 +x1 ·x2 +4 y1 +y2)
t +y1 ·y2 = 0.

Then

t4 -14 t -3 = 0,
t2 -16

or

( t +3)(
t2 +4 t -1)= 0.
t2 -4

Obv
iou
sly, t -1 ≠ 0;ot
t2 -4 herwi
se,wehavet2 - 2·
t -1 = 0,wh
2 i sonx - 2y - 1 = 0,i.
ch meansC i e.,C
18 Ma
thema
tic
alOl
ymp
iadi
nCh
ina

co
inc
ide
swi
the
it rA orB .Sot2 + 4
he t + 3 = 0.Thent1 =
- 1,t2 = -3.
The
ref
ore,t
hecoord
ina
t s(
eofCi 1,-2)or (
9,-6).

æ 1 ön
200·(6) ·ç ÷ ( n =1,2,...,95).Then
3 200-n
8 tan = Cn
Le
è 2ø
Mathematical Olympiad in China (2011–2014) Downloaded from www.worldscientific.com

t
henumbe
roft
ermst
hata
rei
ntege
r n{
si an }
is .

= C200·3 ·2 .Whenan (
200-n 400-5n
So
lut
ion.Wehavean n 3 6
1 ≤n ≤
200 -n 400 -5
n
95)
isani
nt r,
ege and mu
stbei
ntege
rs.Then
3 6
by 91.124.253.25 on 05/31/18. For personal use only.

6|n +4.
Whenn = 2,8,14,20,26,32,38,44,50,56,62,68,
200 -n 400 -5
n
74,80, and a
rea
llnon
-nega
tivei
ntege
rs.So
3 6
t
hecor
res
pond
ingan ,
tot
aly14,a
l rei
ntege
rs.
Whenn =86,wehavea86 = C8
200·3 ·2 .
6 38 -5
Thenumbe
rof
t
hef
act
orso n200! i
f2i s

200 200 200 200 200 200


[200
2 ] [2 ] [2 ] [2 ] [2 ] [2 ] [2 ]
+ 2 + 3+ + 4+ + 5 =197. 6 7

Byt hesamer
eaon,t
s henumbersoft
hef ac
tor
sof2i n86!
and114! a
re82and110,re
spec
tive
ly.Ther
efore,thenumber
200!
o
fthef
act
orso nC8
f2i 6
200 = is197 -82 -110 =5.So
86!·114!
a86i
sani
ntege
r.
Whenn = 92,wehavea92 = C9
200 ·3 ·2
2 36 -10
.Int
hes
ame
way,wef
indt
henumbe
rsoft
hef
act
or n92! and108!
sof2i
a
re88and105,r
espe
cti
vey,wh
l ich meanst
ha nC8
ti 6
s197 -
200i

88 -105 = 4.The
ref e,
or a92i
sno
tani
ntege
r.
Ove
ral,t
l her
equ
irednumbe
ris14 +1 = 15.
Ch
inaMa
thema
tic
alCompe
tit
ion 19

Pa
rtI
I Wo
rdPr
obems (
l 16 ma
rksf
or Qu
est
ion9,20 ma
rks
e
achf
or Qu
est
i s10and11,andt
on hen56 ma
rksi
n
t
otl)
a
9 Suppo
sef(
x)=|l
g(x +1)
|andr
ealnumbe
rsa,
b(a <b)
æ b +1÷ö , (
s
ati
sfyf(
a)= f ç -
è b +2ø
b + 21) = 4
f 10a + 6 lg2.

fa,b.
Mathematical Olympiad in China (2011–2014) Downloaded from www.worldscientific.com

Fi
ndt
heva
lue
so
æ b +1ö÷ ,
So
lut
ion.Asf(
a)= f ç - wehave
è b +2ø

æ b +1 æ 1 ö÷
g(
|l a +1)|= l +1÷ = l g(
=|l b +2)|.
ö
gç - gç
by 91.124.253.25 on 05/31/18. For personal use only.

è b +2 ø èb +2ø

Thene
it ra +1 =b +2or(
he a +1)(
b +2)=1.S
i ea <
nc
b,s
oa +1 ≠b +2.The
re e,(
for a +1)(
b +2)= 1.
Fromf(
a)=|l
g(a +1)|weknow0 < a +1.Then

0 < a +1 <b +1 <b +2,

wh
ichimp
lie
s

0 < a +1 < 1 <b +2.

The
ref e,
or

(
10 b +21)+1 = 10(
a +6 a +1)+6(
b +2)
10
b +2)+
= 6( > 1.
b +2

Then

10
f(
10a +6 g 6(
b +21)= l b +2)+[ ]
b +2
10
g 6(
=l [b +2)+ ].
b +2

Ont
heo
t rhand,
he

f(
10 b +21)= 4
a +6 lg2.
20 Ma
thema
tic
alOl
ymp
iadi
nCh
ina

So

10
g 6(
l b +2)+
[ =4
lg2, ]
b +2

10 1
wh
ich means6(
b +2)+ =16.Thene
it rb = - orb =
he
b +2 3
-1 (
dis
carded).
Mathematical Olympiad in China (2011–2014) Downloaded from www.worldscientific.com

1
Sub
sti
tut
ingb = - in
to(
a +1)(
b +2)= 1,wef
inda =
3
2
- .
5
by 91.124.253.25 on 05/31/18. For personal use only.

2, 1
The
ref
orea = - b =- .
5 3

10 Suppo
ses
equ
ene{
c an }
sat
isf
ie t-3(
sa1 =2 t ∈ Randt ≠±1),

(
2tn+1 -3)
an +2(t -1)
tn -1(
an+1 = n ∈ N* ).
an +2
tn -1

(
1)Fi
ndt
heformu
laofgene
ralt
e t{
rmabou an }.
(
2)I
ft > 0,
findou
twh
ichi
sla
rge tweenan+1andan .
rbe
So
lut
ion.(
1)Theg
ivenexpr
ess
ioncanber
ewr
itt
ena
s

2(
tn+1 -1)(an +1)
an+1 = -1.
an +2 tn -1

Then

2(an +1)
an+1 +1 (
2 an +1) tn -1
= = .
t -1 an +2
n+1
t -1 an +1
n
+2
tn -1

an +1 2bn , a1 +1
Le
tn =bn .Thenbn+1 = wi
thb1 = =
t -1 bn +2 t -1
t -2
2
= 2.
t -1
Ch
inaMa
thema
tic
alCompe
tit
ion 21

1 1 1 ,1 1
Fur
the e,
rmor = + = .Then
bn+1 bn 2 b1 2

1 1 ( ·1 = n .
= + n -1)
bn b1 2 2

an +1 2 , tn -1)
2(
The
ref
ore, n = wh
ichmeansan = -1.
t -1 n n
(
2)Wehave
Mathematical Olympiad in China (2011–2014) Downloaded from www.worldscientific.com

2(
tn+1 -1) 2(
tn -1)
an+1 -an = -
n +1 n
2(
t -1)[ (
= n 1 +t + … +tn-1 +tn )-
n(
n +1)
by 91.124.253.25 on 05/31/18. For personal use only.

(
n +1)(
1 +t + … +tn-1)]
t -1)[ n (
2(
= nt - 1 +t + … +tn-1)]
n(
n +1)
t -1)[(n
2(
= t -1)+ (
tn -t)+ … + (
tn -tn-1)]
n(
n +1)
2(
t -1)2
[(
=
( ) tn-1 +tn-2 + … +1)+
n n +1
t(
tn-2 +tn-3 + … +1)+ … +tn-1].

I
tisobv
iou
st tan+1 -an >0fort >0(
ha t ≠1).The
re e,
for
an+1 > an .

1
11 S
tra
igh
tlnel wi
i ths
lope
3
2
x2 y
i
nte
rcep
tse
lli
pseC : +
36 4
= 1a
t po
ins A ,B ,and
t

pointP (
3 2, 2)i
sinthe
F
ig.11
.1
top
-lef
to (
fl asshownin
Fig .1).
.11
(1)Provet
hatt
hecen
teroft
hei
nsc
ribedc
irc
leof△PAB
22 Ma
thema
tic
alOl
ymp
iadi
nCh
ina

i
sonag
ivenl
ine;
(
2)When ∠APB = 60
°,f
indt
hea
reaof△PAB .
1
So
lut
ion.(
1)Le
tlbeas
tra
igh
tli
nes
ucht
haty = x +m,
3
andA (
x1 ,y1),B (
x2 ,y2).
1
2
x2 y
Sub
sti
tut
ingy = x +mi
nto + = 1,ands
imp
lif
ying
3 36 4
Mathematical Olympiad in China (2011–2014) Downloaded from www.worldscientific.com

t,wehave
i

2x2 +6mx +9m2 -36 = 0.

9m2 -36, y1 - 2 ,
Thenx1 +x2 = -3m ,x1x2 =
by 91.124.253.25 on 05/31/18. For personal use only.

kPA =
2 x1 -3 2

y2 - 2
kPB = .The
ref
ore,
x2 -3 2

y1 - 2 y2 - 2
kPA +kPB = +
x1 -3 2 x2 -3 2
(
y1 - 2)(
x2 -3 2)+ (
y2 - 2)(
x1 -3 2)
= .
(
x1 -3 2)(
x2 -3 2)

Int
heexpr
ess
ionabove,t
henume
rat
ori
sequa
lto

æç 1 æ1
x1 + m - 2 ÷ (
x2 -3 2)+ ç x2 + m - 2 ÷ (
x1 -3 2)
ö ö
è3 ø è3 ø
2
= x1x2 + (
m -2 2)(
x1 +x2)-6 2(
m - 2)
3
2 ·9m2 -36 (
= + m -2 2)(-3m )-6 2(
m - 2)
3 2
= 3m -12 -3m +6 2m -6 2m +12 = 0.
2 2

The
re e,
for kPA +kPB =0.S
i ePi
nc sint
het
op-
lef
tofl,we
know t
hatt
he b
isec
tor of ∠APB i
s pa
ral
lelt
ot -ax
he y is.
The
re e,t
for hecen
teroft
hei
nsc
ribedc
irc
l f△PABi
eo sonl
ine
x = 3 2.
Ch
inaMa
thema
tic
alCompe
tit
ion 23

(
2)When ∠APB = 60
°,byt
her
esu
l n(
ti 1),wehav
ekPA =

3,kPB = - 3.Thent
heequa
tionforl
i sy - 2 =
nePA i
2
x2 y
3(
x -3 2).Sub
sti
tut
ingi
tin
to + = 1,ande
limi
nat
ing
36 4
y,wege
t

14x2 +9 6(
1 -3 3)
x +18(
13 -3 3)= 0,
Mathematical Olympiad in China (2011–2014) Downloaded from www.worldscientific.com

18(
13 -3 3),
wh
ichha
sroo
tsx1 and3 2.Sox1 ·3 2 = i.e.
14

3 2(
13 -3 3)
x1 = .Thenwef
ind
14
by 91.124.253.25 on 05/31/18. For personal use only.

3 2(
3 3 +1)
|PA |= 1 + (3)
2
·|x1 -3 2|= .
7

3 2(
3 3 -1)
In t ame way, we have | PB |=
he s .
7
The
ref e,
or


S△PAB = |PA |·|PB |·s
in60
°
2

1 ·3 2(
3 3 +1)·3 2(
3 3 -1)· 3
=
2 7 7 2

117 3
= .
49

2012 (
Shaanxi)

Commi
ssi
oned by Ch
inese Ma
thema
tica
l Soc
iey, Shaanx
t i
Ma
thema
tica
l Soc
iet
yor
gan
ized t
he 2012 Ch
ina Ma
thema
tica
l
24 Ma
thema
tic
alOl
ymp
iadi
nCh
ina

Compe
tit
ionhe
ldonOc
t r14,2012.
obe

Pa
rtI Sho
rt-An
swe
r Qu
est
i s(
on Qu
est
i s1 8,e
on igh
tma
rks
e
ach)
2(
1 tP beapo
Le intont fy = x +
heimageo x > 0).
x
Mathematical Olympiad in China (2011–2014) Downloaded from www.worldscientific.com

ThroughP dr
awlinesperpendicul
artoy = x andy-ax
is
wit
hf ootpoi
ntsA ,B ,respe
ctively.Thent
heva l
ueof
→ →
PA ·PBi s .
2 ö÷
So
lut
i tP çx0 ,x0 +
on1.Le
æ
.Theexpr
ess
ionforl
inePA
x0 ø
by 91.124.253.25 on 05/31/18. For personal use only.

è
i
sthen

2 ö÷
=- (
x -x0),
æ
y - çx0 +
è x0 ø
2
or y = -x +2x0 + .
x0

Fr
om

ìïy = x,
ï
í 2
ïy = -x +2x0 + ,
î x0

1 1 ö÷
tA çx0 + ,x0 +
æ
wege .
è x0 x0 ø
2 ÷ö
rhand,wehaveB ç0,x0 + .ThenP→
æ
Ont
heo
the A =
è x0 ø
æç 1 , 1 ö÷
- andP→
B = (-x0 ,0).The
ref
ore,
èx0 x0 ø

1 ·(
P→
A ·P→
B = -x0)= -1.
x0

Thean
swe
ris -1.
Ch
inaMa
thema
tic
alCompe
tit
ion 25

So
lut
ion2.Ass
eeni
nFi
g .1,
.1
2 ÷ö
om P çx0 ,x0 +
æ
t
hed
ist
anc
esf
r t
o
è x0 ø
l
i sy = x andy
ne -ax
is,r
espec
tive
ly,
a
re

æ 2 ö÷
x0 - çx0 +
2
Mathematical Olympiad in China (2011–2014) Downloaded from www.worldscientific.com

è x0 ø
|PA |= =
2 x0 F
ig.1
.1

and

|PB |= x0 .
by 91.124.253.25 on 05/31/18. For personal use only.

S
inceO ,A ,P andB a
reconcyc
licpo
ins,t
t hen

∠APB = π - ∠AOB =

.
4


The
ref
ore,P→
A ·P→
B =|P→
A |·|P→
B |·cos = -1.
4

2 Suppo
se △ABC wi
th ang
les A , B and C ,and t
he
cor
res
pond
ings
i sa,bandcs
de ati
sfi
esequa
tionaco
sB -
3 t
anA
bcosA = c.Thent
heva
lueo
f i
s .
5 t
anB
So
lut
ion1.Byt
heg
ive
ncond
it
ionandt
heLawo
fCo
sin
es,weha
ve

c2 +a2 -b2 b2 +c2 -a2 3


a· -b· = c,
2
ca 2
bc 5

3 2
ora2 -b2 = c .The
ref e,
or
5

c2 +a2 -b2

t
anA si
nAcosB 2
ca
= =
t
anB si
nBc
osA b2 2
+c -a
2

2
bc
26 Ma
thema
tic
alOl
ymp
iadi
nCh
ina

8 2
c
c2 +a2 -b2 5
= 2 = = 4.
b +c2 -a2 2 2
c
5

Thean
swe
ris4.
So
lut
ion 2. As s
een i
n Fi
g .1,
.2
t
hroughC dr
aw CD ⊥ AB wi
thfoo
t
Mathematical Olympiad in China (2011–2014) Downloaded from www.worldscientific.com

poitD .Wehaveac
n o
sB = DB and
bc
osA = AD .Byt
hegiv
enc
ond
iton,
i
3
wehaveDB -AD = c.Comb
ini
ng
5
by 91.124.253.25 on 05/31/18. For personal use only.

F
ig.2
.1
i
twi
thDB + AD = c,wege
tAD =
1 4
candDB = c.The
ref
ore,
5 5

CD
t
anA AD BD
= = = 4.
t
anB CD AD
BD

So
lut
ion3.Byt
hep
roe
jct
iont
heo
rem,wehav
eac
osB +bc
osA =

3 4
c.Comb
ini
ngi
twi
thac
osB -bc
osA = c,weg
etac
osB = c
5 5
1
andbcosA = c.The
ref
ore,
5

4
c
t
anA si
nAcosB a·cosB 5
= = = = 4.
t
anB si osA b·co
nBc sA 1
c
5

3 tx,y,z ∈ [
Le 0,1].Thent
hemax
imumva
l fM =
ueo

|x -y| + |y -z| + |z -x|i


s .
So
lut
ion.Wemayas
sume0 ≤ x ≤ y ≤z ≤ 1.Then

M = y -x + z -y + z -x .
Ch
inaMa
thema
tic
alCompe
tit
ion 27

S
ince

y -x + z -y ≤ y -x)+ (
2[( z -y)] = z -x),
2(

wehave

M ≤ 2(
z -x)+ z -x = (2 +1) z -x ≤ 2 +1.

Theequa
lit
yho
ldsi
fandon
l fy -x =z -y,x =0,z =
yi
Mathematical Olympiad in China (2011–2014) Downloaded from www.worldscientific.com

1
1i e.x = 0,y = ,z = 1÷ .
æç ö
.
è 2 ø
The
re e,t
for hean
swe
risM max = 2 +1.
by 91.124.253.25 on 05/31/18. For personal use only.

4 Le
tthef
ocu
sandd
ire
ctr
ixofpa
rabo
l p > 0)
ay2 = 2px(
beFandl,
res
pec
tive
ly.A andBa
remov
ingpo
int
sont
he
π
pa
rabo
las
ati
sfi
yng ∠AFB = .Le
tthepro
jec
ti fM
ono
3
t
he mi
dpo
intofs t AB on l be N .Then t
egmen he
MN |
max
imumva
lueof| is .
|AB |
2πö÷
So
lut
ion1.Suppos = θç0 <θ <
æ
e ∠ABF .Thenbyt
he
è 3ø
Lawo
fSne,wehave
i

|AF | = |BF | =
|AB |.
s
inθ æ2π π
s
in si
n
ö

è3 3
ç ÷
ø

Andt
hen

|AF |+|BF | = |AB |.


æ2π π
s
inθ +s
inç si
n
ö
-θ÷
è3 ø 3

So

æ2π
s
inθ +sinç
ö
-θ÷
|AF |+|BF | = è3 πö
=2cosçθ - ÷ .
ø æ
|AB | π 3ø
s
in
è
3
28 Ma
thema
tic
alOl
ymp
iadi
nCh
ina

Ass
eeni
nFi
g .1,byu
.4 singt
he
de
fin
iti
on of a parabola and t
he
pr
opert
yofat rape
zo d, we have
i
|AF |+|BF |.Then
|MN |=
2

|MN | = πö
c
osçθ - ÷ .
æ

Mathematical Olympiad in China (2011–2014) Downloaded from www.worldscientific.com

|AB | è

MN |
The
ref
ore,| r
eache
sthe
|AB |
F
ig.4
.1
π
max
imumva
lue1whenθ = .
3
by 91.124.253.25 on 05/31/18. For personal use only.

Thean
swe
ris1.
So
lut
ion2.By us
ing t
he de
fin
iti
on ofa pa
rabo
la andt
he
|AF |+|BF |.In
prope
rtyo
fat
rapezo
id,wehave|MN |=
2
△AFB ,byu
singt
heLawofCo
sine
swehave

π
|AB | =|AF | +|BF | -2|AF |·|BF |cos
2 2 2

3
=(
|AF |+|BF |) -3|AF |·|BF |
2

| æ AF |+|BF |÷ö2
≥(
|AF |+|BF |) -3ç
2
è 2 ø
æç|AF |+|BF |ö÷2 2
= =|MN | .
è 2 ø

The equa
lit
y ho
ldsi
f and on
l f | AF |=| BF |.
yi
MN |
The
ref e,t
or hemax
imumva
lueof| is1.
|AB |

5 Suppo
setwor
egu
lart
riangu
larpyr
ami
dsP ABC andQ
ABCsha
ringt
hes
ameba
sea
rei
nsc
ribedi
nthes
ames
phe
re.
I
ftheang
leb
etwe
ent
hes
ide
-fa
ceandt
heba
s fP ABCi
eo s
°,
45 t
hent
het
ang
entva
lueo
ftheang
leb
etwe
ent
hes
ide
-fa
ce
Ch
inaMa
thema
tic
alCompe
tit
ion 29

andt
heba
s fQ ABCi
eo s .
So
lut
ion. As s
een i
n F
ig .1,
.5
c
onne
ctngPQ,
i thenPQi
spe
rpend
icu
lar
t
opaneABC wi
l tht
hef
ootpo
intH
be
ingt
hec
ent
e f△ABC .Thec
ro ent
er
o
fthes
phe
reOi
sal
soonPQ .Conne
ct
Mathematical Olympiad in China (2011–2014) Downloaded from www.worldscientific.com

andex
tendCH t
ole
titi
nte
rsec
twi
th
ABa
tpo
intM .M i
sthent
hemi
dpo
int
ofAB ,andCM ⊥ AB .
Iti
sea
syt
osee
t
hat ∠PMH and ∠QMH a
ret
hep
lane F
ig.5
.1
by 91.124.253.25 on 05/31/18. For personal use only.

ang
lesformed by t
hes
ide
s-f
ace
sand
t
heba
seso
fther
egu
lart
riang
ula
rpy
rami
dsP ABCandQ ABC,
1
r
espe
cti
vel °,
y.Then ∠PMH = 45 oPH = MH =
s AH .
2
S
i °,AH ⊥PQ ,
nce ∠PAQ =90 thenAH2 =PH ·QH .We
1
henhaveAH2 =
t AH ·QH .
2
The
re e,QH = 2AH = 4MH .
for
QH
Fi
nal
ly,
tan∠QMH = = 4.
MH
Thean
swe
ris4.

6 tf(
Le x)beanoddf
unc
tiononR,andf(
x)=x2forx ≥
0.Suppo
seforanyx ∈ [
a,a +2],f(
x +a)≥ 2f(
x).
Thent
her
angeofr
ea rai
lnumbe s .
So
lut
ion.Accord
ingt
otheg
ivencond
iton,wehave
i

x2 (
x ≥ 0),
f(
x)= { -x
2
(
x < 0).

So2f(
x)= f(2x).The
re e,t
for heor
igi
nali
nequa
lit
yis
equ
iva
len
ttof(
x +a)≥ f(2x).
30 Ma
thema
tic
alOl
ymp
iadi
nCh
ina

Asf(
x)i
sinc
rea
si rR,t
ngove henx +a ≥ 2x,
i.e.,

a ≥ (2 -1)
x.

Fur
the e,s
rmor i ex ∈ [
nc a,a + 2],(2 - 1)x r
eache
s
(2 -1)(
a +2)t
hemax
imumva
luewhenx = a +2.

The
re e,
for a ≥ (2 -1)(
a +2),
from wh
ichweob
tana ≥
i
Mathematical Olympiad in China (2011–2014) Downloaded from www.worldscientific.com

2,
i.e.,
a ∈ [2,+ ∞ ).
Thean
swe
ri hen [2,+ ∞ ).
st
by 91.124.253.25 on 05/31/18. For personal use only.

1 π
7 Thes
umofa
llt
hepo
sit
ivei
ntege
rsns
ati
sfi
yng <s
in
4 n
1
< is .
3
π ö÷ ,
So
lut
ion.Ass
inxi ∈ ç0,
æ
saconvexf
unc
ti orx
onf we
è 6ø
3
have x < s
inx < x.Then
π

π π 1, π 3 π 1,
s
in < < s
in > × =
13 13 4 12 π 12 4
π π 1, π 3 π 1,
s
in < < s
in > × =
10 10 3 9 π 9 3

t
hati
s

π 1 π π π 1 π
s
in < <s
in <s
in <s
in < <s
in .
13 4 12 11 10 3 9

The
ref
ore,a
llt
hepo
ssi
bleva
lue
sofpo
sit
ivei
ntege
rsna
re
10,11,12,andt
hei
rs s33.
umi
Thean
swe
ris33.

8 Ani
nforma
tions
tat
ionemp
loy
sfou
rdi
ffe
ren s,A ,
tcode
B ,C andD ,
forcommun
ica
tion,bu
teachweeku
seson
ly
Ch
inaMa
thema
tic
alCompe
tit
ion 31

one of t
hem. The code u
sed i
n a de
fin
ite week i
s
r
andoml
yse
lec
tedwi
thequa
lchanceamongt
het
hreeone
s
t
hathave no
tbeen u
sedi
nthel
ast week.Suppo
set
he
codeu
sedi
nthef
irs sA .Thent
tweeki hepr
obab
ili
tyt
hat
Ai
sal
sou
sedi
nthes
even
thweeki
s .(
expr
ess
ed
a
sani
rreduc
ibl
efr
act
ion)
Mathematical Olympiad in China (2011–2014) Downloaded from www.worldscientific.com

So
lut
ion.Le
tPk deno
tet
hepr
obab
ili
tyt
hatcodeA i
sus
edi
n
hekt
t hweek.Thent
heprobab
ili
tyt
hatAi
sno
tus
edi
nthekt
h
s1 -Pk .The
weeki ref
ore,wehave

1(
by 91.124.253.25 on 05/31/18. For personal use only.

Pk+1 = 1 -Pk ).
3

Or

1 1 çæP - 1 ÷ö
Pk+1 - =- .
4 3è 4ø
k

1 3
AsP1 =1, Pk -
{ i
st}
henageome
tri
csequencewi
th
4 4
1
a
sthef
irs
tte
rmand - a
sthecommonr
ati
o.Sowehave
3

1 3 æç - 1 ö÷
k-1
Pk - = .
4 4è 3ø

Or

3 æç - 1 ö÷ 1
k-1
Pk = + .
4è 3ø 4

61
The
ref e,P7 =
or .
243
61
Thean
swe
ris .
243
32 Ma
thema
tic
alOl
ymp
iadi
nCh
ina

Pa
rtI
I Wo
rdPr
obems (
l 56ma
rksi
nto
talf
ort
hre
equ
est
i s)
on
1 3
9 (
16ma
rks)Suppo
sef(
x)= as
inx - cos2x +a - +
2 a
1,
a ∈ R,a ≠ 0.
2
(
1)I
ff(
x)≤ 0foranyx ∈ R,f
indt angeofa.
her
(
2)I
fa ≥2andt
her
eex
issx ∈ Rs
t ucht
hatf(
x)≤0,
find
Mathematical Olympiad in China (2011–2014) Downloaded from www.worldscientific.com

t angeofa.
her
3
So
lut
ion.(
1)Wehavef(
x)= s
in2x +as
inx +a - .Le
t
a
t =s
inx(-1 ≤t ≤ 1).Then
by 91.124.253.25 on 05/31/18. For personal use only.

3
g(
t)=t2 +a
t +a - .
a

Thes
uff
ici
entandnec
ess
arycond
iti orf(
onf x)≤ 0,∀x
∈ Ri
s

ìï (-1)= 1 - 3 ≤ 0,
g
ï a
í
ï () 3
ïg 1 = 1 +2
a- ≤ 0.
î a

The
ref e,weob
or tai
nther s(
angeaofi 0,1].

( a
2)Asa ≥ 2,
then - ≤ -1.Wehave
2

3
g(
t)min = g(-1)= 1 - .
a

3
Thenf(
x)min = 1 - .The
re e,t
for he s
uff
ici
ent and
a
3
nece
ssa
rycond
iti orf(
onf x)≤0,∃x ∈ Ri
s1- ≤0,or0 <
a
a ≤ 3.
Fi
nal
ly,weob
tai
ntha
tther s[
angeofai 2,3].
Ch
inaMa
thema
tic
alCompe
tit
ion 33

10 (
20ma
rks)I
tisknownt
hateacht
e equence {
rmofs an }i
s
anon
-ze
ror
ea r,andforany po
lnumbe sit
ivei
nt rn
ege
ho
ldst
heequa
tion

(
a1 +a2 + … +an )
2
= a1 +a2 + … +an .
3 3 3

(
1)Whenn = 3,f
indou
tal
lthes
equence
scons
ist
ingof
Mathematical Olympiad in China (2011–2014) Downloaded from www.worldscientific.com

t
hreet
ermsa1 ,a2 ,a3 .
(
2)Doe
sthe
reex
istani
nfi
nit
es an }s
equence { ucht
hat
a2013 = -2012?I
fitex
iss,wr
t iteou
ttheformu
laofgene
ral
t
erm;i t,g
fno iveyourr
eas
on.
by 91.124.253.25 on 05/31/18. For personal use only.

So
lut
ion.(
1)Whenn =1,wehavea2
1 =a1 .
3
S
incea1 ≠0,we
ta1 = 1.
ge
Whenn =2,wehave(
1 +a2)
2
= 1 +a2 .
3
S
incea2 ≠0,we
ta2 = 2ora2 = -1.
ge
Whenn = 3,weha
ve(
1+a2 +a3)
2
=1+a2 +a3 .F
3 3
ora2 =
2,wege
ta3 = 3ora3 = -2;f
ora2 = -1,wege
ta3 = 1.
Ins
umma
ry,we ge
tthr
ees
equence
scons
ist
ing o
fthr
ee
t
ermst
hats
ati
sfyt
heg
ivencond
iton:
i

{
1,2,3},{
1,2,-2},and {
1,-1,1}.

(
2)Le
tSn = a1 +a2 + … +an .Thenwehave

= a1 +a2 + … +an (
n ∈ N),
2 3 3 3
Sn
(
Sn +an+1)
2
= a1 +a2 + … +an +an+1 .
3 3 3 3

Fi
ndi
ngou
tthed
iff
erenceo
fthetwoexpr
ess
ion
saboveand
byan+1 ≠ 0,wehave 2Sn = an
2
+1 -an+1 .

Whenn = 1,weknowf
rom (
1)t
hata1 = 1.
Whenn ≥ 2,wehave

an = 2(
2 Sn -Sn-1)= (
an2
+1 -an+1 )- (
an2
-an ).
34 Ma
thema
tic
alOl
ymp
iadi
nCh
ina

Andt
hati
s

( an+1 -an -1)= 0.


an+1 +an )(

tan+1 = -an oran+1 = an +1.


Thenwege
Fi
nal
l roma1 =1anda2013 = -2012,wef
y,f indt
heformu
la
ofgene
ralt
ermforar
equ
ireds
equenc
eas
Mathematical Olympiad in China (2011–2014) Downloaded from www.worldscientific.com

n, 1 ≤ n ≤ 2012,
an = {
2012(-1), n ≥ 2013.
n
by 91.124.253.25 on 05/31/18. For personal use only.

11 (
20ma
rks)Ass
eeni
nFi
g .1,
.11
i
n t
he r
ect
angu
lar coord
ina
te
s
ysem XOY ,t
t he si
de oft he
r sABCDi
hombu ,
s4 and|OB |=
|OD |= 6.
(
1)Pr
ovet
hat|OA|·|OC|i
sa
cons
tan
t.
(
2) When po
intA i
smov
ingon
F
ig.11
.1
t
heha
lfc
irc
le(
x - 2)
2 2
+y =

4(
2 ≤ x ≤ 4),f
indt
het
rac
eof
C.
So
lut
ion.(
1)S
ince|OB |=|OD |
and|AB|=|AD |=|BC|=|CD |,
henO ,A ,C a
t reco
lli
nea
r.
Ass
eeni
nFi
g .2,connec
.11 ting
BD ,t
henBD i
spe
rpend
icu
la oAC
rt
andt
hroughi
ts mi
dpo
int K .So we
have

F
ig.11
.2
Ch
inaMa
thema
tic
alCompe
tit
ion 35

|OA |·|OC | = (
|OK |-|AK |)(
|OK |+|AK |)
2 2
=|OK | -|AK |

|OB | -|BK | )- (
=( |AB | -|BK | )
2 2 2 2

=|OB | -|AB | = 6 -4 = 2
2 2 2 2
0(
acons
t t).
an

(
2)Le
tC(
x,y),A (
2 +2c
osα,2s
inα),whe
re
Mathematical Olympiad in China (2011–2014) Downloaded from www.worldscientific.com

æ π π ÷ö
α = ∠XMA ç - ≤α ≤ .
è 2 2ø

α
Then ∠XOC = .As
2
by 91.124.253.25 on 05/31/18. For personal use only.

α
|OA | = ( osα)
2 +2c +(
2snα)
i = 8( osα)= 16cos2 ,
1 +c
2 2 2

α
hen|OA |= 4co
t s .Comb
ini
ngi
twi
tht
her
esu
l n(
ti 1),we
2
α
t|OC |co
ge s = 5.
2
α α
Thenwehavex =|OC|cos =5,andy =|OC|s
in =
2 2
α
5
tan ∈[-5,5].
2
The
ref e,t
or het
raceo
fpo
intCi
sas
egmen
twi
tht
heend
s
(
5,5)and (
5,-5).

2013 (
Jil
in)

Comm
iss
ion
ed b
y Ch
ine
se Ma
thema
tica
lSo
cie
ty,J
il
in Ma
thema
tica
l
Soc
iet
yor
gan
izedt
he2013Ch
inaMa
thema
tica
lCompe
tit
ionhe
ldon
Oc
t r13,2013.
obe
36 Ma
thema
tic
alOl
ymp
iadi
nCh
ina

Pa
rtI Sho
rt-An
swe
r Qu
est
i s(
on Qu
est
i s1 8,e
on igh
tma
rks
e
ach)
1 venA = {
Gi 2,0,1,3},
letB = {
x|-x ∈ A ,2 -x2 ∉
A }.Thent
hes
umofe
lemen
t nBi
si s .
So
lut
ion.I
tisea
syt
ofi
ndt
hatB ⊆ {-2,0,-1,-3}.We
have2 -x2 ∉ A whenx = -2,-3,and2 -x2 ∈ A whenx =
Mathematical Olympiad in China (2011–2014) Downloaded from www.worldscientific.com

0,-1.The
re e,B = {-2,-3},
for thes
umofwho
see
lemen
ts
s -5.
i
Thean
swe
ris -5.
by 91.124.253.25 on 05/31/18. For personal use only.

2 Inaplanerect
angularcoord
ina
tes ys
temxOy,po in
tsA ,
Bareont heparabo
lay2 =4x, sa
tis
fyingO→
A ·O→
B = -4,
andpo
intF i
sthefocu
soft
he pa
rabo
la.ThenS△OFA ·
S△OFB = .
So
lut
i tF (
on.Le 1,0),A (
x1 ,y1),B (
x2 ,y2).Thenx1 =
2 2
y ,
1 y , 2
x2 = and
4 4

→ → 1(
-4 = OA ·OB = x1x2 +y1y2 = y1y2) +y1y2 ,
2

16

1(
y1y2 + 8) = 0,ory1y2 = - 8.
2
f
rom wh
ich we have
16
The
re e,
for

æç 1 ö æ1
S△OFA ·S△OFB = |OF |·|y1 |÷· ç |OF |·|y2 |÷
ö
è2 ø è2 ø

|OF | ·|y1y2 |= 2.
2
=
4

Thean
swe
ris2.

3 Suppo
s n△ABC wehaves
ei inA = 10s
inBs
inC ,cosA =
10c
osBc
osC .Thent
anA = .
Ch
inaMa
thema
tic
alCompe
tit
ion 37

So
lut
ion.Ass
inA -cosA = 10(
sinBs
inC -c sC)=
osBco
-1 s(
0co B +C)=10cosA ,wehaves
inA =11c
osA .The
ref
ore
t
anA = 11.
Thean
swe
ris11.

4 Suppo
set
hes
ideo
ftheba
seandt
hehe
ighto
fregu
lar
Mathematical Olympiad in China (2011–2014) Downloaded from www.worldscientific.com

t
riangu
larpy
r dP
ami ABC a
re1and 2,r
espec
tive
ly.
Thent
her
adi
uso
fthei
nsc
ribeds
phe
reo
fthepyr
ami
dis
.
So
lut
ion.Ass
eeni
nFi
g .1,s
.4 uppo
set
he
by 91.124.253.25 on 05/31/18. For personal use only.

pr
oje
c t
ionsoft
hein
scr
ibeds
phe
re􀆳
scen
terO
on faces ABC and ABP are H , K,
r
espec
tive
ly,t
he mi
dpo
into sM ,and
fAB i
t
her
adi
usoft
hes
phe
r sr.ThenP ,K ,M
ei
π
a
reco
lli r,∠PHM = ∠PKO = ,and
nea
2
F
ig.4
.1
OH = OK =r,PO = PH -OH = 2 -r,

3 3, 1 53
MH = AB = PM = MH2 +PH2 = +2 = .
6 6 12 6

Thenwehave

r OK MH 1
= =s
in∠KPO = = .
2 -r PO PM 5

2
The
ref
ore,
r= .
6

2
Thean
swe
ris .
6

5 ta,b ber
Le ealnumbe
rs,andf(
x) = ax +bs
ati
sfi
es
|f(
x)| ≤1foranyx ∈ [
0,1].Thent
hemax fab
imumo
38 Ma
thema
tic
alOl
ymp
iadi
nCh
ina

i
s .
So
lut
ion.I
tisea
syt
ofi
nd t
hata = f(
1) - f(
0),b =
f(
0).Then

1
1)÷ + 1 (
ö2
ab = f(
0)·(
f( 0))= - çf(
1)-f( 0)- f( f(1))
æ 2
è 2 ø 4
1 ( ( )) 1
f1
2
≤ ≤ .
Mathematical Olympiad in China (2011–2014) Downloaded from www.worldscientific.com

4 4

1 1
When2f(
0)=f(
1)= ±1,
i.e.,
a =b = ± ,wege
tab = .
2 4
1
The
ref e,t
or hemax fabi
imumo s .
by 91.124.253.25 on 05/31/18. For personal use only.

4
1
Thean
swe
ris .
4

6 Taker
andoml
yfi
ved
iff
eren
tnumbe
r rom 1,2,...,
sf
20.Then t
he pr
obab
ili
tyt
hatt
her
ear
eatl
eas
ttwo
ad
jac
entnumbe
rsamongt
hemi
s .
So
lut
ion.Suppos
ea1 < a2 < a3 < a4 < a5 t rom 1,
akenf
2,...,20.I
fa1 ,a2 ,a3 ,a4 ,a5 a
re no
tad
jacen
tto each
o
t r,t
he henwehave

1 ≤ a1 < a2 -1 < a3 -2 < a4 -3 < a5 -4 ≤ 16,

f
rom wh
ich weknow t
hatt
henumbe
rof way
stos
ele
ctf
ive
numbe
rsno
tad
jacen
ttoeacho
the
rfom 1,2,...,20i
r sthe
s
amea
sse
lec
tingf
ived
iff
eren
tnumbe
r rom 1,2, ...,16,
sf
e.,C5
i. 16 .The
re e,t
for her
equ
iredpr
obab
ili
tyi
s

C5
20 - C
5
C5 232
=1- 5 =
16 16
.
C20
5
C20 323

232
Thean
swe
ris .
323
Ch
inaMa
thema
tic
alCompe
tit
ion 39

7 Suppo
ser
ealnumbe
rsx,ys
ati
sfyx -4 y = 2 x -y .
Thent
her fxi
angeo s .

So
lut
ion.Le
ty =a, x -y =b(
a,b ≥0).Thenx =y +
(
x -y)= a2 +b2 .Theequa
tioni
ntheque
sti sa2 +
onbecome
b,wh
b2 -4a =2 ichi
sequ
iva
len
tto

(
a -2)
2
+(
b -1)
2
a,b ≥ 0).
=5 (
Mathematical Olympiad in China (2011–2014) Downloaded from www.worldscientific.com

Ass
eeni
nFi
g .1,t
.7 het
rac
eof
poi
nt(a,b)i
np laneaObi
st hepa
rt
ofthec
irc
lewit
hc ente ( , )
r 2 1 and
by 91.124.253.25 on 05/31/18. For personal use only.

r
adi
us 5s
ati
sfi
ynga,b ≥ 0,i.
e.,
t
he union of po
int O and a
rc

ACB .Then

.7
.1
a2 +b2 ∈ {
0}∪ [
2,2 5]. F
ig

The
ref e,x =a2 +b2 ∈ {
or 0}∪
[
4,20].
Thean
swe
ris{
0}∪ [
4,20].

8 Suppo
ses
equ
ene{
c an }c
ons
is
tso
fni
net
erms,wh
ichs
ati
sfy
ai+1 1
a1 =a9 =1and ∈ 2,1,-{ f
oranyi ∈ {
1,2,...,
}
ai 2
8}.Thent
henumbe
rofs
equ
enc
esl
iket
hisi
s .
ai+1 (
So
lut
ion.Le
tbi = 1 ≤ i ≤ 8).Then foreach {
an }
ai
s
ati
sfi
yngt
heg
ivencond
iton,wehave
i
8 8
ai+1 a9 1 (
∏b
i=1
i = ∏
i=1 ai
=
a1
= 1,wi
thbi ∈ 2,1,-
{ 2
1 ≤i ≤ 8).
}

Conve
rse
ly,as
equenceo
fei
ghtt
erms{
bn }
sat
isf
ying ① can
40 Ma
thema
tic
alOl
ymp
iadi
nCh
ina

un
ique
lyde
termi
neas
equence{
an }
int
heque
sti
on.
1
Ineach {
bn },t
her
ear
eobv
iou
sl rof - and
yevennumbe 2
t
hes
amenumbe
rof2,wi
tht
her
ema
inde
rbe
ing1.Or,
ino
the
r
1
s,t
word he numbe
rsof- and 2 a
re bo
t k,wh
h2 ilet
he
2
numbe s8 -4
rof1i k.He
re,i
tisea
syt
oche
ckt
hatkcanon
ly
Mathematical Olympiad in China (2011–2014) Downloaded from www.worldscientific.com

be0,1,2.Onceki
sgven,t
i her
ear 8 C
eC2k 2k
8-2k way
stocons
truc
t
{
bn }.The
re e,t
for het
ota
lnumbe
rof{
bn }s
ati
sfi
yng ①i
s

N = 1 + C2
8C6 +C
2
8C4 = 1 +2
4 4
8 ×15 +70 ×1 = 491.
by 91.124.253.25 on 05/31/18. For personal use only.

Thean
swe
ris491.

Pa
rtI
I Wo
rdPr
obems (
l 56ma
rksi
nto
talf
ort
hre
equ
est
i s)
on
9 (16 ma s) Suppo
rk s
e pos
iti
ve numbe
rs e {
equenc xn }
s
ati
sfi
esSn ≥2Sn-1 ,n =2,3,...,whereSn =x1 + … +
xn .Provet
hatt
her
eex
ist
sacons
t tC > 0,s
an ucht
hat

xn ≥ C ·2n ,n = 1,2,....

So
lut
ion.Whenn ≥ 2,Sn ≥ 2Sn-1i
sequ
iva
len
tto

xn ≥ x1 + … +xn-1 . ①

1
tC =
Le x1 .Wewi
llpr
ove
4

xn ≥ C ·2n ,n = 1,2,.... ②

byi
nduc
tion.
Whenn = 1,
iti
sobv
iou
slyt
rue.Whenn = 2,wehavex2
≥ x1 = C ·2 .
2

Whenn ≥ 3,a
ssumexk ≥ C ·2k ,k = 1,2,...,n -1.
rom ① ,wehave
Thenf
Ch
inaMa
thema
tic
alCompe
tit
ion 41

xn ≥ x1 + (
x2 + … +xn-1)
≥ x1 + (
C ·22 + … +C ·2n-1)
= C(
22 +22 +23 + … +2n-1)= C ·2n .

The
ref e,② ho
or ldsforeve
ryn.

2
x2 y
10 (
20ma s) Gi on 2 + 2 = 1(
Mathematical Olympiad in China (2011–2014) Downloaded from www.worldscientific.com

rk venane
lli
pseequa
ti a >b
a b
> 0)
inap
laner
ect
angu
larcoord
ina
tes
ysemxOy,
t letA1 ,
A2 ,F1 ,F2 bei
tsl
eftandr
ightend
-po
ins,l
t eftandr
ight
focuss,r
e espec
tive
ly,andP beanypoi
ntont heel
lis
pe
by 91.124.253.25 on 05/31/18. For personal use only.

diff
erentf
rom A1 ,A2 .Suppo
sethe
reare po
insQ ,R
t
s
ati
sfi
yngQA1 ⊥ PA1 ,QA2 ⊥ PA2 ,RF1 ⊥ PF1 ,RF2 ⊥
PF2 .Fi
ndandpr
ovet
her
ela
tionsh
ipbe
tweent
hel
eng
th
ofs tQR andb.
egmen

So
lut
ion.Le
tc = a2 -b2 .ThenA1(- a,0),A2(
a,0),
F1(-c,0),F2(
c,0).
x2
eP (
x0 ,y0),Q (
x1 ,y1),R (
x2 ,y2),whe
0
Deno
t re 2 +
a
2
y0
= 1,y0 ≠ 0.
b2
FromQA1 ⊥ PA1 ,QA2 ⊥ PA2 ,wehave

A1→
Q ·A1→ x1 +a)(
P =( x0 +a)+y1y0 = 0, ①

A2→
Q ·A2→
P =(
x1 -a)(
x0 -a)+y1y0 = 0. ②

Sub
trac
ti a(
ng ① and ② ,wehave2 x1 +x0)=0,
i. x1 =
e.
-x0 .Sub
sti
tut
ingi
tin
to ① ,wege
t-x0 +a +y1y0 = 0,or
2 2

x2
0 -a
2
x2 2
0 -a ö
.ThenQ ç -x0 ,
æ
y1 = ÷
.
y0 è y0 ø
FromRF1 ⊥ PF1 ,RF2 ⊥ PF2 , i
nt hes
amewayweob
tai
n
x2 2
0 -c ö
R ç -x0 , e,
æ ÷
.The
ref
or
è y0 ø
42 Ma
thema
tic
alOl
ymp
iadi
nCh
ina

x2
0 -a
2
x2
0 -c
2
b2
|QR |= - = .
y0 y0 |y0 |

S
i e|y0 |∈ (
nc 0,b],t
hen|QR |≥ b,whe
ret
heequa
lit
y
ho
ldsi
fandon
l f|y0 |=b (
yi i.e.,P (
0,±b)).

11 (
20ma
rks)F
inda
llt
hepo
sit
iver
ealnumbe
rpa
irs(
a,b),
Mathematical Olympiad in China (2011–2014) Downloaded from www.worldscientific.com

s
ucht
hatf(
x)= ax2 +bs
ati
sfi
es

f(
xy)+f(
x +y)≥ f(
x)f(
y)(
foranyr
ealnumbe
rsx,y).

So
lut
ion.Theg
ivencond
iti
oni
sequ
iva
len
tto
by 91.124.253.25 on 05/31/18. For personal use only.

(
ax2y2 +b)+ (
a(x +y)
2
+b)≥ (
ax2 +b)(
ay2 +b). ①

In① ,l
ety =0.Wehaveb + (
ax2 +b)≥ (
ax2 +b)
·b,or

(
1 -b)
ax2 +b(
2 -b)≥ 0.

Asa > 0andax2canbes


uff
ici
ent
lyl
ar hen1 -b ≥ 0,
ge,t
e.,0 <b ≤ 1.
i.
In ① ,l
ety = -x.Wehave(
ax4 +b)+b ≥ (
ax2 +b)
2
,or

(
a -a2)
x4 -2abx2 + (
2b -b2)≥ 0. ②

Deno
tet
hel
eft
-hands
ideo sg(
f② a x).I
tisobv
iou
stha
t
a -a2 ≠0(
othe
rwi
se,f
roma >0weknowa =1.Theng(
x)=
bx2 +(
-2 2b - b2)wi
thb > 0,wh
ich meansg(
x)can be
nega
tive.A con
trad
ict
ion).Then

ab ö÷2 ab)
( 2
g(
x)= (
a -a2)çx - +(
2b -b2)
æ 2
2 -
è a -a ø a -a2
b ö÷2 b (
=(
a -a2)çx - a -b)
2 -2
æ
2
+
è 1 -a ø 1 -a
≥0

ho
ldsforanyr
ea rx.Sowehavea -a2 > 0,
lnumbe i.e.,0 <
a < 1.
Ch
inaMa
thema
tic
alCompe
tit
ion 43

b
Fur
the e,f
rmor rom > 0and
1 -a

æ b ö÷ = b ( a -b)≥ 0,
2 -2

è 1 -a ø 1 -a

wehave2
a +b ≤ 2.
Sof
ar,wege
tthenec
ess
arycond
iti
ont
hata,
b mu
sts
ati
sfy
Mathematical Olympiad in China (2011–2014) Downloaded from www.worldscientific.com

a
sfo
llows:

0 <b ≤ 1,0 < a < 1,2


a +b ≤ 2. ③

Wea
rego
ingt
oprovet
hatf
oranypa
ir(
a,b)
sat
isf
ying ③
by 91.124.253.25 on 05/31/18. For personal use only.

andanyr
ealnumbe
rsx,y,① ho
lds,orequ
iva
len
tly,

h( a -a2)
x,y)= ( x2y2 +a( x2 +y2)+
1 -b)(
axy + (
2 2b -b2)≥ 0.

Asama
tte
roff
act,when ③ ho
lds,wet
henhave

b (
a(
1 -b)≥ 0,a -a2 > 0and a -b)≥ 0.
2 -2
1 -a

Comb
ini
ngi
twi
thx2 +y2 ≥ -2xy,wege
t

h( a -a2)
x,y)≥ ( x2y2 +a( axy + (
1 -b)(-2xy)+2 2b -b2)
=(
a -a2)
x2y2 +2abxy + (
2b -b2)
b ö÷2 b (
=(
a -a2)çxy + a -b)≥ 0.
2 -2
æ
+
è 1 -a ø 1 -a

The
re e,t
for hes
etofa
llt
hepa
irs(
a,b)mee
tingt
heg
iven
cond
iti
oni
s

{(
a,b)|0 <b ≤ 1,0 < a < 1,2a +b ≤ 2}.
Chi
naMa thematica
l
Competi
tion
(
Compl
ementaryTe t)
s
Mathematical Olympiad in China (2011–2014) Downloaded from www.worldscientific.com
by NATIONAL UNIVERSITY OF SINGAPORE on 05/06/18. For personal use only.

2010 (
Fui
jan)

1 (40 ma
rks) As s
een i
n
Fi.1
g .1,the ci
rcumcen
ter
ofacut
etr
iang
leABCi sO ,
K is a po
int (notthe
mi
dpoi
nt)onthesideBC ,
Di
sapo
intont
heex
tended
l
ine o
fs t AK ,l
egmen ine
s
BD and AC i
nte
rsec
t a
t F
ig.1
.1

po
intN ,andl
i sCD and
ne
ABi
nte
rse
cta
tpo
intM .Pr fOK ⊥ MN ,
ovei thenA ,B ,
Ch
inaMa
thema
tic
alCompe
tit
ion (
Comp
lemen
tar
yTe
st) 45

D ,C a
reconcyc
lic.
So
lut
ion. By reduc
tion t
o
ab
surd
ity,a
ssumet
hatA ,B ,D ,
Ca
reno
tconcyc
lic.Le
tthec
ircu
-
mc
irc
le (wi
thr
adi
usr)of ABC
i
nte
rse
ctAD a
tpo
intE .Jo
inBE
Mathematical Olympiad in China (2011–2014) Downloaded from www.worldscientific.com

andex
tendi
ttoi
nte
rsec
tlneAN
i
by NATIONAL UNIVERSITY OF SINGAPORE on 05/06/18. For personal use only.

a
tpo
intQ ;j
oinCE andex
tendi
t F
ig.1
.2

t
oin
ter
s tAM a
ec tP .J
oinPQ ,a
s
s
eeni
nFi
g.1
.2.
Wehave

PK2 =t rofP wi
hepowe thr
espec
tto ☉O +
t rofQ wi
hepowe thr
espec
tto ☉O
=(
PO2 -r2)+ (
KO2 -r2).

(Wewi
llprovei
tint
heappend
ix)
Int ameway,
hes

QK2 = (
QO2 -r2)+ (
KO2 -r2).

Thenwehave

PO2 -PK2 = QO2 -QK2 .

The
ref e,OK ⊥ PQ .Byt
or heg
ivencond
itonOK ⊥ MN ,
i
wege
tt tPQ ‖MN .Thenwehave
ha

AQ AP
= . ①
QN PM

By Mene
lau
s em,weob
􀆳Theor tai
n

NB ·DE ·AQ
= 1, ②
BD EA QN

MC ·DE ·AP
= 1. ③
CD EA PM
46 Ma
thema
tic
alOl
ymp
iadi
nCh
ina

NB MC , ND MD
om ① ,② ,③ ,wege
Fr t = or = .Then
BD CD BD DC
△DMN ∽ △DCB ,wh
ichimp
lis ∠DMN = ∠DCB .Then
e
BC ‖MN .The
ref e,OK ⊥ BC ,andt
or hat means K i
sthe
mi
dpo
intofBC ,wh
ichi
sacon
trad
ict
ion.Th
iscomp
let
est
he
prooft
hatA ,B ,D ,C a
reconcyc
lic.
Append
ix.Wearego
Mathematical Olympiad in China (2011–2014) Downloaded from www.worldscientific.com

ingt
opr
ove
by NATIONAL UNIVERSITY OF SINGAPORE on 05/06/18. For personal use only.

PK2 =t rofP wi
hepowe thr
espec
tto ☉O +
t rofQ wi
hepowe thr
espec
tto ☉O .

Ex
tendPK t
opo
intF ,
sucht
hat

PK ·KF = AK ·KE ④

(
seeF
ig .3).ThenP ,E ,F ,A
.1
a
reconcyc
lic,andwehave

∠PFE = ∠PAE = ∠BCE .

Then E , C , F , K a
re
concyc
lic,andwehave

PK ·PF = PE ·PC . ⑤
.1
.3
om ⑤ and④ ,weg
Fr etPK2 = F
ig

PE ·PC -AK ·KE =t


hepowe
r
ofP wi
thr
espe
c o ☉O +t
tt hepowe
rofQ wi
thr
espe
c o ☉O .
tt

Remark.I
fEi
sont
heex
tendedl
ineofAD ,t
hent
hepr
oofi
s
s
imi
lar.

1
2 (
40ma
rks)Gi
venpo
sit
ivei
nt rk,
ege letr =k + .De
fine
2

f ( r)= f(
r)=rr ,f l (
r)= f( (r)),x ∈ R+ ,l≥ 2.
(
1) () (
l-1)
f
Ch
inaMa
thema
tic
alCompe
tit
ion (
Comp
lemen
tar
yTe
st) 47

(
Hee x deno
r test
he mi
nimumi
ntege
rno
tle
s hanx;e.
st g.,
1
= 1,1 = 1.)Pr
ovet
hatt
her
eex
ist
sapo
sit
ivei
nt rm
ege
2
tf m (r)
( )
s
ucht
ha isani
ntege
r.
So
lut
ion.Def
inev2(
n)a
st tof2i
heexponen npo
sit
ivei
ntege
r
tf m (r) rform =v2(
k)+1.
( )
n.Wewi
llprovet
ha isani
ntege
ononv2(
k)=v.
Mathematical Olympiad in China (2011–2014) Downloaded from www.worldscientific.com

Weu
sema
thema
tica
linduc
ti
by NATIONAL UNIVERSITY OF SINGAPORE on 05/06/18. For personal use only.

Whenv = 0,ki
soddandk +1i
seven.Then

1ö 1 1 ö÷ (
f(
r)= f 1 (
r)= çk + ÷ k + k +1)
() æ æ
= çk +
è 2ø 2 è 2ø

i
sani
ntege
r.
As
sumet
hepropo
sit
ioni
str
ueforv -1(
v ≥1).Thenforv
≥ 1,
let

k = 2v +αv+1 ·2v+1 +αv+2 ·2v+2 + …,

whe
reαi ∈ {
0,1}fori =v +1,v +2,....Wehave

1ö 1 1 ö÷ (
f(
r)= çk + ÷ k + k +1)
æ æ
= çk +
è 2ø 2 è 2ø
1 k 2
= + +k +k
2 2
1
+2 - + (
αv+1 +1)·2v + (
αv+1 +αv+2)·
v 1
=
2
2v+1 + … +22v + …
1,
=k
'+ ①
2

wi
th

' = 2v-1 + (
k αv+1 +1)·2v + (
αv+1 +αv+2)·2v+1 + … +22v + … .

1
Obv
iou
sly,v2(
k') = v - 1.Le
tr' = k
' + .By
2
on,weknowf v (')i r,wh
()
a
ssump
ti r sani
ntege ichi
sequa
lto
48 Ma
thema
tic
alOl
ymp
iadi
nCh
ina

f
(
v+1)
(
r)by ①.Theproofi
sthencomp
let
ed.

3 (
50 ma
rks)Gi
veni
nt rn > 2,s
ege uppo
se po
sit
iver
eal
numbe
rsa1 ,a2 ,...,ans
ati
sfyak ≤1,k =1,2,...,n.
a1 +a2 + … +ak ,
tAk =
Le k = 1,2,...,n.
k
Mathematical Olympiad in China (2011–2014) Downloaded from www.worldscientific.com

n n
n -1
Prove ∑ ak - ∑ Ak < .
by NATIONAL UNIVERSITY OF SINGAPORE on 05/06/18. For personal use only.

k=1 k=1 2

on.For1 ≤k ≤n -1,wehave0 < ∑i=1ai


k
So
lut
i ≤kand0 <

∑ t|x -y|< max{


x,y}
n
i=k+1 i
a ≤n -k.Byu
singt
hef
actt
ha

forx,y > 0,wege


t
k n
æç 1 1 ö÷ 1
|An - Ak | = -
èn k ø i ∑
=1
ai + ∑ ai
ni=k+1

n k
1 æ1 1 ö
ni∑
= ai - ç - ÷ ∑ ai
=k+1 è k n ø i=1
n k
1 æ1 1 ö
ni∑
< max { ai ,ç - ÷ ∑ ai }
=k+1 è k n ø i=1

1 æ 1 1 ÷ö
≤ max { ( n -k),ç - k }
n èk n ø
k
=1- .
n

The
ref
ore,
n n n

∑ ak - ∑Ak = nAn - ∑ Ak
k=1 k=1 k=1

n-1 n-1

= ∑ (An -Ak ) ≤
k=1
∑ |A
k=1
n - Ak |

n-1
k ö÷ n -1
∑ è1 - n ø =
æç
< .
k=1 2

Th
iscomp
let
est
hepr
oof.
Ch
inaMa
thema
tic
alCompe
tit
ion (
Comp
lemen
tar
yTe
st) 49

4 (
50ma
rks)Thecodes
ett
ingo
fac
iphe
rlocki
ses
tab
lished
onann
-regu
lar
-po
lygon wi
thve
rti
cesA1 ,A2 , ...,An :
eachve
rtexi
sas
si r(
gnedanumbe 0or1)andaco
lor (
red
orb
lue),s
ucht
hate
ithe
rthenumbe
rsort
heco
lor
son
eachpa
irofad
jacen
tve
rti
cesa
ret
hes sk:How
ame.Wea
manycode
-se
tscanber
eal
izedfort
hi ock?
sl
Mathematical Olympiad in China (2011–2014) Downloaded from www.worldscientific.com

So
lut
ion.Gi
ven an a
rbi
tra
ry code
-se
tfort ock,i
hel ftwo
by NATIONAL UNIVERSITY OF SINGAPORE on 05/06/18. For personal use only.

ad
jac
entve
rti
ceshave d
iff
eren
tnumbe
rs,wel
abe
lthes
ide
s
l
ink
ingt
hembyl
ett
era;
ift
heyhaved
iff
eren
tco
l s,wel
or abe
l
tbyb;
i ifbo
tht
henumbe
rsandco
lor
sar
et ame,wel
hes abe
lit
byc.Oncethenumberandco
loronver
texA1ares
et(ther
ea re
fourd
iff
eren
tset
sfori ),
t wecanthense , , ,
tA2 A3 ... An one
byoneaccord
ingtothelet
ter
slabe
lledoneachsi
de.Inorde
rto
le
titr
eturntothein
itia
lsetofA1final
ly,thenumber
sofside
s
l
abe
lledaandb mu
stbebo
theven.Sot
henumbe
rofcode
-se
ts
fort
he l
ock i
s fou
rtime
soft
he numbe
r of l
abe
lled
-si
de
s
equenc
eswh
ichs
ati
sfyt
hecond
iti
ont
hatt
henumbe
rsofs
ide
s
l
abe
lledbyaandba
rebo
theven.
n ö÷
iç0 ≤i ≤ edbya,
æ
Suppos
ethe
rea
re2 s
ide
slabe
ll [ ]
è 2 ø
n -2i ö÷
and2j ç0 ≤ j ≤
æ
[ s
i
]de
slabe
lledbyb.Thent
her
ear
e
è 2 ø
Cn
2i
way
stol l2
abe is
i sbyaf
de r s,Cn
omn one 2j
-2i way
stol
abe
l
2js
i sbyb f
de rom n - 2 s,andt
i one her
ema
ini
ngs
ide
sar
e
l
abe
lled byc.The
ref
ore, by t
he Mu
lti
pli
cat
ion Pr
inc
ipe,
l
t
her e Cn
e ar Cn-2i way
2i 2j
stol
abe
lal
lthe s
ide
s. So t
her
e
ar
etot
all
y

[ n2 ] æ [n-22i ] ö
4∑ çç C 2
i
n ∑ Cn
2j
-2i÷
÷ ①
i=0 è j=0 ø
50 Ma
thema
tic
alOl
ymp
iadi
nCh
ina

code
-se
tsf
ort
hel
ock.He
rewes
tipu
lat 0 = 1.
eC0
sodd,wehaven -2
Whenni i > 0,andt
hen

[n-22i ]
∑ Cn i =2 ②
2 n-2
i-1
j
-2 .
j=0

Sub
sti
tut
ingi
tin
to ① ,wege
t
Mathematical Olympiad in China (2011–2014) Downloaded from www.worldscientific.com

[ n2 ] æ [n-22i ] [2 ] n
[2 ] n
by NATIONAL UNIVERSITY OF SINGAPORE on 05/06/18. For personal use only.

ö
4∑ çç C 2
i
n ∑ C
2j
n-2i ÷÷ = 4
∑ (
C 2
2
i n-2
n
i-1
)= 2
∑ (
Cn2
2 i)
i n-2

i=0 è j=0 ø i=0 i=0

n n

∑Cn2 + ∑Cn2 (-1)


k n-k k n-k k
=
k=0 k=0

=(
2 +1) + (
2 -1)
n
n

= 3 +1.
n

n, n
seven,i
Whenni fi < hen ② r
t ema
inst
rue;i
fi = ,
2 2
t
hena
llt
hes
ide
soft
he po
lygona
rel
abe
lled bya,andt
hat
meanst
her
eison
lyonewayt
olabe
lthes
ide
s.The
re e,t
for her
e
a
ret
ota
lly

[ n2 ] æ [n-22i ] ö æ [ n2 ] -1 ö
4∑ çç C 2
i
n ∑ C
2j
n-2i ÷÷ = 4 × çç1 +
∑ (Cn
2
2 i-1)÷÷
i n-2

i=0 è j=0 ø è i=0 ø


[ n2 ]
= 2 +4 ∑ (C 2 )= 3n +3
i n-2
2 i-1
n
i=0

code
-se
tsfort
hel
ock.
Ins
umma
ry,

sodd,
3n +1whenni
t
henumbe
rofcode
-se
tsfort
hel
ock = {
3n +3whenni
seven.
Ch
inaMa
thema
tic
alCompe
tit
ion (
Comp
lemen
tar
yTe
st) 51

2011 (
Hubei)

1 (40 ma s) As s
rk een i
nF i.1
g .1, poin
ts P,Q are,
r
espec
tively,t
he midpo
ins of AC , BD — t
t he two
d
iagona
lsofcyc
lic quadr
ila
ter
a t ∠BPA =
lABCD .Le
∠DPA .Prove ∠AQB = ∠CQB .
Mathematical Olympiad in China (2011–2014) Downloaded from www.worldscientific.com
by NATIONAL UNIVERSITY OF SINGAPORE on 05/06/18. For personal use only.

F
ig.1
.1 F
ig.1
.2

So
lut
ion.Asshownin Fi
g .2,weex
.1 tends tDP t
egmen o
i
nte
rcep
twitht
hecirc
lea
tpointE .Then ∠CPE = ∠DPA =
︵ ︵
∠BPA .Si
ncePist
hemidpo
intofAC ,wegetAB =CE ,wh
ich
means ∠CDP = ∠BDA .Fur
the e, ∠ABD = ∠PCD .
rmor
AB PC ,
The
ref e,△ABD ∽ △PCB .Then
or = e.,AB ·
i.
BD CD
CD =PC ·BD .
Thenwehave

1 æ1
AB ·CD = AC ·BD = AC · ç BD ÷ = AC ·BQ ,
ö
2 è2 ø

AB BQ
or = .Comb
ini
ngi
twi
th ∠ABQ = ∠ACD ,wede
rive
AC CD
t
hat△ABQ ∽ △ACD .So ∠QAB = ∠DAC .
Ex
tend
ings tAQ t
egmen oin
ter
cep
twi
tht
hec
irc
lea
tpo
int
F ,wet
henhave
52 Ma
thema
tic
alOl
ymp
iadi
nCh
ina

∠CAB = ∠QAB - ∠QAC = ∠DAC - ∠QAC = ∠DAF ,


︵ ︵
wh
ich meansBC = DF .Fu
rthe e,a
rmor sQ i
sthe mi
dpo
intof
BD ,
then ∠CQB = ∠DQF .
S
ince ∠AQB = ∠DQF ,wet
henhave ∠AQB = ∠CQB .
Theproo
fiscomp
let
ed.
Mathematical Olympiad in China (2011–2014) Downloaded from www.worldscientific.com

2 (
40 ma
rks)Proveforanyi
nt rn ≥ 4,t
ege her
eex
ist
sa
by NATIONAL UNIVERSITY OF SINGAPORE on 05/06/18. For personal use only.

po
lynomi
alofdeg
reen,

f(
x)= xn +an-1xn-1 + … +a1x +a0

wi
tht
hefo
llowi
ngprope
rti
es.
(
1)a0 ,a1 ,...,an-1 a
rea
llpo
sit
ivei
ntege
rs;
(
2)Foranypo
sit
ivei
nt rm anda
ege rbi
tra
ryk(
k ≥ 2)
posi
tiveint
ege
rsr1 ,r2 , ...,rk t
hata
red
iff
eren
tfrom
eacho t
her,wehave

f(
m )≠ f(
r1)
f(r2)…f(
rk ).

So
lut
ion.Le
t

f(
x)= (
x +1)(
x +2)…(
x +n)+2. ①

Obv
iou
sly,f(
x)i
sa mon
icpo
lynomi
alofdeg
ree n wi
th
po
sit
iveintege
rcoe f
fic
ien
ts.Wea
rego
ingt
oprovet
hatf(
x)
ha
spr ope
rty( 2).
Foranyi
nt rt,s
ege i en ≥ 4,weknowt
nc hatt
her
eex
ist
s
de
fin
ite
lyamu
lti
pleo nanyn con
f4i secu
tivenumbe
rst +1,
t
+2,...,
t +n.Thenf
rom ① ,wehavef(
t)≡ 2(
mod4).
oranyk (
Thenf k ≥2)po
sit
ivei
ntege
rsr1 ,
r2 ,...,
rk ,we
have

f(
r1)
f(r2)…f(
rk )≡ 2k ≡ 0(
mod4).

Ont
heo
t rhand,f
he oranypo
sit
ivei
nt r m ,wehave
ege
Ch
inaMa
thema
tic
alCompe
tit
ion (
Comp
lemen
tar
yTe
st) 53

f(
m )≡ 2(
mod4).The
ref e,
or

f( r1)
m )≢ f( f(r2)…f(
rk )(
mod4),

wh
ichimp
lie
st tf(
ha m )≠ f( f(
r1) r2)…f(
rk ).Wet
henf
ind
t
her
equ
iredf(
x)andcomp
let
etheproof.

3 (
50 ma
rk ta1 ,a2 ,...,an (
s)Le n ≥ 4)bepo
sit
iver
eal
Mathematical Olympiad in China (2011–2014) Downloaded from www.worldscientific.com
by NATIONAL UNIVERSITY OF SINGAPORE on 05/06/18. For personal use only.

numbe
rswi
tha1 < a2 < … < an .Foranypo
sit
iver
eal
r r,t
numbe he numbe
roft
erna
r r
ygoups (
i,j,k)
aj -ai
s
ati
sfi
yng =r (
1 ≤i <j < k ≤ n)i
sdeno
teda
s
ak -aj
n2
fn (
r).Provefn (
r)< .
4
So
lut
i venj(
on.Gi 1 <j < n),t
henumbe
roft
erna
r r
ygoups
(
i,j,k)s
ati
sfi
yng1 ≤i <j <k ≤ nand

aj -ai
=r ①
ak -aj

i
sdeno
tedasgj (
r).Forfxedi,j wi
i thi <j,t
her
ei sa
tmo s
t
oneks
ati
sfi
y ;
ng ① s ot
hereaej -1way
r st
ochoos ,
ei wh
ich
meansgj (
r)≤j -1.
Inas
imi
larway,forf
ixedj,k wi
thk >
j,therei
satmostoneisatisf
ying ① ;s
othe
rea ren -j way
sto
chooek,wh
s ich meansgj (
r)≤ n -j.The re e,
for

gj (
r)≤ mi
n{j -1,n -j}.

Then,whenni
seven (
i.e.,
n = 2m ),wehave
n-1 m-1 2m-1

fn (
r)= ∑gj (
j=2
r)= ∑gj (
j=2
r)+ ∑gj (
r)
j=m

m(
m -1) m (
m -1)
m 2m-1

≤ ∑(
j -1)+ ∑ (
j=2
2m -j)=
j=m+1 2
+
2

2 2 n2
= m -m < m = .
4
54 Ma
thema
tic
alOl
ymp
iadi
nCh
ina

sodd (
Whenni i. n = 2m +1),wehave
e.
n-1 m 2m

fn (
r)= ∑gj (
j=2
r)=
j=2
∑gj (
r)+ ∑g (
r)
j=m+1
j

m 2m

≤ ∑(
j -1)+
j=2
∑ (2m +1 -j)
j=m+1

2 n2
=m < .
Mathematical Olympiad in China (2011–2014) Downloaded from www.worldscientific.com

4
by NATIONAL UNIVERSITY OF SINGAPORE on 05/06/18. For personal use only.

Thepr
oofi
scomp
let
ed.

4 (
50ma
rks)Gi
vena3×9a
rrayA wi
theachce
llcon
tai
ning
apo
siti
vei
nt r,wes
ege ayam ×n (1 ≤ m ≤3,1 ≤n ≤9)
s
ubar
rayofA isa “goodr
ectang
le”i fthesum ofthe
numbe
rsi
nit
sce
llsi
samu
lti
pleof10,andca
lla1×1ce
ll
ofA “
bad”i
fiti
sno
tcon
tai nany “
nedi goodr
ect
ang
le”.
F
indt
hemax
imumnumbe
rof“
badce
lls”i
nA .
So
lut
ion.Wef
irs
tcl
aimt
hatt rof “
henumbe badce
lls”i
nAi
s
no mor
ethan25.Ot
herwi
se,t
her
ewi
llbea
tmo
stonec
el nA
li
t
hati t“
sno bad”.Wi
thou
tlo
sso
fgene
ral
ity,wea
ssumet
he
c
ell
sint
hef
irs
tr fA a
owo rea
ll“
bad”.Thenl
ett
henumbe
rs
f
romt
op t
o bo
ttom i
ntheit
hco
l i = 1,
umn beai ,bi ,ci(
2,...,9)
int
urn,andde
fine
k k

Sk = ∑ai ,Tk =
i=1
∑(
b
i=1
i +ci ),k = 1,2,...,9,

wi
thS0 = T0 = 0.Wea
rego
ingt
o pr
ovet
hatt
hreenumbe
r
r
goupsS0 ,S1 ,...,S9 ,T0 ,T1 ,...,T9 ,andS0 +T0 ,S1 +
T1 , ...,S9 + T9 each form a comp le t
es e
to fre si
dues
modu
lo10:
I
fthe
reex
istm ,n,0 ≤ m <n ≤9s
ucht
hatSm ≡Sn (
mod
10),t
hen
Ch
inaMa
thema
tic
alCompe
tit
ion (
Comp
lemen
tar
yTe
st) 55

∑a
i=m+1
i = Sn -Sm ≡ 0(
mod10),

wh
ich meanst
hatt
hec
ell
sint
hef
irs
trow andf
rom co
lumns
m +1t orma “
onf goodr
ect
ang
le”.Bu
titi
sacon
trad
ict
iont
o
t
hea
ssump
tiont
hatt
hec
ell
sint
hef
irs
trowa
rea
ll“
bad”.
I
fthe
reex
istm ,n,0 ≤ m <n ≤9s
ucht
hatT m ≡Tn (
mod
Mathematical Olympiad in China (2011–2014) Downloaded from www.worldscientific.com

10),t
hen
by NATIONAL UNIVERSITY OF SINGAPORE on 05/06/18. For personal use only.

∑(
b
i=m+1
i +ci )= Tn -T m ≡ 0(
mod10).

Sot
hece
llsr
ang
ingf
romrows2t
o3andco
lumnsm +1t
o
n forma “
goodr
ect
ang
le”,wh
ich meanst
her
ear
eatl
eas
ttwo
c
ell
stha
tar t“
eno bad”.Bu
titi
sal
soacon
trad
ict
ion.
Inas
imi
larway,wecana
lsopr
ovet
hatt
her
eaenom ,n,
r
0 ≤ m < n ≤ 9s
ucht
hat

Sm +T m ≡ Sn +Tn (
mod10).

The
ref e,wehave
or
9 9 9

∑Sk ≡
k=0 k=0
∑Tk ≡ ∑ (S
k=0
k +Tk )≡ 0 +1 +2 + … +9

≡ 5(
mod10).

Then
9 9 9

∑ (Sk +Tk )≡
k=0
∑ Sk + ∑Tk ≡ 5 +5 ≡ 0(mod10).
k=0 k=0

I
tisaga
inacon
trad
ict
ion! The
re e,t
for rof “
henumbe bad
ce
lls”i
nAi
snomor
ethan25.
Ont
heo
t rhand,wecancons
he tr ta3 × 9a
uc rrayi
nthe
fo
llowi
ngandche
ckt
hateachc
elli
nitt
hatdoe
sno
tcon
tai
n
s“
r10i
numbe bad”.
56 Ma
thema
tic
alOl
ymp
iadi
nCh
ina

1 1 1 2 1 1 1 1 10

1 1 1 1 1 1 1 1 1

1 1 1 10 1 1 1 1 2

The
ref e,wef
or indou
ttha
tthemax rof “
imum numbe bad
c
els”i
l s25.
nAi
Mathematical Olympiad in China (2011–2014) Downloaded from www.worldscientific.com
by NATIONAL UNIVERSITY OF SINGAPORE on 05/06/18. For personal use only.

2012 (
Shaanxi)

1 (
40ma
rks)Ass
eeni
nFi
g .1,i
.1 nacu
tet
riang
le△ABC ,
AB > AC ,M ,N a
retwod
iff
eren
tpo
insonBCs
t ucht
hat
∠BAM = ∠CAN ,andO1 ,O2 a
ret
hec
ircumcen
ter
sof
△ABC , △AMN ,r
espe
cti
vel ove O1 ,O2 , A a
y.Pr re
co
lli
nea
r.

F
ig.1
.1 F
ig.1
.2

So
lut
ion.AsshowninFi
g .2,weconne
.1 ctAO1 ,AO2 ,and
t
hroughA dr
awal
inepe
rpend
icu
la oAO1 andi
rt nte
rsec
twi
th
t
heex
tendedl
ineofBCa
tpo
intP .ThenAPi
sat
angen
tli
neof
☉O1 ,wh
i s ∠B = ∠PAC .
ch mean
As ∠BAM = ∠CAN ,wehave

∠AMP = ∠B + ∠BAM = ∠PAC + ∠CAN = ∠PAN .

ThenAPi
sal
soat
angen
tli f☉O2 t
neo hec
ircumc
irc
leof
Ch
inaMa
thema
tic
alCompe
tit
ion (
Comp
lemen
tar
yTe
st) 57

△AMN .The
ref e,AP ⊥AO2 ,
or andt
hatmeansO1 ,O2 ,A a
re
co
lli
nea
r.Theproofi
scomp
let
e.

2 (
40ma
rks)Le
tA = {
2,22 ,...,2n ,...}.Prove:
1)Foranya ∈ A ,b ∈ N* ,
( i a -1,
fb < 2 thenb(
b +1)
wi
llno
tbeamu
lti
pleof2
a.
Mathematical Olympiad in China (2011–2014) Downloaded from www.worldscientific.com

2)Foranya ∈ A (= N* - A )s
( ati
sfi
ynga ≠ 1,t
her
e
by NATIONAL UNIVERSITY OF SINGAPORE on 05/06/18. For personal use only.

ex
issb ∈ N* s
t ati
sfi
y a -1,
ngb < 2 sucht
hatb(
b +1)
isa
mu
lti
pleof2
a.
So
lut
ion.(
1)Foranya ∈ A ,a = 2 (
k
k ∈ N* ).Then2a =
2k+1 .Le
tbbeanypo
sit
ivei
ntege
rst
ric
tlyl
es han2
st a -1.Then
(b +1)≤ 2a -1.
tweenbandb +1,onei
Be sanoddnumbe
rtha
tcon
tai
nsno
pr
imefac
tor2,andtheo
the
risanevennumbe
rt ha
tcontai
nsat
mos hekt
tt hpowe f2.The
ro r
ef e,
or b +1)
b( isdef
ini
tel
yno ta
mu
lti
pleof2
a.
2)Fora ∈ Aanda ≠1,
( suppo
sea =2km whe
reki
sanon
-
nega
tivei
nt randm i
ege sanoddnumbe
rgr
eat
ert
han1.Then
2
a = 2k+1m . We wi
ll pr
esen
tthr
ee d
iff
eren
t proof
sint
he
fo
llowi
ng.
Proo tb = mx,
f1.Le b +1 =2k+1y.El
imi
nat
ingb,wehave
2k+1y - mx = 1.S
i e(
nc 2k+1 ,m )= 1,
theequa
tionha
sin
teg
ral
s
olu
tionst
hatcanbeexpr
ess
eda
s
x = x0 +2k+1t,
( et ∈ Z,and (
x0 ,y0)i
{
y = y0 + mt
whe
r sas
pec
ial

s
olu
tiono
ftheequa
tion).
Deno
tet
hesma
lle
sts
olu
tion amongt s(
hem a x* ,y* ).
Thenx* < 2k+1 .
The
ref
ore, a -1andb(
b = mx* <2 b +1)
isamu
lti
pl f2
eo a.
Proo
f2.S
ince(
2k+1 ,m ) = 1,byt
heCh
ine
se Rema
inde
r
58 Ma
thema
tic
alOl
ymp
iadi
nCh
ina

em,t
Theor hecong
ruenc
eequa
tion

x ≡ 0(
mod2k+1),
{
x ≡ m -1(
modm )

ha
sas
olu
tionx =b wi
thb ∈ (
0,2k+1m ).
Iti
sea
syt
oseet
hatb
a -1andb(
<2 b +1)
isamu
lti
pleof2
a.
Proo
f3.S
inc 2k+1 ,m )=1,
e( thent
her
eex
issr ∈ N* ,
t r≤
Mathematical Olympiad in China (2011–2014) Downloaded from www.worldscientific.com

m -1,s modm ).
t2r ≡ 1(
by NATIONAL UNIVERSITY OF SINGAPORE on 05/06/18. For personal use only.

ucht
ha
Taket ∈ N* s
ucht
ha r >k +1.Then2tr ≡ 1(
tt modm ).
It
i
sea
syt
oseet
hatt
her
eex
ist
s

b=(
2tr -1)-q·2k+1m > 0(
q ∈ N),

s
ucht
ha a - 1.Then wehavem |b,2k+1 |b + 1.
t0 < b < 2
The
re e,
for b +1)
b( isamu
lti
pleof2a.

3 (
50ma
rks)Le
tP0 ,P1 ,P2 ,...,Pn ben +1po
int
sona
plane,andtheminimumd i
stancebe
tweeneachtwopo
int
s
oft hemisd(d > 0).Prove

æ d ön (
|P0P1 |·|P0P2 |·…·|P0Pn |> ç ÷ n +1)! .
è3 ø

So
lut
ion1.Wemaya
ssumet
hat|P0P1|≤|P0P2|≤ … ≤|P0Pn |.
d
Atf
irt,wewi
s llprovet
hat|P0Pk |> k +1forany
3
po
sit
ivei
nt rn.
ege
d
Obv
iou
sly,|P0Pk|≥d ≥ k +1fork =1,2,...,8,
3
andt
hes
econdequa
lit
yho
ldson
ly whenk = 8.Then weon
ly
d
needt
oprovet
hat|P0Pk |≥ d ≥ k +1fork ≥ 9.
3
TakeeachPi(
i = 0,1,2,...,k)a
sthecen
tert
odr
awa
d
c
irc
lewi
thr
adi
us .Thent
hes
eci
rcl
esa
ree
ithe
rex
terna
lly
2
Ch
inaMa
thema
tic
alCompe
tit
ion (
Comp
lemen
tar
yTe
st) 59

t
angen
ttoorapa
rtf
rom eacho
t r.TakeP0 a
he sthec
ent
ert
o
d
dr
awac
irc
lewi
thr
adi
us|P0Pk |+ .Thent
hepr
evi
ousk +1
2
sma
lle
rci
rcl
esa
rea
lll
oca
tedi
nth
isl
arge
rone.
d ö2 æ d ö2
Thenπç|P0Pk |+ ÷ > (
k +1)
πç ÷ ,f
æ
r
om wh
ich we
è 2ø è2 ø
d(
have|P0Pk |> k +1 -1).
Mathematical Olympiad in China (2011–2014) Downloaded from www.worldscientific.com

2
by NATIONAL UNIVERSITY OF SINGAPORE on 05/06/18. For personal use only.

k +1 -1 k +1
I
tisea
syt
ocheckt
hat > fork ≥ 9.
2 3
d
Then|P0Pk |> k +1fork ≥ 9.
3
d
Ove
ral,wehave|P0Pk |>
l k +1fork ≥ 9.
3
The
re e,
for

æç d ö÷n (
|P0P1 |·|P0P2 |·…·|P0Pn |> n +1)! .
è3 ø

So
lut
ion2.Wemaya
ssume|P0P1|≤|P0P2|≤ … ≤|P0Pn|.
TakeeachPi(
i = 0,1,2,...,k)a
sthecen
tert
odr
awa
d
c
irc
lewi
thr
adi
us .Thent
hes
eci
rcl
esa
ree
ithe
rex
terna
lly
2
t
angen
ttoorapa
rtf
romeacho
the
r.
tQ beanypo
Le inton☉Pi .S
inc
e

d
|P0Q | ≤|P0Pi |+|PiQ |=|P0Pi |+
2
1 3
≤|P0Pk |+ |P0Pk |= |P0Pk |,
2 2

3
weg
ett
hatt
hec
irc
lewi
thc
ent
erP0andr
adi
us |P0Pk|c
ove
rthe
2
æ3 ö2
r
pev
i sk + 1sma
ou lle
rci
rcl
e eπç |P0Pk |÷ >
s.Then wehav
è2 ø

( æ d ö2
k +1)
πç ÷ ,andt
hati
s
è2 ø
60 Ma
thema
tic
alOl
ymp
iadi
nCh
ina

d
|P0Pk |> k +1(
i = 0,1,2,...,k).
3

The
re e,
for

æ d ön (
|P0P1 |·|P0P2 |·…·|P0Pn |> ç ÷ n +1)! .
è3 ø
Mathematical Olympiad in China (2011–2014) Downloaded from www.worldscientific.com

1 1
4 (
50ma s)Le
tSn =1+ + … + ,wheeni
by NATIONAL UNIVERSITY OF SINGAPORE on 05/06/18. For personal use only.

rk r sapo
sit
ive
2 n
i
ntege
r.Provet
hatforanyr
ealnumbe
rsa,
b wi
th0 ≤a <
b ≤1,
the
rea
rei
nfi
nit
emanyt
ermsi
nthes e{
equenc Sn -
[
Sn ]}t
hata
re wi
thn(
i a,b). (He
re[
x]deno
test
he
l
arge
sti
ntege
rno
tgr
eat
ert
hanr
ea rx.)
lnumbe
So
lut
ion1.Foranyn ∈ N ,weh
*
ave

1 1 … 1 1 æ 1 1 ö÷
S2n = 1 + + + + n =1+ +ç 1 + +
2 3 2 2 è2 +1 22 ø
æç 1 1ö
+… + n÷
è2n-1 +1 2 ø
1 æ1 1 ö æ1 1ö
>1+ + ç 2 + 2 ÷+ … + ç n + … + n ÷
2 è2 2 ø è2 2 ø
1 1 … 1 1
=1+ + + + > n.
2 2 2 2

tN0 =
Le [b 1-a ] +1,m = [SN0 ]+1.Then 1 < N0 ,
b -a
1
<b -a,andSN0 < m ≤ m +a.
N0
tN1 = 22 m+1 .ThenSN1 = S22(m+1) > m +1 ≥ m +b.
( )
Le
Wec
laimt
hatt
her
eex
istn ∈ N* wi
thN0 <n < N1s
ucht
hat
m +a < Sn < m +b (
or,i
not
herwo
rds,Sn - [
Sn ]∈ (
a,b)).
Ot
herwi
se,a
ssumi
ngt
hec
laim i
sfa
lse,t
hent
her
e mu
st
ex
istk > N0s
ucht
hatSk-1 ≤ m +aandSk ≥ m +b.
ThenSk - Sk-1 ≥ b - a.Bu
titcon
trad
ict
sthef
actt
hat
Ch
inaMa
thema
tic
alCompe
tit
ion (
Comp
lemen
tar
yTe
st) 61

1 1
Sk -Sk-1 = < <b -a.Ther
ef e,t
or hec
laimi
str
ue.
k N0
Fur
the e,a
rmor ssumet
her
ear
e on
ly af
ini
te numbe
r of
po
sit
ivei
ntege
rsn1 ,...,nk s
ati
sfi
yng

Snj - [Snj ]∈ (
a,b)(
1 ≤j ≤k).

De
fi n{
nec = mi Snj - [Snj ]}.Thent
her
eex
issnon ∈ N*
t
Mathematical Olympiad in China (2011–2014) Downloaded from www.worldscientific.com

1≤j≤k

tSn - [
Sn ]∈ (
a,c).
by NATIONAL UNIVERSITY OF SINGAPORE on 05/06/18. For personal use only.

s
ucht
ha Itc
ont
rad
ict
stheabov
ecl
aim.
The
re e,t
for her
ear
einf
ini
tet
ermsi
nthes e{
equenc Sn -
[
Sn ]}t
hata
rewi
thn(
i a,b).
Theproofi
scomp
let
e.
So
lut
ion2.Foranyn ∈ N ,weh
*
ave

1 1 … 1 1 æ 1 1 ö÷
S2n = 1 + + + + n =1+ +ç 1 + +
2 3 2 2 è2 +1 22 ø
æç 1 1ö
+… + n÷
è2n-1 +1 2 ø
1 æ1 1 ö æ1 1ö
>1+ + ç 2 + 2 ÷+ … + ç n + … + n ÷
2 è2 2 ø è2 2 ø
1 1 … 1 1
=1+ + + + > n.
2 2 2 2

The
ref e,Sn canbel
or arge
rthananypo
sit
ivenumbe
ras
l snbe
onga come
ssu
ffi
cien
tlyl
arge.

tN0 =
Le [b 1-a ] +1.ThenN1 <b -a,andwhenk > N ,
0
0

wehave

1 1
Sk -Sk-1 = < <b -a.
k N0

Soforanypo
sit
ivei
nt rm > SN0 ,t
ege her
eex
issn > N0
t
s
ucht
hatSn - m ∈ (
a,b),or,i
not
he s,m +a < Sn <
rword
m +b.Ot
herwi
se,t
her
emu
stbek > N0s
ucht
hatSk-1 ≤ m +a
andSk ≥ m +b,
i.e.,Sk -Sk-1 ≥b -a.Bu
titcon
trad
ict
sthe
62 Ma
thema
tic
alOl
ymp
iadi
nCh
ina

1 1
f
actt
hatSk -Sk-1 = < <b -a.
k N0
Nowl
e SN0 ]+i(
tmi = [ i = 1,2,3,...).Thent
her
e
ex
issni > N0s
t ucht
hatmi +a < Sni < mi +b,i.
e.,Sni -
[
Sni ]∈ (
a,b).
The
re e,t
for her
ear
einf
ini
tet
ermsi
nthes e{
equenc Sn -
Mathematical Olympiad in China (2011–2014) Downloaded from www.worldscientific.com

Sn ]}t
[ hata
rewi
thn(
i a,b).
by NATIONAL UNIVERSITY OF SINGAPORE on 05/06/18. For personal use only.

2013 (
Jil
in)

1 (
40ma
rks)Ass
eeni
nFi
g .1,ABi
.1 sachordo
fci
rceω,
l
Pi
sa po
intona
rcAB ,andE ,F a
re2 po
inson AB
t
s
ati
sfi
y tPE ,PF andex
ngAE = EF = FB .Connec tend
t
hemt
oin
ter
sec
twi
t tC ,D ,r
hωa espec
tive
ly.Prove

EF ·CD = AC ·BD .

F
ig.1
.1 F
ig.1
.2

So
lut
ion.Asshownin Fi
g .2,weconnec
.1 tAD ,BC ,CF ,
DE .S
i eAE = EF = FB ,wehave
nc
Ch
inaMa
thema
tic
alCompe
tit
ion (
Comp
lemen
tar
yTe
st) 63

BC ·s
in∠BCE d
ist
anc tweenB andCP
ebe BE
= = = 2.
AC ·s
in∠ACE d
ist tweenA andCP AE
ancebe

Int ameway,
hes

AD ·s
in∠ADF d
ist
anc tweenA andPD
ebe AF
= = = 2.
BD ·s
in∠BDF d
ist tweenB andPD
ancebe BF
Mathematical Olympiad in China (2011–2014) Downloaded from www.worldscientific.com


by NATIONAL UNIVERSITY OF SINGAPORE on 05/06/18. For personal use only.

Ont
heo
t rhand,s
he ince

∠BCE = ∠BCP = ∠BDP = ∠BDF ,

∠ACE = ∠ACP = ∠ADP = ∠ADF ,

BC ·AD
mu
lti
pli
yng ① by ② ,wehave = 4,or
AC ·BD

BC ·AD = 4AC ·BD . ③

ByPt
olemy
􀆳 em,wehave
sTheor

AD ·BC = AC ·BD + AB ·CD . ④

Comb
inng ③ and ④ ,wege
i tAB ·CD = 3AC ·BD ,and
t
hati
s

EF ·CD = AC ·BD .

Theproofi
scomp
let
e.

2 (
40ma
rks)Gi
venpo
sit
ivei
ntege
rsu,v,
thes e{
equenc an }
i
sde
fi s:
neda a1 = u +v,andform ≥ 1,

a2m = am +u,
{
a2m+1 = am +v.

Deno
teSm =a1 +a2 + … +am (
m =1,2,...).Prove
t
hatt
her
ear
einf
ini
tet
ermsi
nsequence {
Sn }t
hata
re
s
qua
renumbe
rs.
64 Ma
thema
tic
alOl
ymp
iadi
nCh
ina

So
lut
ion.Forpos
iti
vei
nt rn,wehave
ege

S2n+1-1 = a1 + (
a2 +a3)+ (
a4 +a5)+ … + (
a2n+1-2 +a2n+1-1)
= u +v + (
a1 +u +a1 +v)+ (
a2 +u +a2 +v)+ … +
a2n-1 +u +a2n-1 +v)
(
=2 (u +v)+2S2n-1 .
n
Mathematical Olympiad in China (2011–2014) Downloaded from www.worldscientific.com

Then
by NATIONAL UNIVERSITY OF SINGAPORE on 05/06/18. For personal use only.

S2n-1 = 2n-1(
u +v)+2S2n-1-1
=2- ( u +v)+2( u +v)+2S2n-2-1)
2n-2(
n 1

= 2·2 - (u +v)+22S2n-2-1
n 1

= … =(
n -1)·2n-1(
u +v)+2n-1(
u +v)
u +v)n·2n-1 .
=(

Suppo
seu +v = 2k ·q,whe
reki
sanon
-nega
tivei
nt r,
ege
andqisanoddnumber.Taken =q·l2 ,wherelisanypositive
ngl ≡k -1(
mod2).ThenS2 -1 =ql ·2 ,
2
2 2 k-1+q·
in
tege
rsat
isf
yi n l

and

k -1 +q·l2 ≡k -1 +l2 ≡k -1 + (
k -1)
2

=k(
k -1)≡ 0(
mod2).

The
re e,S2n-1 i
for sas
qua
re numbe
r.S
ince t
her
ear
e
i
nfi
niel
t 􀆳s,t
her
ear
einf
ini
tet
e n{
rmsi Sn }t
hata
res
qua
re
numbe
rs.Theproo
fiscomp
let
e.

3 (50 marks) Suppo


sethe
re are m ques
tions i
n an
exami
nat
ionat
tendedbyns
tuden
ts,wheem ,n ≥ 2a
r re
given na
tur
al numbe
rs. The ma
rki
ng r ul
e for each
quest
ionisasfolows:i
l ft her
eare exac
tlyx st
udents
f
ail
ingt
oanswe
rthe que
sti
oncor
rec
tly,t
hent
hey wi
ll
t0ma
eachge rks,andt
hos
ewhoan
swe
ritcor
rec
tly wi
ll
tx ma
eachge rks.Thet
ota
lma
rksofas
tuden
tar
ethe
Ch
inaMa
thema
tic
alCompe
tit
ion (
Comp
lemen
tar
yTe
st) 65

s
um of ma
rkshe/she ge
tsfrom the m que
sti
ons.Now
rankt
heto
talmark
soft henstudent
sasp1 ≥ p2 ≥ … ≥
pn .Fi
ndt
hemax
imum po
ssi
bleva
lueo
fp1 +pn .
So
lut
ion.Foranyk = 1,2,...,m ,a
ssumi
ngt
her
eaexk
r
s
tuden
tsf
ail
ingt
oanswe
rt hque
hekt sti
onco
rre
cty,t
l hent
her
e
a
ren -xk one
swhoanswe
ritco
rre
ctl
yande
achge
tsxk ma
rks
Mathematical Olympiad in China (2011–2014) Downloaded from www.worldscientific.com

f
romi
tac
cord
ing
ly.Suppos
ethesum o
fthen s
tuden
ts􀆳t
ota
l
by NATIONAL UNIVERSITY OF SINGAPORE on 05/06/18. For personal use only.

ma
rksi
sS.Thenwehave
n m m m

∑ ∑xk (n -xk )=n∑xk - ∑xk .


2
pi = S =
i=1 k=1 k=1 k=1

As e
ach s
tuden
t ge
tsa
t mos
t xk ma
rks f
rom t
he kt
h
que
ston,wehave
i
m

p1 ≤ ∑x
k=1
k .

p2 +p3 + … +pn S -p1


S
i ep2 ≥ … ≥ pn ,t
nc henpn ≤ = .
n -1 n -1
The
ref
ore,

S -p1 n -2 S
p1 +pn ≤ p1 + = p1 +
n -1 n -1 n -1
m m m
n -2· 1 ·

n -1 k∑
=1
xk +
n -1
(nk∑ xk - k∑ xk2 )
=1 =1

m m
1 ·
=2 ∑x ∑
2
- xk .
n -1 k
k
k=1 =1

Byt
heCauchyI
nequa
liy,wehave
t
m m
1
∑x ( ∑ xk ) .
2 2
k ≥
k=1 m k =1

Then
66 Ma
thema
tic
alOl
ymp
iadi
nCh
ina

m m
1
∑ xk - · ( ∑ xk )
2
p1 +pn ≤ 2
k=1 m(
n -1) k =1

m
1 · ( ∑ xk - m (
n -1)) + m (
n -1)
2
=-
m(
n -1) k =1

≤ m(
n -1).

Ont
heo
t rhand,i
he fthe
rei
sas
tuden
twhoanswe
rsa
llt
he
Mathematical Olympiad in China (2011–2014) Downloaded from www.worldscientific.com

que
sti
onsc
orr
ect
ly,wh
ilet
heo
the
rn -1s
tuden
tsf
ailt
oanswe
r
by NATIONAL UNIVERSITY OF SINGAPORE on 05/06/18. For personal use only.

anyque
stons,t
i henwehave
m

p1 +pn = p1 = ∑ (n -1)= m (n -1).


k=1

The
ref
ore,t
he max
imum pos
sib
leva
lueo
fp1 + pn i
sm
n -1).
(

4 ( rks)Le
50ma tn,kbei
ntege
rsg
rea
tert
han1ands
ati
sfyn
< 2 .P
k
rovet
hatt
her
ear
e2ki
ntege
rsno
tdi
vis
ibebyn,
l
sucht
hati
f wed
ivi
det
hem i
ntotwo g
roups,t
hent
her
e
mus
tex
istag
roupi
nwh
icht
hesumo
fsomei
ntege
rsc
anbe
d
ivi
dedbyn.
So
lut
ion.Atf
irt,wecon
s s
ide
rtheca
set
hatn = 2 ,r ≥ 1.
r

Obv
iou
sly,a
tth
i imer <k.Wet
st aket
hree“
2r-1”sand2
k -3

1”s — each oft
hem canno
tbe d
ivded byn.I
i fthe
s k
e2
numbe
rsa
red
ivi
dedi
ntotwog
r s,
oup t
hent
her
emu
ste
xis
tag
roup
t
hatc
ont
aistwo“
n 2r-1”
s,who
ses s2r — d
umi ivi
sib
lebyn.
t,wecon
Nex side
rtheca
set
hatni
sno
tapowe
rof2.At
t
hi ime,t
st he2
kin
tege
rswet
akea
re

-1,-1,-2,-2 ,...,-2k-2 ,1,2,22 ,...,2k-1 .


2

Thent
heyeachcanno
tbed
ivdedbyn.
i
As
sumet
hes
enumbe
rscanbed
ivi
dedi
nt roups,s
otwog uch
t
hatanypa
rti
als
umofnumbe
rsi
noneg
roupi
sno
tdi
vis
ibl
eby
Ch
inaMa
thema
tic
alCompe
tit
ion (
Comp
lemen
tar
yTe
st) 67

n.Wemays
ayt
hat“
1”i
sint
hef
irs
tgroup.S
ince(-1)+1 =
0i
sdi
vis
ibebyn,
l thetwo “-1”
smu
stbei
nthes roup;
econdg
s
i e(-1)+ (-1)+2 = 0,
nc the “
2”i
sint
hef
irs
tgroup;t
hen
he “-2”i
t sint
hes
econdg
roup.
Nowbyi
nduc
tion,a
ss ng1,2,...,2l a
umi rei
nthef
irs
t
groupand -1,-1,-2,...,-2li
nt econdone(
hes 1 ≤l <
Mathematical Olympiad in China (2011–2014) Downloaded from www.worldscientific.com

k -2),si
nce
by NATIONAL UNIVERSITY OF SINGAPORE on 05/06/18. For personal use only.

(-1)+ (-1)+ (-2)+ … + (-2l )+2l+1 = 0

i
sdi
vis
ibebyn,wege
l tt t2l+1i
ha sint
hef
irs
tgroup,andt
hen -
2l+1i
nthes
econd.
The
ref
ore,1,2,22 ,...,2k-2i
sint
hef
irs
tgoupand -1,
r
-1,-2,-2 ,...,-2 - i y,s
2 k 2
nthes
econd.F
ina
ll inc
e

(-1)+ (-1)+ (-2)+ … + (-2k-2)+2k-1 = 0,

hen2k-1i
t sint
hef
irs
tgroup.The
re e,1,2,22 ,...,2k-1a
for re
a
lli
nthef
irs
tgr
oup.
Ont
heo
t rhand,t
he heknowl
edgeabou
ttheb
ina
rynumbe
r
s
yst
emt e
llsu
stha
teverypos
iti
veint
egerwh i
chi
snotgr
eat
er
han2 - 1can ber
t k
epre
sen
ted a
st he pa
rti
a um of1,2,
ls
22 ,...,2k-1 .S
incen ≤2k -1,
theni
tist
hepa
rti
a umof1,2,
ls
22 ,...,2k-1 t
hati
sofcour
sed
ivi
sib
lebyn i
tse
lf.Th
isi
sa
con
trad
ict
iont
othea
ssump
tion.
The
ref e,wehavefound ou
or t2ki
ntege
rst
hat mee
tthe
r
equ
iremen
tint
heque
sti
on.Theproo
fist
hencomp
let
e.
Ch
inaMathemati
cal
Ol
ymp iad
Mathematical Olympiad in China (2011–2014) Downloaded from www.worldscientific.com
by NATIONAL UNIVERSITY OF SINGAPORE on 05/06/18. For personal use only.

The Ch
ina Ma
thema
tica
lOl
ymp
iad, o
rgan
ized by t
he Ch
ina
Ma
thema
tica
lOl
ymp
iadCommi
ttee,i
she
ldi
nJanua
ryeve
ryyea
r.
Abou
t150wi
nne
rso
ftheCh
inaMa
thema
tica
lCompe
tit
iont
akepa
rt
i
nit.The compe
tit
ionl
ast
sfrtwo days,andt
o her
ear
eth
ree
r
pob
lemst
obecomp
let
edwi
thi
n4.5hou
rseachday.

2011 (
Changchun,J
iln)
i

F
irs
tDay
8:
00~12:
30Janua
ry15,2011

1 ta1 ,a2 ,...,an (


Le n ≥ 3)ber
ealnumbe
rs.Pr
ovet
hat
Ch
inaMa
thema
tic
alOl
ymp
iad 69

n n

∑a
i=1
2
i - ∑ aa
i=1
i i+1 ≤ [ n2 ] (M -m ), 2

whe
rean+1 =a1 ,M = max1≤i≤nai ,m = mi
n1≤i≤nai .[
x]i
sthe
l
arge
sti
ntege
rno
texc
eed
ingx.
So
lut
ion.I
fn k (ki
=2 sapo
sit
ivei
nt r),t
ege hen
n n n

2( ∑ ai2 - ∑ aiai+1 ) = ∑ (a -ai+1) ≤ n(


2
M - m ), 2
Mathematical Olympiad in China (2011–2014) Downloaded from www.worldscientific.com

i
by NATIONAL UNIVERSITY OF SINGAPORE on 05/06/18. For personal use only.

i=1 i=1 i=1

t
her
ef e,
or
n n
n( n (
∑ ai - ∑ aiai+1 ≤ M -m ) M -m )
2 2 2
= . [ ]
i=1 i=1 2 2

I k + 1(
fn = 2 kisa po
sit
ivei
nt r),t
ege k +1
henfor2
numbe
rsa
rrangedi
nacyc
lic way,onecana
lway
sfi
ndt
hree
rms ( ∏ (
2k+1
cons
ecu
tivei
ncr
eas
ing or dec
rea
sing t
e as i=1
ai -

ai-1)(
ai+1 -ai)= ∏ ai -ai-1)
( ≥ 0,
2k+1 2
i=1
soi
tisno
tpo
ssi
ble
t
hatforeve
ryi,ai -ai-1 andai+1 -ai hav
ingopposi
tesigns).
Wit
houtlossof gene
ral
ity, we a ssume thata1 ,a2 ,a3 are
mono
ton
ic,t
hen

(
a1 -a2)
2
+(
a2 -a3)
2
≤(
a1 -a3)
2
.

e,
Henc
n n n

2( ∑ 2
a -
i ∑ aa ) ∑ (ai i+1 = i -ai+1)
2

i=1 i=1 i=1

a1 -a3)
≤( ∑ (a -ai+1),
2 2
+ i
i=3

wh
icht
rans
formedt
heque
sti
oni
ntot
heca
seof2
knumbe
rs.We
have
n n n

2( ∑ ai2 - ∑ aiai+1 ) ≤ (
a1 -a3)
2
+ ∑(
ai -ai+1)
2

i=1 i=1 i=3

k(
≤2 M - m ), 2
70 Ma
thema
tic
alOl
ymp
iadi
nCh
ina

e.,
i.
n n

( ∑a
i=1
2
i - ∑ aa ) ≤k(M
i=1
i i+1 -m ) =
2
[ n2 ] (M -m ).
2

2 Asshowni
nFi
g .1,D i
.2 sthe
Mathematical Olympiad in China (2011–2014) Downloaded from www.worldscientific.com

mi
dpo
into
fac BC o
r ft
he
by NATIONAL UNIVERSITY OF SINGAPORE on 05/06/18. For personal use only.

c
ircumc
irc
leo
ftr
iang
leABC ,
Xl
ies on a
rc BD ,E i
sthe
mi
dpo
intofa
rcAX ,Sl
ieson
a
rcAC ,SD i
nte
rse
ctsBC a
t
R ,SE i
nte
rse
cts AX a
t T.
Provet
ha fRT ‖DE ,t
ti hen F
ig.2
.1

t
hei
ncen
tero
ftr
iang
le ABC
l
iesonl
ineRT .
Proo tAD ,deno
f.Connec tet
hei
nte
rse
ctonofAD andRT by
i
I,
thenAIi
stheb
isec
toro tAS,SI,t
f ∠BAC .Connec henby
RT ‖DE ,wehave

∠STI = ∠SED = ∠SAI,

oA ,T ,
s IandSa
reconcyc
lic,andwedeno
tet
hisc
irc
lebyω1 .
Conne
ctCE ,deno
tet
hei
nte
rse
ctonofCE andRT byJ,
i
conne
ctSC ,
then

∠SRJ = ∠SDE = ∠SCE ,

oS,J,RandCa
s reconcyc
lic,andwedeno
tet
hisc
irc
lebyω2 .
Deno
tebyK t
hei
nte
rsec
tionpo
intofω1 andω2 o
the
rthan
S,weprovenex
tt tK i
ha sthei
nte
rsec
ti fAJ andCI.
ono
Deno
teby K1 t
hei
nte
rsec
tion ofω1 and AJ o
the
rthan
A,
then
Ch
inaMa
thema
tic
alOl
ymp
iad 71

1( 1
∠SK1A = ∠STA = SA + XE )= (SA + AE )
2 2
= ∠SDE = ∠SRT = ∠SRJ,

oS,K1 ,J and R a
s reconcyc
lic,i.e.,K1 be
long
stoω2 .
S
imi
lar
ly,deno
tebyK2t
hei
nte
rse
ctonofω2andCIo
i the
rthan
C ,t
henK2 be
long
st e,K1andK2co
oω1 .Henc inc
ide,andK i
s
Mathematical Olympiad in China (2011–2014) Downloaded from www.worldscientific.com

t
hei
nte
rsec
tionofAJ andCI.
by NATIONAL UNIVERSITY OF SINGAPORE on 05/06/18. For personal use only.

As ∠CAD = ∠CAI,and ∠TJE = ∠CJR = ∠CED =


∠CAD ,oA ,I,J andC a
s reconcyc
lic,t
her
ef e,∠ACI =
or
∠AJI.

Ont
heo
t rhand,byt
he heconcyc
lic
ityofC ,K ,J andR ,
wehave ∠BCI = ∠ICR = ∠AJI,and ∠ACI = ∠BCI,
soIi
s
t
hei
ncen
tero
fthet
riang
leABC .

3 tA1 ,A2 ,...,An bennon


Le -emp
tys
ubs
etso
faf
ini
tes
et
A ofr
ealnumbe
rss
ati
sfi
yngt
hefo
llowi
ngcond
iti
ons:
(
1)Thes
umofe
lemen
t fAi
so sequa
lto0;
(
2)P
icka
rbi
tra
ril
yanumbe
rfromeachAi ,andt
hei
rsum
i
sst
ric
tlypo
sit
ive.
Provet
hatt
her
eex
ists
etsAi1 ,Ai2 ,...,Aik ,1 ≤i1 <i2
< … <ik ≤ n,
sucht
hat

k
|Ai1 ∪ Ai2 ∪ … ∪ Aik |< |A |.
n

|X |deno
test
henumbe
rofe
lemen
tso
faf
ini
tes
etX .
So
lut
ion.Le
tA a1 ,...,am }wi
={ tha1 > … >am .By (
1)we
havea1 + … +am = 0.Cons
ide
rthesma
lle
ste
lemen
tofeach
Ai ,t
hes
um o
fthe
senumbe
rsi
sgr
eat
ert
han0.As
sumet
hat
t
her
ear
eexac
tlyki s
etsamong A1 , ...,An who
se mi
nima
l
e
lemen
ti i = 1,2,...,m .Then,oneha
sai , s
72 Ma
thema
tic
alOl
ymp
iadi
nCh
ina

k1 + … +km = n.

By (
2),wehave

k1a1 + … +kmam > 0.

Fors =1,2,...,m -1,


the
rea
rei
nto
talk1 + … +kss
ets,
who
se mi
nima
lel
emen
tsa
reg
rea
tert
han or equa
lto as .
Mathematical Olympiad in China (2011–2014) Downloaded from www.worldscientific.com

The
ref e,t
or heun
ionoft
hes
ese
tsi
scon
tai n{
nedi a1 ,...,as },
by NATIONAL UNIVERSITY OF SINGAPORE on 05/06/18. For personal use only.

whencet
henumbe
rofe
lemen
tsdoe texceeds.
sno
t,wepr
Nex ovet
hatt
her
eex
isss ∈ {
t 1,2,...,m - 1}
s
n
s
ucht
hatk = k1 + … + ks > .We provet
hisc
laim by
m
con
trad
ict
ion.Suppo
set
hat

s
n
k1 + … +ks ≤ ,s = 1,2,...,m -1.
m

Wi
tht
hehe
lpoft
heAbe
ltr
ans
formandt
hef
actt
hatas -
as+1 > 0,1 ≤s ≤ m -1,weknowt
hat
m

0< ∑ka
j=1
j j

m-1

= ∑ (a
s=1
s -as+1)(
k1 + … +ks )+am (
k1 + … +km )

m-1
s
n
≤ ∑ (a
s=1
s -as+1)
m
+amn

m
n
=
mj∑
=1
aj = 0.

Wet
hen ge
tacon
trad
ict
i uchans,wet
on.Fors aket
hes
ets
amongA1 ,...,An ,who
se mi
nima
lel
emen
tsa
reg
rea
tert
han
as ,s
ay Ai1 ,Ai2 , ...,Aik .Then,byt
heabover
esu
lts,we
s
n
knowt
hatt
het
ota
lnumbe
rofs
uchs
et sk =k1 + … +ks > ,
si
m
andt
henumbe
rofe
lemen
tsoft
hei
run
iondoe texceeds,
sno
Ch
inaMa
thema
tic
alOl
ymp
iad 73

km k
e.,|Ai1 ∪ Ai2 ∪ … ∪ Aik |≤s <
i. = |A |.
n n

S
econdDay
8:
00~12:
30,Janua
ry16,2011
Mathematical Olympiad in China (2011–2014) Downloaded from www.worldscientific.com

4 Gi
venpo
sit
ivei
nt rn,
ege le 1,2,...,n}.Fi
tS = { ndt
he
by NATIONAL UNIVERSITY OF SINGAPORE on 05/06/18. For personal use only.

mi
nimum of|AΔS |+|BΔS |+|CΔS |fornonemp
ty
f
ini
tes
etsA andB o
frea
lnumbe
rs,whe
reC = {
a +b|a
∈ A,b ∈ B },XΔY = {
x|xbe
long
stoexac
tlyoneofX
andY},|X |deno
test
henumbe
rofe
lemen
tso
faf
ini
te
s
etX .
So
lut
i sn +1.
on.Theminimumi
Fi
rst,byt
akngA = B = S,wehave
i

|AΔS|+|BΔS|+|CΔS|= n +1.

cond,wecanpr
Se ovet
hatl =|AΔS|+|BΔS|+|CΔS|≥
Y ={
tX\
n +1.Le x|x ∈ X ,x ∉Y}.Wehave

l =|A\S|+|B\S|+|C\S|+|S\A |+|S\B |+|S\C |.

Al
lweneedt
opr
ovea
ret
hef
oll ng:
owi
(
i)|A\S|+|B\S|+|S\C |≥ 1,
(
ii)|C\S|+|S\A |+|S\B |≥ n.
r(
Fo i).I
nfa
ct,i
f|A\S|=|B\S|= 0,
thenA,B ⊆S.
So
1c
anno
tbeane
leme
n fC,
to hen
ce|S\C|≥1,
the
ref
ore(
i)i
sva
lid.
For (
ii).I
fA ∩ S = ⌀ ,t
hen|S\A |≥ n,t
hec
laimi
s
a
lreadyva
li fA ∩ S ≠ ⌀ ,wea
d.I ssumet
hatt
he max
ima
l
e
lemen
to sn -k,0 ≤k ≤ n -1,
fA ∩ Si then

|S\A |≥k. ①

Ont
heo
t rhand,fori =k +1,k +2,...,n,e
he it ri ∉
he
74 Ma
thema
tic
alOl
ymp
iadi
nCh
ina

B(
theni ∈ S\B )ori ∈ B (
thenn -k +i ∈ C ,
i.e.,
n -k +
i ∈ C\S ),hence

|C\S|+|S\B |≥ n -k. ②

From ① and ② ,weob


tan(
i i).
i
Inconc
lus
ion,(
i)and (
ii)a
reva
lid,s
ol ≥n +1.Hence,
t
hemi
n sn +1.
imumi
Mathematical Olympiad in China (2011–2014) Downloaded from www.worldscientific.com
by NATIONAL UNIVERSITY OF SINGAPORE on 05/06/18. For personal use only.

5 Gi
ven i
nt r n
ege ≥ 4. F
ind t
he max
imum of

∑ a(
ai +bi)
n

s a1 ,
i=1 i
for non
-nega
tive r
eal numbe
r
∑ b(
ai +bi)
n
i=1 i

a2 ,...,an ,b1 ,b2 ,...,bn s


ati
sfi
yng

a1 +a2 + … +an =b1 +b2 + … +bn > 0.

So
lut
ion.The max sn
imum i - 1.By homogene
ity,wecan
t∑i=1ai = ∑
n n
a
ssumewi
thou
tlo
ssofgene
ral
ityt
ha b = 1.
i=1 i

Fi
rst,i
tisc
lea
rtha
tifa1 = 1,a2 =a3 = … =an = 0and
1 ,
b1 =0,b2 =b3 = … =bn = hen ∑i=1ai(
ai +bi)=1,
n
t
n -1
1 ,
∑ b(
ai +bi)=
n
henc
e
i=1 i
n -1
n

∑a (a i i +bi )
= n -1.
i=1
n

∑b (a
i=1
i i +bi )

Now wepr
ovet
hatforanyr
ealnumbe
rsa1 ,a2 ,...,an ,
b1 ,b2 ,...,bn s ng ∑i=1ai = ∑ b = 1,wehave
n n
ati
sfi
y i=1 i

∑a (a i i +bi )
≤ n -1.
i=1
n

∑bi(ai +bi)
i=1
Ch
inaMa
thema
tic
alOl
ymp
iad 75

No
tet
hatt
hedenomi
nat
ori
spo
sit
ive,i
tisequ
iva
len
tto
s
howt
hat
n n

∑ ai(ai +bi)≤ (n -1)∑bi(ai +bi),


i=1 i=1

e.,
i.
Mathematical Olympiad in China (2011–2014) Downloaded from www.worldscientific.com

n n n

n -1)∑bi2 + (
( n -2)∑ aibi ≥ ∑a .
2
by NATIONAL UNIVERSITY OF SINGAPORE on 05/06/18. For personal use only.

i
i=1 i=1 i=1

Bysymme t
ry,wecana s
sumet
hatb1 i
sthesma
lle
stone
amongb1 ,b2 ,...,bn .Then
n n

n -1)∑bi2 + (
( n -2)∑ aibi
i=1 i=1

n n

≥(
n -1)1 +(
b2 n -1) ∑bi + (n -2)∑ aib1
2

i=2 i=1

≥(
n -1) ( ∑b ) +(
n -2)
2
b2
1 + i b1
i=2

=(
n -1)1 +(
b2 1 -b1) +(
n -2)
2
b1
=nb1 + (
2
n -4)
b1 +1
n n

≥1 = ∑ ai ≥ ∑a .
2
i
i=1 i=1

6 Pr
ovet
hatf
oranyg
iven po
sit
ivei
ntege
rsm ,n,t
her
e
ex
isti
nfi
nit
ely manypa
irsofcopr
imepo
sit
ivei
ntege
rsa,
b,s
ucht
hata +b|ama +bnb .
So
lut
i fmn =1,
on.I thent
hec
laimi
sva
lid.Formn ≥2,
since

na (
ama +bnb )= (
a +b)
na+b +a ((
mn)
a
-n + )
a b
,

i
tiss
uff
ici
entt
oprovet
heex
ist
enceofi
nfi
nit
ely manycopr
ime
numbe
rpa
irsa,b,
sucht
hat

a +b| (
mn)
a
-n + ,
a b
(
a +b,n)= 1.
76 Ma
thema
tic
alOl
ymp
iadi
nCh
ina

tp = a + b,we on
Le ly needt
o provet
hatt
her
ear
e
i
nfi
nit
ely manypr
imenumbe
rsp andpo
sit
ivei
nt r1 ≤ a ≤
ege
p -1s
ucht
hat

p| (
mn) -np .
a

ByFe
rma
t􀆳st em,i.
heor e.,whena1 ≡ a2(
modp -1),a1
Mathematical Olympiad in China (2011–2014) Downloaded from www.worldscientific.com

mn)1 ≡ (
≥ 1,a2 ≥ 1,( mn)2 (
modp).
a a
by NATIONAL UNIVERSITY OF SINGAPORE on 05/06/18. For personal use only.

So weon
ly needt
o pr
ovet
hatt
her
ear
einf
ini
tel
y many
pr
imenumbe
rsp andpo
sit
ivei
nt ras
ege ucht
hat

p| (
mn) -n. ①
a

I
ft he
rea re on
lyf
ini
tel
y many such primes,s ay p1 ,
p2 ,...,pr (
as mn ≥ 2,the ex
ist
ence of such pr
imesi s
obv
i s).Suppo
ou set
hat

(
mn)
2
-n = p11p22 .
α α
prr ,αi􀆳sa
.. α
renon
-nega
tivei
ntege
rs
(
1 ≤i≤ r). ②

1 p2 …pr (
ta = pα
Le p1 - 1)…(
pr - 1) + 2,and
1 α2 αr

s
uppo
set
hat

(
mn)
a
1 p2 …pr ,
-n = pβ1 β2 βr
βi􀆳sa
renon
-nega
tivei
ntege
rs

(
1 ≤i ≤r .)

fpi |n,t
I hen,by ③ anda ≥ 2,weknowt
hatpiβi |n,
epiβi | (
henc mn)
2
-n,andby ② wehaveβi ≤αi .
fpi n,t
I henpi m ,s
o(pii ,mn) = 1.By Eu
α +1
ler
􀆳s
em (
Theor asφ(
pii )= pii (
α +1 α
pi -1)
isaf
acorofa -2 )
t

(
mn)
a
-n ≡ (
mn)
2
-n(
modpiαi+1).

Becau
sepiαi+1 (
mn)
2
-n,t
hecong
ruencer
ela
tionabove
imp
lie
st tpiαi+1 (
ha mn)
a
-n.Soβi ≤αi .Hence,
Ch
inaMa
thema
tic
alOl
ymp
iad 77

(
mn)
a
1 p2 …pr
-n = pβ1 β2 βr
≤ p11p22 …prr = (
α α α
mn)
2
-n,

wh
ichi
sincon
trad
ict
ion wi
tha > 2.Sot
her
ear
einf
ini
tel
y
manypr
ime
sp andpo
sit
ivei
ntege
rsas
ucht
hatp| (
mn)
a
-n.
Mathematical Olympiad in China (2011–2014) Downloaded from www.worldscientific.com
by NATIONAL UNIVERSITY OF SINGAPORE on 05/06/18. For personal use only.

2012 (
Xian,Shaanxi)

F
irs
tDay
8:
00~12:
30Janua
ry15,2012

1 Asshowni
nFi
g .1,∠A i
.1 stheb
igge
stang
lei
ntr
iang
le
ABC .Ont
hec
ircumc
irc
leof△ABC ,t
hepo
insD andE
t
a
ret
hemi
dpo
int
sofABC andACB ,r
espec
tive
ly.Deno
te
by ☉O1thecir
clepa
ssi
ngt oughA andB ,
hr andtangen
tto
l
ineAC ,by ☉O2thecirc
lepa s
si hroughA andE ,and
ngt
t
angen
ttol
ineAD .☉O1i
nte
rse
cts☉O2 a
tpo
insA and
t
P .Provet
hatAPi
stheb
isec
t f ∠BAC .
oro
So
lut
ion.Jo
inr
espe
cti
vel
ythe
pa
irsof poi
ntsEP ,AE ,BE ,
BP , CD . For t
he s
ake of
conven
ience,we deno
te by A ,
B, C t
he ang
les ∠BAC ,

∠ABC , ∠ACB ,t
henA + B +
C = 180
°. Take an a
rbi
tra
ry F
ig.1
.1

po
intX ont
heex
tens
ionofCA ,
apoitY ont
n heexten
sionofDA .I
ti sea
sytos eet
hatAD =
DC ,AE = EB .By thef
actthat A ,B ,C ,D and E a
re
78 Ma
thema
tic
alOl
ymp
iadi
nCh
ina

concyc
lic,wege
t

1∠ C
∠BAE = 90
°- °- ,∠CAD
AEB = 90
2 2
1∠ B
=90
°- ADC = 90
°- .
2 2

Asl
ineAC and ☉O1 a
ret
angen
tatpo
intA ,l
ineAD and
Mathematical Olympiad in China (2011–2014) Downloaded from www.worldscientific.com

☉O2 a
ret
angen
tatpo
intA ,wege
t
by NATIONAL UNIVERSITY OF SINGAPORE on 05/06/18. For personal use only.

∠APB = ∠BAX = 1 °- A ,∠ABP = ∠CAP ,


80

and

∠APE = ∠EAY = 1
80°- ∠DAE = 1
80°- (∠BAE + ∠CAD -A)
C ö÷ æç Bö A
=1 °- ç90
80 °- - 9 0
°- ÷ + A = 90
°+ .
æ
è 2ø è 2ø 2

Bycompu
tat
ion,weob
tai
n

∠BPE = 3
A
60
°- ∠APB - ∠APE = 90
°+ = ∠APE .
2

In △APE and △BPE ,weapp


ly Law ofS
ine,andt
ake
i
ntoac
coun
tt tAE = BE ,weob
ha tai
n

s
in∠PAE PE PE s in∠PBE
= = = .
s
in APE AE
∠ BE s
in∠BPE

The
ref e,s
or in∠PAE = s
in∠PBE .Ont
heo
t rhand,
he
∠APE and ∠BPE a
rebo
thob
tue,s
s o ∠PAE and ∠PBE a
re
bo
thacu
te,t
hus ∠PAE = ∠PBE .
e,∠BAP = ∠BAE - ∠PAE = ∠ABE - ∠PBE =
Henc
∠ABP = ∠CAP .

2 Gi
venapr rp,
imenumbe letA beap ×p ma
tri
xsucht
hat
y1,2,...,p i
2
i
tsen
tri
esa
reexac
tl ns
omeorde
r.The
fo
llowi
ng ope
rat
ioni
sal
lowedfora ma
trx:addonet
i o
Ch
inaMa
thema
tic
alOl
ymp
iad 79

eachnumbe
rinar
ow oraco
lumn,ors
ubt
rac
tonef
rom
eachnumbe
rinaroworaco
lumn.Thema
trxAi
i sca
lled

good”i
fonecant
akeaf
ini
tes
eri
eso
fsuchope
rat
ions
r
esu
lti
ngi
nama
tri
xwi
tha
llen
tri
esze
ro.F
indt
henumbe
r
ofgoodma
tri
cesA .
So
lut
ion.Wemaycomb
inet
heope
rat
ionsont
hes
amerowor
Mathematical Olympiad in China (2011–2014) Downloaded from www.worldscientific.com

co
lumn,t
hust
hef
ina
lre
sul
tofas
eri
esofope
rat
ionscanbe
by NATIONAL UNIVERSITY OF SINGAPORE on 05/06/18. For personal use only.

r
eal
izeda
ssub
str
act
ingi
nt rxif
ege r rofi
omeachnumbe -throw
ands
ubs
trac
tingi
ntege
ryj f
rom eachnumbe -t
rofj hco
lumn.
Thu s,thematr
i xAi sgoodifandonl
yifthe
reexis
tin
tege
rsxi
yj ,sucht
hataij = xi +yj f
oral1 ≤i,j ≤ p.
l
S
incet
heen
tri
esofA a
red
ist
i t,x1 ,x2 , ...,xp a
nc re
pa
irwi
sedi
sti
nct,andsoarey1 ,y2 ,...,yp .Wemayconsi
der
on
lytheca
sethatx1 <x2 < … <xps i
nceswapp i
ngt
hevalueof
xi andxjr
esu
ltsi
nswapp
i hei
ngt -thr -t
owandj ow,wh
hr ichi
s
agai
nagood ma t
rix.Simila
rly,we mayconsi
de ronl
yt heca
se
t ty1 < y2 < … < yp ,
ha thusthematri
xisincreas
ingfromlef
t
t
ori
ght,a
lsof
romt
opt
obo
ttom.
Fr
omt
hea
ssump
tionsabove,wehavea11 = 1,a12 ora21
equa
ls2.Wemaycon
side
ron
lyt
heca
set
hata12 = 2s
incet
he
t
rans
pos
e of t
he ma
tri
xis aga
in good. Now we a
rgue by
con
trad
ict
iont
hatt
hef
irs
tr s1,2,...,p.As
owi sumeont
he
con
tra
ryt
hat1,2,...,ki
sont
hef
irtrow,bu
s tk +1i t,2
sno
≤k <p ,
the
ref
orea21 =k +1.Weca
llkcon
secu
tivei
ntege
rsa

block”,andwes
hal
lprovet
hatt
hef
irs
trowcons
ist
sofs
eve
ral
b
locks,t
ha s,t
ti hef
irtk numbe
s rsi
sa b
lock,t tk
he nex
numbe
rsi
saga
inab
lock,ands
oon.
I
fiti
sno
tso,a
ssumet
hef
irtn g
s roupsofk numbe
rsa
re

blocks”,bu
tt tk numbe
henex rsi ta “
sno block”(
ort
her
ear
e
noknumbe
rsr
ema
inng).I
i tfo
llowst
hatforj = 1,2,...,n,
80 Ma
thema
tic
alOl
ymp
iadi
nCh
ina

y(j-1)k+1 ,y(j-1)k+2 ,...,yjki


sa “
block”,t
hefi
rstnkco
lumn sof
t
hema trixcanbed ividedin
topn 1 ×ksubmatr
icsai,(j-1)k+1 ,
e
ai,(j-1)k+2 ,...,ai,jk ,
i =1,2,...,p,j =1,2,...,n,each
s
ubma
tri
xisa “
block”.Now a
ssumea1,nk+1 = a,l
etb bet
he
sma
lle
stpo
sit
ivei
ntege
rss
ucht
hata +bi
sno
tont
hef
irtrow,
s
henb ≤k -1.S
t i ea2,nk+1 -a1,nk+1 =x2 -x1 =a21 -a11 =k,
nc
Mathematical Olympiad in China (2011–2014) Downloaded from www.worldscientific.com

wehavea2,nk+1 = a + k,t
her
ef ea + bl
or iesi
nthef
irtnk
s
by NATIONAL UNIVERSITY OF SINGAPORE on 05/06/18. For personal use only.

co
lumns.
The
ref e,a + b i
or scon
tai
nedi he1 × k
n one oft
s
ubma
tri
cesmen
tionedabove,wh
i sa “
chi bl ra,
ock”,howeve
a +ka
reno
tint
his“
block”,wh
ichi
sacon
trad
ict
ion.
Wes
howed t
hatt
he f
irs
trow i
sformed by b
locks,i
n
par
ticu
lark|p,howeve r,
1 <k <p,andpisaprime,wh
ichi
s
imposs
ibl
e.So weconc
ludetha
tthefi
rstrowis1,2,...,p,
hek
t -thr
ow mu
stbe(
k -1)
p +1,(
k -1)
p +2,...,kp.Thu
s
uptointe
rchang
ing rows,co
lumnsandtran
spoe,t
s he good
que,t p!).
e2( 2
mat
rixi
sun i heanswe
risthe
ref
or

3 Pr
ovet
hatf
orany r
ea r M > 2,t
lnumbe her
eex
ist
sa
s
tri
ctl
yinc
rea
singi
nfi
nit
esequenceofpo
sit
ivei
ntege
rsa1 ,
a2 ,...s
ati
sfi
yngbo
tht
hefo
llowi
ngtwocond
itons:
i
(
1)ai > M iforanypo
sit
ivei
nt ri.
ege
(
2)Ani
nt rni
ege snon
-ze
roi
fandon
lyi
fthe
reex
ist
sa
pos
iti
vei n
tegerm andb1 ,b2 ,...,bm ∈ {-1,1},wi
thn
=b1a1 +b2a2 + … +bmam .

So
lut
ion.For g
iven M > 2,we cons
truc
t by i
nduc
tion a
equence{
s an }t
hatsat
isf
iest
her equ
iremen
ts.Takea1 ,a2 that
s
ati
sfya2 -a1 = 1anda1 > M .Nows
2
uppoea1 ,a2 ,...,a2k
s
a
rea
lreadycho
sen,s
ucht
hatai > M i ,i = 1,2,...,2
kand
s
ucht
hatt
hes
etAk = {
b1a1 + … +bmam |b1 ,...,bm = ±1,1
k}doe
≤ m ≤ 2 s no
t con
tan 0.I
i tis obv
iou
st t Ak i
ha s
Ch
inaMa
thema
tic
alOl
ymp
iad 81

s
ymme
trc,i.
i e.,Ak = -Ak .A1 = {
a1 ,-a1 ,1,-1}.Le
tn be

n Ak ,N = ∑ ai ,now
2k
t
hesma
lle
stpo
sit
ivei
ntege
rno
ti i=1

choo
sepo
sit
ivei
ntege
rsa2k+1 ,a2k+2s
ati
sfi
ynga2k+2 -a2k+1 = N +

n,a2k+1 > M 2k+2 ,a2k+1 > ∑i=1ai .Wenowshowt


2k
hatAk+1doe
s

t,n = - ∑i=1ai -a2k+1 +a2k+2 .


2k
no
tcon
tan0andn ∈ Ak+1 .F
i irs
Mathematical Olympiad in China (2011–2014) Downloaded from www.worldscientific.com

f∑i=1biai = 0,m ≤ 2
rhand,i k +2,a
m
Ont
heo
the s0 ∉
by NATIONAL UNIVERSITY OF SINGAPORE on 05/06/18. For personal use only.

Ak ,wemu
sthavem = 2
k +1or2
k +2.
k +1,hen| ∑i=1biai |≥a2k+1 - ∑i=1ai > 0.
2k+1 2k
fm = 2
I t
fm = 2
I k +2andb2k+1 andb2k+2 a
reoft
hes
ames
ign,t
hen

| ∑i=1biai|≥a2k+1 +a2k+2 - ∑i=1ai >0;


2k+2 2k
ifb2k+1andb2k+2a
re

o
fdi
ffe
ren
tsi
gns,t
hen
k+2
2 2
k 2
k

∑biai =
i=1 i=1
a2k+1 -a2k+2) ≥|a2k+1 -a2k+2|- ∑ ai
∑biai ± ( i=1

= N +n - N = n > 0.

The{
an }t
huscon
str
uct
eds
ati
sfi
est
her
equ
iremen
tss
ince0
i
sno
tcon
tai nanyAk ,andanynon
nedi -ze
roi
ntege
rbe
tween
-kandki
scon
tai nAk .
nedi

S
econdDay
8:
00~12:
30Janua
ry16,2012

tf(
4 Le x)= (
x +a)(
x +b)whe
rea,ba
reg
ivenpo
sit
ive
r
ealnumbe
rs,
n ≥ 2beag
iveni
ntege
r.Fornon
-nega
tive
r
ealnumbe
rsx1 ,x2 ,...,xn t
hats
ati
sfyx1 +x2 + … +
xn = 1,f
indt
he max
imum ofF = ∑ 1≤i<j≤n
n{
mi f(
xi),

f(
xj )}.
So
lut
ion1.As
82 Ma
thema
tic
alOl
ymp
iadi
nCh
ina

n{
mi f(
xi),f(
xj )}= mi
n{(
xi +a)(
xi +b),(
xj +a)(
xj +b)}
≤ (
xi +a)(
xi +b)(
xj +a)(
xj +b)
1 ((
≤ xi +a)(
xj +b)+ (
xi +b)(
xj +a))
2
1(
= xixj + xi +xj )(
a +b)+ab,
2
Mathematical Olympiad in China (2011–2014) Downloaded from www.worldscientific.com

s
o
by NATIONAL UNIVERSITY OF SINGAPORE on 05/06/18. For personal use only.

a +b
∑ ∑ (xi +xj )+ Cn ·ab
2
F ≤ xixj +
1≤i<j≤n 2 1≤i<j≤n

n n n
1 a +b(
( ∑x ) ∑ n -1)∑ xi + Cn
2
= [ - xi2 ] + 2
·ab
2 2
i
i=1 i=1 i=1

n
1 -1(
=
2
(1 - i∑ xi2 ) +n 2 a +b)+ Cn
2
·ab
=1

n
1 æç - 1
∑ xi ) ÷ +n 1(
2ö -
≤ 1 ( a +b)+ Cn
2
·ab
2è n i=1 ø 2
1 çæ1 - 1 ö÷ n -1( n(
n -1)
= + a +b)+ ab
2 è n ø 2 2
n -1æç 1 +a +b +nab ö÷
= .
2 èn ø

1
Theequa
lit
yho
ldswhenx1 = x2 = … = xn = .Sot
he
n
n -1æç 1 +a +b +nab ö÷
max fFi
imumo s .
2 èn ø

So
lut
ion2.Weshowthatthemax
imumva
luei
sat
tai
nedwhen
1, n -1æç 1 +a +b +nab ö÷
x1 =x2 = … =xn = andFmax = .
n 2 èn ø
Wei tonnt
nduc oshowamor
egene
rals
tat t:fornon
emen -
nega
tiver
ealnumbe
rsx1 ,x2 ,...,xns
ati
sfi
yngx1 +x2 + … +
xn = s (whe
res i
saf
ixed non
-nega
tiver
ea r),t
lnumbe he

max
imum va
lue of F = ∑ 1≤i<j≤n
n{
mi f(
xi), f(
xj )}i
s
Ch
inaMa
thema
tic
alOl
ymp
iad 83

s
a
tta
inedwhenx1 = x2 = … = xn = .
n
S
i eFi
nc ssymme
trc,wemaya
i ssumex1 ≤ x2 ≤ … ≤ xn .
No
tet
hat f(
x)i
sst
ric
tlyi
ncr
eas
ing on non
-nega
tive r
eal
numbe
rs,wehave

F =(
n -1)
f(x1)+ (
n -2)
f(x2)+ … +f(
xn-1).
Mathematical Olympiad in China (2011–2014) Downloaded from www.worldscientific.com
by NATIONAL UNIVERSITY OF SINGAPORE on 05/06/18. For personal use only.

æs ö
Whenn =2,F =f(
x1)≤f ç ÷ ,equa
lit
yho
ldswhenx1 =
è2ø
x2 .As
sumet
hatt
hes
tat
emen
tho
ldsforn,cons
ide
rtheca
seof
n +1.App
lyi
ngi
nduc
tivehypo
the
sisonx2 +x3 + … +xn+1 =
s -x1 ,wehave

1 æs -x1 ÷ö
x1)+ n(
F ≤ nf( n -1)
fç x1),
= g(
2 è n ø

whe
re g(
x)i
s a quadr
ati
cfunc
ti f x,t
on o he l
ead
ing
n -1,
coe
ffi
cien
tis1 + andt
hecoe
ffi
cien
tofx i
sa + b -
2n2
n -1æça +b + s ö÷ ,
the
ref e,t
or heax
isofs
ymme
tryi
s
2n è 2nø

n -1æça +b + s ö÷
-a -b
2n è 2nø s
≤ .
n -1 2(
n +1)
2+ 2
n

(
Theabovei
nequa
lit
yisequ
iva
len
tto[(
n -1)
s -2n(
n +1)(
a+
b)](
n + 1) ≤ 2
s(n2 + n - 1);obv
2 iou
sly,l
eft
-hands
ide <

( æ s ÷ö
n2 -1)
s < r
igh
t-hand s
ide.) The
re e,g ç
for i
sthe
èn +1ø
s
max
imumo x)on 0,
fg( [ s,F a
.Thu ] tta
insi
tsmax
imum
n +1
s -x1 s
whenx2 = x3 = … = xn+1 = = = x1 ,comp
let
ing
n n +1
t
hes
olu
tion.
84 Ma
thema
tic
alOl
ymp
iadi
nCh
ina

5 Le
tn be as
qua
re-
free po
sit
i r,k be an
ve even numbe
i
nt r,p beapr
ege r,s
imenumbe ati
sfi
yngp < 2 n ,p n,
2
p|n +k .Provethatncanbewri
ttena c +ca,
sn =ab +b
whe
rea,b,ca r
ed i
sti
ncti
vepo
sit
ivein
t ege
rs.
So
lut
ion.S
i seven,wehavep
eni
nc ≠ 2.Asp n,wehave
p k.Wemaya s
sumewithoutlos
so fgene
ral
ity0 <k < p.Set
Mathematical Olympiad in China (2011–2014) Downloaded from www.worldscientific.com

n -k(p -k) n +k 2
a =k,b = p -k,
by NATIONAL UNIVERSITY OF SINGAPORE on 05/06/18. For personal use only.

thenc = = -k.
p p
By a
ssumpton,c i
i s an int
eger,and a,b a redi
sti
nct
po
sit
ivei
ntege
rs.I
trema
inst
obes
hownt
hatc >0,andc ≠a,
n
b.Byt
heAM GMi
nequa
liy,wehave +k ≥2 n >p,
t thu
s
k
n +k2
n +k2 > pk,hencec > 0.I
fc = a,then -k =k, thus
p
n =k(2p -k).S i
nceniseven,
ki sa
lsoeven,asaconsequence
ni
sdi
vis
ibeby4,wh
l ichcon
trad
ict
sthef
actt
hatni
ssqua
re-
f
r fc = b,t
ee.I henn = p2 - k2 .S
i seven,ki
nceni sodd,
imp
lyi
ngt
hatni
saga
ind
ivi
sib
leby4,wh
ichi
sacon
trad
ict
ion.
Weconc
ludet
hata,b,c s
ati
sfya
llt
he r
equ
iremen
ts,
comp
let
ingt
heproo
f.

6 Fi
ndt
hesma
lle
st po
sit
ivei
nt rk wi
ege tht
hefo
llowi
ng
pr
opery:f
t oranyk e
lemen
ts ub
setA ofthes tS = {
e 1,
2,...,2012},the
reexi
stthr
eepairwi
sed
i s
tincte
lemen
ts
a,b,cofSs
ucht
hata +b,b +c,c +aa
llbe
l oA .
ongt
So
lut
ion.Wi
thou
tlo
ssofgene
ral
ity,wemaya
ssumea <b <

c.Wri
tex = a +b,z = b +c,y = a +c,thenx < y < z,
x +y >z,andx +y +zi seven.Ontheotherhand,i
fthere
ex
istx,y,z ∈ A s
ucht
hatx < y < z,x +y > z,andt
hat
x +y -z x +z -y
seven,s
x +y + zi eta = ,b = ,c =
2 2
Ch
inaMa
thema
tic
alOl
ymp
iad 85

y +z -x ,
iti
scl
eart
hata,b,ca
repa
irwi
sed
ist
inc
tel
emen
ts
2
ofS,andx = a +b,y = a +c,z =b +c.
Ther
equ
ired prope
rtyi
sequ
iva
len
ttot
hefo
ll ng:f
owi or
anyk
-el
emen
tsub
se fS,t
tA o her
eex
istt
hreee
lemen
tsx,y,
z ∈ As
ucht
hat
Mathematical Olympiad in China (2011–2014) Downloaded from www.worldscientific.com

x < y <z,x +y >z,andx +y +zi


seven. (* )
by NATIONAL UNIVERSITY OF SINGAPORE on 05/06/18. For personal use only.

I 1,2,3,5,7,...,2011},|A|=1007,andA doe
fA = { s
no
tcon
tai
nthr
eee
lemen
tss
ati
sfi
yngprope
rty (* ).The
re e,
for
k ≥ 1008.
Wenex
tprovet
hatany1008
-el
emen
tsub
se fS con
to tai
ns
t
hreee
lemen
tss
ati
sfi
yngpr
ope
rty (* ).
Weproveageneralstat
ement:Foranyi
nt rn ≥ 4,any
ege
(
n + 2)
-elemen
ts ub
se t of {
1,2, ...,2n}con t
ainst
hree
e
lemen
tss
ati
sfi
yng (* ).Wei tonn.
nduc
Whenn =4,
letA bea6
-el
emen
tsub
setof{
1,2,...,8},
henA ∩ {
t 3,4,5,6,7,8}
con
tai
nsa
tlea
stfoure
lemen
t fA
s.I
∩{
3,4,5,6,7,8}con
tai
nst
hreeevennumbe
rs,t
hen4,6,
8 ∈ As
ati
sfi
yng (* ).I
fA ∩ {
3,4,5,6,7,8}con
tai
ns
exac
tlytwoevennumbe
rs,t
heni
tcon
tai
nstwooddnumbe
rs.
Foranytwooddnumbe
rsx,y of{
3,5,7}, f(
twoo 4,x,y),
(
6,x,y),(
8,x,y)wou
lds
ati
sfyprope
rty (* ),t
husoneof
t
hemi
scon
tai nA .
nedi IfA ∩ {
3,4,5,6,7,8}
con
tai
nse
xac
tly
onee
v rx,
ennumbe theni
tcon
tai
nsa
llt
hre
eoddnumbe
rs,
then(
x,
5,7)
sat
isf
ies(*).Ther
esu
ltho
ldsf
orn = 4.
As
sumi
ngt
her
esu
ltho
ldsf n ≥4),
orn( cons
ide
rtheca
seof
tA be(
n +1.Le n +3)
-el
emen
tsof {
1,2,...,2
n +2},
if|A
∩{
1,2,...,2
n}|≥n +2.Byi
nduc
tivehypo
the
sis,t
her
esu
lt
fo
llows.I
trema
inst
ocon
si r|A ∩ {
de 1,2,...,2
n}|=n +1,
86 Ma
thema
tic
alOl
ymp
iadi
nCh
ina

n +1,2n +2 ∈ A .I
and2 fA con
tai
n n{
rxi
sanoddnumbe 1,
2,...,2
n},t
henx,2
n + 1,2n + 2s
ati
sfy (* );
ifnoodd
numberof {
1,2,...,2n}grea
tert
han1i scont
ai nA ,
nedi
thenA = {
1,2,4,6,...,2
n,2n +1,2n +2},and4,6,8 ∈
As
ati
sfy (* ).
e,t
Henc hesma
lle
stk wi
tht
her
equ
iredprope
rt s1008.
yi
Mathematical Olympiad in China (2011–2014) Downloaded from www.worldscientific.com
by NATIONAL UNIVERSITY OF SINGAPORE on 05/06/18. For personal use only.

2013 (
Shenyang,L
iaon
ing)

F
irtDay8:
s 00 12:
30
Janua
ry12,2013

1 Twoc
irc
lesK1 andK2 ofd
iff
eren
trad
iii
nte
rsec
tattwo
poin
tsA andB ,l
etC andD betwopoinsonK1 andK2 ,
t
re
spect
ivey,s
l uchthatA i
st he mi
dpo
intoft
hes egmen
t
CD .Theex
ten
sionofDB mee
tsK1 a
tano
the
rpo
intE ,
andt
heex
tens
i fCB mee
ono tsK2 a
tano
the
rpo
intF .Le
t
l1 andl2 bet
he pe
rpend
icu
larb
isec
tor
sofCD andEF ,
r
espec
tive
ly.
(
1)Show t
hatl1 andl2
hav
eaun
iqu
ecommon
po
int(
denot
e yP )
db .
(
2)Pr
ovetha
tt heleng
ths
fCA ,APandPEa
o re
t
hes
idel
eng
thso
fa
r
igh
ttr
iang
le. F
i .1
.1
g
Ch
inaMa
thema
tic
alOl
ymp
iad 87

So
lut
ion (
1)S
inceC ,A ,B ,E a
reconcyc
lic,andD ,A ,B ,
Fa
reconcyc
lic,CA = AD ,andbyt
het
heor
em o
fpowe
rofa
po
int,wehave

CB ·CF = CA ·CD = DA ·DC = DB ·DE . ①

Suppo
seont
hecon
tra
ryt
hatl1andl2dono
tin
ter
set,t
c hen
Mathematical Olympiad in China (2011–2014) Downloaded from www.worldscientific.com

CF DE
CD ‖EF ,hence = .P
lugg
ingi
nto ① ,wege
tCB2 =
by NATIONAL UNIVERSITY OF SINGAPORE on 05/06/18. For personal use only.

CB DB
DB2 ,
t sCB = DB ,henceBA ⊥ CD .I
hu tfo
llowst
hatCB and
DB a
ret
hed
iame
ter
sofK1andK2 ,
res
pec
tive
ly,henceK1and
K2 haves
amer
adi,wh
i ichcon
trad
ict
swi
tha
ssump
ti s,
on.Thu
l1 andl2 haveaun
iquecommonpo
int.
(
2)J
oinAE ,AF andPF ,wehave

∠CAE = ∠CBE = ∠DBF = ∠DAF .

S
inceAP ⊥CD ,APi
stheb
isec
torof ∠EAF .S
incePi
son
t
he pe
rpend
icu
larb
isec
tor o
fthes tEF ,P i
egmen son t
he
c
ircumc
irc
l f△AEF .Wehave
eo

∠EPF = 180
°- ∠EAF = ∠CAE + ∠DAF
= 2∠CAE = 2∠CBE .

e,B i
Henc sont
hec
irc
le wi
thcen
terP andr
adi
usPE ,
deno
tingt
hisc
irc
le byΓ.Le
tR bet
her
adi
u fΓ.By t
so he
t
heor
emofpowe
rofapo
int,wehave

2CA2 = CA ·CD = CB ·CF = CP2 -R2 ,

t
hus

AP2 = CP2 -CA2 = (


2CA2 +R2)-CA2 = CA2 +PE2 .

I
tfo
llowst
hatCA ,AP ,PE f
orm t
hes
idel
eng
thso
fa
r
ightt
riang
le.
88 Ma
thema
tic
alOl
ymp
iadi
nCh
ina

2 F
inda
llnon
-emp
tys
etsS o
fin
tege
rss
ucht
hat3m -2
n∈
Sfora
ll(
notnec
ess
ari
lyd
ist
i t)m ,n ∈ S.
nc
So
lut
ion Ca
llas
etS “
good”i
fits
ati
sfi
est
heprope
rtya
sst
ated
i
ntheprob
lem.
(
1)I
fS ha
son
lyonee
l t,Si
emen s“good”.
(
2)Now we can a
ssume t
hat S con
tai
nsa
tlea
st two
Mathematical Olympiad in China (2011–2014) Downloaded from www.worldscientific.com

e
lemen
ts.Le
t
by NATIONAL UNIVERSITY OF SINGAPORE on 05/06/18. For personal use only.

n{
d = mi |m -n|∶m ,n ∈ S,m ≠ n}.

Thent
her
eisani
nt ra,s
ege ucht
hata +d,a +2d ∈ S.
No
tet
hat

a +4d = 3(
a +2d)-2(
a +d)∈ S,
a +d)-2(
a -d = 3( a +2d)∈ S,
a +d)-2(
a +5d = 3( a -d)∈ S,
a -2d = 3(
a +2d)-2(
a +4d)∈ S.

Sowehavepr
ovedt
hati
f

a +d,a +2d ∈ S,

t
hen

a -2d,a -d,a +4d,a +5d ∈ S.

Con
tinu
ingt
hispr
oc e,wecandeduc
edur etha
t

{
a +kd|k ∈ Z,3 k}⊆ S.

tS0 = {
Le a +kd|k ∈ Z,3 k},
iti
sea
syt
ove
rif
yt tS0
ha
s“
i good”.
(
3)Now wehaveprovedt
ha fS ≠ S0 ,p
tS0 ⊆ S.I icka
rb ∈ S\S0 ,
numbe thent
her
eex
ist
sani
nt rl,s
ege ucht
hata +
ld ≤ b < a + (
l + 1)
d.S
inc
eatl
eatoneofl andl + 1i
s s
i
ndi
vis
ibeby3,weknowt
l hata
tlea
stoneofa +ld,a + (
l+
1)
discon
tai
nedi fa +ld ∈ S0 ,no
nS0 .I tet
hat0 ≤|b - (
a+
Ch
inaMa
thema
tic
alOl
ymp
iad 89

ld)|< d,byt
hede
fin
iti
onofd,wemu
sthaveb = a +ld.I
f
a +( d ∈ S0 ,no
l +1) tet
hat

0 <|a + (
l +1)
d -b|≤ d,

byt
hedefin
iti
ono fd,weal
sohaveb = a +ld.
Inbo
thca ss,wehaveprovedt
e hattherei rb ∈
sanumbe
S\S0oft eby3,hence
Mathematical Olympiad in China (2011–2014) Downloaded from www.worldscientific.com

ormb =a +ld.Th
hef islmu
stbed
ivi
sib
l
by NATIONAL UNIVERSITY OF SINGAPORE on 05/06/18. For personal use only.

a +ld,a + (
l +1)
d,a + (
l +2)
d ∈ S,
a+( d,a + (
l -2) l -1)
d ∈ S.

So

a+(
l +3)
d = 3(
a+(
l +1)
d)-2(
a +ld)∈ S,
a+(
l -3)
d = 3(
a+( d)-2(
l -1) a +ld)∈ S.

Con
tinu
ingt
hisproc e,wehavea + (
edur j)
l +3 d ∈S,j ∈
Z,wh
ichimp
lie
st t{
ha a +kd|d ∈ Z}⊆S.Wec
laimt
hatS =
{
a +kd ∣k ∈ Z},
sinc
ef rx ∉ {
oranynumbe a +kd|d ∈ Z},
t
her
eisane
lemen
tiny ∈ {
a +kd|k ∈ Z}s
ucht
hat0 <|x -
y|< d.Byt
hede
fin
iti fd,we mu
ono sthavex ∉ S.SoS =
{
a +kd ∣k ∈ Z},andi
tisea
syt
ove
rif
ytha
ts s“
uchSi good”.
Weconc
ludet
hatt
her
ear
ethr
eec
las
sesof “
good”s
ets:
(
1)S = {
a};
( a +kd ∣k ∈ Z,3 k};
2)S = {
(
3)S = {
a +kd ∣k ∈ Z},he
rea,d ∈ Z,d > 0.

3 Find al
l pos
iti
ve r
eal numberst wi
ththe fo
llowi
ng
prope
rty:the
reexis
tsani nf
ini
testX o
e frea
lnumbe rs
s
ucht
hatt
hei
nequa
lit
y

max{
|x - (
a -d)|,|y -a|,|z - (
a +d)|}>td

ho
ldsfora
ll(
notnec
ess
ari
lyd
ist
i t)x,y,z ∈ X ,a
nc ll
90 Ma
thema
tic
alOl
ymp
iadi
nCh
ina

r
ealnumbe
rsaanda
llpo
sit
iver
ealnumbe
rsd.
1
So
lut
i s0 <t < .
onTheansweri
2
1 1 -2t ö÷ ,
y,f
or0 <t < ,chooeλ ∈ ç0, (
æ
F
irs
tl s letxi =
2 è 2 1 +t)ø
λi ,X = {
x1 ,x2 ,...}.Wec
laimt
hatfora
ll(
notnece
ssa
ril
y
t)x,y,z ∈ X ,a
Mathematical Olympiad in China (2011–2014) Downloaded from www.worldscientific.com

d
ist
inc llr
ealnumbe
rsa anda
llpo
sit
iver
eal
by NATIONAL UNIVERSITY OF SINGAPORE on 05/06/18. For personal use only.

numbe
rsd,wehavet
hef
oll
owi
ngi
nequa
liy:
t

max{
|x - (
a -d)|,|y -a|,|z - (
a +d)|}>td.

Suppo
seont
hecon
tra
ryt
hatt
her
eex
ista ∈ R,d ∈ R+ and
xi ,xj ,xk ,s
ucht
hat

max{
|xi - (
a -d)|,|xj -a|,|xk - (
a +d)|}≤td.

e,
Henc

ìï -td ≤ xi - (
a -d)≤td,
ï
í -td ≤ xj -a ≤td,
ïï
d ≤ xk - (
î -t a +d)≤td,

e.,
i.

ìïxi + (
1 -t)d ≤ a ≤ xi + (
1 +t)
d,
ï
íxj -td ≤ a ≤ xj +td, (* )
ïï
îxk - (
1 +t)d ≤ a ≤ xk - (
1 -t)
d,

wh
ichimp
lie
stha
t

ìïxk - (1 +t) 1 +t)


d ≤ a ≤ xi + ( d,
ï
íxi + (1 -t)
d ≤ a ≤ xj +td,
ïï
d ≤ a ≤ xk - (
îxj -t 1 -t)d,

1
no
tet
hat0 <t < .I
tfo
llowst
hat
2
Ch
inaMa
thema
tic
alOl
ymp
iad 91

ìï xk -xi , ①
d ≥ (
ï 2 1 +t)
ï
ï xj -xi
íd ≤ , ②
ï 1 -2 t
ï
ïïd ≤ xk xj .
-
î 1 -2t ③

By ② ,③ andd > 0,wegetxi < xj < xk ,henc


ei >j >
Mathematical Olympiad in China (2011–2014) Downloaded from www.worldscientific.com
by NATIONAL UNIVERSITY OF SINGAPORE on 05/06/18. For personal use only.

k,λ +λ ≤λ +λ ,wege
j i+1 k+1 i
t

xj -xi λj -λi
= ≤λ. ④
xk -xi λk -λi

xj -xi xk -xi ,
By ① and ② ,wege
t ≥ hence
1 -2t 2(
1 +t)

xj -xi 1 -2t
≥ >λ,
xk -xi 2( 1 +t)

wh
ichcon
trad
ics④ ! Thu
t sourea
rli
erc
la tXi
imabou sproved.
1,
Se
cond
ly,fort ≥ wes
howt
hatf
oranyi
nfi
nit
estX ,
e
2
f nX ,wecanchoo
oranyx < y <zi sea ∈ Randd ∈ R+ s
uch
t
hat

max{
|x - (
a -d)|,|y -a|,|z - (
a +d)|}≤td.

z -x ,
Inf
act,l
etd = henc 1 -t)
ex + ( 1 +t)
d =z - ( d.
2
1
ta = max{
Le x +(
1 -t)
d,y -td}.S
incet ≥ ,weob
tai
n
2

y -x < 2d ≤ ( t)
1 +2 d,
{
x -y < 0 ≤ (
2t -1)d,

e.,
i.

y -td ≤ x + (
1 +t)d,
{
x +(1 -t)d ≤ y +td,
92 Ma
thema
tic
alOl
ymp
iadi
nCh
ina

hence

ìïx + (1 -t) 1 +t)


d ≤a ≤ x + ( d,
ï
íy -t d ≤ a ≤ y +td,
ïï
îz - (
1 +t)d ≤ a ≤z - (
1 -t)
d,

f
rom wh
ichweconc
ludet
hat
Mathematical Olympiad in China (2011–2014) Downloaded from www.worldscientific.com

max{
|x - (
a -d)|,|y -a|,|z - (
a +d)|}≤td.
by NATIONAL UNIVERSITY OF SINGAPORE on 05/06/18. For personal use only.

1
Soeve
ryt ≥ doe
sno
tsa
tis
fyt
her
equ
iremen
toft
he
2
prob
lem.

on,t s ç0, ÷ .
æ
Inconc
lus
i hes
etofa
llr
equ
iredti
è 2ø

S
econdDay8:
00 12:
30
Janua
ry13,2013

4 Gi
venani
nt rn ≥ 2,s
ege uppo
seA1 ,A2 ,...,An a
ren
nonemp
tyf
ini
tes
etss
ati
sfi
yng|AiΔAj|=|i -j|fora
ll
i,j ∈ {
1,2,...,n}.
Fi
ndt
hemi
nimum va
l f|A1 |+|A2 |+ … +|An |.
ueo
(He
re|X |deno
test
henumbe
rofe
lemen
tsofaf
ini
tes
et
X andXΔY = {
a|a ∈ X ,a ∉Y}∪ {
a|a ∈Y ,a ∉ X }
foranys
etsX andY .)
So
lut
ion For each pos
iti
ve i
nt r k, we prove t
ege hatt
he
mi
nimumva
lueo sk2 + 2;t
fS2k i he mi
nimum va
lueofS2k+1 i
s
k(
k +1)+2.
Fi
rst
ly,de
finet
hes
etsA1 ,A2 ,...,A2k ,A2k+1a
sfo
llows:

Ai = {
i,i +1,...,k},
i = 1,2,...,k;Ak+1 = {
k,k +1};
Ak+j = {
k +1,k +2,...,k +j -1},j = 2,3,...,k +1.
Ch
inaMa
thema
tic
alOl
ymp
iad 93

Fort
hisf
ami
lyofs
ets,i
tisea
syt
ove
rif
yt t|AiΔAj|=
ha
j -i =|i -j|ho
ldsi
nthefo
llowi
ngca
ses:
(
1)1 ≤i <j ≤k;
(
2)1 ≤i <j =k +1;
( k +1;
3)1 ≤i <k +1 <j ≤ 2
(
4)k +1 =i <j ≤ 2
k +1;
Mathematical Olympiad in China (2011–2014) Downloaded from www.worldscientific.com

(
5)k +2 ≤i <j ≤ 2
k +1.
by NATIONAL UNIVERSITY OF SINGAPORE on 05/06/18. For personal use only.

Mor r,t
eove hec
aseo
fi =ji
str
ivi
al,andt
hec
aseo
fi >jc
an
ber
edu
cedt
othec
aseo s,f
fi < j.Thu ora
lli,j ∈ {
1,2,...,
k +1},wehaveve
2 rif
iedt
hat

|AiΔAj |=j -i =|i -j|.

heabove(
Fort 2k +1)s
ets,wecanea
sil
yca
lcu
lat
etha
t

k(
k +1) k(
k +1)
S2k+1 = +2 + k +1)+2;
=k(
2 2

i
fwechoo
set
hef
irt2
s ks
ets,wege
ttha
t

S2k = S2k+1 -k =k2 +2.

Se
cond
ly,weshowt
hatS2k ≥k2 +2andS2k+1 ≥k(
k +1)+
2.No
tet
hef
oll
owi
ngf
acs:
t
Fa
ct1.Foranytwof
ini
tes
etsX ,Y ,wehave|X |+|Y |
≥|XΔY |.
Fa
ct2.Foranytwonon
-emp
tyf
ini
tes
etsX ,Y ,
if|XΔY |
= 1,
then|X |+|Y |≥ 3.
k,
Whenn = 2 i
tfo
llowsf
r t1t
om Fac hat

|Ai |+|A2k+1-i | ≥|AiΔA2k+1-i |= 2 i,


k +1 -2 i
= 1,2,...,k -1.

By|AkΔAk+1|= 1andFa
ct2,weha
ve|Ak |+|Ak+1|≥ 3.So
94 Ma
thema
tic
alOl
ymp
iadi
nCh
ina

k-1

S2k =|Ak |+|Ak+1 |+ ∑ (


|Ai |+|A2k+1-i |)
i=1

k-1

≥3+ ∑ (2k +1 -2i)=k +2.


2

i=1

S
imi
lar
l k +1,wege
y,whenn = 2 ttha
t

|Ai |+|A2k+2-i | ≥|AiΔA2k+2-i |= 2 i,


k +2 -2 i
Mathematical Olympiad in China (2011–2014) Downloaded from www.worldscientific.com

= 1,2,...,k -1.
by NATIONAL UNIVERSITY OF SINGAPORE on 05/06/18. For personal use only.

S
ince|AkΔAk+1 |= 1,wehave

(
|Ak |+|Ak+1 |)+|Ak+2 |≥ 3 +1 = 4,

s
o
k-1

S2k+1 =|Ak |+|Ak+1 |+|Ak+2 |+ ∑ (


|Ai |+|A2k+2-i |)
i=1

k-1

≥4+ ∑ (2k +2 -2i)=k(


i=1
k +1)+2.

Inconc
lus
ion,t
hemi
nimumva
l sk2 +2,andt
ueofS2k i he
mi
nimumva
l sk(
ueofS2k+1i k +1)+2.Equ
iva
len
tly,foranyn
2
≥ 2,
themi
nimumva
lueofSni
s [n4 ] +2.

5 Foreachpo
sit
ivei
nt rnandeachi
ege nt ri(
ege 0 ≤i ≤n),
l
etCin ≡ c(
n,i) (
mod2),whe
rec(
n,i) ∈ {
0,1},
andde
fine
n

f(
n,q)= ∑c(n,i)
q.
i

i=0

tm ,nandqbepo
Le sit
ivei
ntege
rswi
thq +1no
tapowe
r
of2.Suppo
set
hatf(
m ,q)|f(
n,q).Provet
hatf(
m,
r)|f(
n,r)foreve
rypo
sit
ivei
nt rr.
ege
So
lut
ion Foreach pos
iti
vei
nt rn,we wr
ege iten i
nbi
nar
y
Ch
inaMa
thema
tic
alOl
ymp
iad 95

r
epr
esen
tat
i sn =2a1 +2a2 + … +2ak ,whe
ona re0 ≤a1 <a2 < …
< ak .De
fineas
etT (
n)= {
2a1 ,...,2ak },T (
0)i
scons
ide
red
anemp
tys
et.
ByLuca
s em,Ci
􀆳Theor nisoddi
fandon
l fT (
yi i)⊆ T (
n),
hence

f(
n,q)= ∑q ∏ (
1 +qa ),
σ(
A)
=
Mathematical Olympiad in China (2011–2014) Downloaded from www.worldscientific.com

A ⊆T (
n) a∈T (
n)
by NATIONAL UNIVERSITY OF SINGAPORE on 05/06/18. For personal use only.

whe
reσ(
A )deno
test
hes
umofa
lle
lemen
tsofA .
Form ,nandqa
sgi
venbya
ssump
tion,weshowt
hati
f

f(
m ,q)= ∏
a∈T (
(
1 +qa )
m)

a∈T (
n)
(
1 +qa )= f(
n,q),

henT (
t m )⊆ T (
n),andcons
equen
tly,f(
m ,r)|f(
n,r)for
eve
ryr.
Foranyi
ntege
rsi,j,0 ≤ i < j,wehavet
hefo
llowi
ng
f
act
ori
zat
ion:

q -1 = ( +1)…( +1)( -1),


j j-1 i i
2 2 2 2
q q q

t
her
ef e,
or

(
q +1,q +1)= (
q +1,2)|2.
j i i
2 2 2

ts(
Le k)bet
hel
arge
stoddd
ivi
soro
fapo
sit
ivei
nt rk,
ege
hens(
q +1)ands(
q +1)a y,
i
q > 1.I
j
2 2
t recopr
ime.Cl
ear
l fi
> 0, r2(
mod4),andq +1 >2, ss( +1)>
i i i
+1 ≡1o
2 2 2
q thu q
fi = 0,
1.I si eq +1i
nc sno rof2,wehaves(
tapowe q +1)>
1.Foranya ∈ T ( q + 1)| ∏b∈T(n)s(
m ),s( q + 1).S
ince
a b

s(
1 +qa )> 1,wehavea ∈ T (
n),henceT (
m )⊆ T (
n),wh
ich
comp
let
est
heproof.

6 Gi
venpo
sit
ivei
ntege
rsm andn,f
indt
hesma
lle
sti
ntege
r
N (≥ m )wi
tht
hef
oll
owi
ngpr
ope
rty:i
fanN-e
lemen
tse
t
96 Ma
thema
tic
alOl
ymp
iadi
nCh
ina

ofi
ntege
rscon
tai
nsacomp
let
ere
sidues
yst
em modu
lom ,
t
heni
tha
sanon
-emp
tys
ubs
ets
ucht
hatt
hes
um o
fit
s
e
lemen
tsi
sdi
vis
ibebyn.
l
So
lut
ionTheansweri
s

1 [( , ) ]
N = max m ,m +n -
{ m m n +1 . }
2
Mathematical Olympiad in China (2011–2014) Downloaded from www.worldscientific.com

1 [
by NATIONAL UNIVERSITY OF SINGAPORE on 05/06/18. For personal use only.

F
irt,wes
s howt
ha x m,m +n -
tN ≥ ma { m (
m,n)+1] .
}
2
1 (
td = (
Le m ,n),andwr
item =dm 1 ,n =dn1 .
Ifn > m d+
2
1),t
her
e ex
ist
s a comp
let
ere
sidue s
yst
em modu
lo m ,x1 ,
x2 ,...,xm ,
sucht
hatt
hei
rre
siduemodu
loncons
ist
sexac
tlyof
m1 g
roupsof1,2, ...,d.Forexamp
le,t
hefo
ll ng m
owi
numbe
rshavet
her
equ
iredprope
rty:

i +dn1j,
i = 1,2,...,d,j = 1,2,...,m 1 .

1 (
F
ind
ingano
the
rse
tofk = n - m d + 1)- 1numbe
rsy1 ,
2
y2 ,...,yk t
hata
recong
ruen
tto1modu
lon,
thes
et

A ={
x1 ,x2 ,...,xm ,y1 ,...,yk }

con
tai
nsacomp
let
ere
sidues
yst
em modu
lom ,howeve
r,noneof
i
tsnon
-emp
tys
ubs
etsha
sit
ssum ofe
lemen
tsd
ivi
sib
lebyn.In
f
act,t
hes
umo
fthe (
sma
lle
stnon
-nega
tive)r
esi
duemodu
lonof
a
lle
lemen
tsofA i
sgr
eat
ert
hanze
roandl
esst
hanorequa
lto
m 1(
1 +2 + … +d)+k = n -1.Thu
s,

1 (
N ≥ m +n - m d +1),
2

e.,
i.

1 [( , ) ]
N ≥ max m ,m +n -
{ m m n +1 . }
2
Ch
inaMa
thema
tic
alOl
ymp
iad 97

1 (, ) ]
Ne
xt,wes
howt
ha x m,m +n - m[
tN = ma { m n +1 }
2
ha
sther
equ
iredprope
rty.
Thefo
llowi
ng key f
acti
sfr
equen
tlyu
sedi
nt oof:
he pr
amonganyki
ntege
rs,onecanf
inda (
non
-emp
ty)s
ubs
etwho
se
s
umi
sdi
vis
ibl ta1 ,a2 ,...,ak bei
ebyk.Le ntege
rs,Si =a1 +
a2 + … +ai .
I omeSii
fs sdi
vis
ibebyk,
l thent
her
esu
lti
str
ue.
Mathematical Olympiad in China (2011–2014) Downloaded from www.worldscientific.com
by NATIONAL UNIVERSITY OF SINGAPORE on 05/06/18. For personal use only.

Ot
herwi
se,t
her
eex
ist1 ≤i <j ≤k,
sucht
hatSi ≡Sj (
modk),
henSj -Si =ai+1 + … +aji
t sdi
vis
ibebyk,
l ther
esu
lti
saga
in
t
rue.Thefo
llowi
ngf
acti
sanea
sy coro
lla
ry oft
he pr
evi
ous
r
esu
lt:amonganyki
ntege
rs,eachofwh
ichi
samu
lti
pleofa,
onecanf
inda (
non
-emp
ty)s
ubs
etwho
ses
umi
sdi
vis
ibebyka.
l
Re
turn
ingt
otheprob
lem,wesha
lld
iscu
sstwoca
ses.
1
Ca
s n ≤ m(
e1: d +1),andN = m .
2
Weca
llaf
ini
tes
eto
fin
tege
rsak
-se
tift
hes
um ofa
lli
ts
e
lemen
tsi
sdi
vis
ibl tx1 ,x2 ,...,xm beacomp
ebyk.Le let
e
r
esi
dues
yst
em modu
lom .Cl
ear
ly,wecand
ivi
det
hes
enumbe
rs
i
ntom 1 g
r s,each g
oup roup cons
ist
ing ofacomp
let
ere
sidue
s
yst
em modu
lod.Le ty1 ,y2 , ...,yd beacomplet
er e
sidue
s
yst
em modu
lod,andyi ≡i (modd).I fdisodd,wecandiv
ide
d +1
e
achg
roupi
nto d-s
et,
fore
xamp
le,{
y1,yd-1},...,{
yd2-1 ,
2
1 1
yd2+1 },{
yd }.Wege d +1)d
t m 1( -se
ts.S
incen1 ≤ m 1(
d+
2 2
1),wecanchoo
ses
omeoft
heed
s -s
etss
ucht
hatt
hes
um of
t
hei
rel
emen
tsi
sdi
vis
ibebyn1d(=n).
l I seven,s
fdi imi
lar
ly,
d
acomp
let
ere
sidues
yst
em modu
lo d can be d
ivi
dedi
nto
2
d
-se
ts,wi
thyd2 r
ema
ini
ng.Twor
ema
ini
ng numbe
rsc
anf
orm
1
ano
t rd
he -s
et.
I heend,wed
nt ivi
dex1,x2,...,xmi
nto m1d +
2
98 Ma
thema
tic
alOl
ymp
iadi
nCh
ina

[m2 ] d-sets(possiblywithanumberleftifm isodd).


1
1

1 1 m1 , 1
S
incen1 ≤ m 1(
d +1)= m 1d + wehaven1 ≤
2 2 2 2

m 1d + [m2 ] ,againwecanfindsomeofthesed-setssuchthat
1

t
hes
umo
fal
lthe
ire
lemen
tsi
sdi
vis
ibebyn1d = n.
l
Mathematical Olympiad in China (2011–2014) Downloaded from www.worldscientific.com

1 1
e2:
n > m(
d +1),N = m +n - m (
d +1).
by NATIONAL UNIVERSITY OF SINGAPORE on 05/06/18. For personal use only.

Ca
s
2 2
tA beanN-e
Le lemen
tst,con
e tai
ningacomp
let
ere
sidue
1
s
yst
em modu
lom ,x1 ,x2 ,...,xm ,wi
ths
omeo
t rn -
he m
2
(
d +1)numbe
rs.I sodd,a
fd i sshowni
n ca
se1,we may
1
d
ivdex1 ,x2 , ...,xm i
i nto m 1(
d + 1)d
-se
ts.Di
videt
he
2
1 ( 1
r
ema
inngn -
i m d +1)numbe
rsa
rbi
tra
ril
yin
ton1 - m 1
2 2
(
d +1)g s,eachwi
roup thd numbe
rs.Amongeachg fd
roupo
numbe
rs,one may f
indad
-st,t
e her
e e,we haveano
for the
r
1
n1 - m 1(
d +1)d
-se
ts,andt
ota
llyn1 d
-se
ts.I seven,a
fdi s
2
1
d
iscu
ssedi
nca
se1,we mayd
ivdex1 ,x2 ,...,xm i
i nto m1
2

d+ [m2 ]d-sets.Ifm isodd,weareleftwithanumberx with


1
1 i

d 1
d|xi - .Di
videt
heo
t rn - m (
he d +1)numbe
rsa
rbi
tra
ril
y
2 2
d
i
nt n1 -m 1(
o2 d +1)g
r s,eachwi
oup th numbe
rs.Fromeach
2
d - ; d
r
goup,wecanf
inda s
et f
rom anytwo -s
ets.wecan
2 2
f
indad
-se
t.I seven,t
fm 1 i hen wecanf
ind ano
t rn1 -
he
1
m 1(
d +1)d
-se
ts,andt
ota
llyn1d
-se
ts.
I sodd,{
fm 1i xi}
isa
2
Ch
inaMa
thema
tic
alOl
ymp
iad 99

d d
-s
e n1 - m 1(
t,wehave2 d + 1)+ 1 -s
ets,anda
lson1 -
2 2
1 1
m 1(
d +1)+ d-s
ets.Aga
in,wecanf
indn1d
-se
ts.Fi
nal
ly,
2 2
wecanchoo
ses
omeoft
heen1 d
s -se
ts,s
ucht
hatt
hes
um ofa
ll
t
hei
rel
emen
tsi
sdi
vis
ibebyn1d(= n).
l
Mathematical Olympiad in China (2011–2014) Downloaded from www.worldscientific.com
by NATIONAL UNIVERSITY OF SINGAPORE on 05/06/18. For personal use only.

2013 (
Nani
jng,J
iangsu)

F
irs
tDay
00 12:
8: 30,Decembe
r21,2013

1 Inanacu
tet
riang
leABC ,AB >
AC ,t
he b
ise
ctor ofang
le BAC
ands
ideBCi
nte
rse
cta
tpo
intD ,
two po
insE and F a
t rei
nsi
des
AB and AC ,r
espec
tive
ly,s
uch
t
hatB ,C ,F ,E a
reconcyc
lic.
Prove t
hatt
he c
ircumcen
tero
f
t
riang
le DEF co
inc
ide
s wi
tht
he F
ig.1
.1
i
nne
rcen
tero
ftr
iang
le ABC i
f
andon
l fBE +CF = BC .
yi
So
lut
ion.Le
tIbet
hei
nne
rcen
terof△ABC .
(Sufi
fci
ency)Suppo
seBC = BE +CF .Le
tK bethepo i
nt
onBC sucht tBK = BE ,t
ha husCK = CF .S nceBI b
i isect
s
∠ABC ,CI b
ise
cts ∠ACB , △BIK and △BIE a
rer
efl
ect
ion
wi
thr
espe
c o BI, △CIK and △CIF a
tt rer
efl
ect
ion wi
th
r
espec
ttoCI,wehave ∠BEI = ∠BKI = π - ∠CKI = π -
100 Ma
thema
tic
alOl
ymp
iadi
nCh
ina

∠CFI = ∠AFI.Ther
ef e,A ,E ,I,
or
Fa
reconcyc
lic.
SinceB ,E ,F ,C a
re
concyc
lic,wehave ∠AIE = ∠AFE =
∠ABC ,and hence B ,E ,I,D a
re
concyc
lic.
S
incet
he b
isec
t f ∠EAF and
oro
Mathematical Olympiad in China (2011–2014) Downloaded from www.worldscientific.com

t
hec
ircumc
irc
l f △AEF mee
eo tatI,
by NATIONAL UNIVERSITY OF SINGAPORE on 05/06/18. For personal use only.

IE =IF .S
inc
etheb
ise
ct f ∠EBD
oro F
ig.1
.2

and t
he c
ircumc
irc
le of △BED a
lso
mee
tatI,IE =ID .So,
ID = IE = IF ,t
ha s,Ii
ti sal
sot
he
c
ircumcen
terof△DEF .
Q.
E.D.
(
Nec
ess
ity)SuppoeIi
s st
heci
rcumc
ent
e f△DEF.S
ro i eB,
nc
E,F,Ca r
ec onc
ycl
ic,AE·AB = AF·AC,AB > AC,wehav
e
AE < AF.The
ref
ore,t
heb
ise
cto
rof ∠EAFandt
hepe
rpend
icu
lar
b
ise
cto
rofEF me
e tI,wh
ta ichl
iesont
hec
irc
umc
irc
l f△AEF.
eo
S
inceBIb
ise
cts ∠ABC ,
letK bet
hes
ymme
tri
cpo
intofE
wi
thr
espe
c oBI,
tt thenwehave ∠BKI = ∠BEI = ∠AFI >
∠ACI = ∠BCI.Ther
ef e,K l
or iesonBC , ∠IKC = ∠IFC ,
∠ICK = ∠ICF ,and△IKC ≌ △IFC .
eBC = BK +CK = BE +CF .
Henc

2 Foranyi
nt rn wi
ege thn > 1,
let

D(
n)= {
a -b|a,ba
repo
sit
ivei
ntege
rswi
thn
=abanda >b}.

Provet
hatf
oranyi
nt rk wi
ege thk > 1,t
her
eex
istk
pa
irwi
sed
ist
inc
tin
tege
rsn1 ,n2 ,...,nk wi
thni >1(
1≤
i ≤k),s
ucht
hatD (
n1)∩ D (
n2)∩ … ∩ D (
nk )ha
sat
l
eas
ttwoe
lemen
ts.
Ch
inaMa
thema
tic
alOl
ymp
iad 101

Proo ta1 ,a2 ,...,ak+1 bek


f.Le + 1d
ist
inc
tpo
sit
i s,
veodd
whe
re each oft
hem i
ssma
lle
rthant
he pr
oduc
tofo
t rk
he
numbe
rs.Wr
iteN =a1a2 …ak+1 .Foreachi =1,2,...,k +1,
1 æç N + ö÷ ,
ai yi = 1 ç -ai ÷ ,t
æN ö
l
etxi = henxi2 - yi2 = N .
2 èai ø 2 èai ø
N
S
inceaiaj < N and >ai ,(
xi ,yi)(
1 ≤i ≤k +1)a
rek +1
ai
Mathematical Olympiad in China (2011–2014) Downloaded from www.worldscientific.com
by NATIONAL UNIVERSITY OF SINGAPORE on 05/06/18. For personal use only.

pos
iti
veintegersolu
tionsofequa t
ionx2 -y2 = N .Withou
tlo
ss
ofgener
ali
tys, upposexk+1 = mi { , , , }
n x1 x2 ... xk+1 .Foreach
i∈{
1,2,...,k},
si exi2 -yi2 = xk
nc 2
+1 -yk+1 ,
2
wehave

(
xi +xk+1)(
xi -xk+1)= xi2 -xk
2 2 2
+1 = yi -yk+1

=(
yi +yk+1)(
yi -yk+1).

tni = (
Le xi +xk+1)(
xi -xk+1)= (
yi +yk+1)(
yi -yk+1),
then

2xk+1 = (
xi +xk+1)- (
xi -xk+1)∈ D (
ni),
2yk+1 = (
yi +yk+1)- (
yi -yk+1)∈ D (
ni).

Soxk+1 > yk+1 ,2xk+1 and2yk+1 a


retwod
iff
eren
tmembe
rsof
n1)∩ D (
D( n2)∩ … ∩ D (
nk ).

3 Le
tN* bet
hes
eto
fal
lpo
sit
ivei
ntege
rs.Pr
ovet
hatt
her
e
ex
ist
saun
iquef
unc
tionf:N* → N* s
ati
sfi
yngf(
1) =
f(
2)= 1and

f(
n)= f(
f(n -1))+f(
n -f(
n -1)),n = 3,4,....

uchf,f
Fors indt
heva
lueoff(
2m )fori
nt rm ≥ 2.
ege
1
So
lut
ion.S
i ef(
nc 1)= 1,wehave ≤ f(
1)≤ 1.
2
Weshowbyi
nduc
tiont
hatforanyi
nt rn > 1,f(
ege n)i
s
un
ique
lyde
termi
nedbyt
heva
l ff(
ueo 1),f(
2),...,f(
n-
n
1),and ≤ f(
n)≤ n.
2
102 Ma
thema
tic
alOl
ymp
iadi
nCh
ina

Forn = 2,f(
2)= 1,
thec
laimi
str
ue.
As
sumet
hatforanyk,1 ≤k <n(
n ≥3),f(
k)i
sun
ique
ly
k n -1
de
te ned,and
rmi ≤ f(
k)≤k,
then1 ≤ ≤ f(
n -1)≤
2 2
n - 1,and1 ≤ n - f(
n - 1) ≤ n - 1,hencebyi
nduc
tion
hypot
hess,t
i heva
l ff(
ueo f(n - 1))andf(n - f(n - 1))i
s
ned,andt ff(n),byde on,i
Mathematical Olympiad in China (2011–2014) Downloaded from www.worldscientific.com

det
ermi heva
lueo fi
nit
i s
by NATIONAL UNIVERSITY OF SINGAPORE on 05/06/18. For personal use only.

f(
n)= f(
f(n -1))+f(
n -f(
n -1)), ①

wh
ichi
sun
ique
lyde
termi
ned.Fu
rthe e,wehave
rmor

1 (
f n -1)≤ f(
f(n -1))≤ f(
n -1),
2
1(
n -f(
n -1))≤ f(
n -f(
n -1))≤ n -f(
n -1).
2

n
Equa
liy ①imp
t lis ≤ f(
e n -1)+ (
n)≤ f( n -1))=
n -f(
2
n.Thec
laimi
sal
sot
r orn.Byi
uef nduc
tion,weprovedt
hat
t
her
eex
ist
s a un
ique f
unc
tion f: N* → N* s
ati
sfi
yng t
he
n
r
equ
iredprope
rti
es,and ≤ f(
n)≤ n.
2
t,wes
Nex howbyi
nduc
tiont
hatforanypo
sit
ivei
nt rn,
ege
wehave

n +1)-f(
f( n)∈ {
0,1}. ②

Whenn = 1,②i
str
ue.
As
sumet
hat②i
str orn ≤k.By ① ,wehave
uef

f(
k +2)-f(k +1)
=(f(f(
k +1))+f(k +2 -f(
k +1)))- (
f(f(
k))+
f(
k +1 -f(
k)))
=(
f(f(
k +1))-f(
f(k)))+ (
f(k +2 -f(
k +1))-

f(
k +1 -f(
k))).
Ch
inaMa
thema
tic
alOl
ymp
iad 103

Byi
nduc
tionhypo
the
sis,f(
k +1)-f( k)∈ {
0,1}.
ff(
I k +1)= f(k)+1,s i
nce1 ≤ f(
k)≤ k,itf
olows,
l
f
rom ③ andi
nduc
tionhypo
the
sis,t
hat

f(
k +2)-f(
k +1)= f(
f(k)+1)-f(
f(k))∈ {
0,1}.

ff(
I k +1)=f(
k),
since1 ≤k +1 -f(
k)≤k,
itf
oll
ows
Mathematical Olympiad in China (2011–2014) Downloaded from www.worldscientific.com

f
rom ③ andt
hei
nduc
tionhypo
the
sist
hat
by NATIONAL UNIVERSITY OF SINGAPORE on 05/06/18. For personal use only.

f(
k +2)-f(k +1)= f(
k +2 -f(k))-
f(k +1 -f(
k))∈ {0,1}.

s,②i
Thu str orn =k +1.Byi
uef nduc
tion,② i
str
uefor
anypo
sit
ivei
nt rn.
ege
Fi
nal
ly,weshowbyi
nduc
tiont
hatf
oranypo
sit
ivei
ntege
r
m ,wehavef(
2m )= 2m-1 .
Form = 1,
ther
esu
lti
scl
ear.
As
sumet
hatt
her
esu
lti
str orm = k,i.
uef e.,f(
2k ) =
2k-1 ,con
side
rtheca
s orm =k +1.
ef
As
sumeont
hecon
tra
ryt
hatf(
2k+1)≠ 2k ,
sincef(
2k+1)≥
2k ,andf(
2k+1)
isani
nt r,wehavef(
ege 2k+1)≥ 2k +1.S
ince
f(1)=1,by ② ,letn bet hesma
lles
tintege
rsucht
hatf(n)=
2k +1,wehaven ≤2k+1 ,bytheminimal
ityofn,f(n -1)=2k .
No
tic
et tn -2k ≤ 2k ,wehave
ha

2k +1 = f(
n)= f(
f(n -1))+f(
n -f(
n -1))
= f(
2k )+f(
n -2k )≤ 2f(
2k )= 2k ,

wh
ichi
sacon
trad
ict
ion.I
tfo
llowst
hatf(
2k+1)=2k ,
ther
esu
lt
i
sal
sot
rueform =k +1.Byi
nduc
tion,f(
2m )= 2m-1 forany
po
sit
ivei
nt rm .
ege
104 Ma
thema
tic
alOl
ymp
iadi
nCh
ina

S
econdDay
8:
00 12: r22,2013
30Decembe

4 Foranyi
nt rn wi
ege thn > 1,l
e 1 …pt b
tn = pα1 αt
ei
ts
s
tanda
rdf
act
ori
zat
ion,wr
ite

ω( n)=α1 + … +αt .
n)=t,Ω(
Mathematical Olympiad in China (2011–2014) Downloaded from www.worldscientific.com
by NATIONAL UNIVERSITY OF SINGAPORE on 05/06/18. For personal use only.

Prove or d
isprove t
he fo
llowi
ng s
tat t: Gi
emen ven any
po
sit
ivei
nt rk andanypo
ege sit
iver
ealnumbe
rsα andβ,
t
her
eex
ist
sapo
sit
ivei
nt rn wi
ege thn > 1s
ucht
hat

ω(
n +k) Ω(
n +k)
>αand <β.
ω(n) Ω(n)

So
lut
ion.Theansweri
sYES.
Fromt
hede
fin
itonofωandΩ,wehave
i

ω(
ab)≤ ω(
a)+ω(
b), ①

Ω(
ab)= Ω(
a)+ Ω(
b), ②

foranypo
sit
ivei
ntege
rsa,b.Gi
venaf
ixedpo
sit
ivei
nt rk
ege
andpos
iti
verealnumbesα,β,wet
r akeapos
iti
vei
nt rm >
ege
(ω(k)+1) α.Asther
ea r
einf
ini
tel
y manypr
imenumbers,we
Ω(
k)+1
cant
akeas
uff
ici
ent
lyl
argepr
imeps
ucht
hat m +l
ogp2
p
<β,a akem pa
ndt irwi
sed
ist
inc
tpr
imenumbe
rsq1 ,q2 ,...,
tn = 2q1q2
…q
qm t
hata
rea
llg
rea
tert
hanp.Wewi
llshowt
ha k
m

ha
sthede
sir
edprope
rty.
ω(
n +k) n +k
t,weprove =2q1q2 qm +

Fi
rs >α.Le
tn1 =
ω(n) k
1.Asq1 ,q2 ,...,qm a
rea
lloddpr
imenumbe
rs,2qi +1|n1
2qi +1
when1 ≤i ≤ m ,
thendi = isani
ntege
rgr
eat
ert
han1.
3
Ch
inaMa
thema
tic
alOl
ymp
iad 105

No
tet
hat

(
2r -1,2s -1)= 2r s -1fora
(, )
llpo
sit
ivei
ntege
rsr,s,③

and (
qi ,qj )= 1(
i ≠j),wehave

( 1 qi 1 2qi
di ,dj )= (2 +1,2qj +1)≤ (2 -1,22qj -1)
3 3
22qi -1
( ,2
qj )
22 -1
Mathematical Olympiad in China (2011–2014) Downloaded from www.worldscientific.com

= = = 1.
3 3
by NATIONAL UNIVERSITY OF SINGAPORE on 05/06/18. For personal use only.

d1 ,d2 , ...,dm a
ret
he pa
irwi
secopr
imef
act
orsofn1 ,and
eachoft
hemi
sgr
eat
ert e,ω(
han1.Henc n1) ≥ m .From ①
andt
hecho
iceofm ,wehave

n +k) ω(
ω( n1) ω(n1) m
≥ ≥ ≥ >α.
(
ωn ) (
ωn ) ω k)+1 ω(
( k)+1

Ω(
n +k)
Ne
xt,wep
rov
e < β.Asq1q2…qm i
saoddnumbe
r
Ω(n)
edby3,wehaen1 =2q1q2 qm +1 ≡±3(
mod9),

andc
anno
tbed
ivi
d v
n1
t
ha s,
ti 3‖n1.S
uppo
seqi
sap
rimef
act
o f andq ≤ p,
ro the
n
3

22q1q2 -1 = (
2q1q2 qm -1)·n1 ≡ 0(
modq).
…q …
m

Fromt
heFe
rma
t􀆳sLi
ttl em,2q-1 ≡ 1(
eTheor modq).From ③ ,
q|2 q1q2 qm q -1.From ( q -1,2q1q2 …qm )= (
q -1,2)≤
(2 … , -1)

2,q -1 <p <qi( i = 1,2,...,m ),henceq|22 -1,q = 3.


n1
Th
iscon
trad
ict
stha
t i sno
tamu
lti
pleo
f3.The
re e,each
for
3
n1 n1
pr
imef
act
orof i sl
arge
rthanp.So Ω(
n /3)
> p 1 .F om ②
r
3 3
andt
hecho
iceofpr sp andq1 ,q2 ,...,qm ,wehave
ime

æn1 ÷ö æn1 ÷ö
Ω(
n +k)= Ω(
k)+ Ω(
3)+ Ωç < Ω(
k)+1 +l
ogp ç
è3ø è3ø
< Ω(
k)+1 +l n1 -1)
ogp (
= Ω(
k)+1 +q1q2 …qml
ogp2,
106 Ma
thema
tic
alOl
ymp
iadi
nCh
ina

n +k) Ω(
Ω( k)+1 +q1q2 …qml
ogp2 Ω(
k)+1
< < +l
ogp2 <β.
Ω(n) …
q1q2 qm p
m

venX = {
5 Gi 1,2,...,100},cons
ide
rfunc
tionf:X → X
s
ati
sfi
yngbo
tht
hefo
llowi
ngcond
itons:
i
Mathematical Olympiad in China (2011–2014) Downloaded from www.worldscientific.com

(
1)f(
x)≠ xfora
llx ∈ X ;
by NATIONAL UNIVERSITY OF SINGAPORE on 05/06/18. For personal use only.

(
2)A ∩ f(
A )≠ ⌀fora
llA ⊆ X wi
th|A |= 40.
Fi
ndt
hesma
lle
stpo
sit
ivei
nt rk,
ege sucht
hatforanys
uch
f
unc
tionft
her
eex
ist
sas
etB ⊆ X s
ati
sfi
yng|B|=kand
B ∪ f(
B )= X .
Remark.Forasubs
e fX ,wede
tT o finef(
T)= {
x |t
her
e
ex
isst ∈ T s
t ucht
hatx = f(
t)}.
So
lut
ion.Fi
rst,wede
fineaf
unc
tionf:X → X wi
th

f(
3i -2)= 3
i -1,f( 3i -1)= 3
i,f(
3i)= 3
i -2,
i = 1,2,...,30,
f(
j)= 100,91 ≤j ≤ 99,f(
100)= 99.

Obv
iou
sly,fs
ati
sfi
escond
iton (
i 1).ForanyA ⊆ X wi
th|A |
=40,
if
(
i)t
her
eex
ist
sani
nt riwi
ege th1 ≤i ≤30s
ucht
hat|A ∩
{
3i -2,3 i}|≥ 2,
i -1,3 thenA ∩ f(
A )≠ ⌀ ;or
(
ii)91,92,...,100 ∈ A ,
thenA ∩f(
A )≠ ⌀a
lsoho
lds.
Inbo
thca
ses,fs
ati
sfi
escond
iton (
i 2).I
fas
ubs
etB o
fX
s
ati
sfi
esf(
B)∪B = X ,
thenweha
ve|B ∩ {
3i -2,3
i -1,3
i}|
≥ 2f
ora
ll1 ≤i ≤ 30,{
91,92,...,98}⊂ B ,andB ∩ {
99,
100}≠ ⌀.Hence,|B |≥ 69.
t,wewi
Nex llshowt
hat,foranyf
unc
tionfs
ati
sfi
yngt
he
de
scr
ibedcond
iti
ons,t
her
eex
ist
sas
ubs
etB ⊆ X wi
th|B|≤69
s
ucht
hatf(
B )∪ B = X .
Ch
inaMa
thema
tic
alOl
ymp
iad 107

Amonga
llt
hes
ubs
etsU ⊆ X wi
thU ∩ f(
U )= ⌀ ,choo
se
ones
ucht
hat|U |i
smax
ima
l.I
fthe
rea
re manyU ⊆ X wi
th
|U |be
ing max l,choo
ima seones
ucht
hat|f(
U)|i
smax
ima
l.
Theex
ist
enc
eofU i
sgua
ran
teedby cond
iton (
i 1).Le
tV =
U ),W = X\(
f( U ∪ V ).No
tet
hatU ,V ,W a
re pa
irwi
se
d
iso
jitandX = U ∪ V ∪ W .Fromcond
n iton (
i 2),|U |≤ 39,
Mathematical Olympiad in China (2011–2014) Downloaded from www.worldscientific.com

|V |≤ 39,|W |≥ 22.Wemaket
hefo
llowi
nga
sse
rti
ons:
by NATIONAL UNIVERSITY OF SINGAPORE on 05/06/18. For personal use only.

i)f(
( w )∈ U ,fora
llw ∈ W .Ot
herwi
se,l
etU' = U ∪
w },s
{ i ef(
nc U )= V ,f(
w )∉ U ,f(
w )≠ w ,wehaveU' ∩
f(
U')= ⌀.I
tisacon
trad
ict
i o|U |be
ont ing max
ima
l.
(
ii)f(
w 1) ≠ f(
w 2)fora
llw 1 ,w 2 ∈ W ,w 1 ≠ w 2 .
Ot
herwi
se,l
e w 1)=f(
tu =f( w 2)
thenbycond
iton (
i 1),
u∈
tU' = (
U .Le U\{ w 1 ,w 2},
u})∪ { si ef(
nc U')⊆V ∪ {
u},U'
∩ (
V ∪ {
u}) = ⌀ ,we have U' ∩ f(
U') = ⌀.I
tisa
con
trad
ict
i o|U |be
ont ing max
ima
l.
tW = {
Le w 1 ,w 2 ,...,w m },ui =f(
wi),1 ≤i ≤ mt
hen
by (
i)and (
ii),u1 ,u2 ,...,um a
red
ist
inc
tel
emen
t fU .
so
(
i
ii)f(
ui)≠ f(
uj )fora
ll1 ≤i <j ≤ m .Ot
herwi
se,l
et
v = f(
ui)= f(
uj )∈V ,U' = (
U\{
ui})∪ {
wi},
thenf(
U')
=V ∪ {
ui},U' ∩f(
U')= ⌀.Howeve
r,|f(
U')
|>|f(
U)|.
I
tisacon
trad
ict
i o|f(
ont U )|be
ing max
ima
l.
The
re e, f(
for u1), f(
u2), ..., f(
um ) a
re d
ist
inc
t
e
lemen
tsofV .Inpa
rti
cul
ar,|V |≥|W |.As|U |≤ 39,we
tB =U ∪ W ,
have|V|+|W |≥61and|V|≥31.Le then|B|≤
69andf(
B )∪ B ⊇ V ∪ B = X .Ove
ral,t
l hede
sir
edsma
lle
st
i
ntege s69.
rki

6 Fornon
-emp
tys
etsS,T o
fnumbe
rs,wede
fine

S +T = {
s +t|s ∈ S,
t ∈ T },2S = {
2s|s ∈ S}.
108 Ma
thema
tic
alOl
ymp
iadi
nCh
ina

tnbeapo
Le sit
ivei
nt r,andA ,B benon
ege -emp
tys
ubs
ets
of{
1,2,...,n}.Pr
ovet
hatt
her
eex
ist
sas
ubs
etD ofA +
Bs
ucht
hat

A ·B
A +B ),and|D |≥ | | | |,
D + D ⊆ 2(
2n

whe
re|X |deno
test
henumb
ero
fel
emen
tso
faf
ini
tes
etX .
Mathematical Olympiad in China (2011–2014) Downloaded from www.worldscientific.com

So
lut
ion.Le
tSy ={a,b)
( |a -b =y,a ∈ A ,
b ∈ B }.S
by NATIONAL UNIVERSITY OF SINGAPORE on 05/06/18. For personal use only.

ince
n-1

∑ |S
y=1-n
y |=|A|·|B|,
the
reex
ist
sani
ntege
ry0s
ucht
hat1-

A ·B A ·B
n ≤ y0 ≤ n -1and|Sy0 |≥ | | | | > | | | |.
2n -1 2n
tD = {
Le 2b +y0 | (
a,b)∈ Sy0 },
then

|A |·|B |.
|D |=|Sy0 |>
2n

Fr
omt
hede
fin
iti
onofSy0 ,foreachd ∈ D ,t
her
eex
ist
s
a,b)∈ Sy0 s
( ucht
hatd = 2
b +y0 =a +b ∈ A +B .SoD ⊆
A +B .Foranyd1 ,d2 ∈ D ,l
e b1 +y0 = 2a1 -y0 ,
td1 = 2
b2 +y0(
d2 =2 b1 ,b2 ∈ B ,a1 ∈ A ),
then

b2 +y0 = 2(
d1 +d2 = 2a1 -y0 +2 a1 +b2)⊆ 2(
A +B ).

The
ref e,D s
or ati
sfi
est
hecond
iti
on.
Ch
inaNationa
l
TeamSe
lec
tionTe s
t
Mathematical Olympiad in China (2011–2014) Downloaded from www.worldscientific.com
by NATIONAL UNIVERSITY OF SINGAPORE on 05/06/18. For personal use only.

2011 (
Fuzhou,Fui
jan)

F
irs
tDay
8:
00 12:
30,Ma
rch27,2011

1 Gi
venani
nt rn ≥3,
ege findt
hemax
imumr
ea rM ,
lnumbe
s
ucht
hatforanypo
sit
ivenumbe
rsx1 ,x2 ,...,xn ,t
her
e
ex
ist
sape
rmu
tat
iony1 ,y2 ,...,yn o
fx1 ,x2 ,...,xn
t
hats
ati
sfi
es
n 2
yi
∑y
i=1
2
i+1
2
-yi+1yi+2 +yi+2
≥ M,
110 Ma
thema
tic
alOl
ymp
iadi
nCh
ina

whe
reyn+1 = y1 ,yn+2 = y2 .(
poedby QuZhenhua)
s
So
lut
ion.Le
t
n
xi2
F(
x1 ,...,xn )= ∑x
i=1
2
i+1
2 .
-xi+1xi+2 +xi+2

F
irt,t
s akex1 = x2 = … = xn-1 = 1,xn = ε,t
hena
ll
permuta
tionsa
ret
hes
amei
nthes
ens
eofc
ircu
lat
ion.Int
his
Mathematical Olympiad in China (2011–2014) Downloaded from www.worldscientific.com
by NATIONAL UNIVERSITY OF SINGAPORE on 05/06/18. For personal use only.

ca
s e,wehave

2
F(
x1 ,...,xn )= n -3 + 2
+ε .
1 -ε +ε2

tε →0+ ,F → n -1,
Le oM ≤ n -1.
s
t,wes
Nex howt
hatforanypo
sit
ivenumbe
rsx1 ,...,xn ,
t
her
eex
ist
sape
rmu
tat
iony1 ,...,yns
ati
sfi
yngF (
y1 ,...,yn )
≥ n -1.
Inf
act,t
aket
hepe
rmu
tat
iony1 ,...,yn wi
thy1 ≥y2
≥ … ≥yn a ya -ab +b ≤ max(
a2 ,b2),we
2 2
ndbyt
hei
nequa
lit
s
eet
hat
2 2 2
y1 y2 yn-1
F(
y1 ,...,yn )≥ 2 + 2 + … + 2 ≥ n -1,
y2 y3 y1

wher
et he l
asti
nequ
ali
tyis ob
tai
ned by AM-GM i
nequa
lit
y.
Summingup,M = n -1.

tn > 1beani
2 Le nt r,
ege kbet
henumbe
rofd
ist
inc
tpr
ime
f
act
orsofn.Pr
ovet
hatt
her
eex
ist
sani
nt ra,1 <a <
ege
n
+1,
sucht
hatn|a2 -a.(
pos
edby YuHongb
ing)
k
So
lut
ion.Le 1 …pk b
tn =pα et
hes
tanda
rdf
act
ori
zat
ionofn.
1 αk

S
inc 1 ,.
epα ..,pk a
repa
irwi
s ime,byt
ecopr heCh
ine
se
1 αk

Rema
inde em,f
r Theor or each i,1 ≤ i ≤ k,cong
ruence
equa
tions
Ch
inaNa
tiona
lTe
amS
ele
cti
onTe
st 111

x ≡ 1(
modpiαi )
{
x ≡ 0(
modpj
αj
),j ≠i

haves
olu
tionxi .
Foranys
olu
ti 0 =x0 (
onofx2 modn),wes
eet
hatx0(
x0 -1)
≡0(
modn).Thenforeachi =1,2,...,k,e
it rx0 ≡0(
he mod
pii )orx0 ≡ 1(modpiαi ).Fu
rt r,l
he etS(A )bethes um of
α
Mathematical Olympiad in China (2011–2014) Downloaded from www.worldscientific.com

el
emen tsofs
ubstA o
e { , , , }(
f x1 x2 ... xk pa rti
cul
ary S(⌀ )
l ,
by NATIONAL UNIVERSITY OF SINGAPORE on 05/06/18. For personal use only.

= 0).Obv
iou
sly,wehave

S(
A )(
S(A )-1)≡ 0(
modn).

(
Thi
sis be
cau
seoft
he s
elec
tion of xi ,s
uch t
hat S(
A)
mod(
pii )
α
ise
it r0or1.)Mor
he eove
rifA ≠ A',t
henS(
A )≢
S(
A')(
modn).The
ref
ore,t
hes
umo
fal
lsub
set
so x1,x2,...,
f{
xn }
ise
xac
tlya
lls
olu
tion
sofx( modn).
x -1)≡ 0(
tS0 =n,Sr b
Le ethel
eas
tnon
-ne
gat
iver
ema
ind
e fx1 +x2 +
ro
… +xr modu
len,r = 1,2,...,k.Thu
sSk = 1.Fora
ll1 ≤r
≤k -1,Sr ≠0.S
incek +1numbe
rsS0 ,S1 ,...,Ska
r n[
ei 1,
n],byDi
rich
let
􀆳sDr
awe
rPr
inc
ipe,t
l her
eex
ist0 ≤l < m ≤k,
æjn (
j +1)
n ,(
s
ucht
hatSl ,Smi
nthes
amei
nte
r lç ,
va 0 ≤j ≤]
èk k
k -1),whe
rel = 0andm =kdono
tho
lds
imu
ltaneou
sly.
n
s,|Sl -Sm |<
Thu .Deno
tey1 = S1 ,yr = Sr - Sr-1
k
(
r = 2,3, ...,k).Soanys fyr ≡ xr (
um o modn)(
r = 1,
2,...,k)mee
tst
her
equ
iremen
t.
fSm -Sl >1,
I thena =yl+1 +yl+2 + … +ym =Sm -Sl ∈
æç ,n ö÷
1 i
sthes
olu
tiono
ftheequa
tionx2 -x ≡ 0(
modn).
è kø
IfSm - Sl = 1,thenn | (
y1 + y2 + … + yl )+ (ym+1 +
ym+2 + … +yk ),t
hatis,n | (
x1 + x2 + … + xl )+ ( xm+1 +
xm+2 + … +xk ).Noti
cet tm > l,wh
ha i
chcon tr
ad i
ctstot he
112 Ma
thema
tic
alOl
ymp
iadi
nCh
ina

de
fin
iti
onofxi .
IfSm -Sl = 0,t henn|yl+1 +yl+2 + … +ym ,tha
tis,n
|xl+1 +xl+2 + … +xm ,wh
ichcontr
adict
st hedef
ini
tionofxi .
fSm -Sl < 0,
I then

y1 +y2 + … +yl )+ (
a=( ym+1 + … +yk )
Sm -Sl )= 1 - (
= Sk - ( Sm -Sl )
Mathematical Olympiad in China (2011–2014) Downloaded from www.worldscientific.com
by NATIONAL UNIVERSITY OF SINGAPORE on 05/06/18. For personal use only.

n
i
sthes
olu
tiono
fequa
ti modn),and1 <a <1+
onx2 -x ≡0( .
k
ngup,t
Summi her
eex
issas
t ati
sfi
yngt
hecond
iti
on.

t3
3 Le n2 bet
heve
rtexnumbe
rofas
imp
leg
raphG (
int
ege
r
n ≥2).
Ift
hedeg
reeofeachve
rtexi
sno
tgr
eat
ert n,
han4
t
her
eex
ist
satl
eas
toneve
rtex wi
thdeg
ree1,andt
her
e
ex
ist
sarou
tewi
thl
eng
thno
tgr
eat
ert
han3be
tweenany
twove
rti
ces.Provet
hatt
hemi
nimum numbe
rofedge
sof
7 3
s n2 - n.
Gi
2 2
Remark.A rout
ebe
tweentwod
ist
inc
tve
rti
cesu andv wi
th
l
eng
thki
sas
equenceofve
rti
cesu = v0 ,v1 ,...,vk = v,
whe
reviandvi+1 ,
i =0,1,...,k -1,a
read
jac
ent.(
pos
edby
Leng Gang
song)
So
lut
ion.Foranytwod
ist
inc
tve
rti
cesuandv,wes
ayt
hatt
he
d
ist tweenu andv i
ancebe sthes
hor
tes
tleng
th oft
herou
te
tweenu andv.Cons
be ide
ra g
raph G * wi
th ve
rtexs
et{
x1 ,
x2 ,...,x3n2-n ,y1 ,y2 ,...,yn },whe
reyiandyja
read
jac
ent
(
1 ≤i <j ≤n),xiandxj a
reno
tad
j t(
acen 1 ≤i <j ≤3
n2 -
n),xiandyja
read
jacen
tifandon
l fi ≡j(
yi modn).Thu
s,t
he
deg
reeo s1,andt
feachxii hedeg
reeofyi doe
sno
texceed

3
n2 -n
n -1 + =4
n -2.
n
Ch
inaNa
tiona
lTe
amS
ele
cti
onTe
st 113

I
tisea
syt
oseet
hatt
hed
ist
anc tweenxi andxj i
ebe sno
t
r
gea
te han 3.So g
rt raph G * s
ati
sfi
est
he cond
iti
on o
fthe
7 2 3
prob
l sN = 3
em.G * ha n2 -n + Cn
2
= n - nedge
s.
2 2
Int
hefo
ll ng,wes
owi howt
hatanyg
raphG = G (
V ,E )
s
ati
sfi
yngt
hecond
iti
onoft
heprob
lemha
satl
eatN edge
s s.Le
t
ee1,Y ⊆ (
V\X )bet
Mathematical Olympiad in China (2011–2014) Downloaded from www.worldscientific.com

X ⊆ V bet
hes
etofve
rti
ceswi
thdeg
r he
by NATIONAL UNIVERSITY OF SINGAPORE on 05/06/18. For personal use only.

s
etofr
ema
ini
ngve
rti
cesad
jacen
ttoX ,andZ ⊆V\(
X ∪Y)be
t
hes
eto
frema
ini
ngve
rti
cesad
jac
entt tW =V\(
oY .Le X ∪Y
∪ Z).Wewi
llpo
intou
tthefo
llowi
ngf
act
s.
Proper
ty1.Anytwover
tice
sinYa
read
jacen
t.Th
isi
sbecau
se
oft
hef
actt
ha fy1 ,y2 ∈Ya
ti retwov
ert
ics,
e t
her
eex
istx1,x2 ∈
Xt
hata
read
jac
en oy1 andy2 ,r
tt espec
tive
ly;hencey1 andy2
a
read
jac
ents
incet
hed
ist tweenx1 andx2i
ancebe sno
tgr
eat
er
t
han3.
Proper
ty2.Thed
ist
ancebe
tweenve
rt nW andve
exi rt nY
exi
i
s2.Th
isi
sbe
cau
seo
fthef
actt
hati
fthed
ist
anc
ebe enw0 ∈W
twe
andy0 ∈Yi
sgr
eat
e han2 (
rt obv
iou
sly,d
isance >1),s
t uppo
se
t
hatx0 ∈ Xi
sad
jac
en oy0 ,
tt thent
hed
ist
anc tweenw 0and
ebe
x0i
sgr
eat
e han3,wh
rt ichi
sacon
trad
ict
ion.Fur
the e,we
rmor
knowt
hisPr
ope
rty2mean
seachve
rt nW i
exi sad
jacen
ttos
ome
ve
rt nZ .
exi
Deno
tebyx,y,zandwt
henumbe
rsofe
lemen
tins
etsX ,
Y ,Z and W ,r
espe
cti
vel
y.Now coun
tthenumbe
ro s:
fedge
t
her
ear 2
eCyedge
sbe
tweenpo
int
sinY ,xedge
sfrompo
insofX
t
oY ,a
t tlea
stzedge
sfr
om po
int
so oY ,anda
fZt tlea
stw edge
s
f
rom po
int
so oZ .So,i
fW t fy ≥ n,
then

|E |≥ Cy +x +z + w = 3
n2 + Cy -y ≥ 3
n2 + Cn -n = N ,
2 2 2

fy ≤ n - 1,s
andi inceeachdeg
reeofve
rtexi
sat mo
s n,
t4
114 Ma
thema
tic
alOl
ymp
iadi
nCh
ina

wehave

x +z ≤ y(
4 y -1))= y(
n-( 4n +1 -y)
≤(
n -1)(
3n +2)= 3n2 -n -2,
w ≥ 3n2 -y -y(
4n +1 -y)≥ 3.

Se
lec
tave
rtexP i
nW s
ucht
hatP i
sad
jacen
tasl
essa
s
pos
sib
letover
tic
esinZ .Suppos
et heleas
tnumberisa,a > 0
Mathematical Olympiad in China (2011–2014) Downloaded from www.worldscientific.com
by NATIONAL UNIVERSITY OF SINGAPORE on 05/06/18. For personal use only.

(byPrope
rty2).Deno
tet hese
to fthe
s eave
rti
cesbyNP ⊆ Z .
Coun
tingt
he numbe
roft
heedge
saga
in,t
her
ear 2
eCy edge
s
be
tweenpo
int
sinY ,x edge
sfrom po
int oY ,a
sofX t tlea
sty
edge
sfrom po
int oY (
sofNP t byPrope
rty2,t
hed
ist
ancef
rom
Pt
ove
rtexi s2),a
nYi tlea
stz -aedge
sfrom po
insofZ\NP
t
oY ,anda
t tlea
staw edge
sfr
om po
insofW t
t s,
oZ .Thu

|E | ≥ Cy +x +y +z -a +aw
2

n -1 + Cy + (
=3
2
a -1)(
2
w -1).

fa > 1,
I then

|E | ≥ 3
n2 -1 + Cy +(
w -1)
2

n -2 + Cy +3n -y -y(
≥3
2 2
4n +1 -y)> N .
2

fa = 1,
I s
incet
hedeg
reeofeachve
rt nW i
exi satl
eat2,
s
whenwecoun
ttheedge
sfr
om po
int oZ ,wes
sofW t hou
ldadd
a
tlea
stw/2edge
s,s
o

1
|E | ≥ 3
n2 -1 + Cy
2
+ w
2
1( 2
≥3
n -1 + Cy +
2 2
n -y -y(
3 4n +1 -y))> N .
2

7 2 3
ngup,t
Summi hel
eas
tnumbe
rofedge
sisN = n - n.
2 2
Ch
inaNa
tiona
lTe
amS
ele
cti
onTe
st 115

S
econdDay
8:
00 12:
30,Ma
rch28,2011

4 Le
t H bet
heor
thocen
terofa
cub
it-
ang
led △ABC , P be a

poi
ntonBC oftheci
rcumc
irc
le
Mathematical Olympiad in China (2011–2014) Downloaded from www.worldscientific.com


of△ABC .PH int
ers
ecsAC a
t t
by NATIONAL UNIVERSITY OF SINGAPORE on 05/06/18. For personal use only.

M .There ex
ist
sa poi
nt K on

AB such t
hatt hel
i s KM
ne i
pa
ral
lelt
ot heS
ims
onl
ineofP
wi
th respe
ctto △ABC , t
he
chordQP ‖BC ,t
hechord KQ i
nte
rsec
tsBC a
tpo
intJ.
Pr
ovet
hat△KMJi
sis
osce
les.(
pos
edby Xi n)
ongBi
So
lut
ion.WeshowthatJK = JM .

Dr
awl
i rom P andl
nef eti
tbe
perpend
icula
rtoBC andint
ers
ectt
he
ci
rcumcirc
leandBCa tpo
intSandL ,
r
espec
tive
l t N be t
y.Le he pr
ojec
t
po
intofP onAB .S
inceB ,P ,L and
Na r
econcyc
lic,

∠SLN = ∠NBP = ∠ABP = ∠ASP .

s,NL ‖AS.S
Thu inceNL ‖KM ,
thenKM ‖SA .
tT bet
Le hei
nte
rsec
tionpo
intofBCandPH .S
incepo
int
s
K ,Q ,P andM a
reconcyc
licandBC ‖PQ ,
soa
reK ,J,T and
M .Suppo
set
hatt
heex
tens
ionofAH i
nte
rsec
tst
hec
ircumc
irc
le
a
tpo
intD .Thenwehave

∠JKM = ∠MTC , ∠KMJ = ∠KTJ.

I
tsu
ffi
cest
oshowt
hat ∠MTC = ∠KTJ.
116 Ma
thema
tic
alOl
ymp
iadi
nCh
ina

I
tisea
syt
oseet
hatD and H a
res
ymme
tri
c ove
rli
ne
BC .Then

∠SPM = ∠SPH = ∠THD = ∠HDT .

Fur
the e KS
rmor = AM , s
o ∠ADM = ∠KPS.

Cons
equen
tly,
Mathematical Olympiad in China (2011–2014) Downloaded from www.worldscientific.com

∠TDM = ∠HDT + ∠ADM = ∠SPM + ∠KPS


by NATIONAL UNIVERSITY OF SINGAPORE on 05/06/18. For personal use only.

= ∠KPM = ∠KDM ,

wh
ich meanst
hatpo
insK ,T andD a
t reco
lli
nea
r.Thu
s

∠KTJ = ∠DTC = ∠MTC .

So ∠JKM = ∠KMJ.Con
sequen
tlyJK = JM .

ta1 ,a2 ,...beape


5 Le rmu
tat
ionofa
llpo
sit
ivei
ntege
rs.
Provet
hatt
her
eex
isti
nfi
nit
e po
sit
ivei
ntege
r 􀆳s,s
si uch
3
t
hat(
ai ,ai+1)≤ i.(
poedbyChenYonggao)
s
4
So
lut
ion. We provethi
s pr
obl
em by con
trad
ict
ion.I
fthe
conc
lus
ionoft
heprob
lemi
sno
ttue,t
r hent
her
eex
issi0 ,and
t
3
wehave(
ai ,ai+1)> ifori ≥i0 .
4
Takeapo
sit
i rM > i0 ,s
venumbe oifi ≥ 4M ,t
hen (
ai ,
3
ai+1)> i ≥ 3M .
4
fi ≥4M ,ai ≥ (
So,i ai ,ai+1)>3M ,
then{
1,2,...,3M }
⊆{
a1 ,a2 ,...,a4M-1}.
Henc
e

|{
1,2,...,3M }∩ {
a2M ,a2M+1 ,...,a4M-1}|
≥ 3M - (
2M -1)= M +1.

ByDi
rich
let
􀆳sDr
awe
rPr
inc
ipe,t
l her
eex
iss2M ≤j0 < 4M -
t
Ch
inaNa
tiona
lTe
amS
ele
cti
onTe
st 117

1s
ucht
hataj0 ,aj0+1 ≤ 3M .Thu
s,

( 1 3M 3· 3
aj0 ,aj0+1)≤ max{
aj0 ,aj0+1}≤ = 2M ≤ j0 ,
2 2 4 4

wh
ichi
sacon
trad
ict
ion.

6 Weca
llapo
ints
equence(
A0 ,A1 ,...,An )
int
ere
stng,
i if
Mathematical Olympiad in China (2011–2014) Downloaded from www.worldscientific.com

t
heab
sci
ssaandord
ina
tea
r lforeachAi ,andt
eequa he
by NATIONAL UNIVERSITY OF SINGAPORE on 05/06/18. For personal use only.

s
lope
sofs tOA0 ,OA1 ,...,OAn s
egmen tri
ctl
yinc
rea
se
(Oi
stheor
ign),andt
i hea
reaofeach△OAiAi+1(
0 ≤i ≤
1
n -1)
is .
2
Forapo i
ntsequence(A0 ,A1 ,...,An ), i
nser
tapo itA
n
adja
cen
ttotwopo i
ntsAi ,Ai+1s
ati
sfi
y
→ →
ngOA = OAi +OAi→+1 ,

thenwecallt
husobta
inednewpo intsequ
ence( A0,...,Ai ,
A ,Ai+1 ,...,An )anexpans
ionof(
A0 ,A1 ,...,An ).
t(
Le A0 ,A1 ,...,An )and(
B0 ,B1 ,...,Bm )beanytwo
i
nte
res
tingpo
ints
equence
s.Provet
ha fA0 = B0 andAn
ti
= Bm ,then wecanexpandbo
thpo
ints
equence
stos
ome
s
amepo
in equence (
ts C0 ,C1 , ...,Ck ).(
pos
edby Qu
Zhenhua)
So
lut
ion.Wes
eet
hatbyt
hecond
iti
on oft
he prob
lem,an
expans
ionofani
nte
res
tings
equenc
eiss
til
lin
ter
est
ing.
Fi
rst,wecons
truc
tthei
nte
res
ti equence(
ngs C0 ,C1 ,...,
Ck )con
tai
ninga
llpo
int
sofs
equenc
es(
A0 ,A1 ,...,An )and
(
B0 ,B1 ,...,Bm ),andC0 = A0 = B0 ,Ck = An = Bm .
BythePi em,weknowt
ck Theor hatt
hea r
eaoft
riang
le
equa
ls1/2ifandon lyift
her
eisnog
ridpoin
tonthet
riang
le
excep
ttr
iang l
e ve
rti
ces. Henc
ethereis no g
rid poi
nt on
△OAiAi+1 exc
epti
tsvert
ice
s.Ther
e e,i
for fthesl sofOAi
ope
andOBj a
r l,t
eequa henAi = Bj .
118 Ma
thema
tic
alOl
ymp
iadi
nCh
ina

Deno
tet
hes
lope
sofs
egmen
ts
f
rom po
int
sof{
Ai}and {
Bj }t
othe
or
igi
nins
tri
ctl
yinc
rea
singorde
rby
D0 ,D1 ,...,Dl ,whe
reD0 =C0 =
A0 =B0andDl =Ck = An =Bm .
If
a sequence (Di , Di+1) i
s no t
Mathematical Olympiad in China (2011–2014) Downloaded from www.worldscientific.com

i
nte
restng, t
i hen we can i ns
ert
by NATIONAL UNIVERSITY OF SINGAPORE on 05/06/18. For personal use only.

s
eve
ralpo
insE1 ,...,Ess
t ucht
hat F
ig.6
.1
t equence (
he s Di , E1 , ..., Es ,
Di+1)
isi
nte
res
ting.Inf
act,con
side
rtheconvexhu
llP o
fthe
gr
id po
inson △ODiDi+1 ,exc
t eptthe or
ign.P i
i sa convex
pol
ygonors tDiDi+1 (
egmen adegene
rat
edpo lygon).Thenthe
sequenceo f ve
rti
ceso fP isinte
res
ti ng. Thus, we have
construc
tedtheint
ere
sti
ngs e(
equenc C0 ,C1 ,...,Ck ).
Fi
nal
ly,i
tsu
ffi
cest
oshow t
hatt
hei
nte
res
tings
equence
(
A0 ,A1 ,...,An )canbeexpandedt
o(C0 ,C1 ,...,Ck ),and
t
hes
amei
str
uefor(
B0 ,B1 ,...,Bm ).Weon
lyneedt
oprove
t
hisfort
heca
seofn =1,
sinc
ewecanapp
lyt
heconc
lus
ionfor
o(
n =1t Ai ,Ai+1),
i =0,1,...,n -1s
ucce
ssi
vel tC0 =
y.Le
A0 andCk = A1 .Byi
nduc
tiononk,fork = 1,weneedno
expans
ion.Suppo
set
hatt
heconc
lus
ioni
str
uefora
llpo
sit
ive
i
ntege
rsl
es hank.
st
Then deno
tethe gri
d po itA
n
s
ati
sf i → → →
ngOA = OA0 + OA1 ,and we
y
s
eethatA mustbeapoi
nto fC1,...,
Ck-1 .S
inc
ei t,t
fno her
eisno g
rid
pointonint
eri
orofsegmentOA ,and
the
reexis
tsi,0 ≤i <k, s
uchthatA
l
oca
tesi
ntheang
lemadebyr
aysOCi
andOCi+1 .Wemays
uppo
set
hati >
F
ig.6
.2
Ch
inaNa
tiona
lTe
amS
ele
cti
onTe
st 119

0,o
the
rwi
set
aket
he g
raphs
ymme
tri
cove
rthel
inex = y.
S
incet
hea
reaoft
hepa
ral
lel
ogr s1,Ci l
am ▱OA0AA1i oca
tes
outs
ideof▱OA0AA1 ,Ci+1l oca
tesout
sideof▱OA0AA1orCi+1
= A1 .Inany way,wet akeBsuchthatO→B = OC→ +O→
i Ci+1 and
takeB's uchthatCi+1B'‖OA0 ,CiB'‖A0A ,t henAlocateson
s,A l
▱OCi+1B'Ci .Thu oca
tesi
nsi f ▱OCiBCi+1 ,wh
de o ich
Mathematical Olympiad in China (2011–2014) Downloaded from www.worldscientific.com

con
trad
ict
sthef
actt
hatt
hea
r s1.The
eaof▱OCiBCi+1i ref
ore
by NATIONAL UNIVERSITY OF SINGAPORE on 05/06/18. For personal use only.

t
hei
nse
rtedpo
in o(
tAt A0 ,A1)
forexpans
ioni
ssomeCi .Then
weu
set
hei
nduc
tion hypo
the
se o(
st A0 ,A )and (
A ,A1),
r
espec
tive
ly.

2012 (
Nanchang,J
ianxi)

F
irs
tDay
8:
00 12:
30,Ma
rch25,2012

1 Le
tH bet
heor
thoc
ent
ero
fan
acu
te-
ang
led△ABC wi
th ∠A >
60
°.Le
tpo
ins M and N beon
t
s
i sAB and AC ,r
de espe
cti
vey,
l
s
uch t
hat ∠HMB = 60
° =
∠HNC.LtOb
e ethec
irc
umc
ent
er
of△HMN .Le
tpo
insD andA
t
F
ig.1
.1
beont
hes
ames
ideo
flneBC ,
i
s
ucht
hat△DBC i
sregu
lar(
see
F
ig.1.1).Provet
ha tpoin
ts H ,O and D a
reco
lli
nea
r.
(
posedbyZhangS ihui)
120 Ma
thema
tic
alOl
ymp
iadi
nCh
ina

So
lut
ion.Le
tT bet
heor
thoc
ent
erof△HMN .Ex
tendedl
ine
s
ofHM andCAi
nte
rsec
tatpo
intP .Ex
tendedl
i sHN andBA
ne
i
nte
rse
cta
tpo
intQ .
Iti
sea
syt
oseet
hatpo
insN ,M ,P andQ
t
a
reconcyc
lic.
Mathematical Olympiad in China (2011–2014) Downloaded from www.worldscientific.com
by NATIONAL UNIVERSITY OF SINGAPORE on 05/06/18. For personal use only.

F
ig.1
.2

By ∠THM = ∠OHN ,weseet


hat ∠PQH - ∠OHN =
∠NMH - ∠THM = 9°,
0 t
ha s,
ti

HO ⊥ PQ . ①

Le
tpo
intR bes
ymme
tri
ctopo
intC ove
rHP .ThenHC =
HR .
By ∠HPC + ∠HCP = (∠BAC -60
°)+ (
90°- ∠BAC)=
°,wes
30 eet
hat ∠CHR = 60
°.Hence△HCRi sregu
lar.
By ∠HPC = ∠HQB and ∠HCP = ∠HBQ ,wes
eet
hat
△PHC ∽ △QHB .Then △PHR ∽ △QHB ,so △QHP ∽
△BHR .
Deno
te by ∠ (
UV ,XY)t
he ang
l tween U→
e be V and X→
Y
(
pos
iti
vean
tic
lockwi
se).
S
inc °,wehave ∠ (
e ∠PHR = 150 PQ ,RB ) = ∠ (
HP ,
HR ) = 150
°, △BCD and △RCH a
rer
egu
lar.So △BRC ≌
e ∠(
△DHC . Henc RB , HD ) = ∠ (
CR , CH ) = - 60
°.
Ch
inaNa
tiona
lTe
amS
ele
cti
onTe
st 121

Cons
equen
tly,

PQ,HD )= ∠ (
∠( PQ,RB)+ ∠ (
RB,HD )= 150
°-60 °,
° = 90

t
hati
s

DH ⊥ PQ . ②

By ① and ② ,po
insH ,O andD a
t reco
lli
nea
r.
Mathematical Olympiad in China (2011–2014) Downloaded from www.worldscientific.com
by NATIONAL UNIVERSITY OF SINGAPORE on 05/06/18. For personal use only.

2 Provet
hat,f
orany g
iveni
nt rk ≥ 2,t
ege her
eex
istk
d
ist
inc
tpo
sit
ivei
ntege
rsa1 ,a2 ,...,ak s
ucht
hatforany
i
ntege
rsb1 ,b2 ,...,bk wi
t ai ,
hai ≤bi ≤2 i =1,2,...,
k,andanynon
-nega
tivei
ntege
rsc1 ,c2 ,...,ck ,wehave

k∏i=1bi ∏ b ,prov ∏ ∏
k k k k
ci
< i=1 i
ided b ci
i=1 i
< i=1 i
b.
(
poedbyChenYonggao)
s
So
lut
ion.Wewi
llpr
oveas
tronge
rpropo
sit
ion:foranyg
iven
r
ea rkandanypo
lnumbe sit
ivei
nt rn,t
ege her
eex
istn po
sit
ive
i
ntege
rsa1 ,a2 ,...,an ,
sat
isf
yi ai ,1 ≤i ≤ n -1,
ngai+1 > 2
andf
oranyr
ealnumbe
rsb1 ,b2 ,...,bn ,ai ≤bi ≤2
ai ,
i =1,
2,...,n,andany non
-nega
tivei
ntege
rsc1 ,c2 ,...,cn ,we

havek ∏i=1bi ∏ b ,provded ∏i=1bi ∏


n n n n
ci
< i=1 i
i ci
< b.
i=1 i

tk >1.Wepr
Le ovet
hepr
opo
sit
ionbyi
nduc
tiononn.
Ifn
= 1,c1 = 0,
t akea1 >k,
hent theconc
lus
ioni
str
ue.Suppo
se
t
hatt
heconc
lus
ioni
stueforn ≥1.The
r rea
rex1 <x2 < … <
xn s
ati
sfi
yngxi+1 >2xi ,1 ≤i ≤n -1.Nowcons
ide
rtheca
seof
n +1.Takeapo
sit
ivei
nt rxn+1 > 2xn ,
ege sat
isf
ying

xn+1 æ2xn ö÷n


>k ç . ①
2xn è x1 ø

Thenl
etai =txi ,
i = 1,2,...,n +1,ti
sas
uff
ici
ent
ly
l
argei
nt r,s
ege ucht
hat
122 Ma
thema
tic
alOl
ymp
iadi
nCh
ina

an+1
> 2a2 …an+1 , ②
n
1

and

k·2n-1an+1 < a1 …an . ③


n-1

Int
hefo
ll ng,wewi
owi lls
howt
hatt
hes 1 ≤i ≤n +1)
eai(
mee
tther
equ
iremen tbi ∈ [
t.Le 1 ≤i ≤n +1)bet
ai ,2ai]( he
Mathematical Olympiad in China (2011–2014) Downloaded from www.worldscientific.com

r
ea r(
lnumbe no
tic
et twehaveb1 <b2 < … <bn+1 ),andci(
ha 1
by NATIONAL UNIVERSITY OF SINGAPORE on 05/06/18. For personal use only.

≤i ≤ n + 1)b s,s ∏
n+1 c
enon
-nega
tivei
ntege
r ati
sfi
yng b
i=1 i
i <


n+1
b.
i=1 i

f∑i=1ci ≤ n,
n+1
I then

n+1

k∏bi
ci
≤kbn
n
+1 ≤ k·2 an+1 bn+1 < a1 …anbn+1
n-1 n-1

i=1

n+1

≤ ∏b .(byusing ③ )
i=1
i

f∑i=1ci ≥ n +2,
n+1
I then

n+1 n+1

∏b ≥b1+ ≥ a1+ b1 > 2a2 …an+1b1 ≥ ∏b .(byusing ② )


ci n 2 n 1 n
i i
i=1 i=1

I
tisimpo
ssi
ble.

f∑i=1ci = n +1,wecons
n+1
I ide
rthr
eeca
ses.

fcn+1 ≥ 2,
I then
n+1

∏b
ci
i
i=1 bn+1 · æçb1 ö÷n-1 an+1 · æç a1 ö÷n-1
≥ ≥
n+1
bn èbn ø an è2an ø
2
∏b
i=1
i

xn+1 · æç x1 ö÷n-1
= > 1.(
byu
sing ① )
2xn è2xn ø

I
tisimpo
ssi
ble.
Ch
inaNa
tiona
lTe
amS
ele
cti
onTe
st 123

fcn+1 = 1,
I then
n+1

∏b ∏
ci n
i=1
i
b
ci
i=1 i
= .

n+1 n
b
∏b
i=1
i i=1 i

bi
No
tet
hat ∈ [
xi ,2xi],1 ≤ i ≤ n.By t
hei
nduc
tion
Mathematical Olympiad in China (2011–2014) Downloaded from www.worldscientific.com

t
by NATIONAL UNIVERSITY OF SINGAPORE on 05/06/18. For personal use only.

hypo
the
ses,wes
eet
hat
n+1 n+1

k∏bi
ci
< ∏b . i
i=1 i=1

fcn+1 = 0,
I then
n+1

∏b
i=1
i
bn+1 · æçbn ö÷n an+1 · æç an ö÷n
≥ ≥
n+1
bn èb1 ø an è2a1 ø
2
∏b
ci
i
i=1

xn+1 · æç xn ö÷n
= >k.(
byu
sing ① )
2xn è2x1 ø

s,wehaveche
Thu ckedt
hata1 ,a2 , ...,an+1 mee
tthe
r
equ
iremen
t.

tP (
3 Le x)=x2012 +a2011x2011 +a2010x2010 + … +a1x +a0be
apo
lynomi
alofdeg
ree2012ofr
ealcoe
ffi
cien
tswi
th1a
s
i
tsl
ead
ingcoe
ffi
cien
t.Fi
ndt
hemi
nimumo
frea rc
lnumbe
s
ucht
hat|Imz|≤ c|Rez|,whe
reRez andImz a
re,
r
espec
tive
ly,t
her
ealandt
heimag
ina
rypa
rtso
fanyr
oot
of a polynomi
al obta
ined by chang
ing s
ome of the
coe
ffi
cien
t ( )
sofP x tothei
ropposi
tenumber (
s. po
sedby
i)
ZhuHuawe
π
So
lut
ion.Fi
rst,wepo
intou
tt tc
ha ≥co
t .Cons
ide
rthe
4022
124 Ma
thema
tic
alOl
ymp
iadi
nCh
ina

pol
ynomi
alP (
x)= x2012 -x.Changingthes
ignofcoe
ffi
cien
ts
ofP (
x),weobta
infourpo lynomi
alsP (
x),- P (
x),Q (
x)=
x2012 + x and - Q (
x).No
tet
hatP (
x)and - P (
x)havet
he
1006 1006
s
amer
oos;oneo
t ft
heroo
t sz1 = co
si s π +is
in π.
2011 2011
Q(
x)and - Q (
x)havet
hes
ameroo
ts,anda
ret
heoppo
sit
e
sofP (
x).Thu
sQ (
x)ha
Mathematical Olympiad in China (2011–2014) Downloaded from www.worldscientific.com

numbe
roft
heroo
t sar
ootz2 = -z1 .
by NATIONAL UNIVERSITY OF SINGAPORE on 05/06/18. For personal use only.

Then,

æ|Imz1 |,|Imz2 |÷ö π


c ≥ mi
nç =co
t .
è|Rez1 | |Rez2 |ø 4022

π
t,wes
Nex howt
hatt
hean
swe
risc = co
t .Forany
4022

x)= x2012 +a2011x2011 +a2010x2010 + … +a1x +a0 ,


P(

weob
tai
napo
lynomi
al

x)=b2012x2012 +b2011x2011 +b2010x2010 + … +b1x +b0 ,


R(

bychang
ings i
gnofsomecoeff
ici
en fP (
to x),whe
reb2012 = 1,
andforj = 1,2,...,2011,

|aj |,j ≡ 0,1(


mod4),
bj = { -|aj |,j ≡ 2,3(
mod4).

Weshowt
hat,foreachroo fR (
tzo x),wehave|Imz|≤
c|Rez|.
Wepr
ovet
hisr
esu
ltbycon
trad
ict
ion.Suppo
set
her
eisa
roo fR (
tz0o x),
sucht
hat|Imz0|>c|Rez0|,
thenz0 ≠0and
π ,
e
ithe
rtheang
l fz0andii
eo sle
s hanθ =
st ort
heang
leof
4022
z0 and -ii
sle
s hanθ.Suppo
st set
hatt
heang
l fz0andii
eo sle
ss
hanθ;f
t ort
heo
the
rca
se,weneedon
lycons
ide
rthecon
juga
te
fz0 .The
o rea
retwoca
ses:
Ch
inaNa
tiona
lTe
amS
ele
cti
onTe
st 125

fz0i
I sont
hef
irs
tquadr
ant(
orimag
ina
ryax
is),s
uppo
se
t
hat ∠(
z0 ,
i)=α <θ,whe
re ∠(
z0 ,i)i
sthel
eas
tang
let
hat
ro
tat
esz0t
oian
tic
lockwi
se.For0 ≤j ≤2012,
ifj ≡0,2(
mod
4),
then ∠ (
bjzj0 ,1)=j
α ≤ 2012
α < 2012
θ.
fj ≡ 1,3(
I mod4),t
hen ∠ (
bjzj0 ,i) = j
α < 2011
θ and
b1z0 ,i)=α.Thu
∠( s,t
hepr
inc
ipa
lar
gumen 0 ∈ [
tofbjzj 2π -
Mathematical Olympiad in China (2011–2014) Downloaded from www.worldscientific.com

1
by NATIONAL UNIVERSITY OF SINGAPORE on 05/06/18. For personal use only.

α,2π) ∪ 0, π -α .Theve
2012 [ r
]texang
leoft
hisang
le-
2
1 1
doma
i s2012
ni α+ α < π.Andbjzj0 ,0 ≤j
π -α = π +2011
2 2
≤2012,a
reno
tal ro,s
lze othe
res
umcanno
tbez
ero.
fz0i
I satt
hes
econdquadr
ant,s
uppo
set
hat ∠(
i,z0)=α
<θ,
ifj ≡0,2(
mod4),
then ∠ ( 0 )=j
1,bjzj α <2012
θ.I
fj ≡
π
1,3(
mod4),
t i,bjzj0)=j
hen ∠ ( α ≤ 2011 s,eve
α < .Thu ry
2

pr
inc
ipl
ear
gumen
to 0 ∈ 0
fbjzj ,π +2011
[α .S
inc
π
e +2011
] α
2 2
< π,a 0 ,0 ≤ j ≤ 2
ndbjzj 012,a
reno
tal
lze
ro,s
othe
irs
um
canno
tbez
ero.
π
ngup,t
Summi hel
eas
trea rc = co
lnumbe t .
4022

S
econdDay
8:
00 12:
30,Ma
rch26,2012

4 Gi
venani
nt rn ≥ 4,Le
ege tA ,B ⊆ {
1,2, ...,n}.
Suppo
set
hatab +1i
sape
rfe
cts
qua
r rforanya ∈
enumbe
A andb ∈ B .Provet
hat

n{
mi |A |,|B |}≤l
og2n.

(
pos
edby Xi n)
ongBi
126 Ma
thema
tic
alOl
ymp
iadi
nCh
ina

So
lut
ion.Fi
rstweproveal
emma.
Lemma.Gi
venani
nt rn
ege ≥ 4,
letA ,B ⊆ {
1,2,...,n}.
Suppo
set
hatab +1i
sape
rfe
cts
qua
r rforanya ∈ A and
enumbe
ta,a
b ∈ B .Le ' ∈ A ,b,b
' ∈ B ,anda <a
',b <b
',hena
t 'b'
>5
.5ab.
Proo
foft
hel
emma.Fi
rstno
tic
et t(
ha ab +1)(
a'' +1)>
b
Mathematical Olympiad in China (2011–2014) Downloaded from www.worldscientific.com

(
ab' +1)(
a'b +1).So
by NATIONAL UNIVERSITY OF SINGAPORE on 05/06/18. For personal use only.

(
ab +1)(
a'' +1) >
b (
ab' +1)(
a'b +1).

S
inc
etwos
ide
sofabovei
nequa
lit
yar
eal
lin
tege
rs,wehave

ab +1)(
( a'' +1)≥ ( (
b ab' +1)(
a'b +1)+1)
2
.

Byexpan
sion,weob
tai
n

ab +a
'b' ≥ ab
' +a
'b +2 (
ab' +1)(
a'b +1)+1
>ab
' +a
' '·a
b +2 ab 'b.

Bya <a',b <b


',wehaveab' +a
'b >2ab.Leta
' ' =λab,
b
andcomb
ini
ngtheabovei
nequa
liy,wehave(
t 1 +λ)ab > (
2+

2 λ)
ab,s
oλ > 3 +2 2 > 5
.5.
Nowt
urnt
otheor
igi
n pr
obl
em.Le a1 ,a2 ,...,
tA = {
am },B = {
b1 ,b2 ,...,bn },a1 <a2 < … <am ,b1 <b2 < …
<bn .We mays
uppo
set
hat2 ≤ m ≤ n.S
incea1b1 + 1i
sa
pe
rfec
tsqua
r r,wehavea1b1 ≥ 3.Byt
enumbe emma,a2b2
hel
>5
.5a1b1 > 4 ,ak+1bk+1 > 4akbk ,k = 2,...,m -1.Thu
2
s,

n2 ≥ ambm ≥ 4m-2a2b2 > 4m .

The
re e,m ≤l
for og2n.

5 Fi
nda
lli
ntege
rsk ≥ 3,wi
tht
hefo
llowi
ngprope
rti
es:
The
reex
isti
ntege
rsm andns
ati
sfi
yng(
m ,k)= (
n,k)=
Ch
inaNa
tiona
lTe
amS
ele
cti
onTe
st 127

1andk| (
m -1)(
n -1)wi
th1 < m <k,1 <n <kand
m +n >k.(
pos
edby YuHongb
ing)
So
lut
ion.I
fkha
saf
act
oro
fsqua
renumbe
rgr
eat
e han1,l
rt et
k
t2 |k,
t > 1,
thent
akngm =n =k -
i +1,wes
eet
hats
uch
t
kha
sthepr
ope
rti
es.
Mathematical Olympiad in China (2011–2014) Downloaded from www.worldscientific.com

fkha
I snof
act
oro
fsqua
r r,
enumbe i
fthe
rea
retwopr
ime
s
by NATIONAL UNIVERSITY OF SINGAPORE on 05/06/18. For personal use only.

p1 ,p2s
ucht
hat(
p1 -2)(
p2 -2)≥ 4andp1p2 |k.Le
tk =
p1p2 …pr andp1 ,p2 ,...,pr be pa
irwi
sed
iff
er t,r ≥ 2.
en
S
incet
her
eisa
tlea
stoneo p1 -1)
f( p2p3 …pr +1and (
p1 -
2)
p2p3 …pr + 1i
scopr
imewi
thp1 (
othe
rwi
sep1 d
ivi
dest
hei
r
d
iff
er ep2p3 …pr ,wh
enc ichi
sa con
trad
ict
ion),t
aki
ng t
his
numbe
rast rm ,t
henumbe hen,1 < m < k,(
m ,k) = 1.
S
imi
lar
ly,wecant
akeanumbe f(
rn o p2 -1)
p1p3 …pr +1or
(
p2 -2)
p1p3 …pr + 1s
ucht
hat1 < n < k,(
n,k) = 1.So
p1p2 …pr | (
m -1)(
n -1),and

m +n ≥ (
p1 -2)
p2p3 …pr +1 + (
p2 -2)
p1p3 …pr +1
=k + ((
p1 -2)(
p2 -2)-4)
p3 …pr +2 >k.

Suchm ,n wi
lls
ati
sfyt
hecond
iti
ons.
I
fthe
rea
renotwopr sp1 ,p2s
ime ucht
hat(
p1 -2)(
p2 -2)
≥ 4,p1p2 |k,theni
tisea
syt
ove
rif
ysuchi
nt rk ≥ 3can
ege
on
lybe15,30ora
sp,2p (
whe
repi ime).I
sanoddpr tisea
sy
t
oseet
ha fk = p,2p,30,
t,i thent
her
eaenom ,ns
r ati
sfi
yng
t
hecond
iti
ons;
ifk = 15,t
hen m = 11,n = 13s
ati
sfyt
he
cond
iti
ons.
ngup,i
Summi nt rk ≥ 3s
ege ati
sfi
est
hecond
iti
onsi
fand
on
l fki
yi sno ime,nordoub
tanoddpr leofanoddpr
imeand
nor30.
128 Ma
thema
tic
alOl
ymp
iadi
nCh
ina

6 Suppo
set
her
ear
ebee
tle
son ache
ssboa
rdcons
ist
ing of
2012 × 2012 un
it square
s. Each un
it square can
accommoda
teatmostonebeetl t,a
e.Atamomen l
lbeetl
es
f
lyandl
andont
heche
ssboa
rdaga
in.Forabee
tle,weca
ll
t
he ve
ctorf
rom i
tsf
lyi
ng un
itt
oit
sland
ing un
itt
he
bee
tle
􀆳s“
dis
plac
emen
tvec
tor”. Weca
llt
hes
um ofa
ll
Mathematical Olympiad in China (2011–2014) Downloaded from www.worldscientific.com

bee
tle
􀆳s“
dis
plac
emen
tve
ct s”t
or he “
tot
ald
isl
pacemen
t
by NATIONAL UNIVERSITY OF SINGAPORE on 05/06/18. For personal use only.

ve
ct s”.
or
F
ind t
he max
imum l
eng
t f “
ho tot
ald
isl
pacemen
t
ve
ctor”cons
ide
ringt
henumbe
rofbee
tle
sanda
llpo
ssi
ble
po
sit
ion
soff
lyi
ngandl
and
ing.(
poedby QuZhenhua)
s
So
lut
ion.Se
tup a coord
ina
te wi
th or
igi
natt
he c
ent
er of
che
ss rdO andt
boa heg
ridl
inea
sthecoord
ina
tel
ine.Deno
te
t
hes
etoft
hecen
ter
sofs
qua
resbyS,andt
hes
etwhe
ret
he
bee
tle
sin
iti
all
ysandonbyM 1 ⊆S,andt
t hes
ett
hatt
hebee
tle
s
l tf:M 1 → M 2bet
andonbyM 2 ⊆S.Le heone
-to
-onemapp
ing
de
finedbyabee
tle
􀆳spo
sit
ionva
tthebeg
inn
ingt
othepo
sit
ion
u = f(
v)off
irs
tland
i s,t
ng.Thu het
ota
ldi
spl
acemen
tvec
tor
i
sgi
venby

V = ∑ (f(
v)-v)= ∑ u - ∑ v.
v∈M 1 u∈M 2 v∈M 1

No
tet
hatt
her
ight
-hands
ideof①i
sindependen
toff.We

needon
lyt
ofi
ndt
hemax f|V|=|∑u∈M u - ∑v∈M v|
imumo
2 1

fora
llM 1 ,M 2 ⊆ S,|M 1|=|M 2|.Wemays
uppo
set
hatM 1
∩ M 2 = ⌀,s
incee
l tofM 1 ∩ M 2 doe
emen tchange|V |.
sno
Suppo
set
hat|V |a
tta
insi
tsmax t(
imuma M 1 ,M 2),obv
iou
sly
V ≠ 0.Le
tlnel ⊥ V bea
i tpo
intO .
Lemma1.Lineldoesnotpas
sanypo
intofS.M 1i
sthes
etof
Sonones
ideofl,andM 2i
sthes
etofS ont
heo
the
rsdeofl.
i
Ch
inaNa
tiona
lTe
amS
ele
cti
onTe
st 129

Proo
fofLemma1.F
irt,M 1
s ∪ M 2 =S.Ot
herwi
se,s
i e|S|
nc
seven,t
i her
ear
eatl
eas
ttwopo
insa andb wh
t icha
reno
tin
M 1 ∪ M 2 .Suppo
set
hatt
heang
l tweena -bandV doe
ebe sno
t
°,t
exceed90 hen|V + (
a -b)|>|V |.Soaddai
ntoM 2 ,add
bi
ntoM 1 ,
then|V |wi
lli
ncr
eae,wh
s ichi
sacon
trad
ict
ion.
cond,M 2 = - M 1 .Ot
Se herwi
se,t
her
eex
ista,b ∈S,
such
Mathematical Olympiad in China (2011–2014) Downloaded from www.worldscientific.com

t
hata,-a ∈ M 1 andb,-b ∈ M 2 .Suppo
set
hatt
heang
le
by NATIONAL UNIVERSITY OF SINGAPORE on 05/06/18. For personal use only.

tweena -bandV doe


be sno eed90
texc °.Pu
tai
ntoM 2 ,andb
i
ntoM 1 ,Vchange
st a -b),and|V +2(
oV +2( a -b)
| >|V |,
wh
ichi
sacon
trad
ict
ion.
Th
ird,
ldoe
sno
tpa
ssanypo
in fS.Ot
to herwi
se,l
etlpa
ss
a,a ∈ M 1 ,-a ∈ M 2 ,
thenchangeai
ntoM 2 ,-ai
ntoM 1 ,V
change
stoV +4
a.No
ticet
hata ⊥V ,
s a|>|V|,wh
o|V +4 ich
i
sacon
trad
ict
ion.
Four
th,weshowt
hatM 2 i
sthes
etofS onones
ideofl
(
thes
idet
hatV i
spo
int
i o),and M 1 i
ngt sthes
etofS ont
he
o
the
rsdeofl.Ot
i herwi
se,t
her
eisa ∈ M 1 a
tthes
idet
hatVi
s
po
int
ingt
o.Andt
her
eisab ∈ M 2 ont
heo
the
rsi
de.Thent
he
ang
l tweena -bandVi
ebe sle
s han90
st °.Changeai
ntoM 2and
bi
ntoM 1 ,
thenVchange
sin
toV +2(
a -b),
thel
eng
thofwh
ich
i
sgr
eat
er,wh
ichi
sacon
trad
ict
ion.Lemma1i
snow proved.

Lemma2.Le
tSk = { (x,y)∈S |x|=k - 1
2
or|y|=k -

1 ,
}k = 1,2,...,1006,lbeal
inepa
ssngO anddoe
i sno
t
2
pa
sspo
int
sofSk .Deno
tea
llpo
int
sofSk onones
i flbyAk ,
deo

a
llpo
int
sofSk ont
heo
the
rsdebyBk .Deno
i teVk = ∑u∈A u -
k

∑v∈B v,thenthe maximum of|Vk |isobtained whenlis


k

hor
izon
tal(
orve
rti
cal),andVki
sve
rti
cal(
orhor
izon
tal).
130 Ma
thema
tic
alOl
ymp
iadi
nCh
ina

Proo
fofLemma2.Sk i
sloca
teda
t
t
hebounda
ryofas
qua
rewi
th2
kpo
int
s
oneachs
ide.Le
tthefourve
rti
cesof
1 1ö
e be A çk - ,k - ÷ ,
æ
t
he s
qua
r
è 2 2ø
1 1ö 1
B ç -k + ,k - ÷ ,C ç -k + ,-
æ æ
2 2ø 2
Mathematical Olympiad in China (2011–2014) Downloaded from www.worldscientific.com

è è
by NATIONAL UNIVERSITY OF SINGAPORE on 05/06/18. For personal use only.

1 ö÷ 1 1ö F
i .6
.1
andD çk - ,-k + ÷ .By g
æ
k+
2ø è 2 2ø
s
ymmetr
icity,wemays uppo
set tli
ha nt
ers
ec sAD a
t tpo
intP
wit
hnon-nega
tives
lope.LetP beloca
tedbe het(
tweent 1 ≤t
)
h(
≤kt f
romt
opt
obo
ttom)o
fSk onAD andt t +1)
he( thofSk
onAD (
seeF
ig .1,i
.6 nca
s fk = 6,
eo t = 3).Thu
s,
→ → →
Vk = (
2k -2)(
2k -1)j + (
2k -t)(- (
2 i +tj)+
k -1)
→ →
t ((
2k -1)
i+(
2k -t)j)
→ →
= -2(
2k -1)(
k -t)
i +2(- (
k -t) +3 k +1)j,
k -3 2 2

→ →
whe
re i and j a
re hor
izon
tal and ve
rti
cal un
it vec
t s,
or
r
espe
cti
vel
y.Deno
te(
k -t)
2
= u,0 ≤ u ≤ (
k -1)
2
,
then

1
|Vk | = (
2k -1)u +u2 -2(
3 k +1)
k2 -3 u+(
3 k +1)
k2 -3
2 2 2

4
=u - (
2 k +1)
k -2 u+(
3 k +1).
k -3
2 2 2 2

1
Asa quadr
ati
cfunc
tion ofu,u = k2 - k + i
sthe
2
s
ymme tr
icaxi
s.I
tiseasytoknowt t|Vk|2t
ha ake
sitsmax
imum
atu = 0.Sot = k,t
ha s,li
ti shori
zon
tal,soVk i
svert
ica
l.
Lemma2i
spr
oved.
Turnt
otheor
igi
nalprob
lem.Bys
ymme
tri
ciy,weneedt
t o
on
lycon
side
rtha
tthes
lopeofli
snon
-nega
tiveandl
esst
han1.
tM 1 ,M 2 bel
Le oca
tedontwos
i sofl.Deno
de teM 2 ∩ Sk =
Ch
inaNa
tiona
lTe
amS
ele
cti
onTe
st 131

Ak ,M 1 ∩ Sk = Bk ,Vk = ∑ u - ∑v∈B v,
u∈Ak
then
k

1006 1006

|V |= ∑ Vk ≤
k=1
∑ |V
k=1
k |. ②

So,|V|max ≤ ∑k=1|Vk|max. l,M 2i


1006
Ifli
shor
izon
ta sal
lthe

poin
tsofS ontheupperha
lf-p
lane,M 1i
sal
lpoin
tsofS onthe
Mathematical Olympiad in China (2011–2014) Downloaded from www.worldscientific.com

lowerha
lf-l
pane;each|Vk |t ake
sit
s maximum,anda lVk
l
by NATIONAL UNIVERSITY OF SINGAPORE on 05/06/18. For personal use only.

po
intupwa
rd.Andt
heequa
lit
yof ② ho
lds.So|V |i
ndeed
t
ake
st imum|V |max = 2 ×10063 .
hemax

2013 (
Jian i
gyn,J
iangsu)

F
irs
tDay
00 12:
8: 30,Ma
rch24,2013

1 venanyn(> 1)copr
Gi imepo
sit
ivei
ntege
rsa1 ,a2 ,...,
an ,deno
teA = a1 +a2 + … +an .Le
tdi = (
A ,ai)(
the
gr
eate
stcommondivi
sor),
i = 1,2,...,n.
LetDi bet
hegrea
testcommond i
viorof {
s a1 ,a2 ,...,
an }
\{ai},i = 1,2, ...,n.Fi
nd t
he mi
nimum of
A -ai (
∏ i)
n
. po
sedbyZhangS
ihu
i=1
diDi
So
lut
ion.Cons
ide
r

D1 = (
a2 ,a3 ,...,an )andd2 = (
a2 ,A )
a2 ,a1 +a2 + … +an ).
=(

t(
Le D1,d2)=d.Thend|a2,d|a3,...,d|an ,d|a1 +
a2 + … +an .Thu
s,d|a1 .Con
sequen
tly,
132 Ma
thema
tic
alOl
ymp
iadi
nCh
ina

d| (
a1 ,a2 ,...,an ).

S
incea1 ,a2 ,...,an a
r ime,wehaved = 1.No
ecopr te
t
hatD1 |a2 ,d2|a2 and (
D1 ,d2)= 1.WehaveD1d2|a2 .So
D1d2 ≤ a2 .S
imi
lar
ly,wehaveD2d3 ≤ a3 ,...,Dnd1 ≤ a1 .
Hence,
Mathematical Olympiad in China (2011–2014) Downloaded from www.worldscientific.com

∏dD D1d2)·(
=( D2d3)·…·(
Dnd1)
by NATIONAL UNIVERSITY OF SINGAPORE on 05/06/18. For personal use only.

i i
i=1

≤ a2a3 …ana1
n

= ∏a .
i=1
i ①

Con
side
ring
n n

∏ (A -ai)=
i=1
∏ ( ∑a )
i=1 j≠i
j

n 1

≥ ∏ ( (n -1)( ∏a ) )
n-1
j
i=1 j≠i
n

=(
n -1) · ∏a ②
n
i
i=1

andby ① and ② ,wes


eet
hat
n
A -ai
∏ ≥(
n -1) .
n

i=1 diDi

Ont
heo
the fa1 =a2 = … =an = 1,
rhand,i

A -ai
∏ =(
n -1)
n n
.
i=1
diDi

A -ai (
ngup,t f∏i=1 s n -1)
n
Summi hemi
nimumo i n
.
diDi

2 Suppo
set
hatO andI a
ret
hec
ent
reso
fthec
ircumc
irc
le
andi
nci
rcl
eof△ABC wi
thr
adi
usRandr,
res
pec
tive
ly,P
Ch
inaNa
tiona
lTe
amS
ele
cti
onTe
st 133


i
sthemi
dpo
intofa
rcBAC .Le
tQP bet
hed
iame
te fO .
ro
tPIi
Le nte
rse
ctBCa
tpo
intD ,andl
ett
hec
ircumc
irc
leof
△AIDi
nte
rse
ctt
heex
tendedl
ineofPA a
tpo
intF .Le
t
po
intE beon PD s
ucht
hatDE = DQ .Provet
hat,i
f

∠AEF = ∠APE ,
2
r (
thens
in2 ∠BAC = . po
sedbyXi
ong
R
n)
Bi
Mathematical Olympiad in China (2011–2014) Downloaded from www.worldscientific.com
by NATIONAL UNIVERSITY OF SINGAPORE on 05/06/18. For personal use only.

F
ig.2
.1 F
ig.2
.2

So
lut
ion.S
i e ∠AEF
nc = ∠APE ,
then△AEF ∽ △EPF .So
AF ·PF = EF .S
2
inc
epo
insA ,I,D andF a
t reconcyc
lic,
s,
PA ·PF =PI·PD .Thu

PF2 = AF ·PF +PA ·PF



= EF +PI·PD .
2

S
incePQi
sthed
ime
terofc
irc
leOandpo
in sonAQ ,we
tIi
s
eet
hatAI ⊥ AP .Cons
equen
tly,

∠IDF = ∠IAP = 90
°.

s,wehave
Thu

PF2 -EF2 = PD2 -ED2 .

Comb
inng ① ,wehave
i

PI·PD = PD2 -ED2 .


134 Ma
thema
tic
alOl
ymp
iadi
nCh
ina

Thu
s

QD2 = ED2 = PD2 -PI·PD =ID ·PD .

Con
sequen
tly,wehave

△QID ∽ △PQD . ②

S
incePQi
sthed
iame
tero
fci
rceO ,wes
l eet
hatBP ⊥BQ .
Mathematical Olympiad in China (2011–2014) Downloaded from www.worldscientific.com

Suppo
set
hatPQi
sthepe
rpend
icu
larb
isec
torofBCa
tpo
intM .
by NATIONAL UNIVERSITY OF SINGAPORE on 05/06/18. For personal use only.

No
tet
hatIi
sthei
ncen
treof△ABC .WehaveQI2 = QB2 =
s,
QM ·QP .Thu

△QMI ∽ △QIP . ③

By ② and ③ ,weseethat ∠IQD = ∠QPD = ∠QPI =


∠QIM .He nce, MI ‖QD .LetIK ⊥ BC be at K .Then
IK ‖PM ;
t s,
hu

PM PD PQ
= = .
IK ID MQ

Byt
he Ci
rcl
e-Powe
rTheor
em andt
heS
i em,we
ne Theor
knowt
hat

PQ ·IK = PM ·MQ = BM ·MC


æç 1 ö2
= BC ÷ = (
Rsn∠BAC)
i 2
,
è2 ø

PQ ·IK 2R ·r 2r
t
hus,s
in2 ∠BAC = = = .
R2 R2 R

3 Suppo
set
her
eae101pe
r rsonss
itt
inga
roundar
oundt
abl
e
i
nana
rbi
tra
r r.Thekt
yorde hpe
rsonpo
sse
ssskp
e iece
sof
ca
rts,k = 1, ...,101.Weca
l tat
li ran
sit
ion i
fone
t
rans
itsoneofh
isca
rtst
ooneofh
isad
jac
entpe
rsons.
Fi
ndt
hemi
nimum po
sit
i rk,s
venumbe ucht
hatwha
teve
r
t
heorde
roft
hes
eat
ing,t
her
eiswayofno mor
ethank
Ch
inaNa
tiona
lTe
amS
ele
cti
onTe
st 135

t
rans
iti
onss
otha
teachpe
rsonpo
sse
sss51ca
e rts.(
pos
ed
by QuZhenhua)
So
lut
ion.Theansweri
sk =42925.
Le
tthec
ircumf
erenceoft
het
abebe101,andt
l hed
ist
ance
be
tweentwo ad
jac
ent pe
rsons be 1.In t
he fo
ll ng, we
owi
cons
ide
rthel
eas
ttr
ans
iti
ont
ime
s.
Mathematical Olympiad in China (2011–2014) Downloaded from www.worldscientific.com

Deno
tet
he pe
rson whoi
nit
ial
lypo
sse
sssica
e rdsby [
i-
by NATIONAL UNIVERSITY OF SINGAPORE on 05/06/18. For personal use only.

51].So,i
fp >0,
thenwecant
hinkofpe
rson[
p]a
sthes
our
ce
whoshou
lds tp ca
endou rds;andi
fp < 0,pe
rs p]i
on [ sthe
s
inkwhos
hou
ldr
ece
ive -pca
rds.
Suppo
set
hata
ttheendo
ftr
ans
itons,eachpe
i rs s51
onha
cards.If pe
rson B poss
es s
esa ca rd u init
ial
ly bel
ong
i ng to
personA ,wecant hinkt hatA carri
esca rdu toB bypa s
sing

throughthe minoracAB ,t
r helengthoft herouteisthea rc

length|AB |mean sthetime softr
an s
iti
on s.
Le
tsea
ting orde
rbea
sshowni
nthef
igu
re.Pe
rs i]
on [
t
ran
sit
sal
lhsica
i rdst
ope
rson [-i](
i = 1,2,...,50)wi
th
rou
tel
eng
thi.Sot
her
ear
eal
ltoge
t r12 + 22 + … + 502 =
he
42925t
ran
sit
ion
s.Int
hef
oll ng,wewi
owi llshowt
hatnol
ess
t
ran
sit
ionscanmee
tther
equ
iremen
tifpe
rson
sar
esea
tedi
nth
is
way.
We u
se t
he no
tion of
“po
tent
ial”.Le t po
tent
iala
t
t
he hi
ghest po
sit
ion [50]be
50; the nei
ghbor po s
iti
ons
[49 ] and [48 ] each has
po
tenti
al49,...,thelowest
po
sit
ion [- 49]and [- 50]
each ha
s po
ten
til 0. Then,
a
F
ig.3
.1
t
he t
ota
l po
ten
tia
l a
t t
he
136 Ma
thema
tic
alOl
ymp
iadi
nCh
ina

beg
inn
ingi
s

S = 101 ×50 + (
100 +99)×49 + … + (
2 +1)×0.

Anda
ttheendo
ftr
ans
iti
ons,t
het
ota
lpo
ten
tia
lis

T = 51 ×50 + (
51 +51)×49 + … + (
51 +51)×0.

Thed
iff
er sS -T = 42,
encei 925.
Mathematical Olympiad in China (2011–2014) Downloaded from www.worldscientific.com

on,t
by NATIONAL UNIVERSITY OF SINGAPORE on 05/06/18. For personal use only.

S
incea
fte
reacht
ran
sit
i het
ota
lpo
ten
tia
lchange
sat
mo
st1,s
oatl
eat42,
s 925t
ran
sit
ionsa
reneeded.
Int
hefo
ll ng,weshowt
owi hatwha
teve
rbet
heorde
rof
s
eat
ing,t
her
eisa
lway
s a way of no mor
ethan 42,925
t
rans
iti
onss
ucht
hateach pe
rson po
sse
sss51ca
e rds.Toshow
t
his,weg
ivetwol
emma
s.
tc,a0 ,a1 ,...,an-1 bei
Lemma1.Le ntege
rswi
tht
hei
rsum
ze
ro,andc ≥ 0,a0 ≤ a1 ≤ … ≤ an-1 .
fn + 1pe
I rsonsdeno
t c],[
edby [ a0],[
a1],...,[
an-1],
po
sse
ssN + c,N + a0 ,N + a1 , ...and N + an-1 ca
rds,
r
espec
tive
ly,whe
reN i
sapo
sit
ivei
nt r,s
ege ucht
hatN +a0 >
0.Le
tthepe
rsonss
tandon0,1,...,n oft
henumbe
rax
is,
s
ucht
hat[
c]s
tand
satn.Thent
her
eisawayofno mor
ethan

∑ ons,s
n-1
cn + i
ai t
i=0
rans
iti ucht
hateach pe
rson po
sse
sssN
e

ca
rds.
P
roo
fofLemma1.Suppo
set
hatan-1 ≥ … ≥as >0 ≥as-1 ≥ …

≥a0 ,
theni
nduc
tiononM =an-1 + … +as .
IfM =0,
t c]
hen[
pa
ssscca
e rd o[
st ai],
sucht
hat[
ai]ob
tans-aica
i rds(
0 ≤i ≤
s -1).Suppo
set
hatpe
rson [
ai]s
tand
satxi(
0 ≤i ≤ n -1),
henx0 ,x1 ,...,xn-1 i
t sape
rmu
tat
ionof0,1,...,n - 1.
s,a ca
Thu rd t
hat pa
sse
sfr c]t
om [ o [
ai]need
s n - xi
t
rans
iti
ons.Sot
het
ota
ltr
ans
iti
onsneededa
re
Ch
inaNa
tiona
lTe
amS
ele
cti
onTe
st 137

s-1 s-1 s-1

∑ (n -xi)(-ai)=cn + ∑ xiai ≤cn + ∑iai


i=0 i=0 i=0

n-1

=c
n+ ∑ia .
i=0
i

Nows
uppo
set
hatt
heconc
lus
ioni
str
uefori
ntege
rssma
lle
r
hanM .Cons
t ide
rtheca
seo
fin
t rM .Suppo
ege set
hat
Mathematical Olympiad in China (2011–2014) Downloaded from www.worldscientific.com

an-1 = … = an-s > an-s-1 ,al > al-1 = … = a0 .


by NATIONAL UNIVERSITY OF SINGAPORE on 05/06/18. For personal use only.

Thent
her
eisape
rs n[
oni an-1],...,[
an-s ]andape
rsoni
n
[ al-1]s
a0],...,[ ucht
hatt
hei
rdi
stancei
snomor
ethann -s
-l +1.We mays
uppo
set
hatt
hed
ist tween [
ancebe an-s ]and
[
al-1]
isnomor
ethann -s -l +1,
thenl
etpe
rson [
an-s ]pa
ss
o[
rdut
aca al-1]bynomor
ethann -s -l+1t
ran
sit
ion t,
s.Nex
byi
nduc
tiononcand

an-1 ≥ … ≥ an-s+1 > an-s -1 ≥ an-s-1


≥ … ≥ al ≥ al-1 +1 > al-2 ≥ … ≥ a0 ,

wes
eet
hatbyt
aki
ngnomor
ethan

L =cn + (
n -1)
an-1 + … + (
n -s +1)
an-s+1 +
(
n -s)(
an-s -1)+ (
n -s -1)
an-s-1 + … +
al + (
l l -1)(
al-1 +1)+ (
l -2)
al-2 + … +0·a0

t
rans
itons,each pe
i rson po
sse
sss N ca
e rds.Add
iti
on oft
he
t
rans
iti
onso rdu,weknowt
fca hatt
her
ear
enomor
ethan
n-1

n -s -l +1)=cn + ∑i
lL + ( ai
i=0

t
rans
iti
ons.TheproofofLemma1i
scomp
let
ed.
Lemma2.Foranypermut
ati f[-50],[-49],...,[
ono 49],
[
50]onac
irc
le,t
her
ealway
sex
ist
sa pe
rson [
c],deno
tet
he
l
inepa
ssi
ngt
hrough [
c]andt
heo
rii
gnbyl,s
ucht
hatt
hes
umo
f
138 Ma
thema
tic
alOl
ymp
iadi
nCh
ina

e
achs
i fl (
deo inc
ludec )i
ntheb
racke
tsha
sthes
ames
i fc.
gno
Proo
fofLemma2.Le
tthepe
rmu
tat
ionont
hec
irc
lebe[
a1],
a2],...,[
[ a101]c
lockwi
se.Thent
her
eisad
irec
tedd
iame
terl
oft
hec
irc
les
ucht
hat[
a1],[
a2],...,[
a50]a
reonones
ideof
land [
a51],[
a52],...,[
a101]a
reont
heo
the
rsi
de.
f∑i=1ai = 0, akec =a51;e f∑i=1ai ≠0,
e,i
50 50
I thent ls then
Mathematical Olympiad in China (2011–2014) Downloaded from www.worldscientific.com

t
hes
umso
fea
chs
i flhav
deo edi
ffe
ren
tsi
gn.I
fwer
ota
tel180
°
by NATIONAL UNIVERSITY OF SINGAPORE on 05/06/18. For personal use only.

c
lockwi
se,wes ha
eet tt
hes
um o
feachs
idechange
ssi
gn.So
t
her
eex
issa[
t c]wh
ich mee
tingt
her
equ
iremen
tofLemma2.
Take[
c]i
nLemma2.Suppo
set
hatc ≥0(
els
echangeeach
[
ai]by [-ai]andchangea
llt
hed
irec
tionso
fthea
r tc =
c.Le
c1 +c2 ,c1 ,c2 ≥ 0,
sucht
hatt umofc1andt
hes henumbe
rson
ones
ideof(
noti
ncudec)
l lisze
ro.Deno
te50numbe
rsont
his
s
idebya0 ≤a1 ≤ … ≤a49 ,anddeno
te50numbe
rsont
heo
the
r
s
idebyb0 ≤b1 ≤ … ≤b49 .Thent
hes fc2andb0 ,b1 ,...,
umo
b49i
sal
soze
ro.Us
i oc1 ,a0 ,a1 ,...,a49 andc2 ,
ngLemma1t
b0 ,b1 , ...,b49 ,r
espec
tive
ly,weob
tai
ntha
tthet
ran
sit
ion
t
ime
sar
enomor
ethan
49 49

c1 + ∑jaj +50
L = 50 c2 + ∑j
bj .
j=0 j=0

S
incec,a0 ,...,a49 ,
b0 ,...,
b49isape
rmu
tat
ionof-50,
-49,...,50,byt heorde ri
nequal
ity
49

c + ∑j(
L = 50 aj +bj )
j=0

49

≤50 + ∑j(2j -50 +2j -49)


2

j=0

=42925.

ngup,t
Summi hel
eas
tpo
sit
ivei
nt rk = 42925.
ege
Ch
inaNa
tiona
lTe
amS
ele
cti
onTe
st 139

S
econdDay
8:
00 12:
30,Ma
rch25,2013

4 Le ime,aandk bepo
tp beapr sit
ivei
ntege
rs,s
ati
sfi
yng
p <k <2p .Provet
a a
ha
tthe
reexi
stsposi
tivei
nt rn,n
ege
2a
<p s ucht tCn ≡ n ≡ k(
ha k
modp ).(
a
pos
ed by Yu
Mathematical Olympiad in China (2011–2014) Downloaded from www.worldscientific.com

Hongb
ing)
by NATIONAL UNIVERSITY OF SINGAPORE on 05/06/18. For personal use only.

So
lut
ion.Wearetoproveanext
endedprob
lem.
Le ime,
tp beapr aandkbepo
sit
ivei
ntege
rs,s
ati
sfi
yngpa
<k < 2p .Provet
a
hatforany non
-nega
tivei
nt rb,t
ege her
e
ex
ist
spo
sit
ivei
nt rn,n < pa+bs
ege ucht
hatn ≡k(
modpa )and
Cn ≡k(
modpb ).
k

fb =0,pb =1,
I taken =k -pa .Wepr
ovebyi
nduc
tion.
Suppo
set
hatt
heconc
lus
ioni
str
uefori
nt rb ≥ 0.Tha
ege tis,
t
her
eex
ist
sapo
sit
ivei
nt rn <pa+bn ≡k(
ege modpa ),andCn
k

k(
modpb ).
t1 ≤t ≤ p -1.Cons
Le ide
r
k-1
n +tpa+b -i
Cn ∏ .
k a+b =
+t
p
i=0 k -i

m ,
rm ,ltP (
m )= pvp ,r(
m )=
(
m)
Fori
ntege e whe
re
P(
m)
vp (
m)i
sthenumbe
rofpi
nthes
tanda
rdf
act
ori
zat
i fm .
ono
S
incek -i < 2pa ≤ pa+1 ,wes
eet
hatvp (
k -i)≤ a and
n -i ≡k -i(
modpa ).Hence,

P(
k -i)|n +tpa+b -i.

Con
sequen
tly,

n -i a+b-vp (
k-i)
+tp
P(
k -i)
k-1

C = ∏ .
k

r(
k -i)
a+b
n+t
p
i=0
140 Ma
thema
tic
alOl
ymp
iadi
nCh
ina

fk -i ≠pa ,
I thenvp (
k -i)≤a -1anda +b -vp (
k -i)
≥b +1.fk -i = pa ,
I thenvp (
k -i)= a.Hence,
k-1
n -i æ k-1 n -i
∏ ç ∏
ö
Cn
k a+b
≡ + ÷ ·t
p
b
+t
p
( )(
i=0 P k -ir k -i
) ç 0≤i≤k-1P(
k -i)r(k -i)÷
è i≠k-pa ø

r(n -i)ö÷ · b (
k-1


æ
≡ Cn + ç
k
tp modpb+1).
ç 0≤i≤k-1 k -i)÷
(
Mathematical Olympiad in China (2011–2014) Downloaded from www.worldscientific.com

r
by NATIONAL UNIVERSITY OF SINGAPORE on 05/06/18. For personal use only.

è i≠k-pa ø

Th
isi
sbecau
set
ha fk -i ≠ pa ,
ti t n -i)- (
henpa | ( k -i).
So

vp (
n -i)=vp (
k -i).

r(
n -i)
∏ o p, we s
k-1
S
ince is copr
ime t ee t
hat
r(
k -i)
0≤i≤k-1
i≠k-pa

Cn
k a+b (
0 ≤t ≤p -1)goe
sthr
ought
hefo
llowi
ngr
ema
inde
rsof
+t
p

modu
larpb+1 :

Cn p ,j = 0,1,...,p -1.
k b
+j

S
inceCn ≡k(
modpb ),
the
reex
issj(
t 0 ≤j ≤ p -1),
such
k

t
hatCn p ≡ k(
modpb+1).Tha
tis,t
her
eex
isst(
t 0 ≤t ≤
k b
+j

p -1),
sucht
hatCn ≡k(
modpb+1).Le
tN =n +tpa+b .Then
k a+b
+t
p

N < pa+b+1 ,N ≡ n ≡k(


modpa )andCkN ≡k(
modpb+1).

Theex
tendedpr
obl
emi
spr
ovedbyi
nduc
tion.

5 tn ≥ 2anda1 ,a2 ,...,an ,b1 ,b2 ,...,bn benon


Le -
nega
tivei
ntege
rs.Provet
hat
n n n
æç n ö÷n-1 æç 1 ö æ1 ö2
∑ ai2 ÷ + ç ∑bi ÷ ≥ ∏ (a +bi )
1
2 2 n
.
èn -1ø è n i=1 ø è n i=1 ø
i
i=1

(
pos
edbyLeng Gang
song)
Ch
inaNa
tiona
lTe
amS
ele
cti
onTe
st 141

æç n ö÷n-1 ,
So
lut
ion.Denot
eλ =
èn -1ø
n ≥ 2.Obv
iou
sly,
λ > 1.

Forg
iveni ∈ {
1, ...,n},f
i 2
xp = ak 2
+bk fork = 1,
2,...,nandf
ixaj andbj (
j ≠i).Thent
hel
eft
-hands
ideof
λ( 1
① = p -bi + ∑j≠iaj )+ 2 (
bi + ∑j≠ibj )
2 2 2
i
saquad
rat
ic
n n
λ 1
fbi ,
bi ∈ [
0, p ],wi
Mathematical Olympiad in China (2011–2014) Downloaded from www.worldscientific.com

f
unc
tiono thl
ead
ingc
oef
fic
ien
t- + <
n n2
by NATIONAL UNIVERSITY OF SINGAPORE on 05/06/18. For personal use only.

s,i
0.Thu tsmi
nimumi
stakena
ttheendpo
ins,t
t ha s,
ti bi = 0o
r
ai = 0.
So,wecans
uppo
set
hataibi = 0,
i = 1,2,...,n.
Ca
se1.Eachai =0,
thenbyt
hemeanva
luei
nequa
liy,we
t
have
n n
æç 1 ö2
∑ ∏b
2
bi ÷ ≥ n
.
è n i=1 ø
i
i=1

Ca
se2.Eachbi =0,
thenbyt
hemeanva
luei
nequa
liy,we
t
have
n n n
æ1 1
λ ç ∑ ai ÷ ≥ ∑ ai2 ≥ ∏a
2
2ö n
.
è n i=1
i
ø n i=1 i=1

Ca
se3.Wemays
uppo
set
hatb1 = … =bk = 0,ak+1 = … =
an = 0,1 ≤k < n.
ta1a2 …ak =ak ,bk+1 …bn =bn-k ,a,b ≥0.Thenbyt
Le he
meanva
luei
nequa
liy,wehave
t

1 +a2 + … +ak ≥ k
a2 a2 ,bk+1 + … +bn ≥ (
n -k)
2 2
b.

I
tsu
ffi
cest
oprovet
hat

λk 2 (n -k)2 n-k)
2(
b2 ≥ an ·b n .
2
k
a + ②
n n 2

Byt
hemeanva
luei
nequa
liy,wes
t eet
hat
Thel
eft
-hands
ideo
f
142 Ma
thema
tic
alOl
ymp
iadi
nCh
ina

λ 2 … λ 2 n -k 2 … n -k 2
② = a + + a + b + + b
n n n2 n2
􀮩􀪁􀪁􀪁􀪁􀮪􀪁􀪁􀪁􀪁􀮫 􀮩
􀪁􀪁􀪁􀪁􀪁􀮪􀪁􀪁􀪁􀪁􀪁 􀮫
kt
erms n-kt
erms
n-k
æn -k ö÷ 2(
n-k)
≥λ a · ç ·b
k 2
k n
n n n
.
è n ø
n-k
æçn -k ö÷
So,i ≥ 1,t s,t
k n
tsu
ffi
cest
oshowt
hatλn hati o
è n ø
Mathematical Olympiad in China (2011–2014) Downloaded from www.worldscientific.com

æ n ö÷n-k
s
how ç ≤λ .
k
by NATIONAL UNIVERSITY OF SINGAPORE on 05/06/18. For personal use only.

èn -k ø
Inf
act,

n · n ·…· n · · ·…·
1 1 1
n -k n -k n -k 􀮩􀪁􀪁􀪁􀮪􀪁􀪁 􀪁􀮫
􀮩
􀪁􀪁􀪁􀪁􀪁 􀪁􀮪􀪁􀪁􀪁􀪁􀪁􀪁􀮫 nk-nt
erms
n-kt
erms

æn + (
nk -n)÷önk-k

è nk -k
ç
ø
( )
æç n ö÷ n-1 k
=λ .
k
=
èn -1ø

6 I
na p
lane wi
thc
art
esi
anc
oor
dina
tes,l
etP andQ b
etwo
r
egi
onso
fc onv
expolygon (
inc
ludi
ngboundar
yandi n
teri
or)
whos
eve
rti
cesarea
llinte
gerpoi
nts( e.,t
i. heircoordinat
es
a
rea
lli
ntege
rs)andT = P ∩ Q .Provet
ha fT i
ti sno
t
emp
tyanddoe
sno
tcon
tai
nin
tege
rpo
int,t
henTi
sanon
-
degene
rat
econvexquadr
ila
ter
al.(
poedby QuZhenhua)
s
So
lut
ion.S
incet
henon
-emp
tyi
nte
rsec
tionT oftwoconvex
c
los
ed po
lygon
sisa c
los
ed convex po
lygon or degene
rat
ed
po
lygon,t
herearethr
eeposs
ibl
ecase
s.
(
1)T isa po
int.ThenT mus
tbet heve
rtexofP orQ ,
con
trad
ict
ingt
hef
actt
hatT con
tai
nsnoi
ntege
rpo
int.
(
2)Ti
sas t.ThenT mu
egmen stbet
hei
nte
rsec
tionofan
edgeofP andanedgeofQ ,wh
ichcon
tai
nst
heve
rtexofP or
Q ,wh
ichi
sacon
trad
ict
ion.
Ch
inaNa
tiona
lTe
amS
ele
cti
onTe
st 143

(
3)Ti
sac
los
edconvexpo
lygon.
So,i
trema
inst
obeshownt
hatTi
saquadr
ila
ter
al.
Fi
rst,weno
tet
ha fT ha
ti stwoad
jacen
tedge
sont
heedge
s
ofP (
orQ ),
thent
hecommonve
rtexo
fth
istwoedge
smu
stbe
t
heve
rtexofP (
orQ ),wh
ichi
sacon
trad
ict
i s,t
on.Thu he
bounda
ryofT i
sformeda
lte
rna
tel
ybyapa
r fanedgeofP
to
Mathematical Olympiad in China (2011–2014) Downloaded from www.worldscientific.com

andt
henapa
rtofanedgeofQ ,andeachve
rtexofT i
sthe
by NATIONAL UNIVERSITY OF SINGAPORE on 05/06/18. For personal use only.

i
nte
rse
ctpo
into
fedge
so s,t
fP andQ .Thu henumbe
rofedge
s
ofTi
seven.
Wes
eet
ha fanedgeeo
ti fPi
nte
rsec
t fQ ,
sanedgefo then
e mu
sti
nte
rsec
tano
the
redgeofQ ,o
the
rwi
seT wi
llcon
tai
nan
i
ntege
rpo
int.
In t
he f
oll ng, we show by con
owi trad
ict
ion t
hatt
he
numbe
rofedge
sofT canbe6ormor
e.
fT ha
I sedge
snol
es han6,t
st hens
uppo
seP con
tansk
i
i
ntege
rpo
int
sexcep
ttheve
rti
ce fP .
so
Ca
s fk =0,
e1.I thenPi
sane
lemen
tin
tege
rtr
iang
le,ora
pa
ral
lelog
ram wi t
ha ea1.So,P canbel
r oca
tedbetweentwo
pa
ral
lelli
nesl1 ,l2 .Andt
her
eisnoi n
tege
rpo n
iti
nt heopen
doma
i tweenl1 andl2 .Atl
nΩbe eas
tthr sofP a
eeedge ret
he
sofT ,becau
edge seT ha
satl
eas
tsi
xedge
s.
(
a)In ca
s f P be
eo ing △ABC
(
SeeF
ig .1),DE ,FG and HI a
.6 re
sofQ .D ,G andE mayco
edge inc
ide
wi
thF ,H andI,r
espec
tive
ly.Li
nes
FG ,HI,l1 andl2 form a convex
quadr
ila
tera
l.Sinc
elneDE doe
i sno t F
i .6
.1
g
int
ers
ects tBC ,wes
egmen eethatt
he
i
nte
rse
ctpo
into
flneDE andFG orofl
i ineDE andHIi
sinΩ.
s,Q ha
Thu sin
tege
rve
rt nΩ,acon
exi trad
ict
ion.
144 Ma
thema
tic
alOl
ymp
iadi
nCh
ina

(
b)Inca
seofP be
ingapa
ral
lel
ogr tAD ,
am ▱ABCD .Le
AB andBC bet
hreeedge
so fP (s
eeFi
g .2).Thei
.6 n t
ers
ect
ion
po
intoflneEF andHGl
i oca
tesinΩ.LetAB ,BC andCD be
t
hreeedge
so fP (s
eeFi.6
g .3),andt
her
eisnoedgeofQ on
AD .S
imilartotheca
seo f(a),wecanseet
heinte
rse
cti
on
po
intofl
ineEF and HG oro
flneEF andIJ l
i oca
te n Ω,
si
Mathematical Olympiad in China (2011–2014) Downloaded from www.worldscientific.com

wh
ichi
sacon
trad
ict
ion.
by NATIONAL UNIVERSITY OF SINGAPORE on 05/06/18. For personal use only.

F
ig.6
.2 F
ig.6
.3

Ca
se 2.k ≥ 1.Cons
ide
r
i
ntege
rpo
intX onP o
the
rthan
t
he ve
rti
ces.S
ince X ∉ T ,
t
her
eex
issanedge MN o
t fT
s
ucht
hatT andX a
res
epa
rat
ed
F
i .6
.4
bylneMN (
i deno
tebyl )(s
ee g

Fi
g.6.4).Thus,MN isa part
ofbounda
ryofQ .M i heedgeAB ofP ,N i
sont sont
heedge
CD ofP .A ,C andX a
reont
hes
ames
ideofl (A andC may
co
inc
ide,bu
tB andD dono
tbyt
hehypo
the
sist
hatT ha
sat
l
eas
tsi s).Thu
xedge s,t
her
eisano
the
rve
rt fT onAB ,
exU o
andt
her
eisano
the
rve
rtexV ofT onCD .
Deno
tet
heconvexhu
l i
lofponsX andve
t rti
ce fP be
so low
lbyP'.ThenP'andP co
inc
idebe
lowBD ,andt
hepa
rtofP'
upBDi s△BXD .ComparngT' = P' ∩ Q wi
i thT ,weseet
hat
,
T' ⊂ T andt
hebounda
ryo fT'i
stheboundar fT wi
yo hMN ,
t
Ch
inaNa
tiona
lTe
amS
ele
cti
onTe
st 145

MU andNV r
epl
acedbyM'N',M'U'andN'V',r
espec
tive
ly.
Tha
tis,T' and T havet
hes
ame numbe s,andt
rofedge he
numbe
rofi
ntege
rpo
int
sofP' o
the
rthanve
rti
cesi
sle
sst
han
t
ha fP .By af
to ini
te procedu
rel
iket
his,wecan ob
tai
na
convexpo
lygonwi
thnoi
nte
riori
ntege
rpo
int.By Ca
se1,i
tis
impo
ssi
ble.
Mathematical Olympiad in China (2011–2014) Downloaded from www.worldscientific.com
by NATIONAL UNIVERSITY OF SINGAPORE on 05/06/18. For personal use only.

2014 (
Nani
jng,J
iangsu)

F
irs
tDay
8:
00 12:
30,Ma
rch23,2014

1 t O be t
Le he c
ircumcen
terof △ABC and H A be t
he
pro
jec
tionofA on
toBC .Theex
tens
i fAO i
ono nte
rsec
ts
t
hec
ircumc
irc
l f △BOC a
eo tA'.The pro
jec
tionsofA'
on
toAB andAC a
reD andE ,r
espec
tive
l tO A bet
y.Le he
c
ircumcen
terof △DH AE .De
fine H B ,OB ,H C andOC
s
imi
lar
ly.
Provethat H AO A , H BOB and
H COC ar
e concu rrent. (posed
byZhangS
i i)
hu
So
lut
ion.Le
t T be t
he s
ymme
try
poi
nt o r BC , F be t
f A ove he
pro
jec
ti fA' on
ono t ,
oBC and M be
t
hepr
ojec
tionofT on
toAC .
S
i eAC =CT ,weha
nc ve ∠TCM =
π
2∠TAM .S
ince ∠TAM = -
2
F
ig.1
.1
146 Ma
thema
tic
alOl
ymp
iadi
nCh
ina

∠ACB = ∠OAB ,wehave

∠TCM = 2∠OAB = ∠A'OB = ∠A'CF ,

and

∠TCH A = ∠A'CF + ∠A'CT = ∠TCM + ∠A'CT = ∠A'CE .

Be
cau
seoft
he f
actt
hat ∠CH AT , ∠CMT , ∠CEA',
Mathematical Olympiad in China (2011–2014) Downloaded from www.worldscientific.com

∠CFA'a
rer
ightang
les,wehave,
by NATIONAL UNIVERSITY OF SINGAPORE on 05/06/18. For personal use only.

CH A CH A ·CT c
os∠TCH A cos∠A'CE
= = =
CM CT CM cos∠TCM cos∠A'CF
CE ·CA' CE ,
= =
CA' CF CF

e.,CH A ·CF = CM ·CE ,s


i. o H A ,F ,M andE a
reont
he
s
amec
irc
leω1 .
S
imi
lar
ly,l
etN bet
hepro
jec
tionofT on
toAB ,
thenH A ,
F ,N and D a
reont
hes
amec
irc
leω2 .S
ince A'FH AT and
A'EMT a
rebo
thr
ightt
rapezo
id,t
hepe
rpend
icu
larb
isec
torof
t
hes
egmen
ts H AF and EM mee
tatt
he mi
dpo
int K oft
he
s
egmentA'T ,i.
e.,K i
st hec
enterofc
irceω1 andKF i
l sthe
rad
iuso
fcir
cleω1 .S
imi
lar
ly,K andKF a
rea l
sothecen
terand
t
her
adi
usofc
irc
leω2 ,r
espec
tive
l s,ω1 andω2 a
y.Thu ret
he
ame,D ,N ,F ,H A ,E andM a
s reont
hes
amec
irc
le.SoO Ai
s
t
hemi
dpo
in fA'T ,O AH A ‖AA'.
tK o
π ∠ π,
S
ince ∠H CAO + ∠AH CH B = - ACB + ∠ACB =
2 2
we have AA' ⊥ H BH C ,t
hus O AH A ⊥ H BH C ,t
her
e e,
for
O AH A ,OBH B andOCH C a
llpa
sst
hrought
heor
thocen
tero
f
△H AH BH C .

2 tA1A2 …A101 bear


Le egu
lar101 gon,andco
loreve
ry
ve
rtexr
ed or b
l t N be t
ue.Le he numbe
r of ob
tus
e
Ch
inaNa
tiona
lTe
amS
ele
cti
onTe
st 147

t
riang
less
ati
sfi
yng t
hefo
llowi
ng cond
itons:Thet
i hree
ve
rtic
esofthetr
iang
lemu s
tbever
tice
softhe101 gon,
boththeve
rti
ceswithacu
teang
leshavethesameco
lor,
andt
heve
rtexwi
thob
tus
eang
leha
sdi
ffe
ren
tco
lor.
(
1)F
indt
hel
arge
stpo
ssi
bleva
l fN .
ueo
(
2)Fi
ndt
henumbe
rof way
stoco
lort
heve
rti
cess
uch
Mathematical Olympiad in China (2011–2014) Downloaded from www.worldscientific.com

t
ha imum N i
t max sach
ieved. (Two co
lor
ing
sar
e
by NATIONAL UNIVERSITY OF SINGAPORE on 05/06/18. For personal use only.

d
iff
eren
ti omeAi t
ffors heco
lor
sar
edi
ffe
ren
ton
t
hetwoco
lor
ings s.)(
cheme poedby QuZhenhua)
s
So
lut
ion.Def
inngxi
i =0or1depend
sonwhe
t rAii
he sredor
b
lue.Forob
tus
etr
iang
leAi-aAiAi+b (
ver
texAii
stheve
rtexof
t
heob
tus
eang
le,i. a +b ≤ 50),
e., the
set
hreeve
rti
cess
ati
sfy
t
hecond
iti
onsoft
hepr
obl
emi
fandon
lyi
f

(
xi -xi-a )(
xi -xi+b )= 1, ①

o
the
rwi
s l0,he
eequa ret
hes
ubs
cri
ptmodu
le s,
s101.Thu
101

N = ∑ ∑ (x
i=1 (
a,b)
i -xi-a )(
xi -xi+b ).

He
re ∑ (a,b)s
tand
sfort
hes
umma
tionofa
llpo
sit
ivei
nte
gerpa
irs
(
a,b)
sat
isf
yinga +b ≤50.The
rea
re49+48+ … +1 =1225s
uch
po
sit
ivei
ntege
rpa
irs.Expand
ing ① ,wehave
101

N = ∑ ∑ (x
i=1 (
a,b)
i -xi-a )(
xi -xi+b )
101

∑ ∑ (x -xixi-a -xixi+b +xi-axi+b )


2
= i
i=1 (
a,b)
101 101 50

=1225 ∑ xi + ∑ ∑ (
k -1 -2(
50 -k))
2
xixi+k
i=1 i=1 k=1
101 50

=1225
n+ ∑ ∑ (3k -101)xx
i=1 k=1
i i+k . ②

He
reni
sthenumbe
roft
heb
lueve
rti
ces.Foranytwove
rtexe
s
148 Ma
thema
tic
alOl
ymp
iadi
nCh
ina

Ai andAj ,1 ≤i ≤j ≤ 101,
let

d(
Ai ,Aj )= d(
Aj ,Ai)= mi
n{j -i,101 -j +i}.

tB ⊆ {
Le A1 ,A2 ,...,A101}bet
hes
eto
fal
lbl
ueve
rti
ces.
Then ② canbewr
itt
ena
s

N = 1225n -101Cn +3 ∑ d(
P ,Q ), ③
2
Mathematical Olympiad in China (2011–2014) Downloaded from www.worldscientific.com

{
P ,Q}⊆B
by NATIONAL UNIVERSITY OF SINGAPORE on 05/06/18. For personal use only.

whe
re{
P,Q}pa
sst
hrougha
llt
hetwo
-el
emen
tsub
set
sofB.
Wi
thou
tlo
sso
fgene
ral
ity,wea
ssumet
hatni
sev
en.Ot
herwi
se,
chang
ethec
olo
rofa
llv
ert
ics,t
e heva
lu fN doe
eo sno
tchang
e.
Wr
it t,0 ≤t ≤ 50,f
en = 2 romonepo
int,r
enumb
era
llt
heb
lue
v
ert
icsbyP1,P2,...,P2t c
e lockwi
se.Then
t t t-1
1
{
∑ d(
P ,Q}⊆B
P ,Q )= ∑
i=1
Pi ,Pi+t)+ ∑ ∑ (
d(
2 i=1 j=1
d(
Pi ,Pi+j )+

d(
Pi+j ,Pi+t)+d(
Pi+t ,Pi-j )+d(
Pi-j ,Pi))
101 (
≤50
t+ tt -1).
2

He
ret
hes
ubs
cri
ptofPi modu
le t,and weu
s2 set
hei
nequa
lit
y
d(
Pi ,Pi+t)≤ 50and

d(
Pi ,Pi+j )+d(
Pi+j ,Pi+t)+d(
Pi+t ,Pi-j )+d(
Pi-j ,Pi)
≤101.

Comb
inng ③ and ④ ,wehave
i

101
N ≤ 1225n -101Cn +3ç5t + t(
0 t -1)÷
2 æ ö
è 2 ø

101 2 5099
=- t + t.
2 2

Ther
ight
-hands
ideof ⑥ a
tta
insi
tsmax
imum va
luewhen
Ch
inaNa
tiona
lTe
amS
ele
cti
onTe
st 149

t = 25,
t sN ≤ 32175.
hu
Ne
xt, c
ons
ide
rth
e n
ece
ssa
ry and s
uff
ici
entc
ond
it
ion o
f
N =3
217
5.F
irt,t = 2
s 5 me
anst
hatt
her
ear
e50b
luev
ert
ice
s.
S
econd
ly,f
or1 ≤i ≤t,weha
ved(
Pi,Pi+t)= 5
0.Whend(
Pi,
Pi+t)= 5
0,e
qua
li
tyo
f⑤ ho
lds.The
ref
ore,t
henumb
ero
fwa
yst
o
choo
se5
0bl
uev
ert
ice
ssu
cht
hatN a
chi
eve
sma
ximumi
sequ
alt
othe
Mathematical Olympiad in China (2011–2014) Downloaded from www.worldscientific.com

numbe
rof way
sto choo
se 25 d
iagona
lsf
rom t
hel
ong
est 101
by NATIONAL UNIVERSITY OF SINGAPORE on 05/06/18. For personal use only.

d
iagona
lss
ucht
hatanytwod
iagona
lsdono
thav
ecommonv
ert
ice
s.
Edg
ingAiandAi+50 ,
i =1,2,...,101,wege
tag
raphG .
No
tet
hat50and101a
rer
ela
tive
l ime,s
ypr n(
o1 +50 0 ≤n ≤
100)f
orm acomp
let
ere
sidues
yst
em modu
le101,i.
e.G i
sa
c
irc
lewi
th101edge
s.The
re e,t
for henumbe
ro s(
fway wr
itt
en
sS )t
a oco
lor50ve
rti
cesb
lueandN ach
ieve
smax
imumi
sequa
l
t
othenumbe
rofway
stochoo
se25edge
sofGs
ucht
hatanytwo
oft
hemdono
thavecommonve
rti
ces.Now,f
ixoneedgeeof
75 ,
sC24
G .S
inc
ethenumbe
rofway
stochoo
seedge
shav
ingei no
t
76 ,
sC2 oS = C2
75 + C y,t
5 4 25
hav
ingei s 76 .Simi
lar
l henumbe
rofway
s
t
oha
ve50r
edv
ert
ice
sisa
lsoS,t
hust
henumbe
rofway
stoc
olo
r
t
hev
ert
ice
ssu
cht
hatt
hel
are
gstva
lu fNi
eo sach
iev s2S.
edi
Ins
umma
ry,t
hel
arge
stpo
ssi
bleva
l s32175,t
ueofN i he
numbe
rofway
stoco
l s2S = 2(
ori C275 + C
4
76 )
25
.

3 Show t
hatt
her
eae no 2
r t
upl
es(
x,y)of po
sit
ive
i
ntege
rss
ati
sfi
yngt
heequa
tion

x +1)(
( x +2)…(
x +2014)= (
y +1)(
y +2)…(
y +4028).
(
pos
edbyLiWe
igu)

Proo
f.Forn = 2 ·m (
kisanon
-nega
tivei
nt r,m i
ege sodd),
k

l
etv(
n)= 2k .
We provet
his by con
trad
ict
ion.As
sume (
x,y)i
s one
2 Ma
thema
tic
alOl
ymp
iadi
nCh
ina

Pa
rtI Sho
rt-An
swe
r Qu
est
i s(
on Qu
est
i s1 8,e
on igh
tma
rks
e
ach)
1 Ther ff(
angeo x)= x -5 - 24 -3x i
s .
So
lut
ion.I
tisea
syt
oseet
hatf(
x)i
sinc
rea
singoni
tsdoma
in
[
5,8].The
ref
ore,i
tsr s [-3, 3].
angei
150 Ma
thema
tic
alOl
ymp
iadi
nCh
ina
2 Themi
nimumo acos2x -3)
fy = ( sinxi
s -3.Thent
he
pr
a
os
in
tg
i
veo
eifr
nt
ee
ga
ln
e u
rs
om
lb
u
te
irai
s
onoft
heequa
t.
ion.Le
t
So
lut
ion.Le
tsi
nxv(
xt+.T
= i)hee
= x
mp
ar
e
xss
i
{oni
v(xst
he
+j )nc
}. hangedt
o
1≤j≤2014
g( t2 +a -3)
t)= (-a t,or
f1 ≤j ≤ 2014,j ≠i,
I t
hen
( )
g t = -a a -3)
t3 + ( t.
v( x +j)=v( x +i + (j -i))=v(
j -i),
F
s
orom -a a -3)
t3 + ( t ≥ -3,wege
t

t(
a t2 -1)-3( )≥ 0,
t -1-

Mathematical Olympiad in China (2011–2014) Downloaded from www.worldscientific.com

-(
v  x +j) =v (( j -1)!)≤v(
2014 j)!·( 2013!).
by NATIONAL UNIVERSITY OF SINGAPORE on 05/06/18. For personal use only.

 1≤j≤2014,j≠(
t -1)(-
i a t +1)-3)≥ 0.
t(

∏-j1=1≤(x0+,j ∏j=1t
)=ave -a(( )) , rf4028!,t
2014 4028
S
s
ii
n
nc
cet
e weh y j1
t++ is- 3u
am≤l
t0
ipo
l
eo hus

a( t2 +t)≥ -3. 4028!  ①


x +i ≥v( x +i)≥v  ! >2 .
1007
2013 
Whent =0,-1, expr e ssion ① a
lwayshol d s;when0 <t ≤

1,wehave0 <t2 +t ≤ 2;andwhen -1 <t < 0,∏


e,x > 21006 .So ( y +4028) > 1 ( y2+j)=
4028 4028
Therefor
- j=1≤t +
4
∏j=1(x +j, )> 21006·2014 .Wehavey +4028 > 2503 ,y > 2502 .
2014

3
t < 0.The re
fore - ≤a ≤ 1 2.
2 1( 1
1 ≤i ≤n). fx = ∑i=1xi ,y =
n
Lemma.Le t0 ≤ xi < I
2 n

3 2ma
Thx1≤u
en m {
nbx
ei}
2
ro ,
ft
ih
n
te
enra
lpo
ins(
t e.,t
i. hepo
int
swho
sex
-
i≤
g
andy-coordi
nate
sareb n
othintege
rs1 )wi
thi
nt hearea (
not
1 -x ≥  ∏ ( 1 -xi) ≥ 1 -x -y.
n

inc
ludi
ngt heboundariy
=1)encl
osedbyt herightbr
ancho f
Pr
hoofo
rbfle
xmm
-a
y.By1a =100 y,t
2 2
ype o
la = thndl
i
eAMnex
GMineqi
us
a
li
t hei
n.
equ
ali
tyon
So
lut
t
hi
eol
n.
eftB
i
s s
ye
as
ymm
yte
ot
pr
y ,
ew
ov .eon
Thel
i n
ne
y e
qe
ua
ldt
it
yoc on
onths
i
ed
le
frt
h
thoe
ld
pa
s,
rto
s
inf
c
e
t
hea
reaa bovet
n
hex-a1xi
s.Supposeliney = k i
nte
rcep
tst
he
 i∏ (1 -xi) n ≥ n n 1 ≥ n ( n
∑i=1 1 -xi ∑i=1 1 +xi +2xi )
2
=1

n
=
n +nx +2∑i=1xi2
n

1

1 +x +y
≥ 1 -x -y.

Thel
emmai
sproved.
Ch
inaNa
tiona
lTe
amS
ele
cti
onTe
st 151

4029
Backtot
hepr ob
lem,letw =x +2015 >21007 ,z =y +
2
>2 ;
502
t
hentheequat
ioni
sequiva
lentto
1
1ö æ 2ö æ 2014ö÷ ö÷ 2014
w · ç ç1 - ÷ ç1 - ÷ … ç1 -
ææ
èè wøè wø è w øø
1
1 ö÷ æç 9 ö æ 40272 ö÷ ö÷ 2014
=z · ç ç1 - 1 - 2 ÷ … ç1 -
2 ææ
.
èè 4 2
z øè 4
z ø è 4z2 ø ø
Mathematical Olympiad in China (2011–2014) Downloaded from www.worldscientific.com
by NATIONAL UNIVERSITY OF SINGAPORE on 05/06/18. For personal use only.

Byt emma,
hel
1
2015ö÷ 1 ö÷ æç 2ö æ 2014ö÷ ö÷ 2014
w ç1 - > w · ç ç1 - 1 - ÷ … ç1 -
æ ææ
è 2w ø èè wøè wø è w øø
2015 2·20142 ö÷
> w ç1 -
æ
-
è 2w w2 ø
2015÷ö 1
> w ç1 -
æ
- .
è 2w ø 8

1 ö÷ æç 2 ö÷ …
e,t
ææ
The
ref
or he de
cima
lofw · ç ç1 - 1 -
èè wøè wø
1

2014ö÷ ö÷ 2014 æ 3 1ö
oç , ÷ .
æ
ç1 - be
long
st
è w øø è 8 2ø
Ont
heo
t rhand,byt
he hel
emma

12 +32 + … +40272 ö÷
z2 ç1 -
æ
è 4z2 ·2014 ø
1
1 ö÷ æç 9 ö æ 40272 ö÷ ö÷ 2014
>z · ç ç1 - 1 - 2 ÷ … ç1 -
2 ææ
èè 4 2
z øè 4
z ø è 4z2 ø ø
12 +32 + … +40272 2·40274 ö÷ ,
>z ç1 -
2 æ
-
è 4z2 ·2014 (
4z2)2
ø

t
hus
1
4·20142 -1 1 ö÷ æç 9 ö æ
1 - 2 ÷ … ç1 -
40272 ö÷ ö÷ 2014
>z · ç ç1 -
ææ
z2 - 2

12 èè 4 2
z øè 4
z ø è 4z2 ø ø
2 4·20142 -1 40274
>z - -
12 8z2
2 4·20142 -1 1
>z - - .
12 8
152 Ma
thema
tic
alOl
ymp
iadi
nCh
ina

4·20142 -1
S
incez2 - isani
nt r,s
ege othede
cima
lpa
rtof
12
1

1 öæ 9 ö æ 40272 ö÷ ö÷ 2014 æ7
z · ç ç1- 2 ÷ ç1- 2 ÷ … ç1- o ç ,1÷ ,
2
ææ ö
be
long
st
èè 4
z øè 4
z ø è 4 2
z øø è8 ø
wh
ichi
sacon
trad
ict
ion.
The
ref
ore,t
her
ear
eno2 t
upl
es(
x,y)o
fpo
sit
ivei
nte
ger
s
s
ati
sfi
yng
Mathematical Olympiad in China (2011–2014) Downloaded from www.worldscientific.com
by NATIONAL UNIVERSITY OF SINGAPORE on 05/06/18. For personal use only.

2014 4028

∏ (x +i)=
j=1
∏ (y +i).
j=1

S
econdDay
8:
00 12:
30,Ma
rch24,2014

4 Gi
ven an odd i
nt rk > 3.Prove t
ege hatt
her
e ex
ist
i
nfi
nit
ely many po
sit
ivei
ntege
rsn,s
ucht
hatt
her
ear
e
n2 +1,
twopo
sit
ivei
ntege
rsd1 ,d2 t
hata
lld
ivi
de and
2
d1 +d2 =n +k.(
pos
edby YuHongb
ing)
So
lut
ion.Cons
ide
rtheDi
ophan
tineequa
tion

((
k -2)
2
+1)
xy = (
x +y -k)
2
+1, ①

wep
rov
etha
t① ha
sin
fin
ite
ly manypo
sit
iveodds
olu
ti s(
on x,y).
Obv
iou
sly,(
1,1)i
sapo
sit
iveodds
olu
tion,l
e x1 ,y1)=
t(
(
1,1).As
sumet
hat(
xi ,yi)i
sapo
sit
iveodds
olu
tionof ① ,
andxi ≤ yi ,
letxi+1 = yi ,yi+1 = (
k -1)(
k -3)
yi +2
k -xi .
S
ince ① canbewr
itt
ena
s

x2 - ((
k -1)(
k -3) k)
y +2 y -k) +1 = 0,
x +( 2

by Vi
eta
􀆳st em,(
heor xi+1 ,yi+1)i
sal
soaintege
rsolut
ionof①.
Si
nc ,
exi yi andk a
rea llpo sit
iveoddi
nteger ,
s andk ≥ 5,s
o
xi+1i
sapo
sit
iveoddi
nt r,and
ege
Ch
inaNa
tiona
lTe
amS
ele
cti
onTe
st 153

yi+1 = (
k -1)(
k -3) k -xi ≡ -xi ≡ 1(
yi +2 mod2),
yi+1 ≥ 8yi +2
k -xi > yi > 0.

Thu xi+1 ,yi+1)


s,( isapo
sit
iveodds
olu
tionof① ,andxi +
yi <xi+1 +yi+1 .By(x1 ,y1)andtheconst
r uct
ionabove,weget
aseri
eso fpositiveodds olu
ti f① :(
onso xi ,yi),i =1,2,...,
s
uchthatx1 +y1 < x2 +y2 < … .
Mathematical Olympiad in China (2011–2014) Downloaded from www.worldscientific.com

Foranyint rig
ege rea
terthank,xi +yi >k.Le
tn =xi +
by NATIONAL UNIVERSITY OF SINGAPORE on 05/06/18. For personal use only.

yi -k,d1 =xi ,d2 =yi .Thenni sapos


iti
veoddint
eger,and
d1 +d2 =n +k.Si
nce(k -2) 2
seven,wecans
+1i howthatd1 ,
n2 +1,
d2a
rebo
thd
ivi
sor
sof andd1 +d2 =n +k.Thu
ssuchn
2
s
ati
sfi
esa
llt
hecond
iti
ons,t
her
efor
ethe
reex
isti
nfi
nit
ely many
po
sit
iveoddi
ntege
rsns
ati
sfi
yngt
hecond
iti
ons.

5 tnbeag
Le iveni
ntege
rwh
ichi
sgr
eat
ert
han1.Fi
ndt
he
gr
ea te
stconstantλ(n)sucht
hatforanynon
-ze
rocomp
lex
z1 ,z2 ,...,zn ,wehave
n

∑ |z | ≥λ(
n)mi
n{|zk+1 -zk | },
2 2
k
k=1 1≤k≤n

whe
rezn+1 =z1 .(
pos
edbyLeng Gang
song)
So
lut
ion.Le
t

ìïn ,2|n,
ïï4
( )
λ0 n = í n
,o
the
rwi
se,
2 π
ï
ï4c
î os 2n

weproveλ0(
n)i
stheg
rea
tes
tcons
tan
t.
I
fthe
reex
issk(
t 1 ≤k ≤n)s
ucht
hat|zk+1 -zk |= 0,
the
i
nequa
lit
yho
ldsobv
iou
sly.Sowi
thou
tlo
sso
fgene
ral
ity,wecan
a
ssumet
hat
154 Ma
thema
tic
alOl
ymp
iadi
nCh
ina

n{
mi |zk+1 -zk | }= 1.
2

1≤k≤n

Unde
rth
isa
ssump
tion, i
tsu
ffi
cest
o show t
hatt
he

f∑k=1 |zk |2i


sλ0(
n).
n
mi
nimumva
lueo

Whenni
seven.S
ince
n n n
1 1
∑ |zk | = ∑ |zk | +|zk+1 | )≥ ∑ |zk+1 -zk |
(
2 2 2 2
Mathematical Olympiad in China (2011–2014) Downloaded from www.worldscientific.com

2k 4 k=1
by NATIONAL UNIVERSITY OF SINGAPORE on 05/06/18. For personal use only.

k=1 =1

n n
≥ min{ |zk+1 -zk | }= ,
2

4 1≤k≤n 4

æ1
andequa
lit
yho
ldswhen (
z1 ,z2 ,...,zn )= ç ,- 1 ,...,
è2 2

1 , 1 ö÷ , n
f∑k=1|zk|2i
s =λ0(
n).
n
- thu
sthemi
nimumva
lueo
2 2ø 4
t,cons
Nex ide
rthecond
itonwhenni
i sodd.Le
t

zk+1
θk = a
rg ∈[
0,2π),k = 1,2,...,n.
zk

π 3π
Fora
llk =1,2,...,n,
ifθk ≤ orθk ≥ , henby ① ,
t
2 2

|zk | +|zk+1 | =|zk -zk+1 | +2|zk ||zk+1 |co


sθk
2 2 2

≥|zk -zk+1 | ≥ 1. ②
2

æç π ,3πö÷ ,
fθk ∈
I thenbyc
osθk < 0and ② ,
è2 2 ø

1 ≤|zk -zk+1 |2
=|zk | +|zk+1 | -2|zk ||zk+1 |cosθk
2 2

≤(
|zk | +|zk+1 | )(
1 + (-2c
osθk ))
2 2

θk
=(
|zk | +|zk+1 | )·2s
in2
2 2
.
2

The
ref
ore,
Ch
inaNa
tiona
lTe
amS
ele
cti
onTe
st 155

1
2 2
|zk | +|zk+1 | ≥ . ③
θk
2s
in2
2

Nowcon
side
rthef
oll
owi
ngtwocond
iti
ons.
æ π 3πö
(
1)I
ffora
l 1 ≤k ≤ n),θk ∈ ç , ÷ ,by ③
lk(
è2 2 ø
n n n
1 1 1
Mathematical Olympiad in China (2011–2014) Downloaded from www.worldscientific.com

∑ |zk | = ∑ |zk | +|zk+1 | )≥ ∑


( .④
2 2 2

2k 4 k=1
by NATIONAL UNIVERSITY OF SINGAPORE on 05/06/18. For personal use only.

θk
k=1 =1
s
in2
2

zk+1 zn+1
e ∏k=1 = 1,weh
n
S
inc = ave
zk z1
n n
zk+1 ö÷
∑ θk = a
rgç ∏ +2mπ = 2mπ, ⑤
æ
k=1 è k=1 zk ø

whe
remi
sapo
sit
ivei
nt r,andm <n.No
ege tet
ha sodd,s
tni o

mπ (
n -1)
π π
0 <s
in ≤s
in =co
s . ⑥
n 2
n 2
n

1 , π ,3π
tf(
Le x)= x ∈ [4 .I
tisea
syt
oshowt
hat
s
in2x 4]
f(x)isaconvexf unct
ion.By ④ andJ
ens
en􀆳
sInequa
liy,and
t
comb
ining ⑤ and ⑥ ,wehave
n n
1 1 n· 1
∑ |zk | ≥ ∑
2

4k 2θk 4 1 θk ö÷
n ç ∑k=1
n
k=1 =1
s
in si 2æ

2 èn 2ø
n· 1 n· 1
= ≥ =λ0(
n).
4 2 mπ 4 π
s
in cos2
n 2
n

æ π 3πö
(
2)I
fthe
reex
issj(
t 1 ≤j ≤n),
sucht
hatθj ∉ ç , ÷ ,
è2 2 ø
l
et

æç π ,3πö÷ ,
{
I = j θj ∉ j = 1,2,...,n .
}
è2 2 ø
156 Ma
thema
tic
alOl
ymp
iadi
nCh
ina

By ② ,forj ∈I,wehave|zj|2 +|zj+1|2 ≥1;andby ③ ,


forj ∉I,wehave

2 2 1 1
|zj | +|zj+1 | ≥ ≥ .
θ 2
2s
in2 j

The
ref
ore,
Mathematical Olympiad in China (2011–2014) Downloaded from www.worldscientific.com

n
1
by NATIONAL UNIVERSITY OF SINGAPORE on 05/06/18. For personal use only.

∑ |z | ( ∑I (|zj |2 +|zj+1|2)+ ∑I (|zj |2 +|zj+1|2))


2
=
2 j
k
k=1 ∈ j∉

1 1
≥ |I|+ (n -|I|)
2 4
1( n +1
= n +|I|)≥ . ⑦
4 4

No
tet
hat

n +1 n · 1 π n
≥ ⇔c
os2 ≥
4 4 2 π 2
n n +1
c
os
2n
π π n 1
⇔s
in2 = 1 -co
s2 ≤1- = .
2
n 2
n n +1 n +1

Theequa
lit
yho
ldswhenn = 3;whenn ≥ 5,

π æ π ö÷2 π2 · 1 1 ,
s
in2 < ç < <
2
n è2nø 2n n +1 n +1

n +1 n ·
t
hei
nequa
lit
yal
soho
lds.Soforoddi
nt rn ≥3,
ege ≥
4 4
1
.Comb
inng ⑦ ,wehave
i
π
c
os2
2
n
n
n· 1
∑ |z | =λ0(
n).
2

4 2 π
k
k=1
c
os
2n

1 i(
n-1)
rhand,whenzk = ·e ,k = 1,

Ont
heo
the n
π
2co
s
2
n
Ch
inaNa
tiona
lTe
amS
ele
cti
onTe
st 157

2,...,n,wehave|zk -zk+1 |= 1,k = 1,2,...,n,and

∑ ueλ0(
n).
n 2
k=1
|zk | ach
ieve
sit
smi
nimumva
l

ngup,t
Summi heg
rea
tetλ(
s n)i
s

ìïn ,2|n,
ïï4
λ0(
n)= í n
,o
the
rwi
se.
Mathematical Olympiad in China (2011–2014) Downloaded from www.worldscientific.com

2 π
ï
ï4c
os
by NATIONAL UNIVERSITY OF SINGAPORE on 05/06/18. For personal use only.

î 2n

6 Forpo
sit
ivei
nt rk >1,
ege letf(
k)bet
henumbe
rofway
s
off
act
orngk i
i nto produc
tof po
sit
ivei
ntege
rsg
rea
ter
han1. (The orde
t r off
act
orsa
re no
tcoun
teed,f
r or
examp
lef(
12) = 4,a
s12canbef
act
oredi
nthe
sef
our
s:12,2 ×6,3 ×4,2 ×2 ×3.)
way
Pr
ovet
hat,i
fni
sapo
sit
ivei
ntege
rgr
eat
e han1,pi
rt sa
n
pr
imef
acorofn,
t t n)≤ .(
henf( pos
edby YaoYi
jun)
p
So
lut
i eP (
on.Weus n)t
ost
andfort
heb
igge
stpr
imed
ivi
sor
ofn,andde
fineP (
1)=f(
1)=1.Wef
irs
tpr
ovetwol
emma
s.
Lemma1.Forpos
iti
vei
ntege imep|n,wehavef(
rnandpr n)

≤ ∑ d
n
p
f(
d).

Proo
fo ence,af
fLemma1.Forconveni act
ori
ngs
ati
sfi
yng
t
hecond
iti
oni
sca
lledaf
act
ori
ngf
orshor
t.
Foranyf
act
ori fn,wr
ngo iten =n1n2 …nk ,
sincep|n,
so
t
her
eex
issi ∈ {
t 1,...,k},s
ucht
hatp |ni (
ift
her
eismor
e
t uchi,choo
hanones s hem ),wi
e any one oft thou
tlo
ssof
gene
ral
ity,a
ssumet
hati = 1.Mapt
hisf
act
ori
ngt
oaf
act
ori
ng
n,
ofd = d = n2n3 …nk .
n1
Fortwod
iff
eren
tfac
tor
ing
sofn,n = n1n2 …nk andn =
158 Ma
thema
tic
alOl
ymp
iadi
nCh
ina

n
'1n2 …n
' 'l(
whe
rep d
ivi
de '1).I
sn1 andn '1 ,t
fn1 = n hend =
n2 …nk andd =n
'2 …n
'ka
retwod
iff
eren
tfac
tor
i sofd (di
ng sa
n n n
d
ivi
sorof );i
fn1 ≠n
'1,
thend = ≠ ',
=d s
othe
setwo
p n1 n '1

f
act
ori
ngsmapt
oaf
act
ori ' ,r
ngofd andd espec
tive
ly (d and
n
d
'ar
edi
vis
orsof ).Thu
p
s,f(
n)≤ ∑ d f(
nd).Lemma1i
s
Mathematical Olympiad in China (2011–2014) Downloaded from www.worldscientific.com

p
by NATIONAL UNIVERSITY OF SINGAPORE on 05/06/18. For personal use only.

proved.
d
Lemma2.Forpos
iti
vei
nt rn,
ege le n)= ∑d|n ( ),
tg( then
Pd
g(
n)≤ n.
Proo
fofLemma 2.Induc
tion ont
he numbe
rofd
iff
eren
t
pr
imedivi
sor fn = 1,
sofn.I theng(
1)= 1.I
fn = pa ,pi
sa
pr ,
ime then

p -1
a

g(
n)= 1 +1 +p + … +pa-1 = 1 + ≤ 1 +p -1 = n.
a

p -1

As
sumet
hatwhent
henumbe
rofd
iff
eren
tpr
imed
ivi
sor
s
o sk,wehaveg(
fni n)≤ n.Cons
ide
rthes
itua
tionwhenn ha
s
k +1d
iff
eren
tpr
imed
ivi
sor
s.Le
tthepr
imef
act
ori
zat
ionofn
1 …pk pk+1 , ep1 < … < pk < pk+1 ,andwr
a
ben = pa1 ak k+1
whe
r iten
ak 1
= mpk++1 .So
a
pk+1 -1
a k+1
dpik+1
k+1

(
gn )= (
gm )+ ∑ ∑
d|m i=1 pk+1
= (
gm )+ (
σm )
pk+1 -1
,

whe
reσ(
m )s
tand
sfort
hes
um ofpo
sit
ived
ivi
sor
sofm .By
a
ssump
tion,g(
m )≤ m ,s
ince
a a
pk+1 -1 pii -1ö÷pk+1 -1
k+1 k k+1
a +1
( ) = ∏
æç
σm
pk+1 -1 è i=1 pi -1 ø pk+1 -1

pii -1ö÷ ( ak+1


k a +1

∏ pk+1 -1)
æç

èi=1 pi+1 -1 ø
Ch
inaNa
tiona
lTe
amS
ele
cti
onTe
st 159

a +1
k
pii -1ö ak+1
∏ ÷( -1)
æ
≤ ç p
èi=1 pi ø k+1
k

≤ ( ∏ p ) (p
i=1
ai
i
ak+1
k+1 -1)

=n -m ,

og(
s n)≤ n.Lemma2i
sproved.
Mathematical Olympiad in China (2011–2014) Downloaded from www.worldscientific.com

Backt
othe pr
obem,i
l tsu
ffi
ce o prove,f
st or po
sit
ive
by NATIONAL UNIVERSITY OF SINGAPORE on 05/06/18. For personal use only.

n
i
nt rn,f(
ege n)≤ ho
lds.
P(
n)
Byi
nduc
tiononn,
ifn =1,
theequa
lit
yho
lds.As
sumet
hat
n
forn = 1,2,...,k,wehavef(
n)≤ ( ).Then,i
fn =k +
Pn
1,byLemma
s1and2andt
hea
ssump
tion,

d
f(
k +1)≤ ∑k+1
f(
d)≤ ∑k+1 P(
d)
d d
P(k+1) P(k+1)

æ k +1 ö÷ k +1
=g ç ≤ .
èP (
k +1)ø P (
k +1)
Ch
inaGi
rls
􀆳Ma thema
tica
l
Olymp
iad
Mathematical Olympiad in China (2011–2014) Downloaded from www.worldscientific.com
by NATIONAL UNIVERSITY OF SINGAPORE on 05/06/18. For personal use only.

2010 Sh
ii
jazhuang

F
irtDay8:
s 00~12:
00
t10,2010
Augus

1 tn b
Le eani
nte
gerg
rea
te hantwo,andl
rt etA1,A2,...,
A2n bepa
irwi
sed
ist
inc
tnonemp
tys
ubs
et f{
so 1,2,...,n}.
|Ai ∩ Ai+1|
f∑i=1
2
n
De
termi
net
hemax
imumva
lueo .
|Ai |·|Ai+1|
(He
re,wes
etA2n+1 =A1 .Foras
etX ,
let|X|deno
tet
he
numbe
rofe
lemen
t nX .)
si
So
lut
ionTheansweri
sn.
Ch
inaGi
rls
􀆳Ma
thema
tic
alOl
ymp
iad 161

|Ai ∩ Ai+1 |
Wecons
ide ummandsi =
reachs .
|Ai |·|Ai+1 |
fAi ∩ Ai+1i
I semp
tys
et,t
hensi = 0.
fAi ∩ Ai+1i
I snonemp
ty,becau
seAi ≠ Ai+1 ,a
tlea
stoneof
Ai andAi+1 ha
smor
ethanonee
l t,t
emen hati |Ai |,
s,max{
|Ai+1 |}≥ 2.Be
cau
seAi ∩ Ai+1i
sas
ubs
eto fAi and
feacho
Ai+1 ,|Ai ∩ Ai+1 |≤ mi
n{|Ai |,|Ai+1 |}and
Mathematical Olympiad in China (2011–2014) Downloaded from www.worldscientific.com
by NATIONAL UNIVERSITY OF SINGAPORE on 05/06/18. For personal use only.

|Ai ∩ Ai+1 |
si =
|Ai |·|Ai+1 |
n{
mi |Ai |,|Ai+1 |} 1
≤ ≤ .
|Ai |,|Ai+1 |}·mi
max{ n{|Ai |,|Ai+1 |} 2

I
tfo
llowst
hat
2n 2n
|Ai ∩ Ai+1 | 1

i=1 |Ai |·|Ai+1 |
≤ ∑2
i=1
= n.

Th
isuppe
rboundcanbeach
ievedwi
ths
ets

A1 = {
1},A2 = {
1,2},A3 = {
2},A4 = {
2,3},...,
A2n-2 = {
n -1,n},A2n-1 = {
n},A2n = {
n,1}.

2 In t
riang
le ABC ,AB = AC .
Po
intD i
sthe mi
dpo
intofs
ide
BC .Po
intE l
ies ou
tsi
de t
he
t
riang
leABCs
ucht
hatCE ⊥ AB
andBE = BD .Le
t M bet
he
mi
dpo i
nto
fs egmentBE .Po
int

Flie
sont he minoracAD o
r f
t
hec
ircumc
irc
leo
ftr
iang
leABD F
ig.2
.1
s
ucht
hatMF ⊥ BE .Pr
ovet
hat

ED ⊥ FD .
162 Ma
thema
tic
alOl
ymp
iadi
nCh
ina

So
lut
ion1.Cons
truc
tpo
intF1 s
ucht
hatEF1 = BF1 andr
ay
DF1i
spe
rpend
icu
lart
olneED .
i Its
uff
ice
stoshowt
hatF = F1
orABDF1i
scyc
lic;t
ha s,
ti

∠BAD = ∠BF1D . ①

S
et ∠BAD = ∠CAD = x.Be
cau
seEC ⊥ ABandAD ⊥ BC,
Mathematical Olympiad in China (2011–2014) Downloaded from www.worldscientific.com

∠ECB = 90
°- ∠ABD = ∠BAD = x.
by NATIONAL UNIVERSITY OF SINGAPORE on 05/06/18. For personal use only.

No
tet
hat MD i
sa mi
dli
neof t
riang
leBCE .In pa
rti
cul
ar,
MD ‖EC and

∠MDB = ∠ECD = x. ②

In i
sos
cel
est
riang
le EF1M ,
wemays
et ∠EF1M = ∠BF1M =
y.Becau
seEM ⊥ MF1 andMD ⊥
DF1 ,

∠EMF1 = ∠EDF1 = 9°,


0

imp
lyi
ng t
hat EMDF1 i
s cyc
lic.
Cons
equen
tly,wehave

∠EDM = ∠EF1M = y. ③ F
ig.2
.2

Comb
inng ② and ③ ,weob
i tai
n

∠BDE = ∠EDM + ∠MDB = x +y.

Becau
se BE = BD ,we conc
lude t
hatt
riang
le BED i
s
i
sos
cel
eswi
th ∠MED = ∠BED = ∠BDE = x + y.Becau
se
EMDF1i
scyc
lic,wehave ∠MF1D = ∠MED = x +y.I
tis
t
henc
lea
rtha
t

∠BF1D = ∠MF1D - ∠MF1B = x = ∠BAD ,

wh
ichi
s①.
Ch
inaGi
rls
􀆳Ma
thema
tic
alOl
ymp
iad 163

So
lut
ion 2. (Bas
ed on work by She
rry Gong and Inna
Zakha
rev
ich)Wema
int
aint
heno
tat
ionsu
sedi
nSo
lut
ion1.Le
t
ωandO deno
tet
hec
ircumc
irc
leandt
hec
ircumcen
teroft
riang
le
ABD ,andl
etT bes
econdi
nte
rse
cton (
i othe
rthanB )be
tween
l
ineBE andω.Ex
tends tDE t
egmen hroughE t tω a
o mee tS.
Po
intF2l
iesonωs
ucht
hatDF2 ⊥ DE .Wewi
lls
howt
hatF2 =
Mathematical Olympiad in China (2011–2014) Downloaded from www.worldscientific.com

F orF2B = F2E .Le


tM 2 deno
tet
hefoo
toft
hepe
rpend
icu
lar
by NATIONAL UNIVERSITY OF SINGAPORE on 05/06/18. For personal use only.

f
romF2t
olneBE .I
i tsu
ffi
cest
oshowt
hatM 2i
sthemi
dpo
int
ofs tBE ,
egmen tha
tis,EM 2 = M 2B .
Be
cau
se ∠SDF2 = ∠EDF2 =
°,O i
90 sthe mi
dpo
intofSF2 .
Be
cau
seBTAD i
scc
ylc,∠BTD =
i
∠BAD = ∠BCE = x.No
tea
lso
t
hat BD = BE and ∠EBC =
∠DBT . We conc
lude t
hat
t
riang
le BDT and BEC a
re
cong
ruen
ttoeacho
t r,imp
he lyi
ng F
ig.2
.3

t
hatBE = ET .Hence,OE ⊥
BT .Le
tN bet
hefoo
toft
hepe
rpend
icu
la romSt
rf olneBE .
i
No
te t
hat s
egmen
ts NE and EM 2 a
re t
he r
espec
tive
pe
rpend
icu
larpro
jec
tionsofs
egmen
tsSOandOF2on
tol
ineBE .
Be
cau
seSO = OF2 ,NE = EM 2 .Becau
seTE = EB ,
its
uff
ice
s
t
oshowt
hatTN = NE ,wh
ichi
sev
iden
tsi
nce

∠STE = ∠SEB = ∠SDB = ∠EDB = ∠BED = ∠SET

ands
otr
iang
leSETi
sis
osce
leswi
thSE = ST .

3 Pr
ovet
hatf
oreve
r i
ygvenpo
sit
ivei
nt rn,
ege the
reex
ista
primep andanin
t rm s
ege ucht
hat
(a)p ≡ 5(mod6);
164 Ma
thema
tic
alOl
ymp
iadi
nCh
ina

(
b)p n;
(
c)n ≡ m 3(
modp).
So
lut
ion 1. There are inf
ini
tel
y many pr
ime
sp t
hata
re
cong
ruen
tto 5 modu
lo 6. (Th
isi
sas
pec
ial ca
se of t
he
Di
rich
let
􀆳st
heor
emonpr
ime
sinana
rit
hme
ticprog
res
siont
hat
canbeea
sil
yprovend
irec
tly.The
rei
satl
eas
tones
uchpr
ime
Mathematical Olympiad in China (2011–2014) Downloaded from www.worldscientific.com

(namey,5).Suppo
l set
her
e wer
e onlyf
inite
ly many pr
ime
s
by NATIONAL UNIVERSITY OF SINGAPORE on 05/06/18. For personal use only.

cong
ruentt
o5 modu
lo6,andle
tt he
irproductbeP .Then6
P -1,wh
ichi
sla
rge
rthan P andcong
ruen
tto5 modu
lo6,
mu
sthave ano
the
r pr
ime d
ivi
sorcong
ruen
tto 5 modu
lo 6,
wh
ichi
sacon
trad
ict
i s,t
on.Thu he or
igi
nala
ssump
tion wa
s
wrong,and t
her
ea rei
nfi
nit
ely many odd pr
imest ha
tare
congr
uen
tto5 modu
lo6.)In par
ticu
lar,ones imep i
uch pr s
l
arge
rt s,
hann.Thu ps
ati
sfi
esbo
thcond
itons(
i a)and (
b).We
canwr
itep = 6
k +5fors
omepo
sit
ivei
nt rk.Wes
ege etm =
n4k+3 .Thent
heFe
rma
t􀆳sLi
ttl em,wehave
eTheor

m 3 ≡ n12k+9 ≡ n6k+4 ·n6k+4 ·n ≡ np-1 ·np-1 ·n ≡ n(


modp).

So
lut
ion2.Se
tm =n -1.Thenm 3
n +2n -1,
-n =n -3
3 2

wh
ichi
sre
lat
ive
lypr on.Anypr
imet imed
ivi
sorp ofm 3 -ni
s
r
ela
tive
lypr on,
imet tha
tis,ps
ati
sfi
est
hecond
iti
ons(
b)and
(
c).I
trema
inst
ofi
ndap t
hati
scong
ruen
tto5 modu
lo6.
No
tet
hat

m3 -m = m (
m2 -1)= (
m -1)
m(m +1),

wh
ichi
sdi
vis
ibeby6.Henc
l e,m 3 -n ≡ m -n ≡ -1 ≡ 5(
mod
6).Thu
s,t
her
eisapr
imed
ivi
s fm 3 -nt
orp o hati
scong
ruen
t
t
o5modu
lo6,andt
hisi
sthepr
imewes
ought.
Comment:Theor
igi
nalve
rsi
onoft
heprob
lemi
sasfo
llows.
Provet
hatf
oreve
r i
ygvenpo
sit
ivei
nt rn,t
ege her
eex
ista
Ch
inaGi
rls
􀆳Ma
thema
tic
alOl
ymp
iad 165

pr
imep andanint rm s
ege ucht
hat
(a)p ≡ 3(
mod4);
(
b)p n;
c)n ≡ m2(
( modp).
I
twa
sins
pir
edbyas
olu
tiont
otheprob
lem3ofIMO2010.
Mathematical Olympiad in China (2011–2014) Downloaded from www.worldscientific.com

4 tx1 ,x2 ,...,xn (


Le whe
ren ≥ 2)ber
ealnumbe
rswi
th
by NATIONAL UNIVERSITY OF SINGAPORE on 05/06/18. For personal use only.

1 +x2 + … +xn = 1.
x2 2 2

Pr
ovet
hat
n
k ö2 xk
2
æçn -1ö÷2
n 2
xk
∑ 1-

æ
· ≤ .
∑i=1ixi ø k èn +1ø k=1 k
ç n 2 ÷
k=1 è

De
termi
newhent
heequa
lit
yho
lds.
Comment: Expand
ing t
he l
eft
-hand s
ide of t
he de
sir
ed
i
nequa
lit
ygi
ves
n
k ö2 xk
2

∑ 1-
æ
·
∑i=1ixi ø k
ç n 2 ÷
k=1 è

n n n
xk2
2xk
2 2
kxk
= ∑ -∑ + ∑(
k=1 k
∑ ∑ ixi2)
n n
k=1
i=1
ixi2 k=1
i=1
2

n n n
1 1 1
2
xk
∑ - 2∑ 2 2 2∑
2
= 2 + kxk
k=1 k
∑i=1 x (
∑i=1 )
n n
i xi k=1 k i xi k=1

n
2 1
2
xk
= ∑ - +
k=1 k
∑i=1 i ∑i=1ixi
n2 n 2
ix
n
1
2
xk
= ∑k - .
∑i=1
n 2
k=1 i xi

Wewan
ttos
howt
hat
n n n n
xk2
1 æn -1÷ö2 2
xk xk2
4
n xk2

∑ - ≤ ç ∑ = ∑ -
( 2∑
)
k=1 k
∑i=1ixi èn 2ø k=1 k k=1 k n +1 k=1 k
n 2 +
166 Ma
thema
tic
alOl
ymp
iadi
nCh
ina

or
n
4 1
2
n xk
2∑ ≤ ,
(
n +1)k=1 k
∑i=1
n2
i xi

t
ha s,
ti

(
n +1)
n 2 n
xk 2

∑ ∑ ①
æç ö÷ 2
( kxk ) ≤ .
Mathematical Olympiad in China (2011–2014) Downloaded from www.worldscientific.com

è k=1 k ø k=1 4
n
by NATIONAL UNIVERSITY OF SINGAPORE on 05/06/18. For personal use only.

Wepr
esen
ttwoproo
fso
ftheabovei
nequa
lit
y.
So
lut
ion1.Werewr
ite① a
s
n
2 n
xk
nç ∑
4 ( ∑kxk2 ) ≤ (n +1)2.
æ ö÷
è k=1 k ø k =1

Byt
heAM GMi
nequa
liy,wehave
t
n 2 n 2 n n
xk nxk
nç ∑
4 ∑ = 4ç ∑ ( ∑kxk2 )
æ ö÷ 2 æ ö÷
( kxk )
è k=1 k ø k=1 è k=1 k ø k =1

n 2 n 2
nxk
∑ + ∑kxk ÷
æç 2ö

è k=1 k k=1 ø
n
æç n ö 2 ö÷2

æç
= +k ÷xk .
èk=1 èk ø ø

I
tsu
ffi
cest
oshowt
hat

n
+k ≤ n +1
k

or

0 ≤ nk +k -k2 -n = (
n -k)(
k -1),

wh
ichi
sev
iden
t.
Now,wecon
side
rtheequa
lit
y ca
se.No
tet
hatt
hel
ast
i
nequa
li
tyi
sst
ric
tfr1 <k <n.Hen
o e,wemu
c s ex2 = … =
thav
xn-1 = 0.Fort
heAM GMi
nequa
lit
ytoho
ld,wemu
sthave
2 Ma
thema
tic
alOl
ymp
iadi
nCh
ina

Pa
rtI Sho
rt-An
swe
r Qu
est
i s(
on Qu
est
i s1 8,e
on igh
tma
rks
e
ach)
Ch
inaGi
rls
􀆳􀆳Ma
thema
tic
alOl
ymp
iad 167
1 Ther ff(
angeo x)= x -5 - 24 -3x i
s .
So
lut
ion.I
ti
n sea
2s
ytos
neet
hatf(
x)i
sinc
rea
singoni
tsdoma
in
nxk
∑ = ∑kxk o
2
rnx2 2 2
1 +xn = x1 +n
2
xn
[
5,8].Th
ker
e
=1 fk
oe,i
r t
s rangei
k=1 s [-3, 3].

1
wi
t 1 +xn = 1.Wem
hx2 2
uthavex
s 2 2
1 = xn = andx2 = … =
2 Themi nimumo acos2x -3)
fy = ( sinx2
is -3.Thenthe
xn-1 =
ra0.
ngeofr
ea rai
lnumbe s .
So
l
uut
ion.
n2.(
Le
tsi
Byn
Lxynn
=e
lt
l.T
eYh)
eee
Wxpr
ewes
rs
i
ti
e ④ st
oniashenchangedt
o
g( t2 +a -3)
t)= (-a t,or n 2
Mathematical Olympiad in China (2011–2014) Downloaded from www.worldscientific.com

n
xk 
n +1 -4n  ∑ ∑
( ) 
2
 2
kxk  ≥ 0.
by NATIONAL UNIVERSITY OF SINGAPORE on 05/06/18. For personal use only.

g(t)= - t3k
 ka
=1 + (ak-=13)
t.

FN
ro
ot
et
m-ha
att
t3hele
(
a ft
-h3ands
)
t ≥ideo
3,ft
we hea
tbovei nequa
lit
yist
he
+ - - ge
d
isc
riminanto ft
hequa dr
ati c(int):
-a t(t2 -1)-3( t -1)≥ 0,
( )
f t =n ( (
tx
2
-11+)2
(x
-a
2
2t+(… 1n
t ++ )x
2
-n3
)
t2
)≥ n +1)
(
-0. t+
 2 x2 …-+xn2

1, ( . ) ,
2
et -1 ≤ x0
nt +1 -3 ≤ 0 or
S
inc +
w eh +
ave at
 2
(
ah
t2
)≥ -3. ①
tft
+(
I
tsu
ffi
cest
oshowt a t)hasar ea
lroo
t.Becau
set
h e
l
ead
in gc
Wheo
ne
tf
fi
=0
c
ie,
n-1
to,
f (
ex
f x)
pri
e
ssn
i
on(
x①2
a
1 +2
lx
wa2
y
2 +…
shol
d+;x
sn h)
2
wn e,0h
nw i
c
<thi
s

po
,si
tive,i
tr ema i
nst o beshownt hatf(t) ≤ 0fors
1ome2t.
1 wehave0 <t2 +t ≤ 2;andwhen -1 <t < 0,- ≤t +
4
1 + … +xn = 1,
ex2 2
Becaus wehave
3
t < 0.Therefore,-2 ≤ a ≤ 1 2.
f(t)= n( x21 +2x22+ … +nxn ) n +1)(
t2 - ( 1 +x2 + … +
2
x2 2

 x2 xn2

) +… +
2
2
xn t + x2
1 +
3 Thenumbe 
ro fint2
egra
lpo
intn
s( i.e.,t
hepo
int
swho
sex
-
n 2
ex )wi ea (
∑ nkxkt - (n +1)xkt +gk 
andy
= -coordin2at
2 e
sa rebo thi 2n t er
ks thinthea r not
k=1 
incl
ud ingt hebounda r y)enc l o
sedbyt her ightbrancho f
n
1 i
∑ 1a nex = 100i
2 2 
hyperbo la2x -y
= x k(
nk t -1) =t
 -ndl . s .
 k 
Solu
tion.Bysymme y,weon
k=1
tr lyneedt oconsiderthepa rtof
theareaa bovet he x-a x
is .S up 1
po el
s k 1
Itise as y t
o s
e e tha t f ≤i
n0ebye
ca=
u ei
s n
ft
oe
r
rc
tept
=sthe,
n  n
eachs
ummand

 1  xk
2
k -1)(
( k -n)
2
xk (
nkt -1)t -  = ≤ 0,
 k nk
168 Ma
thema
tic
alOl
ymp
iadi
nCh
ina

comp
let
ing our proo
f.For t
he equa
lit
y ca
se of t
he g
iven
i
nequa
liy,wemu
t sthaveequa
lit
yca
sefort
heabovei
nequa
lit
y
foreve
ryk;t
ha s,xk = 0f
ti or2 ≤ k ≤ n - 1.I
tist
henno
t
d
iff
icu
ltt
oob
tai
ntha
t

1
x2 2
1 = x2 = .
2
Mathematical Olympiad in China (2011–2014) Downloaded from www.worldscientific.com
by NATIONAL UNIVERSITY OF SINGAPORE on 05/06/18. For personal use only.

S
econdDay
8:
00 12:
00,Augus
t11,2010

5 tf(
Le x)andg(
x)bes
tri
ctl
yinc
rea
singl
inea
rfunc
tions
rom Rt
f o Rs
ucht
hatf(
x)i
sani
ntege
rifandon
lyi
f
g(
x)i
sanin
tege
r.Provetha
tforanyr
ea rx,
lnumbe
f(
x)-g(
x)isanin
teger.
So
lut
ion.Wecanwr
itef(
x)=ax +bandg(
x)=cx +dfor
s
omer
ealnumbe
rsa,b,c,d wi
tha,c > 0.
Bys
ymme
try,wemaya
ssumet
hata ≥c.
Wec
laimt
hata =c.As
sumeont
hecon
tra
ryt
hata >c.
Be
cau
sea >c >0,
ther
ange
soffandga
rebo
thR.The
rei
sa
x0s
ucht
hatf(
x0)=ax0 +bi
sani
nt r.Henceg(
ege x0)=cx0
+di
sal
soani
ntege
r.Bu
tthen,


f çx0 + ÷ = ax0 +b +1
æ
è aø

and

æ 1ö c
g çx0 + ÷ =cx0 +b + .
è aø a

Bu
tth
isi
simpo
ssi
blebe
cau
sewecanno
thavetwoi
ntege
rs
c
g(
x0)andg(
x1)t
hathavepo
sit
ived
iff
erenc
e whi
chi
sle
ss
a
Ch
inaGi
rls
􀆳Ma
thema
tic
alOl
ymp
iad 169

t
han1.
The
ref e,wecanwr
or itef(
x)=ax +bandg(
x)=ax +d
fors
omer
ealnumbe
rsa,b,d wi
tha > 0.
x0)-g(
Thenb -d = f( x0)mu
stbeani
nt r,t
ege ha s,
ti

f(
x)-g(
x)=b -d

i
sani
ntege
r.
Mathematical Olympiad in China (2011–2014) Downloaded from www.worldscientific.com
by NATIONAL UNIVERSITY OF SINGAPORE on 05/06/18. For personal use only.

6 Inacu
tet
riang
leABC ,
AB > AC .Le
t M be
t
he mi
dpo
int of s
ide
BC .Theex
ter
iorang
le
b
ise
ctor of ∠BAC

mee
t ay BC a
sr t P.
F
ig.6
.1
Po
insK and F l
t ie on
t
hel
inePA s
ucht
hatMF ⊥ BC and MK ⊥ PA .Prove
t
hatBC2 = 4PF ·AK .
So
lut
ion. Le
t ω and O deno
tet
he c
ircumc
irc
le and t
he
c
ircumcen
teroftr
iangeABC ,r
l e
spe
cti
vey,andl
l etN bet
he

midpoi
ntofarcBC (notcon
tai
ningA ).Not
etha
tlineMN is
t
hepe
rpend
icu
larb
isec
torofs tBC .
egmen Inpa
rti
cul
ar,bo
thF
andO l
ieonl
ine MN .No
tea
lsot
ha ayAN i
tr sthei
nte
rior
b
ise
ct f ∠BACimp
oro lyi
ngt
hatNA ⊥ FP or ∠NAF = 90
°.I
t
fo
llowst
hatNF i
sad
iame
terofω.I
tisc
lea
rt tMK ‖AN ,
ha
f
rom wh
ichi
tfo
llowst
hat

AK FK
= .
MN FM

Hence,

PK ·FK ·MN
PK ·AK = .
FM
170 Ma
thema
tic
alOl
ymp
iadi
nCh
ina

No
tet
hat MK i
sthea
lti
tudet
othe hypo
tenu
sei
nri
ght
t
riang
leFMP ,
imp
lyi
ngt
hatt
riang
leFMK andFPM a
res
imi
lar
t
oeacho
t randPF ·FK = FM 2 .Comb
he ini
ngt
hel
asttwo
equa
lit
iesy
iel
ds

PF ·FK ·MN
PF ·AK = = FM ·MN .
FM
Mathematical Olympiad in China (2011–2014) Downloaded from www.worldscientific.com
by NATIONAL UNIVERSITY OF SINGAPORE on 05/06/18. For personal use only.

F
ig.6
.2

By t
he powe
r-o
f-a
-po
int t
heor
em (or c
ros
s-chord
BC2
t em),wehaveFM ·MN = BM ·MC =
heor .Comb
ini
ng
4
t
hel
asttwoequa
lit
iesg
ive
sthede
sir
edr
esu
lt.

7 tn b
Le e an i
nte
gerg
rea
tert
han o
r equ
a o 3.Fo
lt ra
pe
rmu
tat
i x1,x2,...,xn )o
onp = ( f(1,2,...,n),wes
ay
t
hatxjl
iesi
nbe
twe fi <j <k.(
enxi andxki Fore
xamp
le,
i
nthepe
rmu
tat
ion (
1,3,2,4),3l
iesi
nbe en1and4,
twe
and4do
esno
tli
einbe
twe )S
en1and2. tS = {
e p1,p2,...,
pm }c
ons
ist
sof(
dis
tin
ct)pe
rmu
tat
ion f(
spi o 1,2,...,n).
Suppo
set
hat among e
ver
yth
reed
ist
inc
t numbe
r n{
si 1,
2,...,n},oneo
fthe
senumb
ersdoe
sno
tli
einbe
twe
ent
he
o
the
rtwonumbe
rsi
nev
erype
rmu
tat
i spi ∈ S.De
on termi
ne
t
hemax
imumva
lu fm .
eo
So
lut
i s2n-1 .
on.Theansweri
Ch
inaGi
rls
􀆳Ma
thema
tic
alOl
ymp
iad 171

Wef
irs
tshowt
hatm ≤2n-1 .Wei tonn.Theba
nduc seca
se
n =3i
str
ivi
al.(
Indeed,s
ay3doe
sno
tli
ei tween1and2,
nbe
henwecanhaveS = {(
t 1,2,3),(
3,1,2),(
2,1,3),(
3,2,
1)}.)As
sumet
hatt
hes
tat
emen
tist
rueforn = k (
whe
rek ≥
3).Nowcon
si rn =k +1andas
de etSk+1s
ati
sfi
est
hecond
iti
ons
oft
heprob
lem.No
tet
hati
fthee
l tk +1i
emen sde
let
edf
rom
Mathematical Olympiad in China (2011–2014) Downloaded from www.worldscientific.com

eachpe
rmu
tat
i nSk+1 ,
onpii ther
esu
lti
ngpe
rmu
tat
ion
sqiforma
by NATIONAL UNIVERSITY OF SINGAPORE on 05/06/18. For personal use only.

s
etSkt
hats
ati
sfi
est
hecond
iti
onsoft
heprob
lem (
forn =k).
It
s
uff
ice
stoshow t
hefo
llowi
ng c
laim:t
her
ear
eat mo
sttwo
d
ist
inc
tpe
rmu
tat
i nSk+1 t
onsp andqi hatcan mapt
othes
ame
pe
rmu
tat
ionr in Sk (
by de
let
ing t
he e
l tk + 1i
emen nthe
pe
rmu
tat
ionsp andq).
Indeed,a
ssumet
hatf
or

x1 ,x2 ,...,xk+1),p2 = (
p1 = ( y1 ,y2 ,...,yk+1),
p3 = (
z1 ,z2 ,...,zk+1)

nSk+1 ,q1 =q2 = q3 = q.Bys


i ymme
try,wemaya
ssumet
hat
q =(1,2,...,k).As
sumethatxa =yb =zc =k +1.Aga
inby
symmet
ry,wemaya ss
umethat1 ≤ a <b <c ≤ k +1.( No
te
t
hatbe
cau
seq1 = q2 = q3 = q,a,b,c a
red
ist
i t.) We
nc
cons
ide
rthr
eenumbe
rsa,b,k +1.Wehavep1 = (...,k +1,
a,...,b,...)(
inpa
rti
cul
ar,al
iesi tweenk +1andb),
nbe
p2 = (...,a,...,k +1,b,...)(
inpa
rti
cul
ar,k +1l
iesi
n
tweenaandb ),andp3 = (...,a,...,b,...,k +1,...)
be
(
inpa
rti
cul
ar,
bli
esi tweenaandk +1).Henceeachoneof
nbe
t
henumbe
rsa,b,k +1l
iei
nbe
tweent
heo
the
rtwonumbe
rsi
n
s
omepe
rmu
tat
ion nSk ,v
si iol
ati
ngt
hecond
itonsofSk .Thu
i s
oura
ssump
tionwa
swr
onganda
tmo
sttwoe
lemen
t nSk+1can
si
bemappedt
oae
lemen
tinSk ,e
stab
lish
ingourc
laim.
I
trema
inst
obeshownt
hatm = 2n-1 i
sach
ievab
le.We
172 Ma
thema
tic
alOl
ymp
iadi
nCh
ina

cons
truc
tpe
rmu
tat
ion p i
nduc
tive
ly:(
1)p
l e1; (
ac 2)a
fte
r
numbe
rs1,2,...,
lar
epl
aced,wep
l el +1e
ac ithe
rtot
hel
eft
ort
her
ightoft
hea
llt
henumbe
rsp
laceds
ofa
r.Becau
set
her
e
a
retwopo
ssi
blep
lace
sforeacho
fthenumbe
rs2,3,...,n,we
cancons
tr t2n-1s
uc uchpe
rmu
tat
ions.Foranyt
hreenumbe
rs1 ≤
a <b <c ≤n,
cdoe
sno
tli
einbe e,t
tweenaandb.Henc hiss
et
Mathematical Olympiad in China (2011–2014) Downloaded from www.worldscientific.com

of2n-1 pe
rmu
tat
ionss
ati
sfi
est
hecond
iti
onsoft
he prob
lem,
by NATIONAL UNIVERSITY OF SINGAPORE on 05/06/18. For personal use only.

comp
let
ingourpr
oof.
Comment:Theor
igi
nalve
rsi
onoft
heprob
lemi
sasfo
llows.
The
rea
renbooksa
rrangedi
nar
owonashe
lf.Al
ibr
ari
an
come
spe
riod
ica
llyandr
ear
range
sthebooksi
naneworde
r.I
t
t
urnsou
tt t,among anyt
ha hree books,t
her
eisonet
hati
s
neve
rpl
acedanywhe
rebe
tweent
heo
the
rtwo.Provet
hatt
he
t
ota
lnumbe
rofd
iff
eren
torde
rst
hatoc
curi
satmo
st2n-1 .
Toenhanc
ethel
eve
lofd
iff
icu
lty oft
het
es r,t
tpape he
prob
lemdec
ide
stoa
skcon
tes
tan
tst
ofi
ndt
hismax
imumva
lue.

8 De
termi
net
hel
eas ra > 5s
todd numbe ati
sfi
yng t
he
fo
llowi
ng cond
itons: The
i rea
re po
sit
ivei
ntege
rs m1,
m 2 ,n1 ,n2 s
ucht
ha 1 +n1 ,
ta = m2 2
2 +n2 ,
a2 = m2 2
and
m1 -n1 = m 2 -n2 .
So
lut
i s261.
on.Theansweri
No
tet
hat

261 = 152 +62 ,2612 = 1892 +1802 ,15 -6 = 189 -180.

Weknowt
hatt
her
eisnonumbe
ri tween5and261t
nbe hat
s
ati
sfi
est
hecond
iti
onoft
hepr
obl
em.As
sumeont
hecon
tra
ry
t
hatai
ssuchanumbe
r.Wemays
etd = m 1 -n1 > 0.Becau
se
sodd,m 1 andn1 haved
ai iff
eren
tpa
riy,ands
t od i
sodd.
Be
cau
sem 1 < 261,d ≤ 15;
tha
tis,t
hepo
ssi
bleva
lue
sofda
re
Ch
inaGi
rls
􀆳Ma
thema
tic
alOl
ymp
iad 173

1,3,5,7,9,11,13,15.Wewi
lle
limi
nat
eeve
ryoneoft
hem.
Wecanwr
item 2 = n2 +danda2 = (
n2 +d)
2 2
+n2 or

a2 -d2 = (
2 2n2 +d)
2
. ①

fd = 1,
I then ① be
come
saPe
ll􀆳
sequa
tionx2 -2y2 = -1
wi
th(
x,y)= (
2n2 +1,a).Th
isPe
ll􀆳
sequa
tionha
smi
nima
l
Mathematical Olympiad in China (2011–2014) Downloaded from www.worldscientific.com

s
olu
tion ( 1,1)andx +y 2 = (
x,y)= ( 1+ 2)
2k-1
forpo
sit
ive
by NATIONAL UNIVERSITY OF SINGAPORE on 05/06/18. For personal use only.

i
ntege
rsk.They va
lue
soft
hes
olu
tionsoft
hisPe
ll􀆳
sequa
tions
a
re5,29,169,985,....Thu
s,t
heon
lypo
ssi
bleva
l sfora
ue
a
re29and169.I
tisea
syt
ocheckt
hatne
it r29nor169can
he
bewr
itt
eni
ntheformof(
n1 +1)
2 2
+n1 .Henced ≠ 1.
fd i
I sa mu
lti
pl f3,t
eo hen m 2 ≡ n2(
mod3)anda2 ≡
2(
2m 2 mod3).Be
cau
se2i
sno
taquadr
ati
cre
siduemodu
lo3,we
conc
ludet
hat0 ≡ m 2 ≡ n2 ≡ a(
mod3).Hence,m 2 2
1 + n1 ≡

0(
mod3),f
rom wh
ichi
tfo
llowst
hatm 1 ≡ n1 ≡ 0(
mod3).
s,
Thu a = m2 2
1 + n1 i
sa mu
lti
pleof9andm 2 2 2
2 + n2 = a isa
mu
lti
pleof81.Wecanwr
item 2 =3m', ',anda =9a
n2 =3n '
fori
ntege
rsm 2 ,n3 ,a '2 = 9a
'.Wehave m'2 + n '2 .Aga
in,
becau
se -1i
sano
taquadr
ati
cre
siduemodu
lo3,wemu
sthave
' ≡ 0(
m' ≡ n mod3).
Itfo
llowst
hatd = 3( ')
m' -n i
sdi
vis
ibl
e
s,d = 9.
by9.Thu
Becau
sea <261 =152 +62 ,n1 <6.The
re e,
for n1 =3and
a =122 +32 =153.Bu
tt s92·577 = (
hen ① become 2n2 +9)
2
,
wh
ichi
simpo
ssi
ble.Hence,d ≠ 3,or9,or15.
fd = 11ord = 13,be
I cau
se2i
sno
taquadr
ati
cre
sidue
modu
lod,f
r 2(
oma2 ≡ 2m 2 modd),weconc
ludet
hat0 ≡ m 2 ≡
n2 ≡a(
modd).
Itf
oll
owst
ha 1 ≡a ≡0(
t2m 2 modd)and0 ≡ m 1
≡ n1 ≡ a(
modd).Inpa
rti
cul
ar,
n1 ≥ d,m 1 ≥ 2dand

1 +n1 ≥ 5
a = m2 d2 > 261.
2
174 Ma
thema
tic
alOl
ymp
iadi
nCh
ina

Hence,d ≠ 11or13.
fd = 5,wea
I lso no
tet
hat2i
sno
ta quadr
ati
cre
side
modu
lo5.Byt
hes
amer
eas
oni
ngbe
f e,wehave
or

m 1 ≡ n1 ≡ m 2 ≡ n2 ≡ 0(
mod5).

fn1 ≥ 10,
I thenm 1 ≥ 15anda = m 2
1 +n1 ≥ 1
2
02 +152 >
s,
261.Thu n1 =5,m 1 =10,anda =125.Bu
tthen ① become
s
Mathematical Olympiad in China (2011–2014) Downloaded from www.worldscientific.com
by NATIONAL UNIVERSITY OF SINGAPORE on 05/06/18. For personal use only.

52 ·1249 = (
2n2 +5)
2
,

wh
ichi
simpo
ssi
ble.Hence,d ≠ 5.
fd =7,
I thenbe
cau
sea <261 =152 +62 ,wehaven1 ≤8.
Thepo
ssi
bleva
l sofa a
ue r hen65,85,109,137,169,205,
et
245.Bu
tthen ① be s(
come n2 +7)
2 2
a2 -49.I
=2 tisea
syt
o
checkt
hatt
her
eisnos
olu
tioni
nth
isca
s e,d ≠ 7.
e.Henc
Comb
ini heabove,weconc
ngt ludet
hat261i
stheanswe
r
oft
hisque
sti
on.

2011 (
Shenzhen,Guangdong)

The10th Ch
inaG
irl
s􀆳Ma
thema
tica
lOl
ymp
iad washe
lddu
ringJu
ly
28 t3,2011 a
Augus tthe No.Th
ree Sen
iorH
igh Schoo
lof
Shen
zheni
n Shen
zhen,Guangdong P
rov
ince,Ch
ina.Ar
ound 39
t
eamsf
rom Ma
inl
andCh
ina,p
lus9t
eamsf
rom Russ
ia,t
heUn
ited
S
taes,S
t ingapo
re,Japan,e
tc.,t
ota
lly18
8gi
rls
tuden
tsa
ttended
t
hecompe
tit
ion.Thecompe
tit
ioncons
ist
so ounds — each
ftwor
l
ast
sfou
rhou
rsandcon
tai
nsf
ourp
rob
lems.Thet
eamf
romShangha
i
Hi
ghSchoo
lwonthefi
rstp
laceint
eamtotalscor
e.20part
ici
pants
wongo
ldmedas,4
l 0wonsil
vermeda
ls,and8 0bron
zemedals.
Ch
inaGi
rls
􀆳Ma
thema
tic
alOl
ymp
iad 175

F
irs
tDay
8:
00 12:
00,Augus
t1,2011

1 1 1
1 F
inda
llpo
sit
ivei
ntege
rsns
ucht
hatequa
tion + =
x y n
ha
sexac
tly2011po
sit
ivei
ntege
rso
lut
ions(
x,y)wi
thx ≤
Mathematical Olympiad in China (2011–2014) Downloaded from www.worldscientific.com

y.(
pos
edby Xi n)
ongBi
by NATIONAL UNIVERSITY OF SINGAPORE on 05/06/18. For personal use only.

So
lut
ion.Fromtheg
ivenequa
tion,wehavexy -nx -ny =

0⇒ (
x -n)(
y -n)= n .Then,be
2
si n,forany
sx = y = 2
de
x -n equa
ltoa pr
ope
rdi
vis
orofn2 ,we wi
llge
ta po
sit
ive
i
nte
gers
olu
tion(
x,y)
sat
isf
yingt
her
equ
iredc
ond
iti
on.The
ref
ore,
n2s
hou
ldhav
eexa
cty2010p
l rope
rdi
vis
orst
hata
rel
es hann.
st
Suppo
s 1 …pk , w
en = pα he
re p1 , ...,pk a
re pr
ime
1 αk

numbe
rsd
iff
eren
tfr
omeacho
the
r.Thent
henumbe
rofpr
ope
r
d
ivi
sor
sofn2l
es hanni
st s

(
2α1 +1)…(
2αk +1)-1
.
2

So (
2α1 +1)…(
2αk +1)= 4021.S
ince4021i ime,we
spr
tk = 1,2
ge α1 +1 = 4021,andα1 = 2010.
There
fore,n = p2010 ,whe
repi
sanypr imenumbe
r.

2 Ass
howni
nFi
g .1,
.2 thed
iagona
ls
AC,BD of quad
ril
ate
ralABCD
i
nte
rsec
t at po int E, t he
mi
dpe
rpend
icu
las of AB , CD
r
(
wihM ,N b
t eingt
hei
rmi
dpo
ins,
t
r
espe
cti
vey)i
l nte
rsec
tat po
int
F ,andl
ine EF i
nte
rsec
ts wi
th F
ig.2
.1

BC , AD a
t po
ins P, Q,
t
176 Ma
thema
tic
alOl
ymp
iadi
nCh
ina

r
espe
cti
vel
y.Suppo
seMF ·CD = NF ·AB ,DQ ·BP =
AQ ·CP .ProvePQ ⊥ BC .(
poedbyZheng Huan)
s
So
lut
ion. As shown in Fi
g .2,
.2
conne
ctpo
insA F ,B F ,C F ,
t
and D F .Byt
heg
ivencond
iton,
i
△AFB and△CFD a
rebo
thi
sos
cel
es
Mathematical Olympiad in China (2011–2014) Downloaded from www.worldscientific.com

t
riang
leswi
thFM andFN be
ingt
he
by NATIONAL UNIVERSITY OF SINGAPORE on 05/06/18. For personal use only.

a
lti
tude
stoeacht
riang
le􀆳
sba
se.
S
ince MF · CD = NF · AB , F
ig.2
.2
△AFB ∽ △DFC .Then ∠AFB =
∠CFD and ∠FAB = ∠FDC .Mor r, ∠BFD = ∠CFA .
eove
FromFB = FA ,FD = FC ,wehave△BFD ≌ △AFC ,wh
ich
meanst
hat ∠FAC = ∠FBD and ∠FCA = ∠FDB .The
re e,
for
po
insA ,B ,F ,E andpo
t in
tsC ,D ,E ,F a
reeachconcyc
lic.
Fromtheabover e
sut,wege
l t

∠FEB = ∠FAB = ∠FDC = ∠FEC ,

wh
ichimp
lie
stha
tlneEPi
i stheang
leb
isec
t f ∠BEC .Then
oro
EB BP ED QD
wehave = .Int ameway,wehave
hes = .
EC CP EA AQ
fDQ ·BP = AQ ·CP ,
I thenEB ·ED = EC·EA ,wh
ich
meansABCDi
sacyc
licquadr
ila
ter
alwi
thF a
sthec
ent
erofi
ts
c
ircumc
irc
le.Att
hi ime,s
st ince

1∠ 1∠
∠EBC = DFC = AFB = ∠ECB ,
2 2

wet tPQ ⊥ BC .
henge

3 Suppo
sepo
sit
iver
ealnumbe
rsa,b,c,ds
ati
sf cd =1.
yab
Prove
1 1 1 1 9 25 (
+ + + + ≥ . po
sedbyZhu
a b c d a +b +c +d 4
Ch
inaGi
rls
􀆳Ma
thema
tic
alOl
ymp
iad 177

i)
Huawe
So
lut
ion1.Fi
rst,wewi
llprovet
hat,wheneve
rthe
rea
retwo
numbe
rsamonga,b,c,dt
hata
r l,t
eequa hei
nequa
lit
yho
lds.
Wemaya
ssumet
hata = b andl
ets = a +b +c + d.Then
wehave

1 1 1 1 9
Mathematical Olympiad in China (2011–2014) Downloaded from www.worldscientific.com

+ + + +
a b c d a +b +c +d
by NATIONAL UNIVERSITY OF SINGAPORE on 05/06/18. For personal use only.

2 c +d 9 2 9
+a (s -2a)+
2
= + + =
a cd s a s
2 9ö
-2
æ 2
= a3 + ças + ÷ .
a è sø

Wede
finet
heexpr
ess
i sf(
onabovea s).Thenwes
eet
hat
3
f(
s)r
eache
sthemi
nimumfors = .
a
2,
Whena ≥ r,wehave
howeve
2

2 3
s = a +b +c +d ≥ 2a + ≥ .
a a

2
Att
hi ime,f(
st s)r
eache
sthe mi
nimumfors = 2
a+ .
a
Wet
henhave

2 9ö 2 2ö 9
-2 a3 +a2 ç2a + ÷ +
-2
æ 2 æ
a3 + ças + ÷ =
a è sø a è aø s
2 9 9
= +2
a+ =s +
a s s
7 9 9 7 9 ·9
= s+ s+ ≥ ×4 +2 s
16 16 s 16 16 s
7 9 25æç ∵s = 2 2
≥ 4÷ .
ö
= + = a+
4 2 4è a ø

2,
When0 < a < wehave
2
178 Ma
thema
tic
alOl
ymp
iadi
nCh
ina

2 9ö 2 2
-2 a3 +6a =
-2 +5 a -2a3)
a+(
æ 2
a3 + ças + ÷ ≥
a è sø a a
2 2· 25
> +5
a ≥2 5a = 2 10 > .
a a 4

cond,wecon
Se side
rtheca
set
hata,b,c,d a
red
iff
eren
t
ad
f
romeacho
the
r.Wemaya
ssumet
ha f ·
ta >b >c > d.I
c
Mathematical Olympiad in China (2011–2014) Downloaded from www.worldscientific.com

cd = 1,byu tabove,wehave
by NATIONAL UNIVERSITY OF SINGAPORE on 05/06/18. For personal use only.

b·c·c =ab singt


her
esu
l

1 1 1 1 9 25
+ + + + ≥ .
ad b c c ad 4
+b +c +c
c c

The
ref
ore,weon
lyneedt
opr
ovet
hat

1 1 1 1 9
+ + + +
a b c d a +b +c +d
1 1 1 1 9
≥ + + + + . ①
ad b c c ad
+b +c +c
c c

Wehave
1 1 9 c 1 9
① ⇔ + + ≥ + +
a d a +b +c +d ad c ad
+b +2
c
c
ac +cd -c2 -ad 9 ·
⇔ ≥
acd ( )æçad
+b +2
ö÷
a +b +c +d c
èc ø
æç ad ö
a +d - -c÷
è c ø
(
a -c)(
c -d) 9
⇔ ≥ ·
acd ( )æçad
+b +2
ö÷
a +b +c +d c
èc ø
(
a -c)(
c -d)
c
1 9
⇔ ≥
ad ( æad
a +b +c +d)ç +b +2
ö

èc ø
Ch
inaGi
rls
􀆳Ma
thema
tic
alOl
ymp
iad 179

æad
a +b +c +d)ç
⇔( +b +2
c÷ ≥ 9ad
ö
èc ø
ad ad
⇐ +b +2 ad ç ∵a +b +c +d >
9 +b +2
æ ö
c≥ c÷
c è c ø
ad
⇐ +3
c≥ 9
ad .
c

Theproo
fiscomp
let
e.
Mathematical Olympiad in China (2011–2014) Downloaded from www.worldscientific.com
by NATIONAL UNIVERSITY OF SINGAPORE on 05/06/18. For personal use only.

So
lut
ion2.Weprovei
tbyu
singt
head
jus
tmen
tme
thod.We
maya
ssumet
hata ≤b ≤c ≤ d,andde
fine

1 1 1 1 9
f(
a,b,c,d)= + + + + .
a b c d a +b +c +d

F
irs
tly,wewi
llpr
ove

f(
a,b,c,d)≥ f( ac ,b, ac ,d). ②

Asama
tte
roff
act,expr
ess
ion ②i
sequ
iva
len
tto

1 1 9 1 1 9
+ + ≥ + +
a c a +b +c +d ac ac 2 ac +b +d
(a - c)
2
9(a - c)2
⇔ ≥
ac (
a +b +c +d)(
2 ac +b +d)
(∵ (a - c)
2
≥ 0)

⇐(
a +b +c +d)( c +b +d)≥ 9ac
2 a
2 ö
ç ∵b +d ≥ 2 b
æ
d = ÷
è ac ø
2 öæ 2 ö
÷ ç2 a
æ
⇐ ça +c + c+ ÷ ≥9
ac.
è ac ø è ac ø

2
Fr cd ≥a·a·c·c⇒ac ≤ 1⇒
om1 = ab ≥2 ac and
ac
a +c ≥ 2 ac ,wehavet
hefo
llowi
ngcond
iton:
i
2 öæ 2 ö
f③ ≥ ç2 a ÷ ç2 a
æ
Thel
eft
-hands
ideo c+ c+ ÷
è ac ø è ac ø
180 Ma
thema
tic
alOl
ymp
iadi
nCh
ina

c·4 ac =16ac >9ac =t


≥4 a her
ight
-hands
ideof③.
The
ref e,② ho
or lds,wh
ich meansf(
a,b,c,d)(wi
th
a ≤b ≤c ≤ d )r
eache
sit
smi
nimumi
fandon
l fa =c (
yi i.e.a
= b = c).So we may a
ssume t
hat (
a, b,c, d) =
æ 1 ,1 ,1 , 3 ÷ö (
t t ≥1).Thenweon
lyneedt
opr
ovet
hat,f
or
èt t t
ç
ø
lt ≥ 1,
Mathematical Olympiad in China (2011–2014) Downloaded from www.worldscientific.com

a
l
by NATIONAL UNIVERSITY OF SINGAPORE on 05/06/18. For personal use only.

æ 1 1 1 3 ö÷ 25
fç , , ,t ≥ . ④
èt t t ø 4

Wehave

æ 1 1 1 3 ö÷ 25
fç , , ,t ≥
èt t t ø 4
1 9 25
⇔3
t+ 3 + ≥
t 3 3 4
t +
t

⇔12
t8 -25
t7 +76
t4 -75
t3 +12 ≥ 0
⇔(
t -1)
2
(
12t6 -t5 -14
t4 -27
t3 +36 t +12)≥ 0
t2 +24
t6 -t5 -14
⇐12 t4 -27
t3 +36
t2 +24
t +12 ≥ 0
t -1)(
⇔( 12t5 +11
t4 -3
t3 -30 t +30)+42 ≥ 0.
t2 +6

S
incet ≥ 1,12 t ≥ 2 12
t5 +6 t5 ·6 t3 ,and
t = 12 2t3 > 3

t4 +30 ≥ 2 11
11 t2 ,
t4 ·30 = 2 330t2 > 30

t t - 1)(
hen ( 12t5 + 11
t4 - 3
t3 - 30 t + 30)+ 42 > 0.
t2 + 6
The
re e,⑤ ho
for lds,wh
ichj
ust
ifi
es④.
Theproofi
scomp
let
e.

4 n(n ≥ 3) t
able t
enni
s player
s have a round-
robin
t
ournamen —
t eachp l
ayerwillpl
ayal
ltheo
thersexac
tly
once,andt
her
eisno dr
aw game.Suppo
se,a
fte
rthe
t
ou t,a
rnamen llt
he p
laye
rscanbea
rrangedi
nac
irc
le
Ch
inaGi
rls
􀆳Ma
thema
tic
alOl
ymp
iad 181

s
ucht
hat,f
oranyt
hreep
laye
rsA ,B ,C ,i
fA ,B a
re
ad
j t,t
acen hena
tlea
stoneoft
hem de
fea
tedC .P
lea
se
f
inda
llpo
ssi
bleva
l sofn.(
ue poedbyFuYunhao)
s
So
lut
ion.Wewi
llprovet
hatncanbeanyoddnumbe
rsno
tle
ss
t
han3.Suppo
s k +1,anoddnumbe
en =2 rgr
eat
e han3,and
rt
npl
aye
rsarerepr
esentedbyA1 ,A2 ,...,A2k+1 .Le
tusa
rr ange
Mathematical Olympiad in China (2011–2014) Downloaded from www.worldscientific.com

t
hecompe
tit
ionresul
ta sfo
llows:Ai(1 ≤i ≤ 2 k +1)defeated
by NATIONAL UNIVERSITY OF SINGAPORE on 05/06/18. For personal use only.

Ai+2 ,Ai+4 ,...,Ai+2k (


wes
tipu
lat
et tA2k+1+j = Aj ,j = 1,
ha
k + 1 )bu
2,...,2 tlo
stt
othe o
the
rpl
aye
rs.Then t
hes
e
l
payerscanbearr
angedi
nac i
rcl
ei rA1 ,A2 ,...,A2k+1 ,
norde
A1 .Now,givenanythr
ee p
layersA ,B ,C wi
thA ,B be ing
ad
jacen
tint
hec
irc
le,wemaya
ssumet
hatA =At ,B =At+1 ,C
= At+r ( k +1,2 ≤r ≤2
1 ≤t ≤2 k).Thene
it rrorr -1i
he san
evennumbe
rno
tle
sst k,wh
han2 ichimp
lie
stha
tatl
eas
toneof
t
hep
laye
rsA ,B de
fea
tedC .
Ont
heo
t rhand,s
he uppo
seni
sanevennumbe
rno
tle
ss
han4,and t
t hen pl
ayerscan be a
rrangedi
n acir
cle A1 ,
A2 ,...,An ,A1 t
hat mee
tstherequi
redcond
iti
on.We may
a
ssumet
hatA1 de
fea
tedA2 .Ac
cord
ingt
other
equ
ir t,a
emen t
l
eas
toneofA2 ,A3 de
fea
tedA1 ,andt
henA3 de
fea
tedA1 ;bu
t
a
tlea
stoneofA1 ,A2 de
fea
tedA3 ,s
oA2 de
fea
tedA3 ,ands
o
for
th.Wet
henge
tt t,f
ha orany1 ≤i ≤n,Aide
fea
tedAi+1and
l
os o Ai-1 (
tt sti
pul
atet
hatAn+1 = A1 ,A0 = An ).Wenow
n -2
d
ivi
det
hep
laye
rsa
ftrAi andbe
e f eAi-1 i
or nto pa
irs—
2
eachcons
ist
softwoad
jac
entp
laye
rs.Thent
her
eisa
tlea
stone
l
paye
rineach pa
irwhode
fea
tedAi ,andt
hatmeans,be
side
s
n -2
Ai-1 ,
the
rea
rea
tlea
st pl
aye
rswhode
fea
tedAi .ThenAi
2
n n2
l
osta
tlea
s s.Sonp
t game laye
rsl
ostt
ota
llya
tlea
st game
s.
2 2
182 Ma
thema
tic
alOl
ymp
iadi
nCh
ina

n(
n -1) n2
Bu
tthenumbe
roft
ota
lgame
sisCn
2
= < .Th
isi
sa
2 2
con
trad
ict
ion.The
ref e,t
or he po
ssi
bleva
lue
sofn a
rea
llt
he
oddnumbe
rsno
tle
sst
han3.

S
econdDay
Mathematical Olympiad in China (2011–2014) Downloaded from www.worldscientific.com

00 12:
8: 00,Augus
t2,2011
by NATIONAL UNIVERSITY OF SINGAPORE on 05/06/18. For personal use only.

5 Gi
venr
ea rα,p
lnumbe lea
sef
ind
t
he mi
nimum r
ea rλ =
lnumbe
λ(
α),s
ucht
hatforanycomp
lex
numbe
rsz1 ,z2andr
ea rx
lnumbe
∈[
0,1],
if|z1 |≤α|z1 -z2|
hen|z1 -xz2 |≤λ|z1 -z2|.
t
(
poedbyLiShenghong)
s
F
ig.5
.1
So
lut
ion.Asshowninthef
igue,i
r n
t
hecomp
lex p
lane,po
ins A ,B and C deno
t tet
he comp
lex
numbersz1 ,z2 andxz2 ,re
spect
ive
ly.C i sobviou
sly onthe
segmentOB .The Vec tor → →
s BA and CA a rerepresen
ted by
complexnumbersz1 -z2andz1 -xz2 ,
respect
ive
ly.As|z1|≤
α|z1 -z2 |,wehave|O→
A |≤α|B→
A |.Then
→ → →
|z1 -xz2 |max =|AC |max = max {|OA |,|BA |}
= max {|z1 |,|z1 -z2 |}
= max {α|z1 -z2 |,|z1 -z2 |} .

The
re e,
for λ(
α)= max{
α,1}.

6 Ar
ethe
reanypo
sit
ivei
ntege
rsm ,ns
ucht
hatm 20 +11ni
s
asqua
r r? Proveyourconc
enumbe lus
ion.(
pos
edby Yuan
Hanhui)
Ch
inaGi
rls
􀆳Ma
thema
tic
alOl
ymp
iad 183

So
lut
ion.As
sumi
ngt
her
ear
epo
sit
ivei
ntege
rsm ,ns
ucht
hat
m 20 +11n =k2 wi
thk ∈ Z,wet
henhave

11n =k2 - m 20 = (
k - m 10)(
k + m 10),

wh
ich meanst
hatt
her
ear
ein
tege
rsα,
β ≥ 0s
ucht
hat

k - m 10 = 11α , ①
{
Mathematical Olympiad in China (2011–2014) Downloaded from www.worldscientific.com

k + m 10 = 11β . ②
by NATIONAL UNIVERSITY OF SINGAPORE on 05/06/18. For personal use only.

I
tisobv
iou
st tα <β.Sub
ha trac
ti rom ② ,wehave
ng ① f

2m 10 = 11α (
11β-α -1).

tm = 11γm 1 ,whe
Le reγ,m 1 ∈ N* ,11 m 1 .Then

1110γ ·2m 1
1 =1
0
1α (
11β-α -1),

wh
ichimp
lis10
e γ =αand2m 1
1 =1
0
1β-α -1.
ByFe
rma
t􀆳sLi
ttl em,wehavem 1
eTheor 1
0
≡ 1(mod11),
1 ≡ 2(
hen2m 1 mod11).Bu
t11β-α -1 ≡ 10(mod11),wh
0
t ichi
s
ac
ont
rad
ict
ion.Sot
he p
rov
ed c
onc
lus
ioni
stha
tthe
rea
re no
po
sit
ivei
nte
gesm ,ns
r ucht
hatm20 +11ni
sas
qua
renumbe
r.

7 Suppo
sen sma
llba
llshavebeen p
lacedi
nton numbe
red
sB1 ,B2 ,...,Bn .Eacht
boxe imewecans
el taboxBk
ec
anddot
hefo
llowi
ngope
rat
ions:
(
1)I
fk = 1andt
her
eisa
tlea
stoneba
l nB1 ,moveone
li
ba
llf
romB1i
ntoB2 .
(
2)I
fk =nandt
her
eisa
tlea
stoneba
l nBn ,moveone
li
ba
llf
romBni
ntoBn-1 .
(
3)I
f2 ≤k ≤n -1andt
her
ear
eatl
eas
ttwoba
ll nBk ,
si
moveoneba
llf
rom Bk i
nto Bk+1 and oneba
lli
nto
Bk-1 ,r
espec
tive
ly.
Pr
ove t
he fo
ll ng: no ma
owi tte
r how t
he ba
llsa
re
184 Ma
thema
tic
alOl
ymp
iadi
nCh
ina

d
ist
ribu
ted among t
he boxe
s or
igi
nal
ly, i
tisa
lway
s
r
eal
izab
let
ole
teachboxcon
tai
nexac
tlyoneba
llbyf
ini
te
ope
rat
ions.(
pos nmao)
edby Wang Xi
So
lut
ion.Foranytwovec
tor x1 ,x2 ,...,xn )andy =
sx = (
(
y1 ,y2 ,...,yn ),
ift
her
eex
iss1 ≤k ≤ ns
t ucht
hat

x1 = y1 ,...,xk-1 = yk-1 ,xk > yk ,


Mathematical Olympiad in China (2011–2014) Downloaded from www.worldscientific.com
by NATIONAL UNIVERSITY OF SINGAPORE on 05/06/18. For personal use only.

wet
hendeno
tei
ta tx = (
sx ≻ y.Le x1 ,x2 ,...,xn )r
epr
esen
t
t
hed
ist
ribu
tionoft
heba
llsamongt
heboxe
s.Thenxi
sanon
-
nega
tivei
ntege
rvec
tor.Theope
rat
ionde
finedi
ntheque
ston,
i
i
fexecu
tab
le,canbeexpr
ess sx +αk ,whe
eda reα1 = (-1,1,
0,...,0),αk = (
0,...,0,1,-2,1,0,...,0)(
2 ≤k ≤
􀮩􀪁􀪁􀮪􀪁􀪁
􀮫
k-2

n -1),αn = (
0,...,0,1,-1).Thenfork ≥ 2,wea
lway
s
havex+αk ≻x.Soforanyi
nit
iald
ist
ribu
tionoft
heba
lls,a
fte
r
af
ini
tenumbe
rofope
rat
ion
soneve
ryBk (
k ≥ 2)t
hatcon
tai
ns
atleas
ttwoba ls,wecana
l rr
iveatabal
ldi
str
ibu
tiony = (
y1 ,
y2 ,...,yn )
sat
isf
yingyk ≤1foralk ≥2.
l Ifatthist
imey2 =
… =yn =1,t
heproblemiss
olved;othe
rwise,wehavey1 ≥2.
As
s ngii
umi sthesma
lles
tnumbersuchthatyi =0,wecanthen
doas
eri
esofope
rat
i sonB1 ,B2 ,...,Bi-1 :
on
B1,B2,...,Bi-1
(
y1 ,1,...,1,0,yi+1 ,...,yn ) →
B1,B2,...,Bi-2
(
y1 ,1,...,1,0,1,yi+1 ,...,yn ) →
y1 ,1,...,1,0,1,1,yi+1 ,...,yn )→ … →
(
B1
y1 ,0,1,...,1,yi+1 ,...,yn ) →
(
(
y1 -1,1,...,1,yi+1 ,...,yn )

t t(
o ge y1 - 1,1, ...,1,yi+1 , ...,yn ).Repea
ting t
he
ope
rat
ionsabove,wecanf
ina
llya
rri
vea
ttheba
lld
ist
ribu
tion
vec
tort
hatmee
tst
her
equ
iremen
t.
Ch
inaGi
rls
􀆳Ma
thema
tic
alOl
ymp
iad 185

8 Ass
howni
nFi
g .1,☉O i
.8 sthee
scr
ibedc
irc
let
ouch
ing
s
ideBC o
f△ABC a
tpo
intM ,andpo
insD ,E a
t reont
he
s
egmen
ts AB , AC ,r
espec
tive
ly,s
ati
sfi
yng DE ‖BC ;
☉O1i
sthei
nsc
ribedc
irc
leof△ADEt
angen
ttos
ideDE a
t
po
intN ;O1B ,DO i
nte
rse
cta
tpo
intF ,andO1C ,EO
i
nte
rsec
tatpo
intG .P
lea
seprovet
hatMN d
ivi
dess
egmen
t
Mathematical Olympiad in China (2011–2014) Downloaded from www.worldscientific.com

FG equa
lly.(
pos
edbyBi
anHongp
ing)
by NATIONAL UNIVERSITY OF SINGAPORE on 05/06/18. For personal use only.

F
ig.8
.1

So
lut
ion1.I
fAB = AC ,t
hent
heg
raphi
ssymme
tri
cabou
t
t
heb
isec
torof ∠BACandt
heconc
lus
ioni
sobv
iou
s.Sowemay
a
ssumet
hatAB > AC .Asshowni
n Fi
g .2,l
.8 etL bet
he
mi
dpo
intofBC ,
lineO1Li
nte
rsec
ting wi
t tR ,andO1N
hFG a
beex
tendedt
oin
ter
sec
twi
thBC a
tK .Dr
awl
ineAT t
hati
s
pe
rpend
icu
la o BC a
rt tT andi
nte
rsec
ts wi
thl
ine DE a
tS.
Connec
ting AO ,obv
iou
sly O1 i
s on t
he s t AO .By
egmen
Mene
lau
s􀆳t em,wehave
heor

F
ig.8
.2
186 Ma
thema
tic
alOl
ymp
iadi
nCh
ina

O1F ·BD · AO O1G ·CE · AO


= 1, = 1. ①
FB DA OO1 GC EA OO1

BD CE O1F O1G ,
S
i eDE ‖BC ,wehave
nc = .So = wh
ich
DA EA FB GC
FR BL
mean
st tFG ‖BC .Then
ha = = 1.Ther
ef e,Ri
or sthe
GR CL
mi
dpo
intofFG .Wenowon
lyneedt
oprovet
hatM ,R ,N a
re
Mathematical Olympiad in China (2011–2014) Downloaded from www.worldscientific.com
by NATIONAL UNIVERSITY OF SINGAPORE on 05/06/18. For personal use only.

co
lli
nea
r. For t
his purpo
se, by t
he i
nve
rse of Mene
lau
s􀆳
t em,weon
heor lyneedt
oprovet
hat

O1R ·LM · KN
= 1. ②
RL MK NO1

O1R O1F OO1 · AD (


S
i eFR ‖BL ,wehave
nc = = the
RL FB AO DB
s
econd equa
lit
yisj
ust
ifed by ① ).So we on
i ly need t
o
provet
hat

OO1 ·AD ·LM · KN


= 1. ③
AO DB MK NO1

S
i eO1K ⊥ DE ,OM ⊥ BC ,AT ⊥ BC ,DE ‖BC ,t
nc hen
l
i sO1K ,OM ,AT a
ne repa
ral
lel.Byt
het
heor
em ofd
ivi
ding
OO1
t
hes
egmen
tsi
ntopr
opor
tiona
lbypa
ral
lell
i s,wehav
ne e =
AO
MK
.Sub
sti
tut
ingi
tin
to ③ ,wet
henon
lyneedt
oprove
MT

AD ·LM · KN
= 1. ④
DB MT NO1

S
i e DE ‖BC , KN ⊥ DE ,ST ⊥ BC ,quadr
nc ila
ter
al
KNST i
sar
ect
ang
l henKN = ST .Fu
eandt rthe e,f
rmor rom
AD AS
DS ‖BT wehave = .Sub
sti
tut
ingt
hes
ere
sul
tsi
nto ④,
DB ST
wenowon
lyneedt
oprove
Ch
inaGi
rls
􀆳Ma
thema
tic
alOl
ymp
iad 187

LM NO1
= . ⑤
MT AS

tBC = a,AC =b,AB =c.Wehave


Le

a +b -c ( a
BM = thepr
ope
rtyofane
scr
ibedc
irc
le),BL = ,
2 2
a2 +c2 -b2 a2 +c2 -b2
BT =ccos∠ABC =c·
Mathematical Olympiad in China (2011–2014) Downloaded from www.worldscientific.com

= .
2
ac 2a
by NATIONAL UNIVERSITY OF SINGAPORE on 05/06/18. For personal use only.

Then

c -b
LM BL -BM 2 a
= = 2 = .
MT BT -BM c -b2 +a(c -b) a +b +c
2a

Ont
heo
t rhand,
he

2S△ADE
NO1 AD + DE + AE DE a
= = = .
AS 2S△ADE AD + DE + AE a +b +c
DE

The
ref
ore,⑤ ho
lds.Thepr
oofi
scomp
let
e.
So
lut
ion2.Le
tthe r
adi
iof ☉O and ☉O1 ber andr1 ,
r
espe
cti
vel
y.Obv
iou
sly,O1 ,O andA a
reco
lli
nea
r.

AB AC
DE ∥ BC ⇒ =
üï
BD CE ï
1 æç
ï
A ö÷·
ABs
in OO1 ï
OF S△BOO1 2è 2ø ï
= =
FD S△DBO1 1 · ï OF OG
r1 BD ý⇒ =
2 ï FD GE
1 æçACs A ö÷· ï
in OO1 ï
OG S△OO1C 2è 2ø
= = ï
GE S△EOO1 1 ·
r1 CE ï
2
ï
þ
⇒FG ‖DE ‖BC .
188 Ma
thema
tic
alOl
ymp
iadi
nCh
ina

Conne
ctON andex
tendi
ttoi
nte
rse
ct wi
th BC a
tK .I
f
∠ABC = ∠ACB ,
thenbys
ymme
try,t
hepropo
sit
ionho
lds.So
wemaya
ssumet
hat ∠ABC < ∠ACBi
nthefo
llowi
ng.
tOM ,O1M ,OB ,MD ,DO1 ,r
Weconnec espe
cti
vey(
l s
ee
F
ig .3).S
.8 inceO1N ‖OM ,wehave
Mathematical Olympiad in China (2011–2014) Downloaded from www.worldscientific.com
by NATIONAL UNIVERSITY OF SINGAPORE on 05/06/18. For personal use only.

F
ig.8
.3

1 · C -B ,
S△ONM = S△MOO1 = r OO1 ·s
in ①
2 2
1 C
BO ·OO1 ·si
n
OG OF S△BOO1 2 2
= = =
GE DF S△BDO1 1· B
BD ·DO1 ·sin
2 2
C
r·OO1 ·s in
2
= ②
B
r1 ·BD ·cos
2

B B ö÷ ,
r = BO ·cos ,r1 = DO1 ·s
in
æç
è 2 2ø

1
S△DMN -S△MEN = NK ·DN - NE
2
1 B Cö
BD ·s
inB · çr1co
t -r1co
t ÷. ③
æ
=
2 è 2 2ø

om ② and ③ ,wehave
Fr
Ch
inaGi
rls
􀆳Ma
thema
tic
alOl
ymp
iad 189

OG
S△DMN -S△MEN
GE
C
r·OO1 ·s in
1 · B C 2
= r1 BD ·s
inB · çco
t -c
ot ÷·
æ ö
2 2 2ø B
r1 ·BD ·cos
è
2
B· C æo B Cö
=r·OO1 ·s
in sn · çc
i t -co
t ÷
2 2 è 2 2ø
Mathematical Olympiad in China (2011–2014) Downloaded from www.worldscientific.com
by NATIONAL UNIVERSITY OF SINGAPORE on 05/06/18. For personal use only.

C -B
=r·OO1 ·s
in .
2

Comb
ini
ngi
twi
th ① ,wege
t

OG (
S△DMN -S△MEN )= 2S△MON .
GE

OG OG æçDF + EG ö÷ ,
S
ince ∶2 = ∶ then
GE OE èOD OE ø

OF · OG · æDF EG ö÷·
S△MND - S△MEN = ç + S△MON ,
OD OE èOD OE ø
OF ·S△MND - DF ·S△MON OG ·S△MEN +EG ·S△MON
= .④
OD OE

Ont
heo
t rhand,
he

S△MEN ·OG +EG ·S△MON


S△NMG = .
OE

OF ·S△MND - DF ·S△MON
The
ref
ore,S△NMG = .
OD

Int ameway,
hes

OF ·S△MND - DF ·S△MON
S△NMF = .
OD

By ④ ,wehaveS△NMG = S△NMF .The


ref
ore,MN d
ivi
des
s tFG equa
egmen lly.
Theproofi
scomp
let
e.
190 Ma
thema
tic
alOl
ymp
iadi
nCh
ina

2012 (
Guangzhou,Guangdong)

F
irs
tDay
8:
00 12:
00,Augus
t,10,2012
Mathematical Olympiad in China (2011–2014) Downloaded from www.worldscientific.com

1 ta1 ,a2 , ...,an ben non


Le -nega
tive r
eal numbe
rs.
by NATIONAL UNIVERSITY OF SINGAPORE on 05/06/18. For personal use only.

Provet
hat

1 a1 a1a2…an-1
+ +… + ≤ 1.
1+a1 (1+a1)
(1+a2) (
1+a1)
(1+a2)…( 1+an )

(
pos nghua)
edby AiYi
So
lut
ion.Le
ta0 = 1.Weprovet
hefo
llowi
ngi
den
tiy:
t
n k n
aj-1 aj
∑∏
k=1 j=1 1 +aj
=1- ∏
j=1 1 +aj

byi
nduc
tiononn.
I
tisev
iden
ttha
t①i
str
ueforn =1.Suppo
set
hat①i
str
ue
forn -1,n ≥ 2,
thenforn,
n k n-1 k n
aj-1 aj-1 aj-1
∑ ∏ 1 +a
k=1 j=1 j
= ∑ ∏ 1 +a
k=1 j=1 j
+ ∏ 1 +a
j=1 j

n-1 n
aj aj-1 ö÷
∏ -∏
æç
=1-
èj=1 1 +aj j=1 1 +aj ø
n
aj
=1- ∏ 1 +a .
j=1 j

2 Ass
howni
nthef
igu
re,c
irc
lesΓ1 andΓ2 a
ret
angen
t
ex
terna
llya
tpo
intT .Po
insA andE a
t reonc
irc
leΓ1 ,
l
i sAB andDE a
ne ret
angen
ttoc
irc
leΓ2 a
tpo
insB and
t
D ,r
espec
tive
ly.Li
nesAE andBD mee
tatpo
intP .Prove
t
hat
Ch
inaGi
rls
􀆳Ma
thema
tic
alOl
ymp
iad 191

( AB ED ;
1) =
AT ET
(
2)∠ATP + ∠ETP = 180
°.
(
pos
edby Xi n)
ongBi
So
lut
ion.Le
ttheex
ten
sionofAT mee
tci
rceΓ2 a
l tpo
intH ,
andt
heex
ten
si fET mee
ono t
Mathematical Olympiad in China (2011–2014) Downloaded from www.worldscientific.com

ci
rcleΓ2a
tpo i
ntG .Theni
tis
by NATIONAL UNIVERSITY OF SINGAPORE on 05/06/18. For personal use only.

easytos ee t
hat AE ‖GH ,
t
hus △ATE ∽ △HTG ,
AT ET
cons
equen
tly = .And
TH TG
AH EG
f
urt
herwehave = . F
ig.2
.1
TH TG
Byt
heCi
rcl
ePowe em,wehave
rTheor

AB2 AT ·AH ET ·EG ED2 ,


2 = = =
TH ·
TH TH TG ·TG TG2

TH AB ;
Henc
e =
TG ED
t
her
ef e,
or

AT HT AB
= = .
ET GT ED

By t
he S
ine Law, we
have F
ig.2
.2

AP s in ∠ABP si
n ∠EDP EP
= = = .
AB s
in ∠APB si
n ∠EPD ED

e,
Henc

AP AB AT ,
= =
EP ED ET

t
ha s,PT i
ti sthe b
ise
ctoro
ftheex
ter
iorang
le of ∠ATE .
The
ref e,
or
192 Ma
thema
tic
alOl
ymp
iadi
nCh
ina

∠ATP + ∠ETP = 180


°.

3 F
ind a
llt
he pa
irsofi
ntege
rs(
a,b)s
ati
sfi
yng t
he
fo
llowi
ngcond
iton:t
i her
eex
ist
sani
nt rd ≥ 2s
ege uch
t
hatan +bn +1i
sdi
vis
ibebydforanypo
l sit
ivei
nt rn.
ege
(
poedbyChenYonggao)
s
Mathematical Olympiad in China (2011–2014) Downloaded from www.worldscientific.com

So
lut
ion.I sodd,t
fa +bi henan +bn seven,t
+1i husd =2.
by NATIONAL UNIVERSITY OF SINGAPORE on 05/06/18. For personal use only.

I seven,t
fa +bi n
sodd,s
hena +b +1in
odi
sodd.
S
inced|a +b +1,(
a +b +1)
2
=a +b +1 +2(
2 2
a +b +
ab -1),andd|a2 +b2 +1,
1)+2( then,d|2(
ab -1),
sowe
haved|ab -1.
Wes
eet
hata3 +b3 + 1 ≡ (
a +b)(
a2 +b2 -ab)+ 1 ≡
(-1)(-1-1)+1 ≡3( andd|a3 +b3 +1,henced|3,
modd),
od = 3.
s
S
ince

(
a -b)
2
ab ≡ -1 -2 ≡ 0(
= a +b -2
2 2
mod3),

wehavea ≡b(
mod3).Thu
s, a +1(
0 ≡a +b +1 ≡2 mod3),we
s
eet
hata ≡ 1(
mod3).
The
ref
ore,a ≡ b ≡ 1(
mod3).Cons
equen
tly,for any
po
sit
ivei
nt rn,wehave
ege

an +bn +1 ≡ 1 +1 +1 ≡ 0(
mod3).

ngup,t
Summi her
equ
iredi
ntege
rpa
irsa
r heforms:
eoft
(
2k,2
l +1),(
2k +1,2
l),(
3k +1,3
l +1),whe
rekandla
re
i
ntege
rs.

4 The
rei
sas
tone (
o ame)a
fgog tea
chv
ert
exo
fag
ivenr
egu
lar
13
-gon,andt
hec
olo
rofe
achs
tonei
sbl
acko
rwh
ite.Pr
ove
t
hatwemaye
xchang
ethepo
sit
iono
ftwos
tone
ssu
cht
hatt
he
Ch
inaGi
rls
􀆳Ma
thema
tic
alOl
ymp
iad 193

c
olo
ringo
fthe
ses
tone
sar
esymme
tri
c wi
thr
espe
ctt
osome
s
ymme
tri
cax
iso
fthe13
-gon.(
poedbyFuYunhao)
s
So
lut
ion.Takeanyver
texA andt
hes
ymme
tri
cax
islpa
ssi
ng
i
t.The
rea
res
ixpa
irsofve
rti
cess
ymme
tri
ctol.
Ift
heco
lorof
s
tone
sateachpa
irofve
rti
cesi
st ame,t
hes hent
heco
lor
ingi
s
s
ymme
tri
ctol.
Mathematical Olympiad in China (2011–2014) Downloaded from www.worldscientific.com

I
fthe
rei
son
ly one pa
iro
fsymme
tri
cve
rti
cest
hatha
s
by NATIONAL UNIVERSITY OF SINGAPORE on 05/06/18. For personal use only.

d
iff
eren
tco
lors
t s,t
one henexchanget
hes
tonea
toneve
rtexof
t
hepa
iri
ndi
ffe
ren
tco
l oA wi
ort tht
hes
toneonA .
I
fthe
rea
reexac
tlytwopa
irsofve
rti
cesi
ndi
ffe
ren
tco
l s,
or
t
henexchanget
hewh
ites
tonei
nve
rtexo
fapa
irwi
tht
heb
lack
s
tonei
nve
rtexoft
heo
the
rpa
ir.
Nows
uppo
set
hatforanyve
rtexA andt
hes
ymme
tri
cax
is
pa
ssngA ,
i the
rea
rea
tlea
stt
hreepa
irsofs
ymme
tri
cve
rti
cesi
n
d
iff
eren
tco
lor
s.Wes
hal
lshowt
hisi
simpo
ssi
ble.
I
fthe
rea
rexb
lacks
tone
sandy wh
ites
t s,t
one henx +y =
13 ;wi
thou
tlo
ssofgene
ral
ity,l
etx beoddandy beeven.
I
fthes
toneonAi
sinb
lack,t
hent
her
ema
ini
ngs
tone
sar
e
eveni
nbl
ackand wh
ite,r
espe
cti
vey.Hence,t
l her
ear
eeven
pa
irso
fve
rti
cesi
ndi
ffe
ren
tco
l s,t
or ha s,t
ti her
ear
eatl
eas
t
fourpa
irsofve
rti
cesi
ndi
ffe
ren
tco
lor
s.
S
imi
lar
ly,i
fthes
toneonA i
sin wh
ite,t
hent
her
ear
eat
l
eas
tthr
eepa
irso
fsymme
tri
cve
rti
cesi
ndi
ffe
ren
tco
lor
s.
S
inc
eeachpa
iro
fve
rti
cesi
ssymme
tri
ctooneax
is,wes
ee
t
hatt
henumbe
rofpa
irsi
ndi
ffe
ren
tco
lor
sisa
tlea
st4x +3y.
Ont
heo
t rhand,t
he henumbe
rofpa
irsi
ndi
ffe
ren
tco
lor
s
i
sexac
tlyxy,
thu
s

xy ≥ 4x +3y = x +39,

t
ha s,x(
ti y -1)≥ 39,bu
titcon
trad
ict
sto
194 Ma
thema
tic
alOl
ymp
iadi
nCh
ina

æx + (
y -1)ö÷
2

x(
y -1)≤ ç =36.
è 2 ø

S
econdDay
00 12:
8: 00,Augus
t,11,2012
Mathematical Olympiad in China (2011–2014) Downloaded from www.worldscientific.com

5 As s
hown i
n Fi
g .1, t
.5 he
by NATIONAL UNIVERSITY OF SINGAPORE on 05/06/18. For personal use only.

i
nsc
ribedc
irc
le☉I o
f△ABC
i
stangen
ttot
hes
i sAB and
de
AC a
t po
ins D and E ,
t
r
espec
tive
ly. And O i
sthe
c
ircumc
ent
reof△BCI.Prove
t
hat ∠ODB = ∠OEC .(
pos
ed
i)
byZhuHuawe
F
ig.5
.1
So
lut
ion.S
ineOi
c sthec
irc
umc
ent
re
of△BCI,wes
eet
hat
∠BOI = 2∠BCI = ∠BCA .

Int
hes r,∠COI = ∠CBA .Hence,
amemanne
∠BOC = ∠BOI + ∠COI = ∠BCA + ∠CBA = π - ∠BAC .

s,fou
Thu rpo
insA ,B ,O andC a
t reconcyc
lic.ByOB =
OC ,weknowt
hat ∠BAO = ∠CAO .(
Itcana
lsobes
eenf
rom
t
hewel
l-knownconc
lus
ionthatpo
int

Oist
he mi dpo
intof ac BC (
r not
con
tai
npo
intA )onc
ircumc
irc
leo
f
△ABC .) Comb
ined wi
tht
hef
act
t
hatAD = AE ,AO = AO ,wehave
△OAD ≅ △OAE .Hence, ∠ODA
= ∠OEA ,t
her
ef e, ∠ODB
or =

∠OEC .
F
ig.5
.2
Ch
inaGi
rls
􀆳Ma
thema
tic
alOl
ymp
iad 195

6 The
rea
renc
iti
es(
n > 3)andtwoa
irl
inecompan
iesi
na
coun
try.Be
tweenanytwoc
iti
es,t
her
eisexac
tlyone2
-
wayf
ligh
tconne
cti
ngt
hem wh
ichi
sope
rat
edby oneof
t
hetwo compan
ies. A f
emale mat
hemat
ician p
lans a
t
rave
lrou
te,sothati
tsta
rtsandendsatthesamec i
ty,
pa
sse
sthr
ougha
tlea
sttwoo
the
rci
tis,andeachc
e ityon
Mathematical Olympiad in China (2011–2014) Downloaded from www.worldscientific.com

t
herou
tei
svi
sit
edonce.Shef
ind
sou
ttha
twhe
reve
rshe
by NATIONAL UNIVERSITY OF SINGAPORE on 05/06/18. For personal use only.

s
tar
tsand wha
teve
rrou
teshe choo
ses,s
he mu
stt
ake
f
light
sofbo
thcompan
ies.F
indt
hemax
imumva
lueofn.
(
pos
edbyLi ngde)
ang Yi
So
lut
ion.Cons
ide
reachc
itya
save
rtexandeacha
irl
inea
san
edgei
naco
lorcor
res
pond
ingt
othea
irl
inecompanyi
tbe
long
s.
Thent
hea
irl
inerou
techa
rtoft
hiscoun
trycanber
ega
rdeda
sa
two
-co
lorcomp
let
egr
aph wi
th n ve
rti
ces.By t
he prob
lem
s
tat
es,any c
irc
lecon
tai
nsedge
sofbo
thco
lor
s.Tha
tis,t
he
s
ubg
rapho
feachco
lorha
snoc
irc
le.I
tiswe
ll-knownt
hatf
or
t
hes
imp
leg
raphwi
thou
tci
rce,t
l henumbe
rofedge
sisl
esst
han
t
henumbe
rofve
rti
ce s,t
s.Thu henumbe
rofedge
soft
hes
ame
co
lori
snomor
ethann -1,
tha
tis,t
het
ota
lnumbe
rofedge
sis
nomor
ethan2(
n - 1).Ont
heo
t rhand,t
he henumbe
rofa
comp
let
egr
aphwi
thnve
rti
ce sn(
si n -1)
/2.Hence,
n(n -1)≤
2(
n -1),
tha
tis,
n ≤ 4.
fn = 4,deno
I tef
ou rci
tisbyA ,B ,C andD .Le
e tthree
rou
tesAB ,BCandCD beoperat
edbyt
hefir
stcompanyandt
he
o
the
rthr
eerou
tesAC ,AD andBD byt
hes
econdcompany.We
s
eet
hatt
herou
techa
rto
feachcompanyha
snoc
irc
le.So,t
he
max
imumnumbe
ro s4.
fni

7 ta1 ≤ a2 ≤ … beas
Le equenc
eofpo
sit
ivei
ntege
rss
uch
t
hatr/
ar =k +1fors
omepo
sit
ivei
ntege
rskandr.Prove
196 Ma
thema
tic
alOl
ymp
iadi
nCh
ina

t
hatt
her
eex
ist
sapo
sit
ivei
nt rss
ege ucht
hats/
as = k.
(
pos
edbyJ
acekFabr i,USA)
ykowsk
So
lut
i tg(
on.Le t)=t -kat .Theng(
r)=r -kar =ar > 0.
No
tet
hatg(
1)= 1 -ka1 ≤ 0.Sot
hes
et{
t|t = 1,2,...,r,
g(t)≤ 0}isno
tempt y.Lets bet
he max
imalel
ementofthe
s
et;thens <r.Hence,g(s +1)> 0.Ontheot
herhand,
Mathematical Olympiad in China (2011–2014) Downloaded from www.worldscientific.com

g(
s +1)=s +1 -kas+1 ≤s +1 -kas = g(
s)+1 ≤ 1. ①
by NATIONAL UNIVERSITY OF SINGAPORE on 05/06/18. For personal use only.

s,0 < g(
Thu s +1)≤ 1.Cons
equen
tly,g(
s +1)= 1.And
by ① ,1 =g(
s +1)≤g(
s)+1 ≤1.Wehaveg(
s)=0,
tha
tis,
s/as =k.

8 F
indt
henumbe
rofi
ntege
rski
nthes
et{
0,1,2,...,
æ2012ö
2012}s uch t
hatthe combi
nat
ion number ç ÷ =
è k ø
2012!
i
sa mu
lti
pleof2012. (
pos
ed by Wang
k 2012 -k)!
!(
Bin)
So
lut
ion.Fac
tor
izng2012 = 4 ×503,wes
i eet
hatp =503i
sa
pr fki
ime.I sno
tamu
lti
pl fp,
eo then

æ2012ö æ4p ö (
4p)! 4p æ4p -1ö
ç ÷= ç ÷= ! ( )!
= ×ç ÷.
è k ø è k ø k × 4p -k k èk -1 ø
æ2012ö 2012!
Hence ç ÷ = !( i
samult
ipl
eo fp.
è k ø k 2012 -k)!
Ifki samu lt
ipl
eo fp, t
her
ea r
eon
lyfivecas
es:
k = 0,p,
2p,3p,4p.And wes
eet
hati
nal
lca
ses,t
hecomb
ina
tion
numbe
rs

æ4p ö æ4p ö
ç ÷= ç ÷ = 1,
è0ø è4p ø
æ4p ö æ4p ö 4(
3p +1)(
3p +2)…(3p + (
p -1))
÷= ç ÷= ≡ 4(
modp),
ç
èp ø è3p ø (p -1)!
Ch
inaGi
rls
􀆳Ma
thema
tic
alOl
ymp
iad 197

and

2p +1)(
6[( 2p +2)…(
2p + (
p -1))]
æ4p ö [(
2p + (
p +1))(2p + (
p +2))…(2p + (
2p -1))]
÷= (p -1)![(p +1)(p +2)…(
2p -1)]
ç
è2p ø
modp)
≡ 6(

a
reno
tmu
lti
pleofp.
Mathematical Olympiad in China (2011–2014) Downloaded from www.worldscientific.com
by NATIONAL UNIVERSITY OF SINGAPORE on 05/06/18. For personal use only.

Now wedeno
tet
heb
ina
rynume
ralofnon
-nega
tivei
ntege
r
nby
r r

n=(
arar-1 …a0) ∑ aj2 ,ands(n)= ∑a ,
j
2 = j
j=0 j=0

whe
reaj = 0or1forj = 0,1,...,r.
Thent rm o
hepowe ff
acor2mi
t nfac
tor
iza
tionofn ! canbe
expr
ess
edby

n n n
+ +… + m +…
2 4 2
=(
arar-1 …a2a1)
2 +(
arar-1 …a3a2)
2 + … +ar

= ar × (
2 -1)+ar-1 × (
2- -1)+ … +a1 ×
r r 1

21 -1)+a0 × (
( 20 -1)
= n -s(
n).

æ2012ö 2012!
I
fthecomb
ina
tionnumbe
rç ÷= ! ( =
è k ø k × 2012 -k)!
(
k + m )!
sodd (
i m =2012 -k),
theni
tmeanst
hatt
hepowe
rs
k! × m !
of2i
nfac
tor
soft
henume
rat
orandt
hedenomi
nat
oroft
he
f
rac
tionabovea
ret
hes
ame.Then

k + m -s(
k + m )=k -s(
k)+ m -s(
m ),

or

s(
k + m )=s(
k)+s(
m ).
198 Ma
thema
tic
alOl
ymp
iadi
nCh
ina

Th
ismeanst
hatt
heb
ina
ryadd
iti fk +m =2012ha
ono sno
ca
rri
yng.
S
ince2012 = (
11111011100)
2,i
tcons
ist
sofe
igh
t1􀆳
sand
t
hree0
􀆳s.I
fthe
rei
snoca
rri
yngi
ntheadd
iti fk + m =
ono
( 2,
11111011100) t
henont
heb
i n(
tof0i 2,
11111011100) k,m
a
re0;
ont
heb
itof1,onei
s0,t
heo
the
ris1,s
othe
rea
retwo
Mathematical Olympiad in China (2011–2014) Downloaded from www.worldscientific.com

cho
ics:1 =1 +0and1 =0 +1.Thu
e s,t
her
eae28 =256ca
r ses
by NATIONAL UNIVERSITY OF SINGAPORE on 05/06/18. For personal use only.

t
hatt
heb
ina
ryadd
iti
ono
ftwonon
-nega
tivei
ntege
rsk + m =
2012 ha
s no ca
rri
yng.Tha
tis,t
her
eae 256 comb
r ina
tion
numbe
rst
hata
reodd,andt
her
ema
inng2013 - 256 = 1757
i
comb
ina
tionnumbe
rsa
reeven.
I
fthecomb
ina
tionnumbe
r

æ2012ö 2012! (
k + m )!
÷= ! ( )!
=
k! × m !
ç
è k ø k × 2012 -k

i
sevenbu
tno
tamu
lti
pl f4,t
eo heni
tmeanst
hatt
hepowe
rof
2i
nthenume
rat
ori
sgr
eat
ert
hant
hati
nthedenomi
naorby1.
t
s,
Thu

k + m -s(
k + m )=k -s(
k)+ m -s(
m )-1,

or

s(
k + m )=s(
k)+s(
m )-1.

Tha
tis,t
her
eison
lyoneca
rri
yngi
ntheb
ina
ryadd
iti
onof
k +m =2012.Theca
rri
ynghappensa
ttwob
it s01+01 =10.
sa
By

k + m = 2012 = (
11111011100)
2,

wes
eet
hatt
heca
rri
yngcanon
lyhappena
tthef
ifh(
t f
romt
he
highes
tb i
ttot
hel owe
stbit)andsix
thbit
soratthenin
thand
t
en thbit
s.Sothe
reexist2 = 128ca
7
ses.Tha
tis,t
her
ea re256
Ch
inaGi
rls
􀆳Ma
thema
tic
alOl
ymp
iad 199

comb
ina
tions who
se va
lue
sar
e even numbe
rs bu
t no
tthe
mu
lti
ple
sof4.
s,t
Thu her
eae2013 - 256 - 256 = 1501comb
r ina
tions
who
seva
lue
sar
e mu
lti
ple
sof4.Now,wegobackt
ocons
ide
r
t
heca
seswhe
reki
sno
tthemu
lti
pleofp = 503.
æ4p ö æ4p ö
Wes
eet
hatç ÷= ç ÷ = 1i
sno
ta mu
lti
pleof4.For
Mathematical Olympiad in China (2011–2014) Downloaded from www.worldscientific.com

è0ø è4p ø
by NATIONAL UNIVERSITY OF SINGAPORE on 05/06/18. For personal use only.

æ4p ö æ4p ö (4p)!


ç ÷= ç ÷ = !( )! ,t
he powe ss(
rof2i p)+s(
3p)-
èp ø è3p ø p 3p
æ4p ö 4p)!
(
s(
4p)=s(
3p)=7,andfor ç ÷ = ( )!( )! ,
thepowe
rof
è2p ø 2p 2p
ss(
2i 2p)+s(
2p)-s(4p)=s( p)= 8.Thu s,therear
ethree
comb
ina
tionnumbe
rswh
icha
remu
lti
plsof4bu
e tno
tmu
lti
ple
s
ofp = 503,
tha
tis,t rofks
henumbe ucht
hatt
hecomb
ina
tions
æ2012ö
ç ÷ar
emu
lti
plsof2012i
e s1501 -3 = 1498.
è k ø

2013 (
Zhenhi,Z
a he
jiang)

F
irs
tDay
(
8:00 12: t12,2013)
00Augus

1 tA bet
Le hec
los
ed doma
in ont
he p
lanede
limi
ted by
t
hreel
i sx = 1,y = 0,andy =t(
ne 2x -t),whe
re0 <
t < 1.Provet
hatt
hes
urf
aceo
fanyt
riang
lei
nsi
det
he
doma
inA wi
thP (
t,t2)andQ (
1,0)a
stwoo
fit
sve
rti
ces
1
canno
texc
eed .
4
200 Ma
thema
tic
alOl
ymp
iadi
nCh
ina

So
lut
ion. I
t i
s ea
sy t
o
ob
ser
vet
hatt
hedoma
ini
sa
c
los
ed t
riang
le. I
ts t
hree
æt
e B ç ,0÷ ,Q (
1,
ö
ve
rti
cesa
r
è2 ø
0)andC(
1,t(
2-t)).P
icka
he △BQC ,
Mathematical Olympiad in China (2011–2014) Downloaded from www.worldscientific.com

po
intX i
nsi
de t
by NATIONAL UNIVERSITY OF SINGAPORE on 05/06/18. For personal use only.

t
hen t
he a
r f △PQX i
ea o s
equa
ltoha
lfo
ftheproduc
tof
F
ig.1
.1
PQ wi
tht
hed
ist
anc
efom X
r
oPQ .Sot
t hea
r fPQX
ea o
t
ake
sit
smax
imum va
lue whent
hed
ist
anc
efrom X t
o PQ i
s
max zed,i.
imi e.,whenX co
inc
ide
swi
thB orC .
Thea
reaof△PQBi
s

1 æç1 - t ö÷ 2 1 1
t = ( 2 -t)
t2 ≤ (2 -t)
t
2è 2ø 4 4
1 æç2 -t +tö÷ 1;
2
≤ =
4è 2 ø 4

t
hea
r f△PQCi
eao s

1( 1 (
1 -t)(
2t -t2)= 2t 1 -t)(
2 -t)
2 4
t +1 -t +2 -tö÷
1 æç2 1
3
≤ = .
4è 3 ø 4

e,i
Henc nthedoma
inA ,anyt
riang
lewi
thP ,Q a
stwoof
1
i
tsve
rti
cescanno
thaveana
reat
hatexc
eed
s .
4

2 Ass
howni
nFi
g .1,Inat
.2 rapezo
idABCD ,AB ‖CD ,
☉O1i
stangen
ttot
hes
egmen
tsDA ,AB ,BC ,☉O2 i
s
t
angen
ttot
hes
egmen
tsBC ,CD ,DA .Le
tP bet
he
Ch
inaGi
rls
􀆳Ma
thema
tic
alOl
ymp
iad 201

t
angen
tpo
in f ☉O1 wi
to th AB ,and Q bet
het
angen
t
po
in f ☉O2 wi
to th CD .Pr
ovet
hatAC ,BD ,PQ a
re
concu
rren
t.
Mathematical Olympiad in China (2011–2014) Downloaded from www.worldscientific.com
by NATIONAL UNIVERSITY OF SINGAPORE on 05/06/18. For personal use only.

F
ig.2
.1 F
ig.2
.2

So
lut
ion.Le
tR bet
hei
nte
rse
cti
onofl
i sAC ,BD ,andj
ne oin
O1A ,O1B ,O1P ,O2C ,O2D ,O2Q ,PR ,QR .Asshown
i
nFi
g.2
.2.
AsBA andBC a
ret
angen
tli
nesof☉O1 ,

∠PBO1 = ∠CBO1 =
1∠
ABC .
2

1∠
S
imi
lar
ly,wehave ∠QCO2 = BCD .
2
From AB ‖CD ,weknowt
hat ∠ABC + ∠BCD = 180
°.
The
ref e,∠PBO1 + ∠QCO2 = 90
or °,anda
sRt△O1BP and
O1P CQ
Rt△CO2Q a
res
imi
lar,wehave = .
BP O2Q
AP O2Q
S
imi
lar
ly,wehave = .By mu
lti
pli
yngt
hes
etwo
O1P DQ
AP CQ , AP
i
den
tit
ies,weob
tai
n = wh
ichi
ntu
rnimp
lie
s =
BP DQ AP +BP
CQ , AP CQ
i.e., = .
CQ + DQ AB CD
Aga
inbyAB ‖CD ,weknowt
hat△ABR and △CDR a
re
202 Ma
thema
tic
alOl
ymp
iadi
nCh
ina

AR CR AP CQ , AR
s
imi
lar,s
o = .Compa
ringwi
th = weha
ve =
AB CD AB CD AP
CR
.Meanwh
ile, △PAR i
ssimi
la o △QCR a
rt s ∠PAR =
CQ
s ∠PRA = ∠QRC .So P ,R ,Q a
∠QCR .Thu reco
lli
nea
r.
The
re e,AC ,BD ,PQ a
for reconcu
rren
t.
Mathematical Olympiad in China (2011–2014) Downloaded from www.worldscientific.com

s,anytwooft
by NATIONAL UNIVERSITY OF SINGAPORE on 05/06/18. For personal use only.

3 Inag
roupofm g
irsandn boy
l heme
ithe
r
know eacho
t r,ordono
he tknow eacho
the
r.Forany
twoboy
sandtwog
irs,a
l tlea
stoneboyandoneg
irldo
no
tknoweacho
the
r.Pr
ovet
hatt
henumbe
rofboy
-gi
rl
n(
n -1)
pa
irst
hatknoweacho
the
risa
tmo
stm + .
2
So
lut
ion.Fromthehypothes
is,f s,t
oranytwoboy her
eisa
t
mo
stoneg
irlt
hatknowsbo
tho
ft txibet
hem.Le henumbe
rof

s,1 ≤i ≤n.So ∑i=1xi = m .By


n
i
grl
stha
tknowexac
tlyiboy
coun
ting the number of t
he above two boy
s-one g
irl
combina
tions,wehave

i -1)
i( n -1)
n(

i≥2 2
xi ≤
2
.

Thenumbe
rofboy
-gi
rlpa
irst
hatknoweacho
the
rist
hen

i(
i -1)
n n n


i=1
ixi = m + ∑ (
i=2
xi ≤ m + ∑
i -1)
i=2 2
xi

n(
n -1)
≤m + .
2

4 F
indt
henumbe
rofpo
lynomi
alsf(
x) = ax3 +bx t
hat
s
ati
sfyt
hefo
llowi
ngcond
iti
onsbe
low:
(
1)a,b ∈ {
1,2,...,2013};
(
2) t
he d
iff
erenc
eofany two numbe
rsamong f(
1),
Ch
inaGi
rls
􀆳Ma
thema
tic
alOl
ymp
iad 203

f(
2),...,f(
2013)
isno
tamu
lti
pleof2013.
So
lut
ion.2013i
sfac
tor
i s2013 =3×11×61.Le
zeda tp1 =3,

p2 = 11,p3 = 61.Wedeno
tebyait
her
esdueofa modu
i lopi ,
bybi t
her e
sidueofb moduopi (
l i = 1,2,3),a,b ∈ {
1,
2,...,2013}.BytheCh i
neseReminde em,wehavea
rTheor
b
ije
cti
onof(
a,b)wi
t a1 ,a2 ,a3 ,b1 ,b2 ,b3).
h(
Mathematical Olympiad in China (2011–2014) Downloaded from www.worldscientific.com

Now,l
e x) = aix3 + bix,i = 1,2,3.Weca
tfi( lla
by NATIONAL UNIVERSITY OF SINGAPORE on 05/06/18. For personal use only.

po
lynomil “good modu
a lo n ”ifthe resi s of f(
due 0),
f( n -1)modu
1),...,f( lonar
eal
ldis
tinc
t.
ff(
I x)= ax3 +bxi
sno
tgood modu
lo2013,t
hent
her
e
ex
issx1 ≢ x2 (
t mod 2013)s
uch t
hatf(
x1) ≡ f(
x2)(
mod
2013).Suppo
sex1 ≢ x2(
modpi).Le
tu1 andu2bet
her
esi
due
s
ofx1 andx2 modu
lopi ,r
espe
cti
vey.Thenu1 ≢ u2(
l modpi)and
u1)≡ fi(
fi( u2)(
modpi),
sofi(
x)i
sno
tgoodmodu
lopi .
ff(
I x)=ax3 +bxi
sgoodmodu
lo2013,t
henforeve
ryi,
fi (
x)i
sgood modu
lopi .Ther
eas
oni
sasfo
llows.Forany
d
ist
inc
tpa
irr1 ,r2 ∈ {
0,1,...,pi -1},
the
reex
istx1 ,x2 ∈
{
1,2,...,2013}
sucht
hatx1 ≡r1 ,x2 ≡r2(
modpi)andx1 ≡
2013ö÷ 2013ö÷ ,
x2 ç mod .Nowf (
x1)≡ f(
x2)ç mod tf(
x1) ≢
æ æ
bu
è pi ø è pi ø
f(
x2)( of(
mod2013),s r1)≢ f ( modpi).
r2)(
Hence, we need to det
ermi
ne t
he numbe
r of good
po
lynomi
alsfi(
x)modu l
opi .
Forp1 = 3,byFe
rma
t􀆳st em,agoodpo
heor lynomi
al

f1(
x)≡ a1x +b1x ≡ (
a1 +b1)
x(mod3)

i
sequ
iva
len
ttos
ayt
hata1 +b1i
sno
tdi
vis
ibeby3.The
l rea
rei
n
t
ota
lsi
xsuchf1(
x).
Fori =2,3,
iffi(
x)i
sgoodmodu
lopi ,
t oranyuand
henf
v ≢ 0(
modpi),fi(
u +v)≢ fi(
u -v)(
modpi),
i.e.,
204 Ma
thema
tic
alOl
ymp
iadi
nCh
ina

fi(
u +v)-fi(
u -v)= 2
v[ai(
3u2 +v2)+bi]

i
sno
tdi
vis
ibl fai ≠ 0,t
e bypi .I her
esi
due
s modu
lopi of
pi -1
e
lemen
tsi
nthes
et aiu2 |u = 0,1,...,
sA = 3 { and }
2
pi -1
B = (-bi -aiv2)|v = 1,2,...,
{ 2 } do notcoincide,
Mathematical Olympiad in China (2011–2014) Downloaded from www.worldscientific.com

and|A|+|B|=pi .SoA ∪Bformsacomp


let
ere
sidues
yst
em
by NATIONAL UNIVERSITY OF SINGAPORE on 05/06/18. For personal use only.

modu
lo pi . The
irs
um mu
st be a mu
lti
pl f pi ,i.
eo e.,
pi-1 pi-1

∑u=03aiu + ∑v=1 (-bi -aiv )≡ 0(modpi).


22 2 2

æpi -1ö÷ 1·pi -1·pi +1·


2

Now,12 +22 + … + ç = pii


s
è 2 ø 6 2 2
pi -1
amu
lti
pleofpi ,
so- ·bii
sal
soamu
lti
pl fpi .Hence,
eo
2
bii
sdi
vis
ibebypi ,
l i.e.,exac
tlyoneofai ,bii
s0.
fai = 0,bi ≠ 0,t
I henfi (
x) = bix i
sobv
iou
slygood.
The
rea
repi -1s
uchgoodpo
lynomi
als.
fai ≠ 0,bi = 0,t
I henfi(
x) = aix3 .Forp2 = 11 ,by
Fe
rma
t􀆳st em,(
heor x3)
7
=x
21
≡ x(
mod11),s
oforx1 ≠
x2(
mod11)andx3
1 ≠x2 (
3
mod11),f2(
x)=a2x3i
sgood.The
re
a
rei
nto
tal10s
uchpo
lynomi
als.
Forp3 =61,a
s43 =64 ≡125 =53(
mod61),f3(
x)=a3x3
canno
tbegood.
The
re e,t
for het
ota
lnumbe
rtha
twea
rel
ook
ingf s6 ×
ori
(
10 +10)×60 = 7200.

S
econdDay
8:
00 12:
00,Augus
t13,2013

5 Gi
venpo
sit
iver
ealnumbe
rsa1 ,a2 ,...,an .Provet
hat
Ch
inaGi
rls
􀆳Ma
thema
tic
alOl
ymp
iad 205

t
her
eex
istpo
sit
iver
ealnumbe
rsx1 ,x2 ,...,xns
ucht
hat

∑ x = 1,andt sy1 ,
n
i=1 i
hatf
oranypo
sit
iver
ealnumbe
r

y2 ,...,yn s ng ∑i=1yi = 1,oneha


n
ati
sfi
y s

n n
aixi 1

i=1 xi +yi

2i∑
=1
ai .
Mathematical Olympiad in China (2011–2014) Downloaded from www.worldscientific.com

n
ai
.Then ∑xi = 1.Mor
by NATIONAL UNIVERSITY OF SINGAPORE on 05/06/18. For personal use only.

So
lut
ion.Le
txi = r,
eove

n
i=1 i
a i=1

n n n
aixi xi2
∑x
i=1 i +yi
= ∑a ∑x
i=1
i
i=1 i +yi
.

sy1 ,y2 ,...,yn wih ∑i=1yi =


n
Foranypo
sit
ivenumbe
r t

1.Byt
heCauchy
-Schwa
rzInequa
liy,
t
n n n n
xi2 xi2
2∑ ∑ (xi +yi)∑ ( ∑x )
2
= ≥ i = 1.
i=1 xi +yi i=1 i=1 xi +yi i=1

Hence,
n n n n
aixi xi2 1

i=1 xi +yi
= ∑ai ∑
i=1 i=1 xi + y
i ≥
2i∑
=1
ai .

6 tS beas
Le ubs
etofm e
lemen
tsof{
0,1,2,...,98},m
≥ 3,
sucht
hatforanyx,y ∈S,
the
reex
issz ∈S wi
t thx
z(
+y ≡ 2 mod99).F
inda
llpo
ssi
bleva
l sofm .
ue
So
lut
i tS = {
on.Le s1 ,
s2 ,...,
sm }.AsS
'={
0,s2 -s1 ,...,
sm -s1}s
ati
sfi
esa
lsot
hehypo
the
sis,we maya
ssume wi
thou
t
l
osso
fgene
ral
ityt
hat0 ∈ S.Foranyx,y ∈ S,50(
x +y)≡
z(
mod99)∈ S.Byt
akngy = 0,wehavet
i hatforanyx ∈ S,
50x ∈ S.As50and99a
r ime,t
ecopr her
eex
ist
sa po
sit
ive
i
nt rks
ege ucht
hat50k ≡ 1 (
mod99).Hence,

x +y ≡ 50k (
x +y)(
mod99)∈ S.
206 Ma
thema
tic
alOl
ymp
iadi
nCh
ina

td = gcd(
Le 99,s1 ,s2 , ...,sm ).Theabovea
rgumen
t
t
henimp
lie
st td ∈ S,t
ha her
e eS = {
for 0,d,2d,...}.For
any po
sit
ive f
acor d < 99 of 99,t
t hisS s
ati
sfi
esa
llt
he
r
equ
iremen
t e,a
s.Henc llt
hepo
ssi
bleva
l sofm a
ue re3,9,11,
33,99.
Mathematical Olympiad in China (2011–2014) Downloaded from www.worldscientific.com

7 As shown i
n Fi
g .1, ☉O1 and ☉O2 a
.7 ret
angen
t
by NATIONAL UNIVERSITY OF SINGAPORE on 05/06/18. For personal use only.

ex
terna
llya
t po
int T . The quadr
ila
ter
al ABCD i
s
i
nsc
ri n ☉O1 .Thel
bedi i sDA andCB a
ne ret
angen
tto
☉O2 a
tpo
insE andF ,r
t espe
cti
vey.BN ,t
l heb
isec
tof
∠ABF ,
int
ers
ect
sthes tEF a
egmen tpo
intN .Li
neFT
i
nte
rse
ctst
he a
rc AT (wh
ich doe
sno
tcon
tan B )a
i t
po
intM .Pr
ovet
hatM i
stheexocen
terof△BCN .

F
ig.7
.1 F
ig.7
.2

So
lut
ion.Le
tP bet
hei
nte
rsec
tionofl
ineAM wi
thEF .Jo
in
AT ,BM ,BP ,BT ,CM ,CT ,ET ,TP .Asshowni
nFi
g.7
.2.
AsBFi
stangen
tt tF ,wehave ∠BFT = ∠FET .
o☉O2 a
As☉O1i
stangen
tt tT ,wehave ∠MBT = ∠FET .
o ☉O2 a
Hence,∠MBT = ∠BFM .So△MBT and△MFB a
res
imi
lar,
t
her
ef eMB2 = MT ·MF .Thes
or amea
rgumen
tgi
vesMC2 =
MT ·MF .
Nowaga
inf
romt
hat☉O1i
stangen
tt tT ,wehave
o☉O2a
∠MAT = ∠FET .So A ,E ,P and T a
reconcyc
lic,wh
ich
Ch
inaGi
rls
􀆳Ma
thema
tic
alOl
ymp
iad 207

imp
lie
st t ∠APT = ∠AET .AsAEi
ha stangen
tt tE ,
o☉O2 a
s, ∠MPT = ∠PFM ,and
wehave ∠AET = ∠EFT .Thu
△MPTi
ssimi
la o△MF .The
rt ref e,MP2 = MT ·MF .
or
Fromt
heabovea
r t,wehave MC = MB = MP ,
gumen
wh
ich meanst
hatM i
stheexcen
terof △BCP .So ∠FBP =
1∠
CMP .Meanwh
ile, ∠CMP = ∠CDA = ∠ABF ,and we
2
Mathematical Olympiad in China (2011–2014) Downloaded from www.worldscientific.com
by NATIONAL UNIVERSITY OF SINGAPORE on 05/06/18. For personal use only.

1∠
have ∠FBN = ABF .Hence,∠FBN = ∠FBP ,i.
e.,P
2
andN co
inc
ide.

8 tn ≥ 4beanevennumbe
Le r.Att
heve
rti
cesofar
egu
lar
n gon we writ
ei n an a
rbi
tra
ry way n d
ist
inctreal
number
s.St
arti
ngf om oneedge,wenamea
r lltheedge
s
i
nac
lockwi
sewaybye1 ,e2 ,...,en .Anedgei
sca
lled
“po
sit
ive”,i
fthe d
iff
erence of t
he numbe
rsa
tit
s
endpo
intandi
tss
tar
tpo
inti
spo
sit
ive.As
etoftwoedge
s
{
ei ,
ej }
isca
lled “
cro
ssng”,i
i f2|(
i +j),andamongt
he
r,t
fou henumbe
rs wr
itt
ena
tthe
irve
rti
ces,t
hel
arge
st
andt
het
hirdl
arge
stone
sbe
longt
othes
ameedge.Prove
t
hatt
henumbe
rofc
ros
sing
sandt
henumbe
rofpo
sit
ive
edge
shaved
iff
eren
tpa
rit
y.
So
lut
ion1.Wi
thou
tlo
sso
fgene
ral
ity,we maya
ssumet
hat
t
henumbe
rswr
itt
enont
heve
rti
cesa
re1,2,...,n.Le
tA be
t
henumbe
rofc
ros
si s,andB bet
ng henumbe
rofpo
sit
iveedge
s.
We wi
ll pr
ovet
hatt
he pa
rit
yofS r
ema
inst
hes
amei
f we
exchangenumbe
rsiandi +1.Wed
ist
ingu
ishtwoca
ses.
CaseI.The number
si andi + 1a
re wr
itt
en on ad
jac
ent
ve
rti
ces,i.
e.,t
heendpo
int
sofedgeek .Once weexchangei
andi +1,t
henumbe
rofpo
sit
iveedge
sismod
ifedby1,t
i hus
208 Ma
thema
tic
alOl
ymp
iadi
nCh
ina

t
hepa
rit
yofB i
schanged.Ont
heo
t rhand,t
he heon
lytwo
-
edges
ubs
ett
hatwi
llbe
comeanewc
ros
sing (
orchangef
rom a
c
ros
singt
oanon
-cro
ssng)i
i s{ek-1 ,ek+1}(
thes
ubi
ndi
cesa
ret
o
beunde
rst
ood modu
lon ),a
llt
heo
the
rtwo
-edges
ubs
etswi
ll
no
tbea
ffec
ted.Sot
hepa
rit
yofA wi
llchange.
CaseI siandi +1a
I.Thenumber rewr
itt
enonnon
-ad
jac
ent
Mathematical Olympiad in China (2011–2014) Downloaded from www.worldscientific.com

ve
rti
ces. As
sume t
hatt
hey a
re wr
itt he (
en on t common)
by NATIONAL UNIVERSITY OF SINGAPORE on 05/06/18. For personal use only.

endpo
int
sofej ,ej+1 and ofek ,ek+1 ,r
espec
tive
ly.Onc
e we
exchangeiandi+1,
eve
rypo
sit
iveedgewi
llr
ema
inpo
sit
ive,s
o
a
renon
-po
sit
iveone
s.The
ref
ore,B i
sunchanged.Now,f
or
numbe
rofc
ros
si s,
ng i
fatwo
-edges
ubs
etdoe
sno
tinvo
lvea
tthe
s imeiandi +1,
amet thenwhe
the
riti
sac
ros
singorno
tisno
t
a
ffec
tedbyt
heope
rat
ion.Sot
heon
lytwo
-edges
ubs
etst
obe
cons
ide
reda
ret
hetwot
hathavebo
thiandi+1wr
itt
enont
hei
r
ve
rti
cesandt
hes
umoft
heedgenumbe
riseven.Theywi
llbo
th
becomec
ros
singa
ftrt
e heexchangei
ftheya
reno
tbe e,and
for
v
ice
-ve
rs e,t
a.Henc hepa
rit
yofA r
ema
inst
hes
ame.
Nowobv
iou
sly,eve
rypa
tte
rncanbeob
tai
nedf
romaf
ini
te
numbe
rofs
uchexchangei
fwes
tar
tfrom wr
itng1,2,...,n
i
cons
ecu
tive
lyi
nac
lockwi
se way,andi
nthei
nit
ials
itua
tion,
B = n -1,A = 0.SoA andB haved
iff
eren
tpa
rit
y.
So
lut
ion2.St
art
ingf
rom oneve
rtex,wedeno
tet
henumbe
rs
wr
itt
enont
heve
rti
cesbyx1 ,x2 ,...,xn i
nac
lockwi
seway.
Wemaya
ssumet
hatt
henumbe
rswr
itt
enont
heendpo
int
sof
edgeei a
rexi ,xi+1 ,i = 1,2, ...,n,whe
rexn+1 = x1 .
Appar
entlyeii
sposi
tiveifandonlyifxi+1 -xi >0.Now,l etA
bethe number of pos
iti
ve edge ,
s and B bet he numbe
r of
c
ros
sing
s.Wr
ite
n

β= ∏ (x
i=1
i+1 -xi ).
Ch
inaGi
rls
􀆳Ma
thema
tic
alOl
ymp
iad 209

seven,t
Asni hes
igno
fβi
sju
st(-1)
A
.
Ont
heo
t rhand,{
he ei ,ej }i
sac
ros
singi
fandon
l f2|
yi
i +jand

(
xj -xi)(
xj -xi+1)(
xj+1 -xi)(
xj+1 -xi+1) < 0.

Wedeno
tebyf(
ei ,ej )t
hel
eft
-hands
ideo
fabovei
nequa
liy,
t
Mathematical Olympiad in China (2011–2014) Downloaded from www.worldscientific.com

obv
iou
slyf(
ei ,
ej )=f(
ej ,
ei).Th
isquan
tit
yisnega
tivei
fand
by NATIONAL UNIVERSITY OF SINGAPORE on 05/06/18. For personal use only.

on
lyi
fthetwo
-edges
eti
sac
ros
sing.Now,de
fine

α= ∏ (
xj -xi)(
1≤i<j≤n
xj -xi+1)(
xj+1 -xi)(
xj+1 -xi+1)
2|
i+j

= ∏ f(
e ,e ),
1≤i<j≤n
i j

2|
i+j

t
hent
hes
igno s(-1)
fαi B
.Le
tusca
lcu
lat
ethes
ignofαβ.For
1 ≤i < j ≤n,con
side
rthet
ime
sofappea
r e,andt
anc hes
ign
o nα andβ,r
fxj - xi i espec
tive
ly. We d
ist
ingu
ish s
eve
ral
ca
ses.
j -i =1,xj -xiappea
CaseI. rsoncei
nβwi
thapo
sit
ives
ign,
andonc nα.
ei Ifi >1,
itappea
rsi ei-1 ,
nf( ei+1)wi
thapo
sit
ive
s
i fi =1,x2 -x1appea
gn.I r nf(
si e2 ,
en )wi
thanega
tives
ign.
Theproduc
toft
hes
enumbe
rsha
sanega
tives
ign.
I.2 ≤j -i < n -1,thenxj
CaseI -xi doe
sno
tappea
rinβ,
andappea
rinαtwi
ce.Among (
i - 1,j - 1)and (
i - 1,j),
t
her
eisexac
tlyonepa
irt
hati
soft
hes
amepa
riy,among (
t i,
j -1)and (
i,j),thereisa
lsoexact
lyonesuchpair.Thesign
ofxj -xiineachappear
anceisalwayspo
sit
ive.Fori = 1,
its
appea
ranceinf(ej-1 ,en )orf(ej ,en )come
swi
thanega
tive
si
gn.Henc e,t
hereisanega ti
vesignforeachpar(
i i,j)(
i=
1,j = 3,4,...,n -1).Th
ispa
rtoft
heproduc
tha
sthes
ign
(-1)
n-3
= -1.
210 Ma
thema
tic
alOl
ymp
iadi
nCh
ina

CaseI
I i = 1,j
I. = n,thenxn -x1 appea
r nβ wi
soncei tha
nega
tives
ign,andonc nα (
ei i en-1 ,e1 ))wi
nf( tha po
sit
ive
s
ign.Thi
spartoftheproduc
thasanega t
ivesi
gn.
Inbr
ief,thes
ignofαβisnega
tive,i.e.,A +Bi
sodd.
Mathematical Olympiad in China (2011–2014) Downloaded from www.worldscientific.com
by NATIONAL UNIVERSITY OF SINGAPORE on 05/06/18. For personal use only.
Ch
inaWe
stern Mathema
tica
l
Ol
ymp iad
Mathematical Olympiad in China (2011–2014) Downloaded from www.worldscientific.com
by NATIONAL UNIVERSITY OF SINGAPORE on 05/06/18. For personal use only.

2010 (
Tai i)
yuanShanx

28 29Oc
tob
er,2010

1 Suppo
set
hatm andka
renon
-nega
tivei
ntege
rs,andp =
m
22 +1i
sapr
imenumbe
r.Provet
hat
(
a)22 ≡ 1( );
m+1 k
p
modp
k+1

(
b)2m+1pki
sthesma
lle
stpo
sit
ivei
nt rns
ege ati
sfi
yngt
he
cong
ruenceequa
tion2n ≡ 1(
modpk+1).
m+1 k
So
lut
ion.Wewanttoprovethat22 p
= p +tk +1f
k 1
ors
ome
i
nt rtk no
ege tdi
vis
ibl
ebyp.We proceedbyi
nduc
tiononk.
212 Ma
thema
tic
alOl
ymp
iadi
nCh
ina

Whenk = 0, t22 = (
p -1) =
m m
itfo
llowsf
rom22 = p -1t
ha 2

p(
p -2)+1,
int
hisca
se,
t0 = p -2.
ep,s
m+1 k
Fori
nduc
tives
t uppo
set
hat22 p
= p +tk +1wh
k 1
er
e
k ≥ 0andp tk ,
then

=( p =(
) p tk +1)
m+1 k+1 m+1 k
22 p
22 p k+1 p

æp ö
Mathematical Olympiad in China (2011–2014) Downloaded from www.worldscientific.com

∑ çès ÷ø (p tk )
k+1 s
=
by NATIONAL UNIVERSITY OF SINGAPORE on 05/06/18. For personal use only.

s=0

p -1)(k+1 )
p( æp ö k+1 s
p

= 1 +p·p +tk + p tk + ∑ ç ÷ ( p tk ).
k 1 2

2 s=3 ès ø

Ask ≥ 0,
thenforanys ≥ 2,wehave (
k +1)
s ≥ 2(
k +1)=
k +2 ≥k +2, =p +tk+1 +1,wh
m+1 k+1
2 so22 p k 2
eetk+1 ∈ Z+ andp
r
tk+1 .I
tfo
llowsf
rom ma
thema
tica
linduc
tiont
hat(
a)ho
lds.
t,wepr
Nex ove (
b).Wr
it ℓ +r ,whe
e2m+1pk =n reℓ,r ∈
rom (
a)t
m+1 k
Zand0 ≤r <n.Theni
tfo
llowsf hat1 ≡22 p
≡2
n
ℓ+r

2n )
≡( ·2r ≡ 2r (
modpk+1).As0 ≤r <n,
itfo
llowsf
romt
he

de
fin
iti
onofnt
hatr =0,
i.e.,
n|2m+1pk .Byt
heFundamen
tal
ft ≤ m ,t
c,n = 2tps .I
m k
Theor
em o
f Ar
ithme
ti hen22 p
=
( ) ≡ 1(
modp).Ont rhand,2 = (
2 )
t k m-t m k m k
2 2p 2
heo
the 2 p 2 p

(-1) ≡ -1(
modp),wh on,andhencet =
k
p
ichi
sacon
trad
ict
i
m +1,i.
e.,n = 2m+1ps .

2 Ass
hown i
nFi
g .1,AB i
.2 sa
d
iame
terofac
irc
le wi
th cen
ter
O .Le
tC andD betwod
iff
eren
t
poi
ntsont
hecir
cleonthesame
s
ideo ,
fAB andthel
ine
stangen
t
t
othec
irc
lea
t po
insC and D
t
mee
tatE .Segmen
tsAD andBC
mee
tatF .Li
nesEF andAB mee
t F
ig.2
.1
Ch
ina We
ste
rn Ma
thema
tic
alOl
ymp
iad 213

tM .Provet
a hatE ,C ,M andD a
reconcyc
lic.
So
lut
ion1.Asshownin Fi
g .2,j
.2 oinOC ,OD andOE .I
t
f
oll rom ∠OCE + ∠EDO = 90
owsf °+90
° = 180
°t tECODi
ha s
cycl
i fO = M ,
c.I thenECMDi scyc
lic.Wemaya s
sumet tO
ha
≠ Minthefo
llowi
ng.LetG betheinte
rsec
tionofBC andAD ,
andH1bet
hei
nte
rsec
tionofGF andAB .Le
tH2 bet
hefoo
tof
Mathematical Olympiad in China (2011–2014) Downloaded from www.worldscientific.com

pe
rpend
icu
larofEt
oAB .I tfo
ll romAC ⊥ BG andBD ⊥
owsf
by NATIONAL UNIVERSITY OF SINGAPORE on 05/06/18. For personal use only.

AG t
hatF ist
heorthocen
terof △BAG ,s
oGH2 ⊥ AB .As
∠CH 2D = ∠CBF + ∠DAF = 1 °-2∠BGA ,and ∠COD =
80
180 °- (
° - ∠BOC - ∠AOD = 180 ° -2∠GBA )- (
180 180
°-
2∠GAB )=2(∠GBA + ∠GAB)-1
80°-2∠BGA ,
then ∠CH2D
= ∠COD ,soC ,O ,H2 andD a
reconcyc
lic.I
tfo
llowsf
rom
∠ECO = ∠EDO = ∠EH 1O =90
°t tE ,C ,O ,H1andD a
ha re
concyc
lic.Hence,H1 = H2 ,i.
e.EF ⊥ AB andt
hey mee
tat
M .Cons
equen
tly,E ,C ,M andD a
reconcyc
lic.
So
lut
ion2.AsshowninFi
g .2,j
.2 oin
EO ,CO , DO ,CA ,CD .I
tfo
llows
f
rom ∠COE = ∠CAFt
hatRt△COE ∽
CE CO
Rt△COF ,ands
o = .As ∠ECF
CF CA
=9°- ∠BCO = ∠OCA ,
0 so△ECF ∽
△OCA ,andhenc
e ∠CFE = ∠CAO .
As ∠BFM = ∠CFE ,s
o ∠FBM +
∠BFM = ∠FBM + ∠CAO = 90
°. F
ig.2
.2

Hence,EM ⊥ AB ,i
tfo
llowst
hatO ,
M ,D ,E ,C a
reconcyc
lic.

3 De
termi
nea
llpo
ssi
bleva
lue
sofpo
sit
ivei
nt rn,s
ege uch
t
hatt
her
eaend
r iff
er t3 e
en lemen
tsub
sesA1 ,A2 ,...,
t
An oft
hes
et{
1,2,...,n},wi
th|Ai ∩ Aj |≠ 1fora
ll
214 Ma
thema
tic
alOl
ymp
iadi
nCh
ina

i ≠j.
So
lut
ion.The s
eto
f po
sit
ivei
ntege
rss
ati
sfi
yng t
he g
iven
cond
iti
oncon
sis
tsofa
llpo
sit
ive mu
lti
ple
sof4.Wef
irs
tpr
ove
t
ha k(
tn =4 k ∈ Z+ )
sat
isf
yingt
hec
ond
iti
on.De
fineA1,A2,...,
A4k a
sfo
llows:A4i-j = {
4i -3,4
i -2,4
i -1,4
i}\{
4i -j},
for
a
ll1 ≤i ≤kand0 ≤j ≤ 3.
Mathematical Olympiad in China (2011–2014) Downloaded from www.worldscientific.com

cond,wewan
Se ttoprovet
ha f4 n ,t
ti uchns
hens ubs
ets
by NATIONAL UNIVERSITY OF SINGAPORE on 05/06/18. For personal use only.

dono
tex
ist.Suppo
set
othe con
tra
ryt
hatA1 , ...,An be
d
iff
er t3 e
en lemen
tsub
set
sfu
lfi
ll
ingt
hecond
iti
ons
tat
edi
nthe
problem.LetA1 = {
a,b,c},consi
dera
llt
heg
iven3 element
subs
etswit
hnon-emp tyi
nte
rse
cton,we maya
i ssumetha
tt hey
a
reA2 ,...,Am a
fte
rre
labe
li tU = A1 ∪ A2 ∪ … ∪ Am .
ng.Le
Wed
ivi
dei
ntot
hefo
llowi
ngd
iff
eren
tca
ses:


f|U |= 3,
I thenm = 1 <|U |.
æ4ö

f|U |= 4,
I thenm ≤ ç ÷ = 4 =|U |.
è3ø

f|U|≥5,
I thenweprovet
hatm <|U|a
sfo
llows.

Suppo
set
hatm ≥|U|,
thenf
orany2 ≤i,
j ≤ m ,wehav
e|A1 ∩
|A1 ∩ Aj|=2.As|A1|=3,wehav
Ai |=2, eAi ∩ Aj ≠ ⌀.
Andi
tfo
llowsf
rom|Ai ∩ Aj |≠ 1t
hat|Ai ∩ Aj |= 2.I
t
meanst
hatanytwod
ist
inc
tsub
set
sofA1 ,...,Am haveon
ly
twocommone
lemen
ts.
Cons
ide
rthefou
rin
ter
sec
tionsA1 ∩ A2 ,A1 ∩ A3 ,A1 ∩ A4 ,
A1 ∩ A5 ,i
tfo
llowsf
romt
hep
igeon
-ho
lepr
inc
ipl
etha
tthe
re
a
retwoi
nte
rse
cti
onst
hata
r l;byr
eequa elabe
ling aga
in,we
maya
ssumet
hatt
heya
reA2 = {
a,b,d},A3 = {
a,b,e}.
Thenforany4 ≤ i ≤ m ,wehavea,b ∈ Ai ;o
the
rwi
se,Ai
c
ont
ain
satl
eas
toneo rb,
fao andt
heo
the
rth
reee
lemen
tsc,dand
Ch
ina We
ste
rn Ma
thema
tic
alOl
ymp
iad 215

e,
int
hisc
as |Ai|≥4,wh
e, ichi
simpo
ssi
ble.He
nce|U|=m +2,
wh
ichc
ont
rad
ict
sto|U|= m.
Fr
omt
hea
rgumen e,onec
tabov and
ivi
det
hes
ubs
etsA1,...,
Ani
ntos
eve
ralg
roups,andanytwos
ubs
etsi
nthes
ameg
roup
havenon
-emp
tyi
nte
rsec
tion
s.Mor r,t
eove henumbe
rofs
ubs
ets
i
nthes
ame g
roupi
sno
t mor
ethant
he numbe
rofe
lemen
ts
Mathematical Olympiad in China (2011–2014) Downloaded from www.worldscientific.com

appear
edinthisgroup.Asthenumberofs
ubse
t sn,wh
si i
chis
by NATIONAL UNIVERSITY OF SINGAPORE on 05/06/18. For personal use only.

equalthenumberofelemen
tsin{1,2,...,n},s
oeachg
roup
ha
sexac
tlyf
ours
ubs
ets.I
tfo
llowst
hat4|n ,wh
ichcon
trad
ict
s
t
heor
igi
nala
ssump
tion4 n.

ta1,a2,...,an ,
4 Le b1,
b2,...,
bn benon
-ne
gat
ivenumb
ers
s
ati
sfi
yngt
hef
oll
owi
ngc
ond
iti
onss
imu
ltaneou
sly:
1)∑i=1(
( ai +bi)= 1 ;
n

2)∑i=1i(
( ai -bi)= 0 ;
n

3)∑i=1i2(
( ai +bi)= 10.
n

10
Provet
hatmax{
ak ,bk }≤ f
ora
ll1 ≤k ≤ n.
10 +k2
So
lut
ion.For any 1 ≤ k ≤ n,i
tfo
llowsf
rom t
he g
iven
cond
iti
onsandCauchy
􀆳sInequa
lit
ytha
t
n n
(
kak ) ( ∑ia ) ( ∑ib )
2 2 2
≤ i = i
i=1 i=1

n n

≤ ( ∑ib ) ( ∑b )
i=1
2
i
i=1
i

n n

= 10- ∑iai ) (1 - ∑ai )


2
( i=1 i=1

≤(
10 -ka2ak )(
1 -ak )
=1 10 +k2)
0- ( ak +k2ak
2
.
216 Ma
thema
tic
alOl
ymp
iadi
nCh
ina

10 10 ,
I
tfo
llowsf
romt
hatak ≤ .S
imi
lar
ly,
bk ≤ and
10 +k2 10 +k2
hencet
her
esu
ltf
oll
ows.

5 tkbeani
Le nt randk > 1.De
ege fi equence {
neas an }a
s
fo
ll a0 = 0,a1 = 1andan+1 =kan +an-1forn = 1,
ows:
2,....
Mathematical Olympiad in China (2011–2014) Downloaded from www.worldscientific.com
by NATIONAL UNIVERSITY OF SINGAPORE on 05/06/18. For personal use only.

De
te ne,wi
rmi t f,a
h proo ll po
ssi
blek for wh
icht
her
e
ex
istnon
-nega
tivei
ntege
rsℓ,m (
ℓ ≠ m )and po
sit
ive
i
ntege
rsp,qs
ucht
hataℓ +kap = am +kaq .
So
lut
ion.Theansweri
sk = 2.

fk = 2 ,t
I hena0 = 0,a1 = 1,a2 = 2,
soa0 +2
a2 =a2 +
a1 = 4.Hence,(
2 ℓ,m )= (
0,2)and (
p,q)= (
2,1).
Fork ≥ 3 ,i
tfo
llowsf
romt
her
ecur
ren
tre
lat
iont
hatt
he
sequence{an }
isst
r i
ctl
yincr
eas
ing,andk|an+1 -an-1fora
lln ≥
1.Inpa rt
icu
l ar,forn ≥ 0,wehave

a2n ≡ a0 ≡ 0(
modk),a2n+1 ≡ a1 ≡ 1(
modk). (* )

Suppo
set
her
eex
istℓ,m ∈ N,andp,
q ∈ Z+s
ucht
hatℓ ≠ m and
aℓ +kap =am +kaq .Wemaya
ssumet
hatℓ < m ,andd
ivi
dei
nto
t
hef
oll
owi
ngca
ses.
(
a)p <ℓ < m :Themaℓ +kap ≤aℓ +kaℓ-1 <kaℓ +aℓ-1 =
aℓ+1 ≤ am < am + kaq , wh
ich con
trad
ict
sthe or
igi
nal
a
ssump
tion.
(
b)ℓ =p < m :
Ifℓ =p = m -1,
thenwehaveam +kaq =
aℓ +kap = (
k +1)
am-1 ,andmodu
lok wehaveam ≡am-1 modk,
wh
ichcon
trad
ics(* ).
t
fℓ = p < m -1,t
I henaℓ +kap < am-2 +kam-2 = am <
am +kaq ,wh
ichcon
trad
ict
stheor
igi
nala
ssump
tion.
(
c)ℓ < p < m :Int
hisca
s romq wehaveaq > 0,t
e,f hen
Ch
ina We
ste
rn Ma
thema
tic
alOl
ymp
iad 217

aℓ +kap ≤kap +ap-1 =ap+1 ≤am <am +kaq ,wh


ichcon
trad
ict
s
t
heor
igi
nala
ssump
tion.
aℓ +kap -am
(
d)ℓ < m ≤ p:Thenap > =aq .
Itf
oll
ows
k
f
rom

kaq +am =kap +aℓ ≥kap ①


Mathematical Olympiad in China (2011–2014) Downloaded from www.worldscientific.com
by NATIONAL UNIVERSITY OF SINGAPORE on 05/06/18. For personal use only.

t
hat

am ap k -1
aq ≥ ap - ≥ ap - = ap . ②
k k k

No
tet
hat

ap =kap-1 +ap-2 ≥kap-ℓ , ③

hence

k -1
ap > aq ≥ ap ≥ (
k -1)
ap-1 ≥ ap-1 . ④
k

Thenbyi
ncr
eas
ingpr
ope
rtyof{
an }and ④ ,wehaveaq =
ap-1 ,s
oinequa
lit
ies① ③ t
urni
ntoequa
lit
ies.I
tfo
llowsf
rom
②t
hatm = p,andf
rom ③t
hatp = 2.Hence,
aq =ap-1 =a1
= 1.Andi
tfo
ll rom ① t
owsf hatℓ = 0.Fromaℓ +kap =am +
kaq ,wehavek2 =k +k = 2
k,s
ok = 2,wh
ichi
simpo
ssi
ble.
Sok = 2i
stheon
lys
olu
tion.

6 As s
hown i
n F
ig .1,
.6
△ABC i
sa r
igh
t-ang
led
t
riang
le,∠C =90
°.Dr
aw
ac
irc
lec
ent
er tB wi
eda th
r
adi
us BC.Le
tD bea
po
intont
hes
ideAC,and
F
ig.6
.1
DE be t
ang
entt
othe
218 Ma
thema
tic
alOl
ymp
iadi
nCh
ina

c
irc
l tE.Thel
ea inet
hroughC pe
rpend
icu
la oAB mee
rt ts
l
ineBE a
tpo
intF .Li
neAF mee
tsDE a
tpo
intG .Thel
ine
hroughA pa
t ral
le oBG mee
lt tsDE a
tH .Pr
ovet
hatGE =
GH .
So
lut
ion. Le
t K be t
he
i
nte
rse
cti fABandDE,and
ono
Mathematical Olympiad in China (2011–2014) Downloaded from www.worldscientific.com

M bet
hei
nte
rse
cti fABand
ono
by NATIONAL UNIVERSITY OF SINGAPORE on 05/06/18. For personal use only.

CF.
JonFK ,AE andME.I
i n
△ABC,
itf
oll
owsf
romCM ⊥
AB t
hatBM ·BA = BC2 =
BE2,s
o △BEM ∽ △BAE, F
ig.6
.2

and△BEM = ∠BAE .
°,s
As ∠FMK = ∠FEK = 90 o MFEK i
scyc
lic,and
∠BEM = ∠FKM .I
t fo
ll rom ∠BAE = ∠BEM
ows f =

∠FKM ,oFK ‖AE ,andhenc


s e

KA EF , KA ·BF
= e.,
i. = 1. ①
KB BF KB FE

Ast
hel
ineEGAi
nte
rse
cts△EBK ,wehave

EG ·KA ·BF
=1 ②
GK AB FE

HK AK ,
AsBG ‖AH ,wehave = so
KG KB

HG AB
= . ③
GK BK

EG
om ① ③ ,wehave
Fr = 1, oEG = HG .
ands
HG

7 The
rea
ren(
n ≥ 3)p
laye
rsi
nat
abl
etenn
ist t,
ournamen
i
nwh
ichanytwop
laye
rshaveama
tch.P
l rAi
aye sca
lled
Ch
ina We
ste
rn Ma
thema
tic
alOl
ymp
iad 219

no
tou
t-pe
rfo
rmedbyp
lay
erB,i
fatl
eas
toneo
fpl
ayrA􀆳s
e
l
ose
rsi taB􀆳sl
sno ose
r.
De
te ne,wi
rmi t f,a
hproo llpo
ssi
bleva
l sofn,
ue sucht
hat
t
hef
oll
owi
ngca
secou
ld happen:a
fte
rfi
nish
ing a
llt
he
ma
t s,eve
che r l
ypaye
risno
tou
t-pe
rformedbyanyo
the
r
l
paye
r.
Mathematical Olympiad in China (2011–2014) Downloaded from www.worldscientific.com

So
lut
ion.Theansweri
sn = 3orn ≥ 5.
by NATIONAL UNIVERSITY OF SINGAPORE on 05/06/18. For personal use only.

(
1)Forn = 3 ,s
uppo
seA ,B andC a
ret
hreep
laye
rs,and
t
her
esu
ltoft
hreema
tche
sar
easf
olows:A wi
l nsB ,B wi
nsC ,
andC wi
nsA .The
ser
esu
ltsobv
iou
slys
ati
sfyt
hecond
iti
on.
( fn = 4 ,s
2)I uppo
set
hatt
hecond
iti
onho
lds,i.
e.,i
n
v
iew o
fther
esu
ltsofa
ll ma
t s,eve
che r l
ypaye
ris no
tou
t-
performedbyanyotherpl
ayer.Itisobvi
ou stha
tnoneofthes
e
fourp l
aye
rswinsi
nh i
sthreegames,otherwie,t
s heo
the
rthr
ee
l
paye
rs wi
llbe no
tou
t-pe
rformed by t
hisp
laye
r.S
imi
lar
ly,
noneo
fthe
set
hreep
laye
rsl
ose
sinh
ist
hreegame
s.I
tfo
llows
t
hateachp
laye
rwi
nsoneortwoma
tche
s.
Fort
hep
l rA ,a
aye ssumet
ha nsB andD ,bu
tA wi tlo
sest
o
C,
thenbo
t nC ;o
hB andD wi the
rwi
se,t
hey wou
ldno
tou
t-
pe
rformedA .Forthel
ose
ri nthema
t s.D ,heon
chB v ly wi
ns
,
C andsotheloseri
simposs
ibl
etobenotou
t-pe
rformedbythe
wi
nne
r.Con
sequen
tly,forn = 4,
theg
ivencond
iti
oncou
ldno
t
happen.
3) For n = 6,one can
(
cons
truc
tthe t
ournamen
tre
sul
ts
by meanso
fthefo
llowingdir
ected
gaph,i
r n whi
ch each bl
ack dot
r
epr
esen
tsa p
l r,and · → 􀳱
aye
r
epr
esen
tsama
tch wi
tht
her
esu
lt
t
hatp
l r·wi
aye nsp
laye
r􀳱. F
ig.7
.1
220 Ma
thema
tic
alOl
ymp
iadi
nCh
ina

(
4)I
fthe
reex
istt
ournamen
tre
sul
tss
ucht
ha fn
teacho
pl
aye
rsAi( 1 ≤i ≤n)i
snotou
t-performedbyanyotherpl
ayer,
wewillprovethatt
hesameholdsforn +2p l
ayer
sa sfo
llows:
Suppo
set
hatM andN a
ret
headd
iti
ona
lpl
aye
rst
otheor
igi
naln
l
paye
rs.Con
str
uctt
hegamer
esu
lt fM andN a
so sfo
llows:

Ai → M ,M → N ,N → Ai
Mathematical Olympiad in China (2011–2014) Downloaded from www.worldscientific.com
by NATIONAL UNIVERSITY OF SINGAPORE on 05/06/18. For personal use only.

f
ora
lli = 1,2,...,n,andt
heg
amer
esu
lts
amongAi􀆳sa
res
ti
llt
heo
rii
gna
lone
s.Now we
wan
ttoche
ckt
hatt
heen +2p
s lay
erss
ati
sfyt
he
i
gvencond
iti
on.Fo rany p
lay
erG ∈ {A1,
A2,...,An },t
heni
tsuf
fic
estoc
ons
ide
rthe F
ig.7
.2
l
pae
yrsG,M ,N ,andi
tredu
cest
othec
asen
= 3.

Onec
anche
ckt
hate
acho
fthe
set
hre
epl
aye
rsG,M ,Ni
sno
t
ou
t-pe
rfo
rmed by any on
eoft
heo
the
rtwo p
lae
yrs.He
nce,t
he
t
our
name
ntr
esu
lto
fth
esen +2p
lae
yrss
ati
sfi
est
her
equ
ire
dcond
it
ion.
Inpa
rti
cul
ar,i
tfo
llowsf
rom (
1)andt
hei
nduc
tions
tepof
(
4)t
hatt
her
equ
iredcond
iti
onho
ldsforanyoddn wi
thn ≥3.
Mor r,i
eove tfo
llowsf
rom (
3)andt
hei
nduc
tions
tep of (
4)
t
hatt
her
equ
iredcond
iti
onho
ldsforanyevenn wi
thn ≥6,and
i
tcomp
let
est
heproo
f.

8 De
termi
nea
llpo
ssi
bleva
lue
sofi
nt rkforwh
ege icht
her
e
b +1 a +1
ex
istpo
sit
ivei
ntege
rsaandbs
ucht
hat + =k.
a b
So
lut
ion.Theansweri
st tk
ha = 3or4.
Fi
xa po
ssi
ble va
luek,among a
llt
he pa
irs(
A ,B )of
po
sit
ivei
ntege
rss
ati
sfi
yng

B +1 A +1
+ =k,
A B
Ch
ina We
ste
rn Ma
thema
tic
alOl
ymp
iad 221

choo
seany(
a,b)
sucht
hatbi
sthesma
lle
st.Thent
hequadr
ati
c
equa
tion

1 -kb)
x2 + ( x +b2 +b = 0

ha
sani
nteg
ralr tx = a.Le
oo tx = a
' bet
hes
e t,i
condroo t
fo
llowsf ' =kb -1t
roma +a ha ' ∈ Z ,andf
ta rom

' =b(
b +1)
Mathematical Olympiad in China (2011–2014) Downloaded from www.worldscientific.com

a ·a
by NATIONAL UNIVERSITY OF SINGAPORE on 05/06/18. For personal use only.

t
ha ' > 0.Hence,wehave
ta

b +1 a' +1
+ =k.
a' b

Andi
tfo
llowsf
romt
hea
ssump
tiononbt
hat

a ≥b,a
' ≥b.

faanda
Sooneo 'isequa
ltob.Wi
thou
tlo
sso
fgene
ral
ity,we
2,
maya
ssumea =b,
sok = 2 + ands e.,
ob|2i. b = 1or2,
b
andk = 3or4,r
espec
tive
ly.
fa = b = 1 ,t
I henk = 4;i
fa = b = 2,t
henk = 3.
Cons
equen
tly,
k = 3or4i
stheon
lys
olu
tion.

2011 (
Yushan,Ji
angi)
x

F
irs
tDay
8:
00 12:
00,Oc
t r29,2011
obe

1 Gi
vent
hat0 < x,y < 1,de
te ne,wi
rmi th proof,t
he
xy(1 -x -y)
max
imumva
lueo
f( .(
pos
edbyLi
u
x +y)(
1 -x)(1 -y)
222 Ma
thema
tic
alOl
ymp
iadi
nCh
ina

Sh
ixong)
i
1, 1
So
lut
ion.Whenx =y = t
heva
lueofexpr
ess
ioni
s .
3 8
xy(1 -x -y) 1
Wewi
llprovet
hat( ≤ f
orany
x +y)(
1 -x)(1 -y) 8
0 < x,y < 1a
sfo
llows.
xy(1 -x -y) 1
Mathematical Olympiad in China (2011–2014) Downloaded from www.worldscientific.com

fx +y ≥ 1,
I then ( ≤0 < .
x +y)(
1 -x)(1 -y) 8
by NATIONAL UNIVERSITY OF SINGAPORE on 05/06/18. For personal use only.

fx +y < 1,
I thenl
et1 -x -y = z > 0,
itfo
llowsf
rom
AM GMInequa
lit
ytha
t

xy(1 -x -y) xyz


=
(
x +y)(
1 -x)(1 -y) (x +y)(
y +z)(
z +x)
xyz 1
≤ = .
· ·
2 xy 2 yz 2 zx 8

1
Inconc
lus
ion,t
hemax
imumva
lueoft
heexpr
ess
ioni
s .
8

2 tM ⊆ {
Le 1,2, ...,2011}beas
ubs
ets
ati
sfi
yngt
he
fo
llowi
ngcond
iton:Foranyt
i hreee
lemen
t nM ,t
si her
e
ex
isttwo o
fthem a andb,s
ucht
hata |b orb |a .
De
te ne,wi
rmi th proof,t
he max
imum va
lueof| M |,
whe
re|M |deno
test
henumbe
rofe
lemen
tsofM .(
pos
ed
byFengZh
igang)
So
lut
ion.OnecancheckthatM ={
1,2,22 ,23 ,...,210 ,3,
3 ×2,3 ×22 ,...,3 ×29}s
ati
sfi
est
hecond
iton,and|M |=
i
21.
Suppo
set
hat|M |≥ 22,andl
eta1 < a2 < … < ak bet
he
e
lemen
tsofM ,whe
re|M |=k ≥22.Wef
irs
tprovet
hatan+2 ≥
2
an fora
lln;o
the
rwi
se,wehavean < an+1 < an+2 < 2
an for
Ch
ina We
ste
rn Ma
thema
tic
alOl
ymp
iad 223

omen <k +2,


s thenanytwooft
hes
ethr
eei
ntege
rsan ,an+1 ,
an+2 dono
thaveany mu
lti
pler
ela
tionsh
ip,wh
ichcon
trad
ict
s
t
hea
ssump
tion.
I
tfo
llowsf
romt
hei
nequa
lit
yabovet
ha a2 ≥ 4,a6
ta4 ≥ 2
a4 ≥8,… a22 ≥2a20 ≥2 >2011,wh
≥2
11
ichi
sacon
trad
ict
ion!
e,t
Henc hemax
imumva
lueo s21.
f|M |i
Mathematical Olympiad in China (2011–2014) Downloaded from www.worldscientific.com
by NATIONAL UNIVERSITY OF SINGAPORE on 05/06/18. For personal use only.

3 tn ≥ 2beag
Le iveni
ntege
r.
(
1)Provet
hatonecana
rrangea
llt
hes
ubs
etsoft
hes
et
{
1,2, ...,n }a
sas
equenceofs
ubs
etsA1 ,A2 , ...,
A2n ,s
ucht
hat|Ai+1|=|Ai|+1or|Ai|-1,whe
rei =
1,2,...,2n andA2n+1 = A1 .
(
2)De
te ne,wi
rmi t f,a
hproo llpo
ssi
bleva
lue
soft
hes
um

∑ Ai)= ∑x∈A xandS(⌀ )


n
(-1)S(
Ai),wheeS (
2
i=1
i
r
i

= 0,foranys
ubs
e equenceA1 ,A2 ,...,A2n s
ts ati
sfi
yng
t
hecond
iti n(
oni 1).(
pos
edbyLi ngde)
ang Yi
So
lut
ion.(
1)Weproveby ma
thema
tica
linduc
tiont
hatt
her
e
ex
ist
sas eA1 ,A2 ,...,A2ns
equenc ucht
hatA1 = {
1},A2n = ⌀
ands
ati
sfi
est
hecond
iti n(
oni 1).
Whenn = 2,t
hes e{
equenc 1},{
1,2},{
2},⌀ of {
1,2}
work
s.
As
sumet
hatwhenn = k,t
her
eex
ist
ssuchas eB1 ,
equenc
B2 ,...,B2k ofs
ubs
etsof{
1,2,...,k}.Asf
orn =k +1,one
cancons
truc
tas
equenceo
fsub
set
sof{
1,2,...,k + 1},a
s
fo
llows:

A1 = B1 = {
1},
Ai = Bi-1 ∪ {
k +1},
i = 2,3,...,2k +1,
Aj = Bj-2k ,j = 2k +2,2k +3,...,2k+1 .

Onecanea
sil
ycheckt
hatt
hes
equencef
ulf
ill
sther
equ
ired
224 Ma
thema
tic
alOl
ymp
iadi
nCh
ina

cond
iti
onss
tat
edabove.Byi
nduc
tionwehaveproved (
1)forn
≥ 2.
(
2) We wi
llshowt
hatt
hes s0,i
um i ndependen
toft
he
a
rrangemen
t.Wi
thou
tlo
ssofgene
ral
ity,we maya
ssumet
hat
A1 = {
1},o
the
rwi
sesh
iftt
hei
ndexcyc
lica
lly.I
tfo
llowsf
rom
|Ai+1 | =|Ai|+1or|Ai|-1t
hatt
hei
rpa
rit
iesa
red
iff
er t,
en
Mathematical Olympiad in China (2011–2014) Downloaded from www.worldscientific.com

andhencet
hepa
rit
iesoft
hei
ndexl
abe
lofanys
ubs
etandi
ts
by NATIONAL UNIVERSITY OF SINGAPORE on 05/06/18. For personal use only.

ca
rdi
nal
itya
ret
hes
ame.

t∑i=1(-1) Ai)= ∑A ∈PS (


A )- ∑A ∈Q
n

S(
2
I
tfo
llowst
ha i

S (
A ),whe
reP cons
ist
sofa
lls
ubs
et f{
so 1,2,...,n}wi
th
evennumbe
rsofe
lemen
ts,andQcons
ist
sofa
lls
ubs
etsof{
1,2,
...,n}wi
thoddnumbe
rsofe
lemen
ts.
1,2,...,n},amonga
Foranyx ∈ { llk
-el
emen
tsub
ses,
t x
appea
rsi
nexac
tlyCn-1 o
k-1
fthem,hencei
tcon
tri
but
est
othes
um

∑ A ∈P
A )- ∑A ∈Q S (
S( A )a
s

- Cn-1 + Cn-1 - Cn-1 + … + (-1)Cn-


-1 = - (
1 -1)- = 0.
0 1 2 n n 1 n 1

e,∑i=1(-1)
n

S(
Ai)= 0.
2
The
refor i

4 Ass
howni
nFi
g .1,AB andCD a
.4 re
twochord
sint
hec
irc
le☉O mee
ting
a
tpo
intE ,and AB ≠ CD . ☉I i
s
t
angen
tto☉Oi
nte
rna
llya
tpo
intF ,
andi
stangen
ttot sAB and
hechord
CD a
tpo
insGandH ,
t res
pec
tive
l l
y.
i
sal
inepa
ssi hroughO ,mee
ngt ting
AB,CD a
tpo
insP,Q,r
t espe
cti
vey,
l
s
ucht
hatEP = EQ .Li
neEF mee
ts
t
hel
inela
tpo
intM .Pr
ovet
hatt
he F
ig.4
.1
Ch
ina We
ste
rn Ma
thema
tic
alOl
ymp
iad 225

l
i hroughM and pa
net ral
lelt
othel
ineAB i
stangen
tto
t
hec
irc
le☉O .(
pos
edbyLiQi
usheng)
So
lut
ion.AsshowninFi
g .2,dr
.4 awal
inepa
ral
le oAB and
lt
t
angen
ttoc
irc
le ☉O a
t po
intL ,wh
ich mee
tst
he common
t
angen
tli
net
othe
setwo c
irc
lesa
ta po
intS.Le
tR bet
he
i
nte
rse
cti
ono
fli
nesFSandBA ,andj
oins
egmen
tsLFandGF.
Mathematical Olympiad in China (2011–2014) Downloaded from www.worldscientific.com
by NATIONAL UNIVERSITY OF SINGAPORE on 05/06/18. For personal use only.

F
ig.4
.2

Fi
rst,wepr
ovet
hatt
hepo
insL ,G andF a
t reco
lli
nea
r.As
bo
thSL andSF a
ret
angen
tto☉O ,SL = SF ;a
sbo
thRG and
RF a
ret
angen
tto☉I,RG = RF .
AsSL ‖RG , we have ∠LSF =
∠GRF ,
so

∠LFS =
180
°- ∠LSF 180
°- ∠GRF
=
2 2
= ∠GFR ,

andhenceL ,G andF a
reco
lli
nea
r.
S
imi
lar
ly,a
sshown i
n Fi
g .3,
.4
dr
awal
inepa
ral
le oCD andt
lt angen
t
o☉O a
t tpo
intJ,t
henF ,H andJ a
re
F
ig.4
.3
226 Ma
thema
tic
alOl
ymp
iadi
nCh
ina

co
lli
nea
r.Le
ttangen
tli
nest
othec
irc
le
☉O a
tthe po
insL andJ mee
t tEF a
t
poinsM 1 andM 2 ,r
t espec
tive
ly.Inthe
fo
llowi ,
ng weprovet hatthepoinsM 1
t
and M 2 co
inc
ide.I
tfo
llowsf
rom t
he
homo
the
tyc
ent
er tF mapp
eda ing ☉O
Mathematical Olympiad in China (2011–2014) Downloaded from www.worldscientific.com

M 1E LG
o☉It
t hatLJ ‖GH ,t
hen
by NATIONAL UNIVERSITY OF SINGAPORE on 05/06/18. For personal use only.

=
EF GF
JH M 2E ,
= = eM 1 and M 2
andhenc
HF EF
co
inc
ide.Deno
tet
hispo
intbyK .
Fi
nal
ly, we wan
tto pr
ove t
hat
F
ig.4
.4
po
insM andK a
t lsoco
inc
ide.I
tsu
ffi
ces
t
oshowt
hatK l
iesont
hel
inel.
Asshowni
nFi
g .4,j
.4 oinKO ,a
sKL andKJ a
ret
angen
t
o☉O ,
t o ∠LKO = ∠JKO .
s
No
tet
hatKO b
isec
ts ∠LKJ,
itf
oll
owsf
rom KL ‖AB and
KJ ‖CD t
hatt
hel
ineKO mee
tsl
i sAB andCD wi
ne tht
hes
ame
ang
lesofi
nte
rse
cton,andhenc
i eKOi
sju
stt
hel
inel,i.
e.,K
l
iesont
hel
inel.
e,K i
Henc sju
stt
hei
nte
rsec
tionpo
intM ofl
ineEF andl.

S
econdDay
8:
00 12:
00,Oc
t r30,2011
obe

5 De
te ne,wi
rmi thpr f,whe
oo the
rthe
rei
sanyoddi
ntege
r
n ≥ 3andn d
ist
inc
tpr
imenumbe
rsp1 ,p2 ,...,pn s
uch
t
hata
llpi + pi+1(
i = 1,2,...,n,andpn+1 = p1)a
re
pe
rfec
tsqua
res? (
pos
edbyTaoP
ing
sheng)
Ch
ina We
ste
rn Ma
thema
tic
alOl
ymp
iad 227

So
lut
ion.Theansweri
snega
tive.Suppo
set
hatt
her
eex
istodd
i
nt rn ≥ 3andn d
ege ist
inc
tpr
imenumbe
rsp1 ,p2 , ...,pn
s
ati
sfi
yngt
heg
ivencond
iti
on.
I
falp1 ,p2 ,...,pn a
l reodd,t
heni
tfo
llowsf
rom t
he
gi
vencond
iti
onthatallthesumspi +pi+1ar
emu lt
ipl
esof4,so
t
heprimenumbe
r sp1 ,p2 ,...,pn modu
lo4appeartobe1and
Mathematical Olympiad in China (2011–2014) Downloaded from www.worldscientific.com

3a
lte
rna
tive
ly,andi
tcon
trad
ict
sthef
actt
hatni
sodd.
by NATIONAL UNIVERSITY OF SINGAPORE on 05/06/18. For personal use only.

Ifone o s2,t
f p1 ,p2 , ...,pn i hen wi
thou
tl oss of
gene
ral
ity,we maya s
sumet tp1 = 2.Asbo
ha thp1 + p2 and
pn +p1 a
repr
efe
cts
qua
resandbo
tha
reodd,i
tfo
llowst
hatp2
andpn a
recong
ruen
tto3modu
lo4.S
imi
lart
othed
iscu
ssi
oni
n
t
hef
irs
tca
se,weknowt
hatt
hepr sp2 ,p3 ,...,pn modu
ime lo
4appea
rtobe1and3a
lte
rna
tive
ly,s sodd,wh
on -1i ichi
sa
con
trad
ict
ion.
e,t
Henc her
ear
e no odd i
nt rn ≥ 3 and n pr
ege ime
s
s
ati
sfi
yngt
heg
ivencond
iti
ons.

6 ta,b,c > 0,pr


Le ovet
hat

(a -b)2
(b -c)2
(
c -a)2
+ +
(
c +a)(c +b) (a +b)(a +c) (b +c)(
b +a)
(a -b)2
≥ .
a +b +c2
2 2

(
poedbyLiShenghong)
s
1 1(
So
lut
ion1.I
tfo
llowsf
rom ( b)
a -2 2
+ c)
a -2 2
+(
b-
2 2
c) ≥ 0t
2
hat

a2 +b2 +c2)≥ 2a2 +2ab +2


3( bc +2ac = 2(
a +b)(
a +c),

3(2
owe have (
s a + b)(
a + c) ≤ a + b2 + c2).S
imi
lar
ly,
2
wehave
228 Ma
thema
tic
alOl
ymp
iadi
nCh
ina

( 3 2
b +a)(
b +c)≤ (a +b2 +c2),
2

3 2
and (
c +a)(
c +b)≤ (a +b2 +c2).
2
e,
Henc

(a -b)2
(b -c)2
(
c -a)2
+ +
( )(
c +a c +b ) ( )(
a +b a +c ) (
b +c b +a)
)(
Mathematical Olympiad in China (2011–2014) Downloaded from www.worldscientific.com
by NATIONAL UNIVERSITY OF SINGAPORE on 05/06/18. For personal use only.

2·(
a -b)
2
+( b -c)2
+(c -a)
2

3 2 2
a +b +c 2

( 1(
a -b)
2
+ b -c +c -a)
2

2· 2

3 a2 +b2 +c2
(a -b)2
= .
a +b +c2
2 2

So
lut
ion2.I
tfo
llowsf
rom CauchyInequa
lit
ytha
t

(a -b)2
(b -c)2
(
c -a)2

[ + + ·
]
(
c +a)(c +b) (a +b)(a +c) (b +c)(
b +a)
c +a)(
[( c +b)+ (
a +b)(
a +c)+ (
b +c)(
b +a)]
≥(
|a -b|+|b -c|+|c -a|)
2

≥(
|a -b|+|b -c +c -a|) = 4(
a -b)
2 2
.

And

(
c +a)(
c +b)+ (
a +b)(
a +c)+ (
b +c)(
b +a)
=(
a2 +b2 +c2)+3( c +ca)≤ 4(
ab +b a2 +b2 +c2),

hence

(a -b)2
(b -c)2
(
c -a)2
+ +
(
c +a)(c +b) (a +b)(a +c) (b +c)(
b +a)
4(
a -b)2

(
c +a)(
c +b)+ (
a +b)(a +c)+ (
b +c)(
b +a)
4( a -b)2
(a -b)2
≥ = 2 .
4(
a +b +c ) a +b +c2
2 2 2 2
Ch
ina We
ste
rn Ma
thema
tic
alOl
ymp
iad 229

7 Ass
howni
nFi
g .1,AB >
.7
AC ,andt
hei
nci
rce ☉I o
l f
△ABCi
stangen
ttoBC ,CA
and AB a
t po
insD ,E and
t
F ,r
espec
tive
l tM bet
y.Le he
mi
dpo
int o
f s
ide BC, and
Mathematical Olympiad in China (2011–2014) Downloaded from www.worldscientific.com

AH ⊥ BC a
tthe po
int H.
by NATIONAL UNIVERSITY OF SINGAPORE on 05/06/18. For personal use only.

The b
ise
cto f ∠BAC
r AI o F
i .7
.1
g
i
nte
rse
ctst
hel
i sDE ,DF a
ne t
poinsK ,L ,r
t espe
cti
vel
y.
Provet tM ,L ,H andK a
ha reconcyc
lic.(
pos
edbyBi
an
Hongp
ing)
So
lut
ion.Jo
inCL ,BI,DI,BK ,ML andKH .Ex
tendCLt
o
tAB a
mee tpo
intN .
Asbo
thCD andCE a
ret
het
angen
tto☉I,s
oCD = CE .
As

1 (∠
∠BIK = ∠BAI + ∠ABI = BAC + ∠ABC)
2
1(
= °- ∠ACB )= ∠EDC = ∠BD ,
180
2

oB ,K ,D andIa
s recyc
lic.
As ∠BKI = ∠BDI = 90 e.,BK ⊥ AK ;s
°,i. imi
lar
ly,
CL ⊥ AL .AsALi
stheb
ise
ctorof ∠BAC ,
soLi
sthemi
dpo
int
ofCN .AsM i
sthemi
dpo
intofBC ,
soML ‖AB .
S
i °,
nce ∠BKA = ∠BHA =90 i
tfo
llowst
hatpo
insB ,K ,
t
H andA a
recyc
lic,s
o ∠MHK = ∠BAK = ∠MLK ,henc
eM ,
L ,H andK a
recyc
lic.

8 De
te ne,wi
rmi thpr f,a
oo llpa
irs(
a,b)ofi
ntege
rs,s
uch
t
hatforanypo
sit
ivei
nt rn,oneha
ege sn | (
an +bn+1).
230 Ma
thema
tic
alOl
ymp
iadi
nCh
ina

(
poedbyChenYonggao)
s
So
lut
ion.Theso
lut
ionpa
irscon
sitof (
s 0,0)and (-1,-1).
I
foneo s0,i
faandbi tisobv
iou
stha
ttheo
the
risa
lso0.
Nowwea
ssumet
hatab ≠0,
sel
ectal
argepr
imeps
ucht
hat
p >|a +b |,
2
itfo
llowsf
rom Fe
rma
t􀆳sLi
ttl
eTheor
emt
hat
ap +bp+1 ≡a +b2(
modp).
Mathematical Olympiad in China (2011–2014) Downloaded from www.worldscientific.com

Asp| (
ap +bp+1)andp >|a +b2|,wehavea +b2 = 0.
by NATIONAL UNIVERSITY OF SINGAPORE on 05/06/18. For personal use only.

Thenwes
elec
tano
the
rpr
imeqs
ucht
hatq >|b +1|and
(
q,b)= 1.Le
tn = 2
q.Thenwehave
an +bn+1 = (-b2)
2q
+b q+ =b q +b q+ =b q+ (
2 1 4 2 1 2 1
b2q-1 +1).
I
tfo
llowsf
romn| ( q,b)= 1t
an +bn+1)and ( hat

q| (
b2q-1 +1).
Asb2q-1 +1 ≡ (
bq-1)
2
·b +1 ≡b +1(
modq)
,andq >|b +1|,
i
tfo
llowst
hatb +1 = 0,
i.e.,
b =-1,ands
oa =-b2 = -1.
Inconc
lus
ion,t
her
ear
eon
lytwos
olu
tionpa
irs(
0,0)and
(-1,-1).

2012 (
Hohhot,I
n nerMongol
ia)

F
irs
tDay
8:
00 12:
00,Sep
t r28,2012
embe

1 Fi
ndt
hel
eas
t po
sit
ivei
nt r m ,s
ege uch t
hatforeve
ry
pr rp > 3,
imenumbe
2
105|9p -29p + m .
(
poedby Yang Hu)
s
Ch
ina We
ste
rn Ma
thema
tic
alOl
ymp
iad 231

So
lut
ion.As105 =3×5×7,
theor
igi
nalprob
lemi
sequ
iva
len
t
2
t
otha
toff
ind
ingt
hel
eas
tpo
sit
ivei
nt rm s
ege ucht
hat9p -
29p + m canbed
ivi
deds
imu
ltaneou
slyby3,5,7.
S
i ep2 andp havet
nc hes
ameodd
-evencha
rac
ter,wehave

9p -29p + m ≡ (-1) -(-1) +m ≡ m (


mod5).
2 2
p p

tm ≡ 0 (
mod5).
Mathematical Olympiad in China (2011–2014) Downloaded from www.worldscientific.com

Thenwege
by NATIONAL UNIVERSITY OF SINGAPORE on 05/06/18. For personal use only.

Asp > 3i r,wehave


sanoddnumbe

9p -29p + m ≡ - (-1) + m ≡ m +1(


mod3).
2
p

The
ref e,m ≡ 2 (
or mod3).
Wehavea
lsop2 ≡1(
mod3)
forp >3,orp2 =3
k +1.Then

9p -29p + m ≡ 23k+1 -1 + m ≡ 8k ·2 -1 + m
2

≡ m +1(
mod7).

The
re e,m ≡ 6 (
for mod7).
Ins
umma
ry,wehave

ìïm ≡ 0(mod5),
ï
ím ≡ 2(mod3),
ïï
îm ≡ 6(
mod7).

Theni
tisea
syt
ofi
ndt
hatt
hel
eas
tpo
sit
ivei
ntege s20.
rmi

2 Pr
ovet
hat,among anyn ve
rti
ceso
far
egu
la n -1
r2
poygon (
l n ≥ 3),ther
ea r
et hr
eeones,whi
ch ar
ethe
ve
rtice
sofani
sosce
lestri
angle.(po
sedbyZouJn)
i
So
lut
ion.S
inc
eiti
sea
syt
ove
rif
ydi
rec
tlyt
hea
sse
rti
onfort
he
ca
sesn = 3 (
a pen
tagon)andn = 4 (
ahep
tagon),we may
a
ssumet
hatn > 4i
nthefo
llowi
ngd
iscu
ssi
on.
Byr
educ
tiont
oab
surd
ity,a
ssumewecans
ele
ctn ve
rti
ces
232 Ma
thema
tic
alOl
ymp
iadi
nCh
ina

i
nar
egu
lar2
n -1po
lygonA1A2A3 …A2n-1s
ucht
hatnot
hreeof
t
hem cons
tit
ute
sani
sos
cel
est
riang
le. We ma
rkt
hes
e po
int
s
wi
th co
lor r
ed and t
he r
ema
i r n - 1 one
nde s wi
thb
lue,
r
espec
tive
ly.
Wemayl
e ed,andd
tA1 ber ivi
det
heo
the
r2n - 2po
int
s
i
nton - 1pa
irs(
seet
hef
igue):(
r A2 ,A2n-1),(
A3 ,A2n-2),
Mathematical Olympiad in China (2011–2014) Downloaded from www.worldscientific.com

...,(
An ,An+1).
by NATIONAL UNIVERSITY OF SINGAPORE on 05/06/18. For personal use only.

Thetwopo
int
sineach
pa
ircanno
tbebo
thr
eda
s
t
hey p
lusA1 cons
tit
ute an
i
sos
cel
est
riang
le,andmu
st F
ig.2
.1
beoner
edandoneb
luefor
t
her
ear
eexac
tlynr
edpo
int
s.
As
s ngA2i
umi sred,t
henA2n-1i
sbue,andA3 mu
l stbeb
lue
s△A1A2A3 i
a sani
sos
cel
est
riang
le.The
re e A2n-2 i
for sred,
f
rom wh
ich we i
nfe
rt t A5 , A2n-4 a
ha re bo
th b
lue, a
s
△A2n-2A2A5 , △A2n-4A2n-2A1 a
retwoi
sos
cel
est
riang
les.Bu
t
A5 ,A2n-4a
ret
hetwopo
int
sinapa
ir,t
hey mu
stbeoner
edand
oneb
lue.Th
isi
sacon
trad
ict
ion! Thea
sse
rtonforn >4i
i sthen
a
lsot
rue.Theproofi
scomp
let
e.

3 tE be a g
Le iven s
et wi
thn e
lemen
ts.Suppo
se A1 ,
A2 ,...,Ak a
rek d
ist
inc
tnon
-emp
tys
ubs
etsofE ,wi
th
t
hep
rope
rtyt
hat,f
orany1 ≤i < j ≤k,e
it rAi ∩ Aj =
he
⌀ oronei
ncl
ude
stheo
t r(
he e.,Ai ⊂ Aj orAj ⊂ Ai ).
i.
Fi
ndt
hemax
imumva
l fk.(
ueo pos
edbyLengGang
song)
So
lut
ion.Wec
laimt
hatt
hemax
imumva
l s2
ueofki n -1.To
provet
his,weatf
irs
tgi
veanexamplet
hats
ati
sfi
esk =2
n -1.
Wemayl etE = {
1,2,...,n},and
Ch
ina We
ste
rn Ma
thema
tic
alOl
ymp
iad 233

i}
{ or1 ≤i ≤ n,
f
Ai = { {
1,2,...,
i -n +1} forn +1 ≤i ≤ 2n -1.

I
tisea
syt
oseet
hatt
heypo
sse
sst
heg
ivenprope
rty.
Now wewi
llprovet
hatk ≤ 2
n -1byi
nduc
tion.
Whenn = 1,
iti
sobv
iou
slyt
rue.
As
sumet
hati
tist
rueforn ≤ m -1.Thenwhenn = m ,we
Mathematical Olympiad in China (2011–2014) Downloaded from www.worldscientific.com

cons
ide
rthes
ett
hatcon
tai
nst
hemo
ste
lemen
tsamongA1 ,A2 ,
by NATIONAL UNIVERSITY OF SINGAPORE on 05/06/18. For personal use only.

...,Ak exc
lud
ingE .We maya
ssumet
hati
tisA1 con
tai
ning
t(≤ m -1)e
lemen
ts.Thenwecand
ivdeA1 ,A2 ,...,Aki
i nto
t
hreeca
tegor
ies:
(
1)t e
hestE ;
(
2)s
etst
hata
rei
ncl nA1 ;and
udedi
(
3)s
etswho
sei
nte
rse
cti
onwi
thA1i
semp
ty.
(Categori
es(1)and (
2)maybeempty.)
Thent henumbe
rofset
sinc
ate
goy(
r 1)isnotg
rea
tert
han1.
Byi
nduc
tion,t
henumbe
rofs
etsi
nca
t y(
egor 2)i
sno
t
r
gea
tert t -1.
han2
Thenumbe
rofe
lemen
tscon
tai
nedi
ntheun
ionoft
hes
ets
i
nca
t y(
egor 3)i
sno
tgr
eat
e hanm -t.Thenbyi
rt nduc
tion,t
he
numbe
rofs
etsi
ntha
tca
tego
ryi
sno
tgr
eat
e han2(
rt m -t)-1.
Int
otl,
a

k ≤1+ (
2t -1)+ [
2(m -t)-1]= 2m -1.

The
ref e,t
or hepropo
sit
ioni
str orn = m .
uef
Byi
nduc
tion,weconc
ludet
hatforanys
etE ofne
lemen
ts,
wea shavek ≤ 2
lway n -1.Th
iscomp
let
est
heproo
f.

4 tP beanyi
Le nne
rpo
intofanacu
te△ABC ;E ,F bet
he
pro
jec
tionpoin
tsofP ontol
i sAC ,AB ,r
ne especti
vey;
l
and the ex
tended l
ine
s of BP , CP in
tersec
tt he
234 Ma
thema
tic
alOl
ymp
iadi
nCh
ina

c
ircumc
irc
leof△ABC a
tpo
insB1 ,C1 (
t B1 ≠ B ,C1 ≠
C),r
espec
tive
l tR andr deno
y.Le tet
her
adi
ioft
he
c
ircumc
irc
leandi
nci
rcl
eof △ABC ,r
espe
cti
vel
y.Prove
EF r, ,
t
hat ≥ and whent
heequa
lit
yho
lds,de
termi
ne
B1C1 R
comp
let
elyt
hepo
sit
ion
sofP .(
pos
edbyLiQi
usheng)
So
lut
ion.Ass .1,
.4
Mathematical Olympiad in China (2011–2014) Downloaded from www.worldscientific.com

eeni
nFi
g
by NATIONAL UNIVERSITY OF SINGAPORE on 05/06/18. For personal use only.

we make PD ⊥ BC wi
th
i
nte
rse
cti
ngpo
intD ,ex
tendl
ine
AP t
o i
nte
rse
ct wi
th t
he
c
ircumc
irc
leof △ABC a
tpo
int
tDE ,DF ,A1B1 ,
A1andconnec
A1C1 .
S
ine P , D , B, F a
c re
F
ig.4
.1
c
onc
ycl
ic, we have ∠PDF =
∠PBF ;s
i eP ,D ,C ,E ,wehave ∠PDE = ∠PCE .Then
nc

∠FDE = ∠PDF + ∠PDE = ∠PBF + ∠PCE

= ∠AA1B1 + ∠AA1C1 = ∠C1A1B1 .

In t ame way, we have ∠DEF


he s = ∠A1B1C1 .

The
ref e,△DEF ~ △A1B1C1 .
or
Ther
adi
uso
fthec
irc
umc
irc
leo
f
△A1B1C1 i
sal
so R ,and l
ett
he
r
adi
uso
fthec
ircumc
irc
l f△DEF
eo
EF R'
beR'.Wet
henhave = .
B1C1 R
Nowl
ett
hei
ncen
te f△DEF b
ro e
O
'.Conne
c ', BO
t AO ', CO
' (
see
F
ig .2).Weha
.4 ve F
ig.4
.2
Ch
ina We
ste
rn Ma
thema
tic
alOl
ymp
iad 235

(
AB +BC +CA )·r
S△ABC =
2
= S△O'AB +S△O'BC +S△O'AC

AB ·O'F BC ·O'D CA ·O'E


≤ + +
2 2 2
AB +BC +CA )·R'
(
= .
2
Mathematical Olympiad in China (2011–2014) Downloaded from www.worldscientific.com

e,R' ≥r.Theequa
by NATIONAL UNIVERSITY OF SINGAPORE on 05/06/18. For personal use only.

The
ref
or lit
yho
ldsi
fandon
lyi
f

O'D ⊥ BC ,O'E ⊥ CA ,O'F ⊥ AB ,

wh
ichimp
lisP = O'andPi
e sthei
ncen
terof△ABC .
EF r
The
re e,
for ≥ andt
heequa
lit
yho
ldsi
fandon
lyP
B1C1 R
i
sthei
ncen
terof△ABC .
Theproo
fiscomp
let
e.

S
econdDay
8:
00 12:
00,Sep
t r29,2012
embe

5 tH andO bet
Le heor
thoc
ent
erandc
irc
umc
ent
ero
fac
ute
t
riang
le△ABC,r
espe
cti
vey (A ,H ,O a
l renon
-co
ll
ine
ar).
Suppo
seD i
sthe pro
jec
tionofA on
tol
ineBC ,andt
he
pe
rpend
icu
larb
isec
toroft
hes tAO mee
egmen tsl
ineBCa
t
E .Provet
hatt
hemi
dpo
intN o
fOH i
sont
hec
ircumc
irc
le
of △ADE . ( po
sed by
FengZh
igang)
So
lut
ion.Ass
eeni
nFi
g .1,
.5
weex
tendHD t
ole
titi
nte
rsec
t
wi
tht
hec
ircumc
irc
leof△ABC
a
t po
int H',l
ink FN , DN ,
BH ,BH',OH',and deno
te
F
ig.5
.1
236 Ma
thema
tic
alOl
ymp
iadi
nCh
ina

t
hemi
dpo
intofAO a
sF .
AsH i
stheor
thocen
ter,wehave

∠CBH' = ∠CAH' = ∠CBH .

The
ref e,Di
or sthemi
dpo
in fHH'.Ont
to heo
t rhand,
he
Ni
sthe mi
dpo
in f HO ,s
to oDN i
sthe med
ianof △HOH'.
1
Mathematical Olympiad in China (2011–2014) Downloaded from www.worldscientific.com

The
ref e,DN =
or OH'.
2
by NATIONAL UNIVERSITY OF SINGAPORE on 05/06/18. For personal use only.

S
inceOH' = OA andFi
sthemi
dpo
in fOA ,
to then

1 1
DN = OH' = OA = AF .
2 2

I
tisea
syt
osee FN ‖AH .Then AFND i
sani
sos
cel
es
t
rapezo
idandA ,F ,N ,D a
reconcyc
lic.
Fur
the e,f
rmor r °= ∠AFE weknowA ,F ,
om ∠ADE =90
D ,E a
rea
lsoconcyc
lic.ThenA ,F ,N ,D ,E a
reconcyc
lic,
wh
ichimp
lie
stha
tthec
ircumc
irc
l f△ADE c
eo ros
sesN a
tthe
mi
dpo
intofOH .Thepr
oofi
scomp
let
e.
Remark.Bythefac
tst
hatt
her
adi
usoft
hen
ine
-po
int
c
irc
leo
fat
riang
lei
sha
lfo
ftha
toft
hec
ircumc
irc
leandN i
s
t
hec
ent
eroft
hen
ine
-po
intc
irc
leof △ABC ,wecan ge
tal
so
t
hatAFNDi
sani
sos
cel
est
rape
zoi
d.

1,
2
an
6 e{
Sequenc an }i
sde
finedbya0 = an+1 = an + ,
2 2012
n = 0,1,2,....Fi
ndi
nt rk,
ege sucht
hatak <1 <ak+1 .
(
pos
edbyBi
anHongp
ing)
So
lut
ion.Fromtheg
ivencond
iton,wehave
i

1
= a0 < a1 < … < a2012 .
2

Weno
tet
hat
Ch
ina We
ste
rn Ma
thema
tic
alOl
ymp
iad 237

1 2012 1 1 ,
= = -
an+1 an (
an +2012) an an +2012
or
1 1 1
- = .
an an+1 an +2012
Byt
het
ele
scop
i um,wehave
ngs
Mathematical Olympiad in China (2011–2014) Downloaded from www.worldscientific.com

n-1
1 1 1
- = ∑a .
by NATIONAL UNIVERSITY OF SINGAPORE on 05/06/18. For personal use only.

a0 an i=0 i +2012
Then
2011 2011
1 1 1
2-
a2012
= ∑
i=0 ai +2012
< ∑ 2012 = 1.
i=0

The
ref
ore,
a0 < a1 < … < a2012 < 1.
Thenwehave
2012 2012
1 1 1
2-
a2013
= ∑
i=0 ai +2012
> ∑ 1 +2012 = 1,
i=0

wh
ich meansa2013 > 1.
Con
sequen
tly,wef
indt
hatk = 2012.

7 venann ×ng
Gi rid,weca
lltwoc
ell
sini
tad
jac
enti
fthey
have a common s
ide. Att
he beg
inn
ing,each ce
lli
s
a
ssi r+1.Anope
gnednumbe rat
ionont
heg
ridi
sde
fined
a
sfo
llows:onechoo
sesac
ell,andt
henchange
sthes
ign
s
o
fev
erynumbe
rini
tsad
jac
entc
els(
l bu
tno
tchang
ethes
ign
o
fthenumb
eri
nit
sef).F
l inda
llt
hei
nte
gesn ≥2,
r sucht
hat
a
fte
raf
ini
teo
fope
rat
i s,a
on llt
henumbe
rsi
nthec
ell
soft
he
r
gida
rechang o -1.(
edt pos e)
edbyShenHuyu
So
lut
ion.Wewi
llpr
ovet
hatn mee
tst
her
equ
iredcond
iti
oni
f
andon
lyi
fiti
sanevennumbe
r.
Wedeno
tet
hece
lli
nthei
-rowandj
-co
lumnoft
heg
rida
s
238 Ma
thema
tic
alOl
ymp
iadi
nCh
ina

Aij (
i,j ∈ {
1,2,...,n}).
k,k ∈ N* ,wema
Whenn =2 rkeachAijs
ati
sfi
yngi +j ≡
0(
mod2)wi
thco
l ed (
orr pre
sen
tedbyshadeda
r s),andt
ea hat
s
a t
isf
yingj -i ≡ 3(mod4)andj -i ≢j +i(mod4)wi
thb
lue
(pres
entedbyobliquel
inearea )(
s s eeFi
g.7 )
.1 .
Mathematical Olympiad in China (2011–2014) Downloaded from www.worldscientific.com
by NATIONAL UNIVERSITY OF SINGAPORE on 05/06/18. For personal use only.

F
ig.7
.1 F
ig.7
.2

I
nth
i y,wec
swa ans
eet
hate
ver
yce
llad
jac
entt
oab
lueonei
s
ed,andt
r her
eise
xac
tlyoneb
luec
ella
rounde
achr
edone.
Now wedot
heope
rat
iononeachb
luece
ll.Thenumbe
rin
eachr
edc
elli
sthenchangedf
r o -1,wh
om +1t ilet
henumbe
rs
i
nther
ema
ini
ngce
llsa
reunchanged.
S
i eni
nc r,wecanro
sanevennumbe tat
etheg
rida
roundi
ts
cen
terO an
tic
lockwi
se by 90
°.Then a
llt
her
edc
ell
soft
he
ro
tat
edg
ridcove
rexac
tlya
llt
hece
llst
hata
reno
tredi
nthe
or
igi
nalg
rid(
seeFi
g .2).
.7
Wedot
heope
rat
ionsaga
inf
ort
heor
igi
nalg
ridona
llt
he
c
ell
stha
tar
ecove
redbyb
luece
llso
fthero
tat
edg
rid.Thenwe
s
eet
hata
llt
henumbe
rswi
thva
lue +1i
nther
ema
ini
ngce
llsof
t
heor
igi
nalg
rida
r o -1,wh
echangedt ilet
henumbe
rsi
nthe
o
the
rce
llsa
reunchanged.
The
ref
ore,whenni
sane
v r,a
ennumbe llt
henumb
ersi
nthe
c
ell
soft
heg
ridc
anb
echang o -1byaf
edt ini
teo
fope
rat
ion
s.
sodd,wedeno
Whenni tet
henumbe
rince
l sM i(
lAiia i=
Ch
ina We
ste
rn Ma
thema
tic
alOl
ymp
iad 239

1,2,...,
n),anddeno
tet
henumbe
r
ofope
rat
ionsoneachoft
hei
rad
jac
ent
c
ell
sasx1,x2,...,xn-1 ,y1,y2,...,
yn-1 ,r
espec
tivey(
l s
eeF i.7
g .3).
Afte
rfinit
eopera
tions,iti
sea
sy
t
oseet
hat:
Mathematical Olympiad in China (2011–2014) Downloaded from www.worldscientific.com

M 1i
schangedf
rom +1t
o -1i
f F
ig.7
.3
by NATIONAL UNIVERSITY OF SINGAPORE on 05/06/18. For personal use only.

andon
l sodd;
yx1 +y1i
M 2i rom +1t
schangedf o -1i
fandon
lyx1 +y1 +x2 +y2
sodd;
i
M 3i rom +1t
schangedf o -1i
fandon
lyx2 +y2 +x3 +y3
sodd;
i

M n-1i
schangedf
rom +1t
o -1i
fandon
lyxn-2 +yn-2 +
sodd,and
xn-1 +yn-1i
M ni
schangedf
rom +1t
o -1i
fandon
lyxn-1 +yn-1i
sodd.
S
i sodd,t
nceni hes
umofnoddnumbe
rsi
sst
ilodd.Then,
l

(x1 +y1 ) + (x1 +y1 +x2 +y2 ) + (x2 +y2 +x3 +y3 )
+ … + (xn-2 +yn-2 +xn-1 +yn-1 ) + (xn-1 +yn-1 )

= 2(
x1 +x2 + … +xn-1 +y1 +y2 + … +yn-1)

i
sodd.I
tisimpo
ssi
ble!
The
ref e,a
or llt
henumbe
rsi
nthec
ell
sofag
ivenn ×ng
rid
canbechangedf
rom +1t
o -1a
fte
raf
ini
teo
fope
rat
ionsi
f
andon
l fni
yi sanevennumbe
r.

8 Find al
lt he pr
ime numbers p,for which t
herea r
e
i
nfini
tely manypos
iti
vein
tegersn,sucht
ha tp |nn+1
+
(
n +1)
n
.(po
sedbyChenYonggao)
So
lut
ion.nn+1 + (
n + 1) i
salway
sanodd numbe
rforany
n
240 Ma
thema
tic
alOl
ymp
iadi
nCh
ina

pos
iti
vein t
egern,sot
herequi
redp wi
llno
tbe2.Now wear
e
go
ingt o provet t,f
ha oranyprimenumberp ≥ 3,t
her
ear
e
i
nfi
nit
e many po
sit
ivei
ntege
rst
hat mee
tthe cond
iti
on. We
pr
esen
ttwoproo
fsi
nthef
oll
owi
ng.
Proo
f1.Foranypr rp ≥3,
imenumbe letn =pk -2beanodd
numbe
r.Then
Mathematical Olympiad in China (2011–2014) Downloaded from www.worldscientific.com

nn+1 + (
n +1)
n
≡(-2)
pk-1
+(-1)
pk-2
≡ 2p - -1
k 1
by NATIONAL UNIVERSITY OF SINGAPORE on 05/06/18. For personal use only.

≡(
2p-1) modp).
·2k-1 -1 ≡ 2k-1 -1(
k

t,l
Nex etk -1 = (
p -1)
t.
n +1)
Thennn+1 + ( n
≡ 0(
modp).
The
re e,whenn = p(
for p - 1)
t + p - 2 (whe
reti
sany
po
sit
ivei
nt r),wehavep|nn+1 + (
ege n +1)
n
.
Proo
f2.Gi
venanypr
imenumbe
rp ≥3,wehave(
2,p)=1.
ByFe rma
t􀆳sLit
tl em,weknow2p-1 ≡ 1(
eTheor modp).Le
tn
,
= p -2 wh
t
e
r , , ,
et = 1 2 3 ....Wehave

n +1)
nn+1 + ( ≡(-2) +(-1)
t t
n p -1 p -2

≡ 2p - -1 ≡ (
2p-1)
t t-1 t-2 …
-1
1 p +p + +p+1

≡ 1 -1 ≡ 0(
modp).

Theproofi
scomp
let
e.

2013 (
Lanzhou,Gansu)

F
irs
tDay
8:
00 12:
00,Augus
t17,2013

1 Dot
her
eex
isti
ntege
rsa,b andc s
ucht
ha c + 2,
ta2b
Ch
ina We
ste
rn Ma
thema
tic
alOl
ymp
iad 241

ab2c +2,ab
c2 + 2 a
re pe
rfec
tsqua
res. (
pos
ed by Li
Qi
usheng )
So
lut
ion. No.Suppos
ethe con
tra
ryt
hatt
her
ear
esuch
i
ntege
rsa,bandc.
I
foneo
ft seven,s
hemi aya,
t c +2 ≡ 2 (
hena2b mod4),
wh
ichcon
trad
ict
sthe a
ssump
tiont
ha c + 2i
ta2b sa pe
rfec
t
Mathematical Olympiad in China (2011–2014) Downloaded from www.worldscientific.com

s
quare.Wemaythenass
umet
hata,bandca reodd,s ot hey
by NATIONAL UNIVERSITY OF SINGAPORE on 05/06/18. For personal use only.

aree
it r1 or3 (mod4).I
he tfol
lowsfrom t
he Pigeon-hole
Pr
inc
ipl
etha
ttwooft
hem a
recong
ruen
tmodu
lo4.Re
labe
lif
nece
ssa
ry,wemaya
ssumet
hata ≡b(
mod4),
s c2 +2 ≡c2 +
oab
2 ≡ 1 +2 ≡ 3 (
mod 4),wh
ich v
iol
ate
sthe pe
rfec
tsqua
re
a
ssump
tion.

2 tnbeani
Le nt r,
ege n ≥2,andx1 ,x2 ,...,xn ∈ [
0,1].
Provet
hat
n
n -1

1≤k<l≤n
kxkxl ≤
3 k∑=1
kxk .

(
pos
edbyLeng Gang
song )
So
lut
ion.Asx1 ,x2 ,...,xn 0,1],xixj ≤xi ,
∈[ sowehave

3 ∑ kx x
1≤k<l≤n
k l = ∑ 3
kxkxl ≤
1≤k<l≤n

1≤k<l≤n
( kxl ).
kxk +2

For1 ≤k ≤ n,
thecoe
ffi
cien
tofxki
nthel
asts
umi
s

2[
1 +2 + … + (
k -1)]+k(
n -k)=k(
n -1),

s
owehave
n

3 ∑
1≤k<l≤n
kxkxl ≤ ∑
1≤k<l≤n
( kxl )=
kxk +2 ∑k(n -1)x
k=1
k

=(
n -1) ∑kx
k=1
k ,

andhencet
hede
sir
edi
nequa
lit
yho
lds.
242 Ma
thema
tic
alOl
ymp
iadi
nCh
ina

3 In △ABC ,po
intB2 i
sther
efl
ect
ion oft
hecen
terof
B-exc
irc
lewi
thr
espec
ttot
he mi
dpo
intofs
ideAC ,and
po
intC2i
sther
efl
ect
iono
fthecen
terofC-exc
irc
lewi
th
r
espec
ttot
he mi
dpo
intofs
ide AB .The A-exc
irc
le
t
ouche
ssde BC a
i t po
intD .Pr
ovet
hatAD ⊥ B2C2 .
(
pos
edbyBi
anHongp
ing)
Mathematical Olympiad in China (2011–2014) Downloaded from www.worldscientific.com

So
lut
ion.Le
tA1bet
hec
ent
erof
by NATIONAL UNIVERSITY OF SINGAPORE on 05/06/18. For personal use only.

A-exci
rcle. By the prope
rti
es
aboutc ent
ersof ex
-ci
rcl
es,the
fo
llowi
ngs
etsoft
hree po
int
sar
e
co
lli
nea B1 ,A ,C1},{
r:{ A1 ,C ,
B1}and{
C1 ,B ,A1},andA1A ⊥
B1C1 .
Onthepane,choo
l sepoitP
n Fi
g.3.1

s
uch tha → →
t C2P = B2C ,t hen i
t
fo
llowsf
r omB2→ →t
C = AB1 hatC2→ → .AsBC
P = AB1
→ =C →
2 1A and
thepo i
ntsC1 ,B ,A1 a
reco l
li r,t
nea he po
insB ,C2 ,P a
t re
col
linea oB→
r,s P =C B → .I
1tfol
1lowsf
rom

æç180
°- ∠BAC ö÷ æç180
°- ∠ABC ö÷
∠AC1B = 180
°- -
è 2 ø è 2 ø
∠BAC + ∠ABC 180
°- ∠ACB
= = = ∠BCA1
2 2

t
hat△A1BC ∽ △A1B1C1 .Le
tA1D andA1A bet
hea
lti
tude
sof
he △A1BC and △A1B1C1 ,r
t espec
tive
ly,wi
thr
espe
ctt
othe
B1C1 A1A
oppo
sit
esi
des,s
o = .
BC A1D

fB→ →, BP A1A
I P =C1B henA1A ⊥ B1C1 ,
1 t so = I
.fBP ⊥
BC A1D
A1A ,thenBC ⊥ A1D ,s
o wehave △BPC ∽ △A1AD ,and
henceCP ⊥ AD .Aga
inbyC2→
P = B2→
C ,oneha → = C→
sB2C2 P,
Ch
ina We
ste
rn Ma
thema
tic
alOl
ymp
iad 243

andhenceAD ⊥ B2C2 .

4 The
rea
ren(
n ≥ 2)co
insi
nar
ow.I
foneoft
heco
insi
s
head,s
elec
tanoddnumbe
rofcons
ecu
tiveco
ins(
oreven
1co
in)wi
tht
heonei
nheadont
hel
eftmo
st,andt
hen
f
lipa
llt
hes
ele
cted co
insups
ide down s
imu
ltaneou
sly.
Mathematical Olympiad in China (2011–2014) Downloaded from www.worldscientific.com

Th
i samove.Nomovei
si sal
lowedi
falnco
l insa
ret
ail
s.
by NATIONAL UNIVERSITY OF SINGAPORE on 05/06/18. For personal use only.

Suppo
sen c
oin
sar
ehe
adsa
tthei
nit
ials
t e,de
ag termi
nei
f
2n+1
t
her
eisawayt
oca
rryou
t mov
es.(
pos
edbyGuB
in)
3
So
lut
ion.Theansweri
spo
ssi
ble.
Fo
ranyc
onf
igu
rat
iono
fthec
ois,wede
n fineac
orr
espond
ing
01 s
equencec1c2 ...
cn ofl
engthnasfo
llows:ci =1,i
fthei
-th
co
infromthelefti shead,o t
herwi
se,ci = 0.Itisea
sytosee
t
hatt
hes
tat
uso hen co
ft insa
sone
-to
-onecor
res
pondencet
o
uch01 s
s equenc
es,s
oint
hefo
ll ng,we wi
owi llcons
ide
rth
is
s
equencemodelin
stead.
Ini
tia
lly,t
hes equenc
eis11…11,deno
tedby1n (wi
thn
cons
ecu
tived
igi
tsof1).S
imi
lar
ly,00…00,deno
tedby0n (
wit
h
ndigi
tsof0).Forany01 s equencewit
hatlea
stadigt“
i 1”,
con
sidert
hefol
lowing move:locatethefi
rstdi
git“
1”f rom
r
ightt
ol e
ftint hes e,t
equenc hentakethe01 subs
equence
f
romlefttor
ightsta
rtingths“
i 1”ofmaximaloddl
eng
th,and
he01 pa
changet rit
yint
hiss
ubs
equencej
ustl
ikef
li i
ppngt
he
co
insi
namove.Deno
tebyant
het
ota
lnumbe
rofmove
sint
he
2n+1
ways
tat
edabove.Wec
laim:
an = .Whenn =1,
iti
sea
sy
3
22 ,
t
oseet
hata1 =1 = pr
oceedbyi
nduc
tion.As
sumef
ormu
la
3
2n+1 2k+1
an = ho
ld orn =k,
sf i.e.,
ak = .
3 3
244 Ma
thema
tic
alOl
ymp
iadi
nCh
ina

Now,wed
iscu
sst
heca
sen =k +1,
i.e.,wewan
ttof
ind
t
het
ota
lnumbe
rofmove
sasde
scr
ibedabovei
fthes
equencei
s
1k+1 ,
i.e,t
her
eaek +1co
r insi
narow.
sodd,t
fki
I henbyi
nduc
tionhypo
the
sis,t equence1k+1
hes
(
wihk +1d
t igi
t f1)change
so sto10k (
wihkd
t igi
t f0)a
so fte
r
ak move
sass
tat
edabove.Af
teranadd
iti lmove,i
ona tchange
s
Mathematical Olympiad in China (2011–2014) Downloaded from www.worldscientific.com

o01k-10 (wi
t thk - 1d
igi
tsof1),t
hena
fte
rapp
lyngak - 1
i
by NATIONAL UNIVERSITY OF SINGAPORE on 05/06/18. For personal use only.

s,t
move hes
equencechange
sto0k+1 (
wihk +1d
t igi
t f0).In
so
f
act,r
eca
llt
hati
nsequenc fak move
eo sf o0k by,t
rom1k t he
f
irs
tmovei
sfr o1k-10.The
om1k t ref
ore,wehave

2k+1 2k+1 -1 2k+2 -2 2k+2


ak+1 = 2ak = 2 = 2· = = .
3 3 3 3

I seven,byt
fki hei
nduc
tiona
ssump
tioni
tt sak move
ake s
f
r o10k ,t
om1k+1t henapp
lyanadd
iti
ona
lf o01k ,and
rom10k t
f
ina
lly by t
hei
nduc
tion a
ssump
tion aga
in,i
tt sak move
ake s
f
rom01k t
o0k+1 .Thenwehave

2k+1 2k+1 -2
ak+1 = 2ak +1 = 2 +1 = 2· +1
3 3
2k+2 -1 2k+2
= = .
3 3

2n+1 ,
Byi
nduc
tiont
hatan = hencet
her
eex
ist
sa wayt
o
3
maket
her
equ
irednumbe
rofmove
s.

S
econdDay
8:
00 12:
00,Augus
t18,2013

5 Anon
-emp
tys
etA ⊆ {
1,2,3,...,n}
isca
lledagoods
et
Ch
ina We
ste
rn Ma
thema
tic
alOl
ymp
iad 245

o
fde
gre f|A |≤ mi
eni nx.Deno
tebyan t
henumbe
rof
x∈A

goods
etsofdegreen.Provet tan+2 = an+1 +an +1f
ha or
anypos
iti
veint
egern.(posedbyLiWe igu )
So
lut
ion1.Le
tA beagoods
etofdeg
reen,and|A |= k,
nx∈Ax ≥ k,s
henmi
t k,k + 1, ...,n}.Hencet
oA ⊆ { he
numbe
rofgoods
etsofdeg
reen wi
thk e
lemen
tsi
sCn
k
-k+1 .It
Mathematical Olympiad in China (2011–2014) Downloaded from www.worldscientific.com

[ n2+1 ]

by NATIONAL UNIVERSITY OF SINGAPORE on 05/06/18. For personal use only.

Cn-k+1 = Cn +Cn-1 + Cn-2 + … .


1 2 3
f
oll
owst
hatan = k=1
k

I seven,
fni n = 2m ,
then

a2m+2 = C1
2m+2 + C2m+1 + … + Cm+2
2 m+1

=(
C12m+1 + C2m+1 )+ (
C22m + C2m )+ … + (
Cmm+1 + Cm+1 )
0 1 +1 m

=(
C12m+1 + C2m + … + Cm+1 )+ (
C12m + C2m-1 + …
2 m+1 2

+ Cm+1)+ C2m+1
m 0

= a2m+1 +a2m +1.

I sodd,
fni n = 2m -1,
then

a2m+1 = C1
2m+1 + C2m + … + Cm+2 + Cm+1
2 m m+1

=(
C12m + C2m )+ (
C22m-1 + C2m-1)+ … + (
Cm +1 + Cm+1)+ C
0 1 m m-1 m
m

=(
C12m + C2m-1 + … + Cm+1 )+ (
C12m-1 + C2m-2 + …
2 m 2

+1 + Cm )+ C
+ Cm-
m 1 m 0
2m

= a2m +a2m-1 +1.

Ins
umma
ry,t
heequa
liyan+2 = an+1 +an +1ho
t ldsfora
ll
po
sit
ivei
ntege
rsn.
Remark.Le
tFn bet
hen
-tht
ermo
fFi
bonac
cis
equence.From

y ∑k=0 Cn-k = Fn ,onecan de


[ n2 ] k
t
hecomb
ina
tor
iali
den
tit rive

t
hatan = Fn+1 -1.
So
lut
ion2.I
fn = 1,the
rei
son
lyonegoods
eto
fdeg
ree1,
name
ly{
1},
soa1 = 1.
fn = 2,t
I hent
her
ear
eon
lytwogoods
etsofdeg
ree2:
246 Ma
thema
tic
alOl
ymp
iadi
nCh
ina

{
1},{
2},s
oa2 = 2.
Re
cal
lt tan ,an+1a
ha ret
henumbe
rsoft
henon
-emp
tygood
s
ubs
et f{
sA o 1,2,...,n +1},
1,2,...,n}and{ res
pec
tive
ly,
s
ati
sfi
yng|A |≤ mi
nx∈Ax.
Con
side
rtheca
sen +2:Foranynon
-emp
tygoods
etA of
deg
reen +2,Ai
sas
ubs
etof{
1,2,...,n +2},
thenwehave
Mathematical Olympiad in China (2011–2014) Downloaded from www.worldscientific.com

t
hef
oll
owi
ngt
hreeca
ses:
by NATIONAL UNIVERSITY OF SINGAPORE on 05/06/18. For personal use only.

(
a)A doe
sno
tcon
tai
nthee
l tn +2;
emen
(
b)A con
tai
nsn +2,andha
satl
eat2e
s lemen
ts;
(
c)A = {
n +2}.
Int
hefo
ll ng,wefocu
owi sont
henumbe
rofgoods
etsof
t s(
ype a)and (
b).Foranygoods
e n(
tA i a),i
tfo
llowsf
rom
|A | ≤ mi
nx∈Ax andmaxx∈Ax ≤ n +1t
hatA i
sagoods
etof
deg
reen +1.Conve
rse
ly,anygoods
etofdeg
reen +1i
sal
soa
goods
etofdeg
reen +2,
the
ref
ore,t
her
ear
eexac
tlyan+1 good
s
etso
ftype (
a).
Foranygoods
etA = {
a1 ,a2 ,...,ak ,n +2}ofdeg
ree
ype (
n +2oft b),whe
rea1 < a2 < … < ak < n +2.Asa1 =
nx∈Ax ≥|A |≥ 2,onecancons
mi ide
rthenon
-emp
tys
etA' =
{
a1 -1,a2 -1,...,ak -1},whe
re1 ≤a1 -1 <a2 -1 < …
< ak -1 <n +1,andA's
ati
sfi
es|A'|=|A |-1 ≤a1 -1 =
nx∈A'x,henc
mi eA'i
sagoods
etofdeg
reen.Conve
rse
ly,any
goods
etofdeg
reencanber
epr
esen
tedi
ntheformA' = {
a1 -
1,a2 -1,...,ak -1},whe
reA = {
a1 ,a2 ,...,ak ,n +2}
is
agoods
etofdeg
reen +2o
ftype (
b),s
othecor
res
pondencei
s
one
-to
- tweenA andA',andt
onebe her
ear
eexac
tlyan goods
ets
ofdeg
reen +2o
ftype (
b).
Ac
cord
ingt
othed
iscu
ssi
onont
henumbe
rofgoods
etsof
t s(
ype a),(
b)and (
c),wehavean+2 = an+1 +an +1.
Ch
ina We
ste
rn Ma
thema
tic
alOl
ymp
iad 247

6 Ass
howni
nFi
g .1,PA ,PB a
.6 re
t
angen
ttot
hec
irc
le wi
thc
ent
erO
tA andB ,po
a intC (
dif
fer
entf
rom
A ,B )
isonmi
nora
rcAB .Thel
ine
lt
hroughpo
intC andpe
rpend
icu
lar
o PC mee
t tst
he ang
leb
isec
tor
s
Mathematical Olympiad in China (2011–2014) Downloaded from www.worldscientific.com

∠AOC and ∠BOC a


tpo
insD and
t
by NATIONAL UNIVERSITY OF SINGAPORE on 05/06/18. For personal use only.

E ,r
espec
tive
ly.Pr
ovet
hatCD =
CE .(
pos
edby HeYi
jie)
F
ig.6
.1
So
lut
ion1.AsshowninFi
g .2,Li
.6 ne
PC mee
tst
hec
irc
le ☉O a
tano
the
rpo
int
F .Jo
inBC ,BE ,BF ,OF ,r
espec
tive
ly.
S
ince B, C a
re on ☉O , OE b
isec
ts
∠BOC ,a e OE pe
nd henc rpend
icu
lar
ly
b
isec
tsBC ,
soCE = BE .ByFO = BO and
PC ⊥ DE ,wehave
∠ECB = 90
° - ∠FCB = 90
° -
1∠
BOF = ∠OBF ,and s
o△CEB ∽
2 F
ig.6
.2
△BOF ,hence

CE CB
= . ①
BO BF

AsPBi
stang
en o☉O,
tt so ∠PBC = ∠PFB,and△PCB ∽
△PBF ,andhence

CB PC
= . ②
BF PB

PC
By ① and ② ,we have CE = BO · .Bys
ymme
try,
PB
PC
CD =AO· . I
tfo
llowsf
romAO =BO,PA =PBt
hatCD =CE.
PA
248 Ma
thema
tic
alOl
ymp
iadi
nCh
ina

So
lut
ion2.Asshownin Fi
g.6
.3.Jo
in
AB ,AC ,BC .OD mee
tsAC a
tpo
intM .
As A , C a
re on ☉O ,s
o OD b
isec
ts

∠AOC , ∠DMC = 9
1
0
°andMC = AC .
2
AsPC ⊥ DE ,
socos∠ACD =s
in∠ACP ,
Mathematical Olympiad in China (2011–2014) Downloaded from www.worldscientific.com

and
by NATIONAL UNIVERSITY OF SINGAPORE on 05/06/18. For personal use only.

MC AC
CD = = .③
c
os∠ACD 2s
in ∠ACP F
ig.6
.3

tR bet
Le her
adi
usof☉O .AsPA i
st o ☉O ,s
angentt o
∠ABC = ∠CAP ,andt
heni
tfo
ll rom ③ andt
owsf heS
ine
Lawt
hat

2Rsin ∠ABC si
n ∠CAP CP
CD = = R· = R· .
2s
in ∠ACP si
n ∠ACP AP

CP ,
Bys
ymme
try,oneha
sCE = R · andi
tfo
llowsf
rom
BP
AP = BPt
hatCD = CE .

7 Labe
lthes
ide
sofar
egu
larn
-goni
ncl
ockwi
sed
irec
tioni
n
orde
rwi
th1,2,...,n.De
termi
nea
lli
ntege
rsn (
n ≥ 4)
s
ati
sfi
yngt
hefo
llowi
ngtwocond
itons:
i
(
1)n -3non
-in
ter
sec
tingd
iagona
lsi
nthen
-gona
res
ele
cted,
wh
ichs
ubd
ivi
d hen
et -goni
nton - 2non
-ov
erl
app
ing
t
riang
les,and
(
2)eacho
fthecho
senn -3d
iagona
lsi
slabe
ledwi
than
i
nt r,s
ege ucht
hatt
hes
um ofl
abe
led numbe
rson
t
hrees
ide
sofeacht
riang
le n(
si 1)i
sequa
ltot
he
o
t r.(
he pos
edbyZouJ
in)
So
lut
ion. The requ
ired i
ntege
rsa
ret
hos
esa
tis
fyi
ng bo
th
cond
iti
on n ≥4andn ≢2(
s: mod4).Suppo
set
hatns
ati
sfi
est
he
Ch
ina We
ste
rn Ma
thema
tic
alOl
ymp
iad 249

cond
itons (
i 1)and (
2),wef
irs
tprovet
hatn ≢ 2(
mod4)a
s
fol
lows.Deno
tebySthes
umo fthelabel
sinthr
eesi
desofany
tr
iange,andbym t
l hesum oft
helabelsinthen -3di
agonal
s
cho
sen.Add
ing up t
hes
ums ofl
abe
lsi
nthr
ees
ide
s of a
ll
t
riang
lesi
nthes
ubd
ivi
si n(
oni a),eve
ryd
iagona
lappea
rstwi
ce
i
ntho
set
riang
les,i
tfo
llowst
ha n -2)
t( S =(
1+2+ … +n)+
Mathematical Olympiad in China (2011–2014) Downloaded from www.worldscientific.com

2m .Suppo
sen ≡2(
mod4),
then(
n -2)
Si t(
sevenbu 1+2+ …
by NATIONAL UNIVERSITY OF SINGAPORE on 05/06/18. For personal use only.

sodd,wh
+n)+2m i ichi
sacon
trad
ict
ion,s
on ≢ 2(
mod4).
Le
tusr
ema
rkt
hatt
hecond
iti
onont
her
egu
larn
-goni
nthe
prob
lemcanber
elaxedt -gon,
oconvexn sowecans
imp
lif
ythe
wr
iti
ng-
upoft
hes
olu
tion.

F
ig.7
.1

Int
hefo
ll ng,wepr
owi ovet
hata
lli
ntege
rsn wi
thn ≥4and
n ≢ 2(
mod4)s
ati
sfycond
itons (
i 1)and (
2).
TheFi
g.7
.1showsl
abe
lingfort
heca
sen =4,5and7,and
onecanve
rif
ydi
rec
tlyt
hatbo
thcond
iti
onsho
ld.
In t
he f
oll ng, we show t
owi ha fn s
ti ati
sfi
es bo
th
cond
iti
ons,s sn +4.
odoe
Asshowni
nFi
g .2,l
.7 abe
loned
iagona
lwh
ichs
ubd
ivi
des
t
heg
iven (
n +4)
-goni
ntoaconvexn
-gonandaconvexhexagon
(
non
-regu
la e).Asns
ranymor ati
sfi
escond
itons (
i 1)and (
2),
ubd
onecans ivi heconvexn
det -goni
nto(
n - 2)t
riang
less
uch
t
hatt
hes
umoft
het
hreel
abe
lsi
neacht
riang
lei
sequa
ltot
he
250 Ma
thema
tic
alOl
ymp
iadi
nCh
ina

s rS.Onecana
amenumbe lsos
ubd
ivi
det
hehexagonwi
tht
hree
d
iagona
lswi
thl
abe
ls:S -2
n -1,n +1andS -2n -5.Onecan
checkt
hatt
hes
um oft
het
hreel
abe
lsi
neach oft
hes
efour
t
riang
lesi
sal
soS,henc
en +4a
lsos
ati
sfi
escond
iti
ons(
1)and
(
2).Soi
tfo
llowst
hatanyi
nt rns
ege ucht
hatn ≥ 4andn ≢
2(
mod4)s
ati
sfi
escond
iti
ons(
1)and (
2).
Mathematical Olympiad in China (2011–2014) Downloaded from www.worldscientific.com
by NATIONAL UNIVERSITY OF SINGAPORE on 05/06/18. For personal use only.

F
ig.7
.2

Ins
umma
ry,t
hos
ein
tege
rsns
ati
sfi
yngcond
iti
ons(
1)and
(
2)a
reexac
tl i
ygvenbyn ≥ 4andn ≢ 2(
mod4).

8 F
inda
llpo
sit
ivei
ntege
rsas
ucht
hat(
2n -n2)|(
an -na )
f
ora
llpo
sit
ivei
ntege
rsn ≥5.(
pos
edbyYang Mi
ngl
iang)
So
lut
ion.Theonl
yan
swe
r oraa
sf re2and4.
Fi
rst,weprovet
hatai
seven.I
tfo
llowsf
rom t
heg
iven
cond
iti
onbychoo
singaneveni
nt rn ≥ 6.
ege
t,weprovet
Nex hata ha
snoodd pr
imef
act
or.Suppo
se
t
hecon
tra
ry,l
etpbeanoddpr
imef
acorofa.
t Ifp =3,
letn =
8,
then2n -n2 = 192ha
saf
acor3,bu
t tan -nai
sno
tdi
vis
ibl
e
by3,con
trad
ict
i o(
ngt 2n -n2)| (
an -na ),hencepi t3.
sno
Ch
ina We
ste
rn Ma
thema
tic
alOl
ymp
iad 251

fp = 5,
I letn = 16,
then2n -n2 = 64110ha
saf
acor5,
t
tan -nai
bu sno
tdi
vis
ibeby5,con
l trad
ict
ing(
2n -n2)|(
an -
na ),hencepi t5.
sno
fp ≥ 7,l
I etn = p - 1,i
tfo
llowsf
rom Fe
rma
t􀆳sLi
ttl
e
Theor
emt
ha modp).
t2p-1 ≡ 1(
As(
p -1) ≡1(
2
modp),
sop|(
2n -n2).Mor r,s
eove ince
Mathematical Olympiad in China (2011–2014) Downloaded from www.worldscientific.com

sevenandp|a,
ai sona ≡ (
p -1) ≡ (-1) ≡1(
a a
modp)and
by NATIONAL UNIVERSITY OF SINGAPORE on 05/06/18. For personal use only.

p|a ,andhencep doe


snotdiv
ide (
an - na ),contr
adi
cti
ng
n

2n -n2)| (
( an -na ).
Fi
nal
ly,wepr
ovet
ha s2or4.Fort
tai his,
leta =2t whe
re
r,t rom (
2n -n2)
|(2tn -n2 )
t
ti
sapo
sit
ivei
ntege heni
tfo
llowsf
and ( 2tn -n2t)t
2n -n2)| ( t( n2 -n2t).
2n -n2)| (
t
ha
I
fwechoo
sent
obes
uff
ici
ent
lyl
arge,i
tfo
llowsf
romt
he
t
n2
tn2 -n2t =0,hence2t =2
t
f
actl
im n =0tha t.t =1andt =
n→ ∞ 2

2a
reobv
iou
sso
lut
ions.
ft ≥3,
I thenbyt
heBi
nomi
a em,wehavet =2t-1 =
lTheor
(
1 +1)
t-1
t -1)=t,wh
>1 + ( ichi
simpo
ssi
ble.Atl
ast,one
canea
sil
ycheckt
hata =2anda =4s
ati
sfyt
hecond
iti
oni
nthe
prob
lem,s
othes
olu
ti oraa
onsf re2and4.
ChinaSouthea
stern
Ma
themati
calOl ympiad
Mathematical Olympiad in China (2011–2014) Downloaded from www.worldscientific.com
by NATIONAL UNIVERSITY OF SINGAPORE on 05/06/18. For personal use only.

2010 (
Lukang,Changhua,T
aiwan)

F
irs
tDay
8:00 12:00,Augues
t17,2010

1 ta,b,c ∈ {
Le 0,1,2,...,9}.Thequadr
ati
cequa
tion
ax2 +bx +c =0ha
sar
ati
ona
lroo
t.Provet
hatt
het
hree
-
d
igi rab
tnumbe cisno
tapr
imenumbe
r.
So
lut
ion.Weprovebycont
rad
ict
i fab
on.I c = pi
sa pr
ime
r,t
numbe her
ati
ona
lroo
tofqu
adr
ati
cequ
atonf(
i x)=ax2 +bx +
-b ± b2 -4ac
sx1 ,x2 =
c = 0i .Obv
iou
sly,b - 4
2
aci
sa
2
a
Ch
inaSou
the
ast
ern Ma
thema
tic
alOl
ymp
iad 253

pe
rfec
tsqua
r r,andx1 ,x2 a
enumbe rea
llnega
tive,and

f( x -x1)(
x)= a( x -x2).

s,
Thu

p = f(
10)= a(
10 -x1)(
10 -x2).

So,
Mathematical Olympiad in China (2011–2014) Downloaded from www.worldscientific.com
by NATIONAL UNIVERSITY OF SINGAPORE on 05/06/18. For personal use only.

ap = (
4 20a -2ax1)(
20a -2ax2).

I
tisea
syt
oseet
hat(
20a -2ax1)and (
20a -2ax2)a
rea
ll
pos
iti
veint
ege
rs.Consequent
ly,p|(20a -2ax1)orp|( 20
a-
2ax2).I
fp | ( a - 2ax1),t
20 henp ≤ 20a - 2ax1 ,s
o,80 -
8x1 -10x2 +x1x2 ≤ 0,wh
ich con
trad
ict
sto x1 ,x2 < 0.
S
imi
lar
ly,p| (
20a -2ax1)
isno
ttr
ue.

2 Forany s
etA = {
a1 ,a2 , ...,am },deno
teP(
A) =
a1a2 ... tA1 ,A2 ,...,andAn bea
am .Le ll99 e
lemen
t
s
ubs
et f{
so 1,2, ...,2010},n = C99
2010 .Prove t
hat

∑ P(
Ai).
n
2010 i=1

So
lut
ion1.Foreach99
-el
emen
tss
ubs
et,Ai a1 ,a2 ,...,
= {

a99}of{
1,2,...,2010}unique
lycor
respondsto99 e lement
s
subse b1,
tBi = { b2,...,
b99}of{1,2,...,2010}bybk =2011-
ak ,k = 1,2,...,99.

e∑k=1(
ak +bk )=99×2011i
sodd,wes tAi ,Bi
99
S
inc eet
ha

a
red
iff
eren
tsub
set
sof{
1,2,...,2010}.WhenAi t
akea
ll99
e
lemen
tss
ubs
e f{
to 1,2,...,2010},
soa
reBi .Mor
eove
r

P(
Ai)+P (
Bi)
= a1a2 …a99 + (
2011 -a1)(
2011 -a2)…(
2011 -a99)
≡ a1a2 …a99 + (-a1)(-a2)…(-a99)(
mod2011)
≡ 0(
mod2011).
254 Ma
thema
tic
alOl
ymp
iadi
nCh
ina

s,
Thu
n n n

2∑P (
Ai)= ∑P (Ai)+ ∑P (Bi)≡ 0(mod2011),
i=1 i=1 i=1

hence2011 ∑P (A ).
i=1
i

So
lut
i tf(
on2.Le n -1)(
n)= ( n -2)…(
n -2010)-n2010 -
Mathematical Olympiad in China (2011–2014) Downloaded from www.worldscientific.com

2010!,whe
by NATIONAL UNIVERSITY OF SINGAPORE on 05/06/18. For personal use only.

ren ∈ Z.
S
i e2011i
nc ime,by Fe
spr rma
t􀆳sLi
ttl em,n2010 ≡
eTheor
1(
mod 2011). By Wi
lson
􀆳 em, we have 2010! ≡
s Theor
-1(
mod2011).Thu
s,
(
i)I
f2011 n,
then

f(
n)≡ (
n -1)(
n -2)…(
n -2010)≡ 0(
mod2011).

(
ii)I
f2011|n,
then

f(
n)≡ (
2011 -1)(2011 -2)…(2011 -2010)-20112010 -2010!
≡2010! -2010!(mod2011)
≡ 0(
mod2011).

Sof(
n)≡ 0(
mod2011)ha
s2011s
olu
tionsi
nthes
ens
eof
mod2011.
S
incef(
n)i
sapo
lynomi
a r2009,andf
loforde ora
lln ∈
Z,2011|f(
n),wes
eet
hateachcoe
ffi
ci toff(
en n)canbe
d
ivdedby2011.
i

em, ∑ P(
Ai) i
n
Turn t
o t
he or
igi
nal pr
obl i=1
s t
he

coe
ffi
cien
t of t
erm wi
t r 1911 of f(
h orde n), t
hus

∑ P(
Ai).
n
2011 i=1

3 Asshowni
nFi
g.3
.1.Le
tthei
nsc
ribedc
irc
leIof△ABC
t tD andF,
ouchBCandABa res
pec
tiv
el tIi
y.Le nte
rse
ctt
he
Ch
inaSou
the
ast
ern Ma
thema
tic
alOl
ymp
iad 255

s
egmensAD andCFa
t tH and
K ,r
espec
tiv
ely.Prove t
hat
FD × HK
= 3.
FH × DK
So
lut
ion 1. Suppos
etha
tthe
l
eng
thsofs
egmen
tsa
reAF = x,
BF = y, CD = z,t
Mathematical Olympiad in China (2011–2014) Downloaded from www.worldscientific.com

hen by
F
i .3
.1
by NATIONAL UNIVERSITY OF SINGAPORE on 05/06/18. For personal use only.

g
St
ewa
rt􀆳 em,wehave
sTheor

BD · CD ·
AD2 = AC2 + AB2 -BD ·DC
BC BC
y(
x +z) +z(x +y)
2 2

= -yz
y +z
2 4xyz
=x + .
y +z

AF2 x2
ByTangen
t-Secan em,wehaveAH =
tTheor = .
AD AD
s,
Thu

AD2 -x2 4xyz


HD = AD - AH = = .
AD AD (
y +z)

S
imi
lar
ly,wehave

4xyz
KF = .
CF (
x +y)

S
i e△CDK ∽ △CFD ,wes
nc eet
hat

DF ×CD DF
DK = = z.
CF CF

Byt
hef
actof△AFH ∽ △ADF ,wehave

DF × AF DF
FH = = x.
AD AD
256 Ma
thema
tic
alOl
ymp
iadi
nCh
ina

Byt
heCo
sineLaw,

DF2 = BD2 +BF2 -2BD ·BFcosB


æ y +z) + (
( 2
x +y)2
-(
x +z)
2
ö÷
=2y ç1 -
2

è 2(x +y)(
y +z) ø
4xy2z
=
( .
x +y)(y +z)
Mathematical Olympiad in China (2011–2014) Downloaded from www.worldscientific.com

Thu
s
by NATIONAL UNIVERSITY OF SINGAPORE on 05/06/18. For personal use only.

4xyz 4xyz
·
KF × HD CF (
x +y) AD (
y +z)
=
FH × DK DF ·DF
x z
AD CF
16xy2z
= = 4.
DF (
2
x +y)( y +z)

App
lingPt
olemy
􀆳sTheor
emt
ocyc
licquadr
ila
ter
alDKHF ,
wehave

KF ·HD = DF ·HK +FH ·DK .

KF × HD FD × HK
Comb
ini
ngwi
th = 4,weob
tai
n = 3.
FH × DK FH × DK
So
lut
ion 2. Fi
rst we prove a
l
emma.
Lemma.Asshownin Fi
g.3
.2.
I
fthec
irc
let sAB andACa
ouche t
B and C ,r
espe
cti
vey,Q i
l sa
poin
tont hec
irc
le.AQ i
nte
rsec
ts F
ig.3
.2

the ci
rcl
e at point P, then
wehave

PQ ·BC = 2BP ·QC = 2BQ ·PC .

Proo
foft
hel
emma.ByPto
lemy
􀆳 em,wes
sTheor eet
hat

PQ ·BC = BP ·QC +BQ ·PC .


Ch
inaSou
the
ast
ern Ma
thema
tic
alOl
ymp
iad 257

S
ineABt
c ang
ent
stot
hec
irc
le,wes
eet
hat ∠ABP = ∠AQB.
Fur
t rby ∠BAP = ∠QAB ,wehave△ABP ∽ △AQB .
he
Cons
equen
tly,
BP AP AB
= = .
BQ AB AQ

s,
Thu
Mathematical Olympiad in China (2011–2014) Downloaded from www.worldscientific.com

æçBP ö÷2 AP AB AP
by NATIONAL UNIVERSITY OF SINGAPORE on 05/06/18. For personal use only.

= × = .
èBQ ø AB AQ AQ

S
imi
lar
ly,
æçCP ö÷2 AP
= .
èCQ ø AQ

æCP ö÷2 æBP ö÷2 ,


s,ç
Thu = ç tha
tisBP ·QC = BQ ·PC .So
èCQ ø èBQ ø

PQ ·BC = 2BP ·QC = 2BQ ·PC .

Thel
emmai
sproved.
Tu
rnt
otheor
igi
nalprob
lem.
Le
tci
rceIi
l nte
rse
ctACa
tpo
intO
anddr
aws
egmen
tsHO ,OK ,OD
andFO .Asshowni
nFi
g.3
.3.
By Pt
olemy
􀆳 em, we
s Theor
have
F
ig.3
.3

KF ·HD = DF ·HK +FH ·DK .

s,
Thu

FD × HK KF × HD
= 3⇔ = 4.
FH × DK FH × DK

S
ineCQandCDa
c ret
ang
entt
oci
rceI,byLemma,wes
l eet
hat

KF ·DO = 2DK ·FO ,


HD ·FO = 2FH ·DO .
258 Ma
thema
tic
alOl
ymp
iadi
nCh
ina

Mu
lti
plyabovetwoequa
tion
sbyeachs
ide,wehave

KF ·HD ·DO ·FO = 4DK ·FH ·DO ·FO ,

KF × HD
t
ha s,
ti = 4.
FH × DK

4 taandbbepo
Le sit
ivei
ntege
rss
ucht
hat1 ≤a <b ≤100.
Mathematical Olympiad in China (2011–2014) Downloaded from www.worldscientific.com

tab|(
ak +bk ),
by NATIONAL UNIVERSITY OF SINGAPORE on 05/06/18. For personal use only.

I
fthe
reex
ist
sapo
sit
ivei
nt rks
ege ucht
ha
t
henwes
ayt
hatt
hepa
i a,b)i
r( sgo
od.De
termi
net
he
numbe
rofgoodpa
irs.
So
lut
i t(
on.Le a,b)=d,a =sd,b =td,(
s,t)=1,
t >1,
td2|dk (
hens
t sk +tk ).Sok ≥2ands sk +tk ).S
t|dk-2( i e(
nc t,
s
sk +tk )= 1,wehaves
t|dk-2 .The
re e,anypr
for imef
act
orof
s
tcanbed
ivdedbyd.
i
I
fthe
rei
sapr
imef
act fsortnol
orp o es han11,t
st henp
d
ivi
de tp2 > 100,wh
sd.So p2 |a orp2 |b,bu ichi
sa
con
trad
ict
ion.
Sot
hepr
imef
act fs
oro t maybe2,3,5or7.
I
fthe
rea
rea
tlea
stt
hreepr
imef
act
or tamong2,3,5,
sofs
7,t
hent
her
eisapr
imef
act fsortnol
oro es han5.Andd >
st
2 × 3 × 5 = 30,s
ot ta orb ≥ 5d > 100,wh
ha ichi
sa
con
trad
ict
ion.The pr
imef
act
ors
etofs tbe {
t canno 3,7},
o
the
rwi
se,
aorb ≥ 7 ×3 ×7 > 100,wh
ichi
sacon
trad
ict
ion.
S
imi
lar
ly,t
hepr
imef
act
ors
etofs tbe {
tcanno 5,7}.
The
ref e,t
or hepr
imef
act
ors
e fs
to tcanon
lybe {
2},{
3},
{
5},{
7},{
2,3},{
2,5},{
2,7}or {
3,5}.
(
i)I
fthep
rimef
act
ors
e fs
to s{
ti 3,5},t
hend c
anon
lybe
15.Then,
s =3,
t =5.
Sot
her
eison
egoodp
air(
a,b)= (
45,7
5).
(
ii)I
fthepr
imef
act
ors
etofs
tis{
2,7},t
hend canon
ly
be14.Thens =2,
t =7ors =4,
t =7.Sot
her
ear
etwogood
Ch
inaSou
the
ast
ern Ma
thema
tic
alOl
ymp
iad 259

pa
irs(a,b)= (
28,98)and (
56,98).
(
ii)I
i fthepr
imefacorofs
t s{
ti 2,5},t
hend canon
lybe
10or20.
Ford = 10,
thens = 2,
t = 5;s = 1,
t = 10;s = 4,
t=
5;s = 5,
t = 8.
Ford = 20,
thens = 2,
t = 5;s = 4,
t = 5.
Mathematical Olympiad in China (2011–2014) Downloaded from www.worldscientific.com

The
rea
res
ixgoodpa
irs.
by NATIONAL UNIVERSITY OF SINGAPORE on 05/06/18. For personal use only.

(
iv)I
fthepr
imef
act fs
oro s{
ti 2,3},t
hend canon
lybe
6,12,18,24or30.
Ford =6,
s =1,
t =6;
s =1,
t =12;
s =2,
t =3;
s=
2,
t = 9;s = 3,
t = 4;s = 3,
t = 8;s = 3,
t = 16;s = 4,
t
= 9;s = 8,
t = 9;s = 9,
t = 16.
Ford =12,
s =1,
t =6;
s =2,
t =3;
s =3,
t =4;
s=
3,
t = 8.
Ford = 18,s = 2,
t = 3;s = 3,
t = 4.
Ford = 24,s = 2,
t = 3;s = 3,t = 4.d = 30,s = 2,
t = 3.
The
rea
re19goodpa
irs.
(
v)I
fthepr
imef
act
ors
etofs
tis{
7},t
hens =1,
t =7,d
canon
lybe7or14.
So,t
her
ear
etwogoodpa
irs.
(
vi)I
fthepr
imef
act
ors
e fs
to s{
ti 5},t
hens = 1,
t = 5,
dcanon
lybe5,10,15or20.So,t
her
ear
efourgoodpa
irs.
(
vii)I
fthepr
imef
act
ors
etofs
tis{
3}t
hen wehavet
he
fo
ll ng:
owi
whens = 1,
t = 3,dcanon
lybe3,6,...,or33;
whens = 1,
t = 9,dcanon
lybe3,6or9;
whens = 1,
t = 27,dcanon
lybe3.
The
rea
re15goodpa
irs.
(
vii)I
i fthepr
imef
act
ors
etofs
tis{
2},t
henwehavet
he
260 Ma
thema
tic
alOl
ymp
iadi
nCh
ina

fo
ll ng:
owi
whens = 1,
t = 2,dcanon
lybe2,4,...,or50;
whens = 1,
t = 4,
thendcanon
lybe2,4,...,or24;
whens = 1,
t = 8,
thendcanon
lybe2,4,...,or12;
whens = 1,
t = 16,
thendcanon
lybe2,4or6;
whens = 1,
t = 32,
thendcanon
lybe2.
Mathematical Olympiad in China (2011–2014) Downloaded from www.worldscientific.com

The
rea
re47goodpa
irs.
by NATIONAL UNIVERSITY OF SINGAPORE on 05/06/18. For personal use only.

The
re e,t
for her
ear
eal
ltoge
t r1 +2 +6 +19 +2 +4 +
he
15 +47 =96goodpa
irs.

S
econdDay
8:00 12:00,Augues
t18,2010

5 Ass
howni
nFi
g.5
.1.Le
tC
be t
he r
ight ang
le o
f
△ABC . M 1 and M 2 a
re
twoa
rbi
tra
ry po
int
sins
ide
△ABC , and M i
s t
he
mi
dpo
in f M 1M 2. The
t o
e
xten
sion
sofBM 1,BM and F
ig.5
.1
BM 2i
nte
rse
c tN1,N
tAC a
M 1N1 M 2N2 MN
andN2r
espe
cti
vel
y.Pr
ovet
hat + ≥2 .
BM 1 BM 2 BM
So
lut
ion.AsshowninFi
g.5
.2.
t H1 , H2 and H be t
Le he
pr
ojec
tionpoinsofM 1 ,M 2 and
t
M onlineBC ,
respec
tive
ly.Then

M 1N1 H1C ,
=
BM 1 BH1

F
ig.5
.2
Ch
inaSou
the
ast
ern Ma
thema
tic
alOl
ymp
iad 261

M 2N2 H2C ,
=
BM 2 BH2
MN HC H1C + H2C
= = .
BM BH BH1 +BH2

Suppo
set
hatBC = 1,BH1 = xandBH2 = y.Wehave

M 1N1 H1C 1 -x ,
= =
Mathematical Olympiad in China (2011–2014) Downloaded from www.worldscientific.com

BM 1 BH1 x
by NATIONAL UNIVERSITY OF SINGAPORE on 05/06/18. For personal use only.

M 2N2 H2C 1 -y ,
= =
BM 2 BH2 y
MN HC 1 -x +1 -y
= = .
BM BH x +y

s,t
Thu hei
nequa
lit
y wea
ret
oprovei
sequ
iva
len
tto

1 -x 1 -y 1 -x +1 -y
,
+ ≥2
x y x +y

1 1 4 ,
wh
ichi
sequ
iva
len
tto + ≥ t
ha s,(
ti x -y)
2
≥0
x y x +y
wh
ichi
sobv
iou
slyt
rue.

6 tN* bet
Le hes
etofpo
sit
ivei
ntege
rs.De
finea1 =2,and
forn = 1,2,...,

1 1 … 1 1
an+1 = mi

{ + + + + < 1,λ ∈ N* . }
a1 a2 an λ

Provet
ha 2
tan+1 = an orn = 1,2,....
-an +1f

1 1
So
lut
ion.Bya1 = 2,a2 = minλ { + < 1,λ ∈ N* ,}
a1 λ
1 1 1 1 1,
con
side
r + <1,
then <1- = λ >2,hencea2 =
a1 λ λ 2 2
3.Sot
heconc
lus
ioni
str orn = 1.
uef
Suppo
set
hatt
heconc
lus
ioni
str
uefora
lli
nt rn ≤ k -
ege
1 1 … 1 1
1(
k ≥ 2).I
fn =k,
thenak+1 = mi
nλ + {+ + +
a1 a2 ak λ
262 Ma
thema
tic
alOl
ymp
iadi
nCh
ina

< 1,λ ∈N } .Considering


*

1 1 … 1 1
+ + + + < 1,
a1 a2 ak λ

1 æ1 1 … 1 ö÷ ,
t
ha s,0 <
ti <1- ç + + + wehave
λ èa1 a2 ak ø
Mathematical Olympiad in China (2011–2014) Downloaded from www.worldscientific.com

1
by NATIONAL UNIVERSITY OF SINGAPORE on 05/06/18. For personal use only.

λ> .
1 1 … 1
1- - - -
a1 a2 ak

Int
hef
oll ng,wes
owi howt
hat

1
= ak (
ak -1).
1 1 … 1
1- - - -
a1 a2 ak

Byt hei
nduc
tion hypo
the
ses,f
or2 ≤ n ≤ k,an = an-1
an-1 -1)+1,wehave
(

1 1 1 1 ,
= = -
an -1 an-1(
an-1 -1) an-1 -1 an-1

1 1 1
t
her
efor
e = - .Byt
aki
ngt um,wehave
hes
an-1 an-1 -1 an -1
k
1 1 ,
∑a
i=2 i-1
=1-
ak -1
t
hati
s

k
1 1 1 1
∑a
i=1 i
=1- +
ak -1 ak
=1-
ak (
ak -1)
.

1
Con
sequen
tly, = ak (
ak - 1).
1 1 … 1
1- - - -
a1 a2 ak
The
re e,
for

1 1 … 1 1
ak+1 = mi
n λ| + {
+ + + < 1,λ ∈ N* }
a1 a2 ak λ
ak -1)+1.
= ak (
Ch
inaSou
the
ast
ern Ma
thema
tic
alOl
ymp
iad 263

Byi
nduc
tiononn,f
ora
llpo
sit
ivei
nt rn,wehave,an+1 =
ege
-an +1.
2
an

7 The
rea
re2
nrea
lnumbe
rsa1 ,a2 ,...,an ,r1 ,r2 ,...,
andrn s
ati
sfi
ynga1 ≤a2 ≤ … ≤an and0 ≤r1 ≤r2 ≤ …

∑ ∑ n(ri ,rj )≥ 0.
n n
≤rn .Provet
hat aiaj mi
Mathematical Olympiad in China (2011–2014) Downloaded from www.worldscientific.com

i=1 j=1
by NATIONAL UNIVERSITY OF SINGAPORE on 05/06/18. For personal use only.

So
lut
ion.Wr
iteama
tri
xofn ×ne
lemen
tsa
sfo
llows:

æa1a1r1 a1a2r1 a1a3r1 … a1anr1 ö


ç
… a2anr2 ÷
÷
ça2a1r1 a2a2r2 a2a3r2
ç
A1 = ça3a1r1 a3a2r2 a3a3r3 … a3anr3 ÷ .
÷
ç ︙ ︙ ︙ ⋱ ︙ ÷
ç ÷
èana1r1 ana2r2 ana3r3 … ananrn ø

S
ince
n n n n

∑ ∑aiajmin(
i=1 j=1
ri ,rj )= ∑a1ajmin(
j=1
r1 ,rj )+ ∑a2ajmi
n(r2 ,rj )
j=1

+… + ∑aa min(
j=1
r ,r )+ …
k j k j

+ ∑aa min(
j=1
r ,r ),
n j n j

i
tskt
hte
rmi
s
n

∑aa min(
j=1
r ,r )= aar
k j k j k 1 1 +aka2r2 + … +akakrk

+akak+1rk + … +akanrk

wh
ichi
sthes
um o
fel
emen
tso
fthekt fA1 ,k = 1,
hrow o
2,...,n.

e,∑i=1 ∑j=1aiajmi
n(ri ,rj )i
n n
The
refor sthes
um ofa
ll

e
lemen
tsofA1 .
264 Ma
thema
tic
alOl
ymp
iadi
nCh
ina

Ont
he o
t rhand,t
he hes
umma
tion can a
lso be donea
s
fo
llows:Taket
hee
lemen
tsoff
irs
tco
lumnandt
hef
irs
trowof
A1 ,
sumup;deno
tet
her
est(
n -1)× (
n -1)e
lemen
tby ma
tri
x
A2 ,
thent
aket
hef
irs
tco
lumnandt
hef
irtrowofA2 ,
s sumup,
deno
tet
her
estby ma
trxA3 ,...,s
i owege
t
n n n

∑ ∑aiajmin(
ri ,rj )= ∑r (a ak (
+2 ak+1 +ak+2 + … +an ))
2
Mathematical Olympiad in China (2011–2014) Downloaded from www.worldscientific.com

k k
by NATIONAL UNIVERSITY OF SINGAPORE on 05/06/18. For personal use only.

i=1 j=1 k=1

n n n

∑rk ( (ak + ∑ai ) - ( ∑a ) )


2 2
= i
k=1 i=k+1 i=k+1

n n n

∑r ( ( ∑a ) ( ∑a ) )
2 2
= k i - i
k=1 i=k i=k+1

n n n

( ∑a ) ( ∑a ) ( ∑a ) +…
2 2 2
=r1 i +r2 i +r3 i
i=1 i=2 i=3

n n n

( ∑a ) ( ∑a ) ( ∑a )
2 2 2
+rn i -r1 i -r2 i
i=n i=2 i=3

- … -rn-1 ( ∑a )
2
i
i=n

n n

∑( -rk-1) ( ∑a )
2
= r k i ≥0
k=1 i=k

(
whe
rer0 = 0).

8 Gi
vene
igh
tpo
insA1 ,A2 ,...,A8onac
t irc
le,de
termi
ne
t
hesma
lle
st po
sit
ivei
nt rn s
ege uch t
hatamong anyn
t
riang
leswi
thve
rti
cesi
nthe
see
ightpo
ins,t
t her
ear
etwo
wh
ichhaveacommons
ide.
So
lut
ion.Fi
rst,wecons
ide
rthe max
ima
lnumbe
roft
riang
les
wi
thnocommons
ide.
Con
side
rthemax
ima
lnumbe
roft
riang
leswi
thnocommon
s
idepa
irwi
se.The
rea
r 8 = 2
eC2 8chord
sby conne
cti
ng e
igh
t
Ch
inaSou
the
ast
ern Ma
thema
tic
alOl
ymp
iad 265

poi
nts.Ifeachchordonlybe
long
sto
onetri
angle,thenthes
echordscan

on
l ormr ≤
yf [238] =9triangleswith
no common s
ide pai
rwise. Bu
ti f
the
reareninesuch t
riangl
es,then
t
her
eae27ve
r rti
ces.So,t
her
eisone
Mathematical Olympiad in China (2011–2014) Downloaded from www.worldscientific.com
by NATIONAL UNIVERSITY OF SINGAPORE on 05/06/18. For personal use only.

po
inti
nei
ght po
ins, wh
t ichi
sthe
F
ig.8
.1
common ve
rtex offou
rtr
iang
les.
Suppo
set
hatpo
in sA8 ,t
ti hen e
ightedge
sar
econne
ctedt
o
s
evenpo
insA1 ,A2 , ...,A7 .So,t
t her
e mu
stex
istanedge
A8Ak ,wh
ichi
sthecommons
ideo
ftwot
riang
les,wh
ichi
sa
con
trad
ict
ion.Sor ≤ 8.
Ont
heo
t rhand,whenr = 8,wecan makes
he uche
ight
t
riang
les,s
eef
igur
e.Deno
tet
het
riang
lesbyt
hreeve
rti
ce s:
sa
(
1,2,8),(
1,3,6),(
1,4,7),(
2,3,4),(
2,5,7),(
3,5,
8),(
4,5,6)and (
6,7,8).Sot
hemi
nima
lnumbe
ro s9.
fni

2011 (
Ningbo,Zhei
jang)

F
irs
tDay
8:00 12:00,Ju
ly27,2011

ax2 +b
fmi
1 I nx∈R = 3,f
ind
x2 +1
(
1)t
her
angeofb;
266 Ma
thema
tic
alOl
ymp
iadi
nCh
ina

(
2)t
heva
lueofaforg
ivenb.(
pos
edbyLuXi
ngi
jang)
ax2 +b
So
lut
i ef(
on1.Denot x)= I
.ti
sea
syt
oseet
hata >
2
x +1
0.Byf(
0)=b,wes
eet
hatb ≥ 3.
(
i)I a ≥ 0,
fb -2

ax2 +b b -a
f(
x)= = a x +1 + b -a) = 3,
≥ 2 a(
2
Mathematical Olympiad in China (2011–2014) Downloaded from www.worldscientific.com

x +1
2
x2 +1
by NATIONAL UNIVERSITY OF SINGAPORE on 05/06/18. For personal use only.

b -a , b -2a
e
qua
li
tyho
ld fa x2 +1 =
si tha
tis,
ifx = ± .
x +1
2 a

b - b2 -9,
Theva
l faf
ueo org
ivenbi
sa = es
pec
ial
ly
2
3
whenb = 3,a = .
2
(
ii)I a <0,
fb -2 let x2 +1 =t(
t ≥1).f(
x)=g(
t)=
b -a
a
t+ mono
ton
ica
llyi
ncr
eas
ingwhent ≥ 1,s
o,
t

3
nf(
mi x)= g(
1)= a +b -a =b = 3,whena > .
x∈R 2

ngup,wege
Summi t(1)t s[
angeofbi
her 3,+ ∞ ).

( 3 b - b2 -9
2)I
fb =3,
thena ≥ ;fb >3,
i thena = .
2 2
ax2 +b
So
lut
i tf(
on2.Le x)= I
.ti
sea
syt
oseet
hata > 0.
2
x +1
2
ax +b
S
incemi
nx∈R = 3,andf(
0)=b,wes
eet
hatb ≥ 3.
x2 +1

æ 2 b -2 a ö÷
ax çx -
a
'(x)=
è ø
f .
(x2 +1)
3/2

(
i)I a ≤0,
fb -2 le '(
tf x)=0,wehavet
hes
olu
tionx0 =
0,andi
fx <0,
t '(
henf x)<0;andi
fx >0,
t '(
henf x)>0.
Ch
inaSou
the
ast
ern Ma
thema
tic
alOl
ymp
iad 267

b
f(
0)=bi
sthemi
nima
lva
lue.Henceb = 3,anda ≥ .
2
(
ii)I a >0,
fb -2 le '(
tf x)=0,wehavet
hes
olu
tionsx0 =

b -2a
0,x1,2 = ± .
a
I
tisea
syt
oseef(
0)= bi
sno
tthe mi
nima
lva
lue,wh
ich
imp
lisb > 3;andf(
e x1,2)
ist
hemi
nima
lva
lue
Mathematical Olympiad in China (2011–2014) Downloaded from www.worldscientific.com
by NATIONAL UNIVERSITY OF SINGAPORE on 05/06/18. For personal use only.

b -2a
a· +b
( ) a
x
f 12
, = = 3⇒2 a(
b -a) = 3
b -2a
+1
a

9 b - b2 -9,
⇒a2 -ab + = 0⇒a =
4 2

b - b2 -9
t
ha s,
ti b > 3anda = .
2
ngup,wege
Summi t
(
1)t
her s[
fbi
angeo 3,+ ∞ ).

( 3 b - b2 -9
2)I
fb =3,
thena ≥ ;fb >3,
i thena = .
2 2

2 ta,bandcb
Le ecop
rimepo
sit
ivei
nte
ger
ssot
hata2|(
b3 +
c3),b2|(
a3 +c3)andc2|(
a3 +b3).Fi
ndt
heva
l sofa,
ue
bandc.(
pos
edby Yang Xi ng)
aomi
So
lut
ion.Bythecond
iti
onoft
heprob
lem,wehavea2|(
a3 +
b3 +c3),b2 | (
a3 +b3 +c3)andc2| (
a3 +b3 +c3).S
incea,b
andca
r ime,wes
ecopr eet
hata2b2c2 | (
a3 +b3 +c3).
Wi
thou
tlo
ssofgene
ral
ity,s
uppo
set
hata ≥b ≥c,s
o

b2c2 ,
3
a3 ≥ a3 +b3 +c3 ≥ a2b2c2 ⇒a ≥
3
268 Ma
thema
tic
alOl
ymp
iadi
nCh
ina

and

b4c4 18
2
b3 ≥b3 +c3 ≥ a2 ⇒2
b3 ≥ ⇒b ≤ 4 .
9 c

Wes
eet
ha fc ≥ 2⇒b ≤ 1,t
ti her
esu
ltcon
trad
icsb ≥c.
t
s,
Thu c = 1.
fc = 1andb = 1,
I thena = 1,
s a,b,c)= (
o( 1,1,1)
is
Mathematical Olympiad in China (2011–2014) Downloaded from www.worldscientific.com
by NATIONAL UNIVERSITY OF SINGAPORE on 05/06/18. For personal use only.

as
olu
tion.
fc = 1,b ≥ 2anda = b,t
I henb2 |b3 + 1,wh
ichi
sa
con
trad
ict
ion!
fb ≥ 2anda >b >c = 1,
I then
2
b
a2b2 | (
a3 +b3 +1)⇒2a3 ≥ a3 +b3 +1 ≥ a2b2 ⇒a ≥ ,
2

andbyc = 1,
4
b
a2 | (
b3 +1)⇒b3 +1 ≥ a2 ≥ ⇒4
b3 +4 ≥b4 .
4

Forb >5,
thei
nequa
lit
yha
snos
olu
tion.Takeb =2,3,4,
5,wes
eet
hatt
hes
olu
tion
saec = 1,b = 2,a = 3.
r
The
ref e,a
or lls
olu
tionsa
re(
a,b,c)= (
1,1,1),(
1,2,
3),(
1,3,2),(
2,1,3),(
2,3,1),(
3,2,1)and (
3,1,2).

3 Le
tstM = {
e 1,2,3,...,50}.Fi
nda
llpo
sit
ivei
ntege
r
n,
sucht
hatt
her
ear
eatl
eas
ttwod
iff
eren
tel
emen
tsaand
bi
nanys
ubs
etwi
th35e
lemen
tsofM ,
sucht
hata +b =n
ora -b = n.(
poedbyLiShenghong)
s
So
lut
ion.T
akeA ={
1,2,3,...,35},
thenf
oranya,b ∈ A ,

a -b ≤ 34,a +b ≤ 34 +35 = 69.

Int
hefo
ll ng,wes
owi howt
ha tA = {
t1 ≤ n ≤ 69.Le a1 ,
Ch
inaSou
the
ast
ern Ma
thema
tic
alOl
ymp
iad 269

a2 ,...,a35},wi
thou
tlo
ssofgene
ral
ity,s
uppo
set
hata1 < a2
< … < a35 .
(
i)I
f1 ≤ n ≤ 19,by

1 ≤ a1 < a2 < … < a35 ≤ 50,


2 ≤ a1 +n < a2 +n < … < a35 +n ≤ 50 +19 = 69,

andby Di
rich
let
􀆳sDr
awe em,t
rTheor her
eex
ist1 ≤i,j ≤ 35
Mathematical Olympiad in China (2011–2014) Downloaded from www.worldscientific.com

(
i ≠j)s
ucht
hatai +n = aj ,
tha
tis,
ai -aj = n.
by NATIONAL UNIVERSITY OF SINGAPORE on 05/06/18. For personal use only.

(
ii)I
f51 ≤ n ≤ 69,by

1 ≤ a1 < a2 < … < a35 ≤ 50,


1 ≤ n -a35 < n -a34 < … < n -a1 ≤ 68,

andby Di
rich
let
􀆳sDr
awe em,t
rTheor her
eex
ista
tlea
st1 ≤i,
j ≤ 35(i ≠j)suchtha tn -ai = aj ,
tha
tis,
ai +aj = n.
(i
ii)If20 ≤ n ≤ 24,s
ince

50 - (
2n +1)+1 = 50 -2n ≤ 50 -40 = 10,
wes
eet
hatt
her
ear
eatl
eat25e
s lemen
t na1 ,a2 ,...,a35t
si hat
be
l o[
ongt 1,2
n].
The
rea
rea
tmo
st24e
lemen
t n{
si 1,n +1},{
2,n +2},...,
{
n,2
n}s
ucht
hat{
ai ,aj }= {
i,n +i}.Henc
e,aj -ai =n.
(
iv)I
f25 ≤n ≤34,
since{
1,n +1},{
2,n +2},...,{
n,
n}havea
2 tmo
st34e
lemen
ts,by Di
rich
letDr
awe em,
rTheor
t
her
eex
ist1 ≤i,j ≤ 35(
i ≠j)s
ucht
hatai =i,aj = n +i,
t
ha saj -ai = n.
ti
(
v)I
fn =35,
the
rea
re33e
lemen
ts{
1,34},{
2,33},...,
{
17,18},{
35},{
36},...,{
50}.Hence,t
her
eex
ist1 ≤i,
j≤
35(
i ≠j)s
ucht
hatai +aj = 35.
(
vi)I
f36 ≤ n ≤ 50,
k + 1,{
fn = 2
i k},{
1,2 2,2 k,k + 1},
k - 1},...,{
{
2k +1},...,{
50};
f18 ≤k ≤20,50- (
i 2k +1)+1 =50-2
k ≤50-36 =14;
270 Ma
thema
tic
alOl
ymp
iadi
nCh
ina

f21 ≤k ≤24,50- (
i 2k +1)+1 =50-2
k ≤50-42 =8,
t
her
eex
ist1 ≤i,
j ≤35(
i ≠j)
sucht
hatai +aj =2
k +1 = n.
k,{
fn = 2
I 1,2
k -1},{
2,2
k -2},...,{
k -1,k +1},
{
k},{
2k},{
2k +1},...,{
50};
f18 ≤k ≤19,50- (
i 2k +1)+3 ≤16
k -1 ≤19-1 =18;
f20 ≤k ≤ 23,50 - (
i 2k +1)+3 ≤ 50 -2
k +2 ≤ 12
k -1
Mathematical Olympiad in China (2011–2014) Downloaded from www.worldscientific.com

≤23 -1 = 22;
by NATIONAL UNIVERSITY OF SINGAPORE on 05/06/18. For personal use only.

f24 ≤k ≤25,50- (
i 2k +1)+3 ≤50-2
k +2 ≤4
k -1 ≤
25 -1 = 24,
t
her
eex
ist1 ≤i,j ≤ 35(
i ≠j)
sucht
hatai +aj = 2
k.

4 Suppo
set
hatal
inepa
ssi
ngt
hec
ircumc
ent
reO of△ABC
i
nte
rsec
tsABandACa
tpo
insM andN ,
t res
pec
tive
ly,and
E andF a
ret
he mi
dpo
int
sofBN andCM ,r
espec
tive
ly.
Provet
hat ∠EOF = ∠A .(
pos
edbyTaoP
i sheng)
ng
So
lut
ion.Wes
howt
hatt
hea
bov
econ
clu
sioni
str
uef
oranyt
riang
le.
f △ABC i
I sri
ght
-ang
led.Theconcl
usi
onisobv i
ous.In
f
ac ,
ts eeFi
g.4 ,
.1 where ∠ABC = 90 ,
°.So thec
ircumcen
treO
i
sthemi
dpo
intofAC ,OA = OB andN = O .S
inceF i
sthe
mi
dpo
intofCM ,wes
eet
hatt
hemed
i i
anlneOF ‖AM .Hence
∠EOF = ∠OBA = ∠OAB = ∠A .

f△ABCi
I sno
tri
ght
-ang
led,s
eeFi
g.4
.2andFi
g.4
.3.

F
ig.4
.1 F
ig.4
.2 F
ig.4
.3
Ch
inaSou
the
ast
ern Ma
thema
tic
alOl
ymp
iad 271

Fi
rstweg
iveal
emma.
Lemma.Le
tA andB betwo po
int
sont
hed
iame
terKL of
c
irc
le☉O wi
thr
adi
usR ,andOA = OB = a.Seef
igur
e.
Let CD and EF be two chords pas
sing A and B ,
r
espe
cti
vely.Suppo
seCE and DF i
nte
rs tKL a
ec t M and N ,
r
espe
cti
vey.ThenMA = NB .
l
Mathematical Olympiad in China (2011–2014) Downloaded from www.worldscientific.com

Proo
foft
hel
emma.Asshownin
by NATIONAL UNIVERSITY OF SINGAPORE on 05/06/18. For personal use only.

F
ig.4
.4.Suppo
set
hatCD ∩ EF = P .
Th
inko
ftha
tli
nesCE andDFi
nte
rse
ct
△PAB.By Mene
lau
s􀆳The
orem,wehav
e

AC ·PE ·BM
= 1,
CP EB MA
BF ·PD ·AN .4
.4
= 1. F
ig
FP DA NB

Then

MA AC ·AD ·PE ·PF ·BM


= . ①
NB BE BF PC PD AN

Byt
heIn
ter
sec
ti em,wege
ngChordTheor t

PC ·PD = PE ·PF . ②

So

AC ·AD = AK ·AL = R2 -a2 = BK ·BL = BE ·BF .


MA MB ,
By ① ,③ ,wehave = tha
tis
NB NA

MA MA + AB AB
= = = 1.
NB NB + AB AB

s,MA = NB .
Thu
Now,r
eturn t
othe or
igi
nal prob
lem,s
ee F
ig.4
.5 and
272 Ma
thema
tic
alOl
ymp
iadi
nCh
ina

Fi.4
g .6.Ex
tendMN tod i
amet
erKK1 ,takepo
intM 1 onKK1
s
ucht tOM 1 = OM .Le
ha tCM 1 ∩ ☉O = A1 ,andletA1B
i
nte
rse
ctKK1 a
tN1 .Byt emma,MN1 = M 1N (
hel orM 1N1 =
MN ont
her
igh
tfi e).So,Oi
gur sthemi
dpo
in s,
tofNN1 .Thu
OE and OF a
ret
he med
ianl
ine of △NBN1 and △MCM 1 ,
r
espe
cti
vel s,wehave ∠EOF = ∠BA1C = ∠A .
y.Thu
Mathematical Olympiad in China (2011–2014) Downloaded from www.worldscientific.com
by NATIONAL UNIVERSITY OF SINGAPORE on 05/06/18. For personal use only.

F
ig.4
.5 F
ig.4
.6

S
econdDay
8:00 12:00,Ju
ly28,2011

5 tAA0 ,BB0 andCC0 beangu


Le larb
isec
t sof △ABC .
or
tA0A1 ‖BB0 andA0A2 ‖CC0 ,whe
Le reA1 andA2l
ieon
AC andAB,r
espe
cti
vey,
l
andl
etl
ineA1A2i
nte
rse
ct
BC a
tA3.The po
insB3
t
and C3 a
re ob
tai
ned
s
imi
lar
ly. Pr
ove t
hat
po
ins A3, B3, C3 a
t re
co
lli r.(
nea pos
edby Tao
P
i sheng)
ng
So
lut
ion. By the Mene
lau
s
F
ig.5
.1
Ch
inaSou
the
ast
ern Ma
thema
tic
alOl
ymp
iad 273

Inve
rs em,weneedon
eTheor lyt
oshowt
hat

AB3 ·CA3 ·BC3


= 1. ①
B3C A3B C3A

S
inc
elne A1A2A3 i
i nte
rsec
ts △ABC , by Mene
lau
s􀆳
CA3 ·BA2 ·AA1
em,wehave
Theor = 1.So
A3B A2A A1C
Mathematical Olympiad in China (2011–2014) Downloaded from www.worldscientific.com
by NATIONAL UNIVERSITY OF SINGAPORE on 05/06/18. For personal use only.

CA3 A2A ·A1C


= . ②
A3B BA2 AA1

S
imi
lar
ly,wehave

AB3 B2B ·B1A ,


= ③
B3C CB2 BB1
BC3 C2C ·C1B
= . ④
C3A AC2 CC1

BC0 · AA0 ·
ByBA2 = BA0 andAA2 = AC0 ,wehave
BC AI

AA2 AA0 ·AC0 ·BC


= . ⑤
BA2 BA0 ·BC0 AI

AA0· CA0·
Mor r,byAA1 =
eove AB0andCA1 = CB0 ,we
AI CB
have

A1C CA0 ·CB0 ·AI


= . ⑥
AA1 AA0 ·AB0 BC

Thenby ② ,⑤ and ⑥ ,wehave

CA3 CA0 ·AC0 ·CB0 æCA0 ÷ö2


= = ç .
A3B BA0 BC0 AB0 èA0B ø

S
imi
lar
ly,wehave

CB3 æ CB0 ö÷2 ,


= ç
B3A èB0A ø
274 Ma
thema
tic
alOl
ymp
iadi
nCh
ina

AC3 æAC0 ö÷2


= ç . ⑦
C3B èC0B ø

S
inc
ethr
eeang
leb
ise
ct sAA0 ,BB0 ,CC0 o
or f△ABC a
re
concur
r t,byCeva
en 􀆳 em,wehave
sTheor

AB0 ·CA0 ·BC0


= 1. ⑧
B0C A0B C0A
Mathematical Olympiad in China (2011–2014) Downloaded from www.worldscientific.com
by NATIONAL UNIVERSITY OF SINGAPORE on 05/06/18. For personal use only.

s,by ⑦ and ⑧ ,wehave


Thu

AB3 ·CA3 ·BC3 æAB0 ·CA0 ·BC0 ö÷2


= ç = 1,
B3C A3B C3A èB0C A0B C0A ø

t
hati
s①.

6 vennpo
Gi insP1 ,P2 ,...,Pn onap
t lane,l
etM beany
po
intons tAB ont
egmen hep
lane.Deno
teby|PiM |t
he
d
ist
ancebe enPi andM ,
twe i = 1,2,3,...,
n.Pr
ovet
hat

∑ x{ ∑i=1|PiA |,∑i=1|PiB|} .
n n n
i=1
| PiM |≤ ma
(
pos
edbyJ
i i)
n Mengwe
So
lut
ion.Le
tO bet
heor
i →
n.Then wehaveOM
i →
= tOA +
g
1 -t)O→
( B,t∈(
0,1).
→ →
|PiM | =|OM -OPi |
→ 1 -t)O→
=|tOA + ( B -tOP→ -( 1 -t)OP→
i i|

→ → 1 -t)|O→
≤t|OA -OPi |+ ( B -OP→
i|

=t|PiA |+ (
1 -t)|PiB |.

e,
Henc
n n n
→ →
∑ |P M | ≤t∑ |P A |+ (1 -t)∑ |P B |
i=1
i
i=1
i
i=1
i

n n

≤ max { ∑ |P A |,∑ |P B |} .
i=1
i
i=1
i
Ch
inaSou
the
ast
ern Ma
thema
tic
alOl
ymp
iad 275

7 Suppo
set
hatt
hes e{
equenc an }de
finedbya1 = a2 = 1,
an =7an-1 -an-2 ,n ≥ 3.Provet
hatan + an+1 + 2i
sa
per
fectsquareforanypo
sit
ivei
nt rn.(
ege pos
edby Tao
Pi
ngsheng )
So
lut
ion.I
tiswe
llknownt
hatt
hes
olu
tionoft
hes
equence
canbeob
tai
nedbys
olv
ingtwogeome
tri
csequenc tan
e.Wege
Mathematical Olympiad in China (2011–2014) Downloaded from www.worldscientific.com

= C1λ1 +C2λ2 ,wher


e
n n
by NATIONAL UNIVERSITY OF SINGAPORE on 05/06/18. For personal use only.

7 + 45 æ3 + 5 ö2 7 - 45 æ3 - 5 ö2
λ1 = = ç ÷ , λ2 = = ç ÷ ,
2 è 2 ø 2 è 2 ø

λ1 andλ2a
res
olu
tion
soft
heequa
ti λ +1 =0,a1 =
onλ2 -7
1 = C1λ1 + C2λ2 ,a2 = 1 = C1λ2 λ2
1 +C2 2.

The
re e,
for

1 C1 (
an +an+1 +2 =λn 1 )+λ2 C2 ( 2 )+2
-1
λ1 +λ2 n-1
λ2 +λ2
=λ1- +λ2- +2
n 1 n 1

n-1 2
æ æ3 + 5 ö æ æ - 5 ön-1 ö2
÷ + ç ç3
ö
= çç ÷ ÷ ÷ +2
èè 2 ø ø èè 2 ø ø
n-1 n-1 2
é æ3 + 5 ö æ3 - 5 ö
ú = xn ,
ùú 2
= êê ç ÷ +ç ÷
ëè 2 ø è 2 ø û

ön-1 æ3 - 5 ön-1
exn = ç3 + 5 ÷
æ
whe
r + ç ÷ i
sthes
olu
tionoft
he
è 2 ø è 2 ø
s xn }ofpo
equence { sit
ivei
ntege
rsx1 =2,x2 =3,xn =3xn-1 -
xn-2 ,n ≥ 3.

8 Cons
i r12f
de igur
esont
hec
lockf s12po
acea int
s.Co
lor
t
hemi
nfou
rco
l s:r
or ed,ye
llow,b
lueandg
reen.Each
col
or isu s
ed for three poi
n t
s. Conf
igure n convex
quadr
ila
ter
alswi
t hve
r t
icesinthesepo
int
ss, ucht
hat
(
1)t
her
ear
enos
amec
olo
rofv
ert
ice
sfo
rea
chquad
ril
ate
ral.
(
2)amonganyt
hreeoft
hes
e quadr
ila
ter
als,t
her
eisa
276 Ma
thema
tic
alOl
ymp
iadi
nCh
ina

co
loro
fve
rti
cess
ucht
hatt
heve
rti
cesoft
hatco
lor
a
red
iff
eren
t.
F
indt
hel
arge
stnumbe
rofn.(
pos
edbyTaoP
i sheng)
ng
So
lut
i eA ,B ,C,D t
on.Weus orepr
esen
tthe
sefourco
l s,
or
r
espec
tive
ly,andt
hepo
int
sint
hes
ameco
lorbyl
owe
rle
tte
rsa
s
a1,a2,a3;b1,b2,b3;c1,c2,c3 andd1,d2,d3r
espe
cti
vel
y.
Mathematical Olympiad in China (2011–2014) Downloaded from www.worldscientific.com

Nowcon
side
rco
l f,i
orA .I nn quadr
ila
ter
als,t
henumbe
r
by NATIONAL UNIVERSITY OF SINGAPORE on 05/06/18. For personal use only.

ofpo
insa1 ,a2 ,a3i
t nco
lorA a
ren1 ,n2 ,n3r
espe
cti
vey,t
l hen
n1 +n2 +n3 = n.Suppo
set
ha fn ≥ 10,
tn1 ≥n2 ≥n3 .I then
n1 +n2 ≥ 7.Cons
ide
rthe
ses
even quadr
ila
ter
als(
itsA co
lor
ve
rtexi
sei
t ra1 ora2 ),i
he fthenumbe
rsofpo
insb1 ,b2 ,b3i
t n
co
lorB a
rem 1 ,m 2 ,m 3 ,
res
pec
tive
ly,t
henm 1 +m 2 +m 3 =7.
Bysymmet
ric
ity,wemays uppo
set
hatm 1 ≥ m 2 ≥ m 3 ,
thenm 3
≤ 2,t
ha s,m 1 + m 2 ≥ 5.
ti
Con
side
rthe
sef
ivequadr
ila
ter
als(
isAco
t lorpo
inti
sei
the
r
a1 ora2 ,andi
tsBco
lorpo
inti
sei
t rb1orb2 ),i
he fthenumbe
rs
ofpo
insc1 ,c2 ,c3i
t nco
lorC a
rek1 ,k2 ,k3 ,r
espe
cti
vey,t
l hen
k1 +k2 +k3 =5.Bys
ymme
tri
ciy,wemays
t uppo
set
hatk1 ≥k2
≥k3 ,
thenk3 ≤ 1,
tha
tis,
k1 +k2 ≥ 4.
Con
side
rthe
sef
our quadr
ila
ter
als,deno
t s T1 ,T2 ,
ed a
T3 ,T4 (
it
sco
lorA po
inti
sei
t ra1 ora2 ,
he itsco
lorB po
inti
s
e
it rb1 orb2 ,andi
he tsco
lorC po
inti
sei
t rc1 orc2 ).S
he ince
t
her
ear
e on
lyt
hree po
int
sin co
lor D ,t
her
e a
re two
quadr
ila
ter
alstha
thavet hes
amecoorD po
l int.Suppo
set
hat
thesamecolorD po
in fT1 ,T2i
to sd1 .
Then,i
nthr
eequadr
ila
ter
alsT1 ,T2 ,T3 ,wha
teve
rbet
he
col
or oft he vertex, the
re a
re repea
ted po
ins, wh
t ich
contr
adi
ctscond
iti ( )
on 2 .Henc ,
e n ≤ 9.
Wes
how t
he max
ima rn = 9by cons
lnumbe truc
tion of
t
hes
eni
nequadr
ila
ter
als.
Ch
inaSou
the
ast
ern Ma
thema
tic
alOl
ymp
iad 277
Mathematical Olympiad in China (2011–2014) Downloaded from www.worldscientific.com
by NATIONAL UNIVERSITY OF SINGAPORE on 05/06/18. For personal use only.

Wedr
aw t
hree “
conc
ent
ricannu
lus”wi
thf
our po
int
son
eachr
adi
usr
epr
esen
tingf
ourve
rti
cesandt
heco
lor.So n
ine
r
adi
irepr
esen
tni
nequadr
ila
ter
als,wh
ichs
ati
sfycond
iton (
i 1).
t,wes
Nex howt
hatt
heya
lsos
ati
sfycond
iton (
i 2).Take
anyt
hreer
adi(
i ort
hreequadr
ila
ter
als).
I
fthe
set
hreer
adi
icomef
rom aconcen
tri
cannu
lus,f
or
eachco
lorexc
eptA ,
the
rea
ret
hreepo
int
s.
I
fthe
set
hreer
adi
icomef
rom t
hree concen
tri
c annu
li,
t
henf
orco
lorA ,
the
rea
ret
hreepo
int
s.
I
fthe
set
hreer
adi
icomef
romtwoconcen
tri
cannu
li,ca
ll
t
he set hr
ee fi
gures Fi
g.1, F ig.2 and Fi.3. The r
g adius
dir
ectionsar
e cal
led “up radius”, “l
eftradi
us”and “r
ight
radis”,anddeno
u ted,res
pe c
tively,byS,Z andY .Ifthree
r
adi
ihavet
hreed
ire
cti
ons,t
hent
her
ear
ethr
eepo
int
sofco
lor
Bi
nthr
ee quadr
ila
ter
als.I
fthet
hree r
adi
ihave on
ly two
d
ire
cti
ons,t
hent
her
ear
eal
lca
sesa
sshowni
nthet
abl
esbe
low,
whe
re1,2and3s
tandf
orF
ig.1,F
ig.2andF
ig.3,r
espe
cti
vel
y.
He
re, t
he co
lor i
n f
igur
e means t
hat t
he t
hree
quadr
ila
ter
alshaveco
lorwi
thd
iff
eren
tfi
gur
es.

S 1,21,2 2 1 S 1,21,2 1 2

Z 1 1,21,2 2 Z 2 1,21,2 1

Y 2 1 1,21,2 Y 1 2 1,21,2

C D
278 Ma
thema
tic
alOl
ymp
iadi
nCh
ina

S 1,31,3 1 3 S 1,31,3 3 1

Z 3 1,31,3 1 Z 1 1,31,3 3

Y 1 3 1,31,3 Y 3 1 1,31,3

C D

S 2,32,3 3 2 S 2,32,3 2 3

2 2,32,3 3 3 2,32,3 2
Mathematical Olympiad in China (2011–2014) Downloaded from www.worldscientific.com

Z Z
by NATIONAL UNIVERSITY OF SINGAPORE on 05/06/18. For personal use only.

Y 3 2 2,32,3 Y 2 3 2,32,3

C D

s,t
Thu hemax
ima
lnumbe
ro s9.
fni

2012 (
Put
ian,Fui
jan)

F
irs
tDay
8:00 12:00,Ju
ly27,2012

1 F
ind at
ril
pe(
l,m ,n)(
1 < l < m < n)o
f po
sit
ive

t ∑k=1k, ∑k=l+1k, ∑k=m+1k f


l m n
i
ntege
rss
ucht
ha orm a

geome
trics
equencei r.(
norde po
sedbyTaoPing
sheng)
t(
t +1)

You might also like